You are on page 1of 146

‫ﺍﻝﺠﻤﻬﻭﺭﻴﺔ ﺍﻝﺠﺯﺍﺌﺭﻴﺔ ﺍﻝﺩﻴﻤﻘﺭﺍﻁﻴﺔ ﺍﻝﺸﻌﺒﻴﺔ‬

‫ﻭﺯﺍﺭﺓ ﺍﻝﺘﺭﺒﻴﺔ ﺍﻝﻭﻁﻨﻴﺔ‬

‫ﺍﻝﺩﻴﻭﺍﻥ ﺍﻝﻭﻁﻨﻲ ﻝﻠﻤﻁﺒﻭﻋﺎﺕ ﺍﻝﻤﺩﺭﺴﻴﺔ‬

‫ﺍﻝﺭﻴﺎﻀﻴﺎﺕ‬
‫ﺍﻝﺴﻨﺔ ﺍﻝﺜﺎﻝﺜﺔ ﻤﻥ ﺍﻝﺘﻌﻠﻴﻡ ﺍﻝﺜﺎﻨﻭﻱ ﺍﻝﻌﺎﻡ‬
‫ﻭ ﺍﻝﺘﻜﻨﻭﻝﻭﺠﻲ‬

‫ﻤﻔﺘﺵ ﺍﻝﺘﺭﺒﻴﺔ ﻭﺍﻝﺘﻜﻭﻴﻥ‬ ‫ﻤﺤﻤﺩ ﻓﺎﺘﺢ ﻤﺭﺍﺩ‬ ‫ﺍﻝﻤﺅﻝﻔﻭﻥ‪:‬‬


‫ﻤﻔﺘﺵ ﺍﻝﺘﺭﺒﻴﺔ ﻭﺍﻝﺘﻜﻭﻴﻥ‬ ‫ﺠﻤﺎل ﺘﺎﻭﺭﻴﺭﺕ‬
‫ﻤﻔﺘﺵ ﺍﻝﺘﺭﺒﻴﺔ ﻭﺍﻝﺘﻜﻭﻴﻥ‬ ‫ﻤﺤﻤﺩ ﻗﻭﺭﻴﻥ‬
‫ﺃﺴﺘﺎﺫ ﺍﻝﺘﻌﻠﻴﻡ ﺍﻝﺜﺎﻨﻭﻱ‬ ‫ﻋﺒﺩ ﺍﻝﺤﻔﻴﻅ ﻓﻼﺡ‬
‫ﺃﺴﺘﺎﺫ ﺍﻝﺘﻌﻠﻴﻡ ﺍﻝﺜﺎﻨﻭﻱ‬ ‫ﻋﺒﺩ ﺍﻝﻤﺅﻤﻥ ﻤﻭﺱ‬
‫ﺃﺴﺘﺎﺫ ﺍﻝﺘﻌﻠﻴﻡ ﺍﻝﺜﺎﻨﻭﻱ‬ ‫ﻏﺭﻴﺴﻲ ﺒﻠﺠﻴﻼﻝﻲ‬
‫ﺍﻝﺸﻌﺏ‪ • :‬ﺭﻴﺎﻀﻴﺎﺕ‬
‫ﺘﻘﻨﻲ ﺭﻴﺎﻀﻲ‬ ‫•‬

‫ﻋﻠﻭﻡ ﺘﺠﺭﻴﺒﻴﺔ‬ ‫•‬

‫ﺍﻝﺠﺯﺀ ﺍﻷﻭل‬
‫ﺍﻷﻧﺸﻄﺔ‬
‫ا
ط اول‬
‫‪: /‬‬
‫ا
ف‪ :‬ر
 م  
  

  د ‪.‬‬
‫ ت‪ :‬م ا ‪-‬ط آ*) ('ة "  
  
   " و ‪ #‬إ!ز  أاج‪.‬‬
‫ا
‪./ :‬‬
‫ا
ط ا
‬
‫ ‪/ :‬‬
‫ا
ف‪ :‬ر
 
دا
'آ‪.
1‬‬
‫ ت‪ :‬م ا ‪-‬ط آ*) ('ة "  ت دا
'آ‪ "
1‬و و ‪ #‬إ!ز  أاج آ ‪ #‬ا‪67‬ل ‪ 4‬ز ا ا‪.23‬‬
‫ا
‪./ :‬‬
‫ا
ط ا

‬
‫ ‪/ :‬‬
‫ا
ف‪ :‬ر
 م ا‪'79‬ار
‪.‬‬
‫ ت‪ :‬م ا ‪-‬ط آ*) ('ة " ا‪'79‬ار
" و ‪  #‬أاج آ ‪ #‬ا‪:;7‬ل ‪ 4‬ز ا ا‪.23‬‬
‫ا
‪../ :‬‬
‫ا
ط ا
ا‬
‫ ‪/ :‬‬
‫ا
ف‪ :‬ر
‪'1‬ه
ا ‪ #‬ا ‪
<7‬و ‪. 31<3‬‬
‫ ت‪ :‬م ا ‪-‬ط آ*) ('ة " ‪'1‬ه
ا ‪ #‬ا ‪ "
<7‬و ‪  #‬أاج‪.‬‬
‫ا
‪./ :‬‬

‫ﺍﻷﻋﻤﺎﻝ ﺍﻟﻤﻮﺟﻬﺔ‬
‫إزا
‪$ %
& %‬م ا
"! ‬
‫ ‪/ :‬‬
‫ا
ف‪ >?3 :‬ا ت‪.‬‬
‫ ت‪ :‬م ا ‪ )@2  )6‬أاج ‪.‬‬
‫ا
‪./ :‬‬
‫إ‪,-‬د &
 ‪!+‬د
‪)  "
 %‬‬
‫ ‪/ :‬‬
‫ا
ف‪'1 >?3 :‬ه
ا ‪ #‬ا ‪.
<7‬‬
‫ ت‪ #3 @ :‬ا ‪ )@2  )6‬أاج ‪ A‬ا‪67‬ل ‪ 4‬ز ا ا‪.23‬‬
‫ا
‪ @ :‬إ‪13‬ع ‪ >(F‬ا 'ا) ‪(1‬غ ا ‪ CD‬ا *ة‪.,‬‬

‫ﺍﻟﺘﻤﺎﺭﻳﻦ‬
‫‪2‬ر‪% . /0 -‬‬
‫‪ 1‬ـ ‪ % " + %-‬أو ‪ +∞  $ % " +  4‬أو ∞‪−‬‬

‫‪ 2,9 < f ( x ) < 3,1 (1 1‬ﻤﻌﻨﺎﻩ )‪ 2,9 ( x + 1) < 3x − 2 < 3,1( x + 1‬ﻷﻥ ‪x + 1 > 0‬‬
‫ﻭ ﻤﻨﻪ ‪2,9 x + 4,9 < 3 x < 3,1x + 5,1‬‬ ‫ﻭ ﻤﻨﻪ ‪2,9 ( x + 1) + 2 < 3x < 3,1( x + 1) + 2‬‬
( 2,9 x + 4,9 − 3x < 0 ) ‫ﻭ‬ ( 3x − 3,1x − 5,1 < 0 ) ‫ﻭ ﻤﻨﻪ‬
 4, 9   5,1 
A = 49 ‫ﺇﺫﻥ‬  x > 0,1  ‫ ﻭ‬ x > −0,1  ‫ﻭ ﻤﻨﻪ‬ ( −0,1x < −4,9 ) ‫ ( ﻭ‬−0,1x < 5,1) ‫ﻭ ﻤﻨﻪ‬
   
lim f ( x ) = 3 (2
x →+∞

3x − 2 −5
f ( x) − 3 = −3 = (3
x +1 x +1
. ∆ ‫ ﺃﺴﻔل‬C f ‫ ﻭ ﻤﻨﻪ‬f ( x ) − 3 < 0
f ( x ) − y ‫ ﻨﻡ ﻝﺩﺭﺍﺴﺔ ﺍﻝﻭﻀﻌﻴﺔ ﻨﺩﺭﺱ ﺇﺸﺎﺭﺓ‬lim  f ( x ) − y  ‫ ﻭ‬lim  f ( x ) − y  ‫ ﻨﺤﺴﺏ‬11
x →+∞ x →−∞

.10 ‫ ﻭ‬9 ، 8 ‫ ﻨﻔﺱ ﺍﻝﻁﺭﻴﻘﺔ ﻤﻊ ﺍﻝﺘﻤﺎﺭﻴﻥ‬:‫ﻤﻼﺤﻅﺔ‬


. .& ‫د‬$  $ % " +  4 ‫ أو‬% " + %- ‫ ـ‬2

، lim f ( x ) = 3 13
x→4

x+2
2,95 ( x − 2 ) − 2 ≤ x ≤ 3, 05 ( x − 2 ) − 2 ‫ ﻭ ﻤﻨﻪ‬2,95 ≤ ≤ 3, 05 ‫ ﻴﻜﺎﻓﺊ‬2,95 ≤ f ( x ) ≤ 3, 05
x−2
 7, 9   8,1 
3, 951219512... ≤ x ≤ 4, 051282051... ‫ ﺇﺫﻥ‬،  x ≤  ‫ ﻭ‬x≥ ‫ﺃﻱ‬
 1,95   2, 05 
I = ] 3,95; 4, 05[ ‫ﻴﻤﻜﻥ ﺃﺨﺫ‬
1000 x 2 − 4003 x + 3996 < 0 ‫ﻭ ﻤﻨﻪ‬ 3 x + 4 > 103 ( x − 2 ) ‫ ﻤﻌﻨﺎﻩ‬f ( x ) > 103 ، lim f ( x ) = +∞ 14
2

x→2

4003 − 40009 4003 + 40009


. 1,901488751 < x < 2,101511249 ‫ﻭ ﻤﻨﻪ‬ <x< ‫ﻭ ﻤﻨﻪ‬
2000 2000
a = 0,1 ‫ﻴﻤﻜﻥ ﺃﺨﺫ‬
‫ت‬-
‫ ا‬9:$ ‫ت‬2" ‫ ـ‬3

+∞ ‫ﻋﻨﺩ‬ −∞ ‫ﻋﻨﺩ‬ ‫ﺍﻝﻨﻬﺎﻴﺔ‬ 18

+∞ −∞ (‫ﺃ‬

−∞ −∞ (‫ﺏ‬

−∞ +∞ (‫ﺝ‬
lim f ( x ) = −∞ ،
> →−1
lim
<
f ( x ) = +∞ ، lim f ( x ) = 1 ، lim f ( x ) = 1 (‫ ﺃ‬19
x  x →−1 x →+∞ x →−∞

lim f ( x ) = +∞ ، lim
>→ 2 <
f ( x ) = −∞ ، lim f ( x ) = +∞ ، lim f ( x ) = −∞ (‫ﺏ‬
x  x → 2 x →+∞ x →−∞

lim f ( x ) = +∞ ، lim
>→ 3 <
f ( x ) = −∞ ، lim f ( x ) = 4 ، lim f ( x ) = 4 (‫ﺝ‬
x  x → 3 x →+∞ x →−∞

lim f ( x ) = 0 ، lim f ( x ) = 0 (‫ﺃ‬ 20


x →+∞ x →−∞

lim f ( x ) = +∞ ، lim f ( x ) = 0 ، lim f ( x ) = 0 (‫ﺏ‬


x→2 x →+∞ x →−∞

. lim f ( x ) = +∞ ، lim f ( x ) = +∞ ، lim f ( x ) = −∞ (‫ﺝ‬


x →0 x →+∞ x →−∞

lim f ( x ) = +∞ ، lim− f ( x ) = −∞ ، lim f ( x ) = +∞ ، lim f ( x ) = −∞ (‫ﺃ‬ 21


x →0+ x →0 x →+∞ x →−∞
lim f ( x ) = −∞ ، lim f ( x ) = 3 ، lim f ( x ) = 3 (‫ﺏ‬
x →−1 x →+∞ x →−∞

lim f ( x ) = −∞ ، lim− f ( x ) = +∞ ، lim f ( x ) = +∞ ، lim f ( x ) = +∞ (‫ﺝ‬


x →3+ x →3 x →+∞ x →−∞

، lim− f ( x ) = −∞ ، lim f ( x ) = 0 ، lim f ( x ) = 0 (‫ ﺃ‬22


x →1 x →+∞ x →−∞

lim f ( x ) = −∞ ، lim− f ( x ) = +∞ ، lim+ f ( x ) = +∞


x → 4+ x→ 4 x →1

lim f ( x ) = −∞ ، lim f ( x ) = +∞ ، lim f ( x ) = −∞ (‫ﺏ‬


x →−1− x →+∞ x →−∞

lim f ( x ) = +∞ ، lim− f ( x ) = −∞ ، lim+ f ( x ) = +∞


x →3+ x →3 x →−1

. lim f ( x ) = +∞ ، lim f ( x ) = +∞ ، lim f ( x ) = +∞ (‫ﺝ‬


x →−2 x →+∞ x →−∞

lim f ( x ) = +∞ (‫ﺃ‬ 26
x →+∞

 2
x 1 + 
x+2  x  :‫ ﻝﺩﻴﻨﺎ‬x > 0 ‫ﺏ( ﻤﻥ ﺃﺠل‬
، lim f ( x ) = lim = lim
x →+∞ x →+∞ 3 − x x →+∞  3 1 
x − 
x x
2
1+
lim f ( x ) = lim x =1
x →+∞ x →+∞ 3 1 3

x x
lim f ( x ) = lim x + 1 − x − 1 ( ‫ ﺃ‬28
x →+∞ x →+∞

= lim
( x +1 − x −1 )( x +1 + x −1 ) = lim
x +1− x +1
= lim
2
=0
x →+∞ x +1 + x −1 x →+∞ x +1 + x −1 x →+∞ x +1 + x −1

lim − x = +∞ ‫ ﻭ‬lim x 2 − x + 1 = +∞ ‫ ﻷﻥ‬lim f ( x ) = lim x 2 − x + 1 − x = +∞ (‫ﺏ‬


x →−∞ x →−∞ x →−∞ x →−∞

 1
x  −1 + 
x − x +1− x
2 2
 x
= lim = lim
x →+∞  
x →+∞
x2 − x + 1 + x 1 1
x  1 − + 2 + 1
 x x 

lim f ( x ) = lim
( x2 − x + 1 − x )(
x2 − x + 1 + x
= lim
) 

1
 −1 + 
x
=−
1
x →+∞  
x →+∞ x →+∞
x − x +1 + x
2
1 1 2
 1 − + 2 + 1
 x x 

D = ℝ − {0; 2} :(1)‫ ﺍﻝﺤﺎﻝﺔ‬29


، lim− f ( x ) = −∞ ، lim+ f ( x ) = −∞ ، lim− f ( x ) = +∞ ، lim f ( x ) = 0 ، lim f ( x ) = 0
x→ 2 x →0 x →0 x →+∞ x →−∞

lim f ( x ) = +∞
x → 2+
D = ℝ − {0} :(2)‫ﺍﻝﺤﺎﻝﺔ‬
lim f ( x ) = +∞ ، lim− f ( x ) = −∞ ، lim f ( x ) = +∞ ، lim f ( x ) = +∞
x →0+ x →0 x →+∞ x →−∞

، lim+ f ( x ) = +∞ ، lim− f ( x ) = −∞ lim f ( x ) = +∞ ، lim f ( x ) = −∞ D = ℝ − {−1;1} :(3)‫ﺍﻝﺤﺎﻝﺔ‬


x →−1 x →−1 x →+∞ x →−∞

lim f ( x ) = +∞ ، lim− f ( x ) = −∞
x →1+ x →1
%‫ر‬.2
 ‫ت‬-
‫ – ا‬%/‫آ‬+ %
‫ دا‬%- ‫ ـ‬4

3x + 4 3x + 4
lim
>
‫ ﺇﺫﻥ‬lim X = +∞ ‫ ﻭ‬lim>
= +∞ (1 30
x → 3 x −3 X →+∞ x → 3 x − 3

lim x 2 + x + 1 = +∞ ‫ ﺇﺫﻥ‬، lim X = +∞ ‫ ﻭ‬lim x 2 + x + 1 = +∞ (3


x →+∞ X →+∞ x →+∞

lim −2 x + x − 3 = +∞ ‫ ﺇﺫﻥ‬، lim


3
X = +∞ ‫ ﻭ‬lim − 2 x3 + x − 3 = +∞ (4
x →−∞ X →+∞ x →−∞

lim
X →0 +
+
(
X = 0 ‫ ﻭ‬lim 4 − x
x→2
2
)=0 +
‫ ﻭ‬lim 4 − x
x →−2
( 2
)=0 +
‫ ﻭ ﻤﻨﻪ‬4 − x > 0 ] − 2; 2 [ ‫ ﻋﻠﻰ‬:‫ ﻝﺩﻴﻨﺎ‬: 31
2

−3 −3
lim = −∞ ‫ ﻭ‬lim = −∞ ‫ﺇﺫﻥ‬
x→2
4 − x2 x →−2
4 − x2
x+4 1
= lim = 0 :‫( ﻝﺩﻴﻨﺎ‬1 32
lim
x −3
2 x →+∞ x x →+∞

 x+4 
lim cos  2  = 1 ‫ ﺇﺫﻥ‬، lim cos X = 1 ‫ﻭ‬
x →+∞
 x −3 X →0

 π x −1  π x −1 πx π
lim cos   = 0 ‫ ﺇﺫﻥ‬، lim cos X = 0 ‫ ﻭ‬lim = lim = (2
π
x →+∞
 2x  X→
2
x →+∞ 2x x →+∞ 2 x 2
 π   π  π
lim sin  − x  = 1 ‫ ﺇﺫﻥ‬، lim sin X = 1 ‫ ﻭ ﻤﻨﻪ‬lim  − x  = (3
π
x →−1
 2  X→
2
x →−1
 2  2
 π  1 1
lim sin  − x + = +∞ : ‫ ﻭ ﺒﺎﻝﺘﺎﻝﻲ‬lim = +∞ ‫ﻭ‬
 ( x + 1) ( x + 1)
2 2
x →−1
 2 x →−1

 sin x   sin x   sin x 


lim cos  π  = −1 ‫ ﺇﺫﻥ‬، lim cos X = −1 ‫ﻭ‬ lim  π  = π ‫ ﻭ ﻤﻨﻪ‬lim   = 1 (4
X →π
x →0
 x  x →0
 x  x → 0
 x 
1
lim f ( x ) = 3 ‫ ﺃﻱ‬lim f ( x ) − 3 = 0 ‫ ﻭ ﻤﻨﻪ‬lim 2 = 0 ‫ ﻝﺩﻴﻨﺎ‬35
x →+∞ x →+∞ x →+∞ x + 1

lim f ( x ) = −∞ ‫ ﻓﺈﻥ‬f ( x ) ≤ −2 x3 ‫ ﻭ ﺒﻤﺎ ﺃﻥ‬lim − 2 x3 = −∞ ‫ ﺩﻴﻨﺎ‬36


x →+∞ x →+∞

1 4 1 
lim f ( x ) = +∞ ‫ ﻓﺈﻥ‬f ( x ) ≥ x + x ‫ ﻭ ﺒﻤﺎ ﺃﻥ‬lim  x 4 + x  = +∞ ‫ ﻝﺩﻴﻨﺎ‬37
x →+∞ 2 
x →+∞ 2

1 ≤ 3 + 2 cos x ≤ 5 ‫ ﻭ ﻤﻨﻪ‬−2 ≤ 2 cos x ≤ 2 ‫ ﻭ ﻤﻨﻪ‬−1 ≤ cos x ≤ 1 ‫( ﻝﺩﻴﻨﺎ‬1 38
1 1
≤ ≤ 1 ‫ ﺘﻜﺎﻓﺊ‬1 ≤ 3 + 2 cos x ≤ 5 . x − 1 → +∞ ‫ ﻓﺈﻥ‬x → +∞ : ‫( ﺇﺫﺍ ﻜﺎﻥ‬2
5 3 + 2 cos x
x −1 x −1 x −1 x −1
lim = +∞ ‫ ﻭ ﺒﺎﻝﺘﺎﻝﻲ‬lim = lim x − 1 = +∞ : ‫ ﻝﺩﻴﻨﺎ‬. ≤ ≤ x − 1 ‫ﻭ ﻤﻨﻪ‬
x →+∞ 3 + 2 cos x x →+∞ 5 x →+∞ 5 3 + 2 cos x
‫‪ x 2 − 3sin x − ( x 2 − 3) = −3sin x + 3 = 3 (1 − sin x ) (1 39‬ﺒﻤﺎ ﺃﻥ ‪ −1 ≤ sin x ≤ 1‬ﻓﺈﻥ ‪ −1 ≤ − sin x ≤ 1‬ﻭ‬

‫(‬ ‫)‬
‫ﻤﻨﻪ ‪ ، 0 ≤ 1 − sin x ≤ 2‬ﺇﺫﻥ ‪x 2 − 3sin x − x 2 − 3 ≥ 0‬‬
‫ﻭ ﺒﺎﻝﺘﺎﻝﻲ ‪x 2 − 3sin x ≥ x 2 − 3 :‬‬
‫‪ lim x 2 − 3 = +∞ (2‬ﻭ ‪ x 2 − 3sin x ≥ x 2 − 3‬ﻓﺈﻥ ∞‪lim x 2 − 3sin x = +‬‬
‫∞‪x →+‬‬ ‫∞‪x →+‬‬

‫‪ • 40‬ﻋﻨﺩ ∞‪ −1 ≤ sin x ≤ 1 : ( x > 0 ) +‬ﻭ ﻤﻨﻪ ‪ −2 x ≤ 2 x sin x ≤ 2 x‬ﻭ ﻤﻨﻪ‬


‫‪x 2 − 2 x ≤ x 2 + 2 x sin x ≤ x 2 + 2 x‬‬
‫ﺒﻤﺎ ﺃﻥ ∞‪ lim x 2 − 2 x = lim x 2 + 2 x = +‬ﻓﺈﻥ ∞‪. lim f ( x ) = +‬‬
‫∞‪x →+‬‬ ‫∞‪x →+‬‬ ‫∞‪x →+‬‬

‫• ﻋﻨﺩ ∞‪ −1 ≤ sin x ≤ 1 : ( x < 0 ) −‬ﻭ ﻤﻨﻪ ‪ 2 x ≤ 2 x sin x ≤ −2 x‬ﻭ ﻤﻨﻪ‬


‫‪x 2 + 2 x ≤ x 2 + 2 x sin x ≤ x 2 − 2 x‬‬
‫ﺒﻤﺎ ﺃﻥ ∞‪ lim x 2 − 2 x = lim x 2 + 2 x = +‬ﻓﺈﻥ ∞‪. lim f ( x ) = +‬‬
‫∞‪x →−‬‬ ‫∞‪x →−‬‬ ‫∞‪x →−‬‬

‫‪x − 1 x + sin x x + 1‬‬


‫≤‬ ‫≤‬ ‫‪ (1 41‬ﻝﺩﻴﻨﺎ ‪ −1 ≤ sin x ≤ 1 : :‬ﻭ ﻤﻨﻪ ‪ x − 1 ≤ x + sin x ≤ x + 1‬ﻭ ﻤﻨﻪ‬
‫‪2x +1‬‬ ‫‪2x +1‬‬ ‫‪2x + 1‬‬
‫‪1‬‬ ‫‪x −1‬‬ ‫‪x +1 1‬‬
‫‪ lim‬ﻓﺈﻥ = ) ‪lim f ( x‬‬ ‫‪= lim‬‬ ‫‪ (2‬ﺒﻤﺎ ﺃﻥ ‪= :‬‬
‫∞‪x →+‬‬ ‫‪2‬‬ ‫‪x →+∞ 2 x + 1‬‬ ‫‪x →+∞ 2 x + 1‬‬ ‫‪2‬‬
‫‪ 5‬ـ ا‪2"DE‬ار‪%-‬‬

‫; ‪ f ( x ) = x 2 − 2 x + 1‬‬ ‫‪x≤2‬‬


‫‪‬‬ ‫‪ 43‬ﻝﺘﻜﻥ ﺍﻝﺩﺍﻝﺔ ‪ f‬ﺍﻝﻤﻌﺭﻓﺔ ﻋﻠﻰ ‪ ℝ‬ﻜﻤﺎ ﻴﻠﻲ‪:‬‬
‫; ‪ f ( x ) = x + x − 5‬‬ ‫‪x>2‬‬
‫‪2‬‬

‫ﻭ ‪. f ( 2 ) = 1‬ﺇﺫﻥ ﺍﻝﺩﺍﻝﺔ ‪ f‬ﻤﺴﺘﻤﺭﺓ ﻋﻨﺩ ‪ 2‬ﻋﻠﻰ ﺍﻝﻴﺴﺎﺭ‬ ‫‪lim f ( x ) = lim x 2 − 2 x + 1 = 1 (1‬‬


‫<‬ ‫<‬
‫‪x → 2‬‬ ‫‪x → 2‬‬

‫ﻭ ‪. f ( 2 ) = 1‬ﺇﺫﻥ ﺍﻝﺩﺍﻝﺔ ‪ f‬ﻤﺴﺘﻤﺭﺓ ﻋﻨﺩ ‪ 2‬ﻋﻠﻰ ﺍﻝﻴﻤﻴﻥ‬ ‫‪lim f ( x ) = lim x 2 + x − 5 = 1‬‬


‫>‬ ‫>‬
‫‪x → 2‬‬ ‫‪x → 2‬‬

‫ﻭ ﻤﻨﻪ ﺍﻝﺩﺍﻝﺔ ‪ f‬ﻤﺴﺘﻤﺭﺓ ﻋﻨﺩ ‪. 2‬‬


‫‪ (2‬ﻨﻌﻡ ﺍﻝﺩﺍﻝﺔ ‪ f‬ﻤﺴﺘﻤﺭﺓ ﻋﻠﻰ ‪ ℝ‬ﻷﻨﻬﺎ ﻤﺴﺘﻤﺭﺓ ﻋﻠﻰ [‪) ]−∞; 2‬ﻜﺜﻴﺭ ﺤﺩﻭﺩ( ﻭ ﻋﻠﻰ [∞‪) ]2; +‬ﻜﺜﻴﺭ ﺤﺩﻭﺩ( ﻭ ﻤﺴﺘﻤﺭﺓ ﻋﻨﺩ ‪.2‬‬
‫‪ 6‬ـ ‪/+‬ه ‪ %‬ا
‪ F .‬ا
‪%0D"2‬‬
‫‪5‬‬ ‫‪ 1 3‬‬ ‫‪1‬‬ ‫‪3‬‬
‫‪f ( −1) = −‬‬ ‫‪، f  −  = ، f ( 0 ) = − ، f (1) = (1 52‬‬
‫‪4‬‬ ‫‪ 2 8‬‬ ‫‪4‬‬ ‫‪4‬‬
‫‪ 1  ‬‬ ‫‪1‬‬
‫‪ (2‬ﻨﻁﺒﻕ ﻤﺒﺭﻫﻨﺔ ﺍﻝﻘﻴﻡ ﺍﻝﻤﺘﻭﺴﻁﺔ ﻋﻠﻰ ﺍﻝﻤﺠﺎﻻﺕ ‪  − ;0  ،  −1; ‬ﻭ ]‪. [ 0;1‬‬
‫‪ 2  ‬‬ ‫‪2‬‬
‫‪ • 56‬ﺒﻤﺎ ﺃﻥ ‪ f‬ﻤﺴﺘﻤﺭﺓ ﻭ ﺭﺘﻴﺒﺔ ﺘﻤﺎﻤﺎ ﻋﻠﻰ ]‪ ]−3; 0‬ﻭ ﺘﺄﺨﺫ ﻗﻴﻤﻬﺎ ﻓﻲ [ ∞ ‪ [ −2; +‬ﻭ ﺒﻤﺎ ﺃﻥ [ ∞ ‪ 0 ∈ [ −2; +‬ﻓﺈﻥ‬
‫ﺍﻝﻤﻌﺎﺩﻝﺔ ‪ f ( x ) = 0‬ﺘﻘﺒل ﺤﻼ ﻭﺍﺤﺩﺍ ﻓﻲ ﺍﻝﻤﺠﺎل ]‪]−3; 0‬‬
‫ﻓﺈﻥ ﺍﻝﻤﻌﺎﺩﻝﺔ‬ ‫• ﺒﻤﺎ ﺃﻥ ‪ f‬ﻤﺴﺘﻤﺭﺓ ﻭ ﺭﺘﻴﺒﺔ ﺘﻤﺎﻤﺎ ﻋﻠﻰ ] ‪ [ 0; 2‬ﻭ ﺘﺄﺨﺫ ﻗﻴﻤﻬﺎ ﻓﻲ ] ‪ [ −2; 4‬ﻭ ﺒﻤﺎ ﺃﻥ ] ‪0 ∈ [ −2; 4‬‬
‫‪ f ( x ) = 0‬ﺘﻘﺒل ﺤﻼ ﻭﺍﺤﺩﺍ ﻓﻲ ﺍﻝﻤﺠﺎل ] ‪[0; 2‬‬
‫ﺇﺫﻥ ﺍﻝﻤﻌﺎﺩﻝﺔ ‪ f ( x ) = 0‬ﺘﻘﺒل ﺤﻠﻴﻥ ‪ x0‬ﻭ ‪ x1‬ﺤﻴﺙ ‪ −3 < x0 < 0‬ﻭ ‪0 < x1 < 2‬‬
‫‪ 7‬ـ ا
وال ا
‪2"I2‬ة و ا
 ‪+2 %/‬‬
‫‪ lim f ( x ) = +∞ (1‬ﻭ ∞‪lim f ( x ) = −‬‬ ‫‪64‬‬
‫∞‪x →+‬‬ ‫∞‪x →−‬‬

‫‪ (2‬ﺍﻝﺩﺍﻝﺔ ‪ f‬ﺘﻘﺒل ﺍﻻﺸﺘﻘﺎﻕ ﻋﻠﻰ ‪ ، ℝ‬ﻭ ﻤﻥ ﺃﺠل ﻜل ﻋﺩﺩ ﺤﻘﻴﻘﻲ ‪f ' ( x ) = −3x 2 + 6 x x‬‬
‫‪ f ' ( x ) < 0‬ﻤﻌﻨﺎﻩ ) ‪ ( x < 0‬ﺃﻭ ) ‪، ( x > 2‬‬ ‫ﺃ( ‪ f ' ( x ) = 0‬ﻤﻌﻨﺎﻩ ) ‪ ( x = 0‬ﺃﻭ ) ‪، ( x = 2‬‬
‫)‪( 0 < x < 2‬‬ ‫‪ f ' ( x ) > 0‬ﻤﻌﻨﺎﻩ‬

‫‪x‬‬ ‫∞‪-‬‬ ‫‪0‬‬ ‫‪2‬‬ ‫∞‪+‬‬ ‫ﺏ(‬


‫∞‪+‬‬ ‫‪3‬‬
‫)‪f ( x‬‬
‫‪-1‬‬ ‫∞‪−‬‬
‫ﻜل ﻤﺠﺎل ﻤﻥ ﺍﻝﻤﺠﺎﻻﺕ ]‪[ 2;3] ، [0;1] ، [ −1; 0‬‬ ‫‪ (3‬ﻨﻁﺒﻕ ﻤﺒﺭﻫﻨﺔ ﺍﻝﻘﻴﻡ ﺍﻝﻤﺘﻭﺴﻁﺔ ﻋﻠﻰ‬
‫‪ 7 3‬‬
‫‪ 67‬ﻨﻌﺘﺒﺭ ﺍﻝﺩﺍﻝﺔ ) ‪ h : x ֏ f ( x ) − g ( x‬ﻭ ﻨﻁﺒﻕ ﻤﺒﺭﻫﻨﺔ ﺍﻝﻘﻴﻡ ﺍﻝﻤﺘﻭﺴﻁﺔ ﻋﻠﻰ ﺍﻝﻤﺠﺎل ‪ − 8 ; − 4 ‬‬
‫‪2‬ر‪.J2!":
-‬‬
‫‪ 2‬ـ ‪ % " + %-‬أو ‪$  $ % " +  4‬د &‪. .‬‬

‫‪ c = −1 ، b = 3 ، a = 2 71‬ﻭ ‪d = −1‬‬
‫‪ c = 3 ، b = 1 ، a = 1 (1 72‬ﻭ ‪d = −1‬‬
‫‪3‬‬ ‫‪1‬‬ ‫‪3‬‬ ‫‪1‬‬
‫‪lim  f ( x ) − ( x + 1)  = lim‬‬ ‫‪−‬‬ ‫‪= 0 ، f ( x) = x +1+‬‬ ‫‪−‬‬ ‫‪(2‬‬
‫‪x →−∞ x + 1‬‬
‫)‪( x + 1‬‬ ‫‪x + 1 ( x + 1)2‬‬
‫∞‪x →−‬‬ ‫‪2‬‬

‫‪3‬‬ ‫‪1‬‬
‫‪lim  f ( x ) − ( x + 1)  = lim‬‬ ‫‪−‬‬ ‫‪=0‬‬
‫‪x →+∞ x + 1‬‬
‫)‪( x + 1‬‬
‫∞‪x →+‬‬ ‫‪2‬‬

‫ﺍﻝﻤﻤﺜل ﻝﻠﺩﺍﻝﺔ ‪ f‬ﻴﻘﺒل ﻤﺴﺘﻘﻴﻤﺎ ﻤﻘﺎﺭﺒﺎ ﻤﺎﺌﻼ ∆ ﻋﻨﺩ ∞‪ −‬ﻭ ﻋﻨﺩ ∞‪ +‬ﻤﻌﺎﺩﻝﺘﻪ ‪y = x + 1‬‬ ‫ﺇﺫﻥ ﺍﻝﻤﻨﺤﻨﻲ ) ‪( C‬‬
‫‪3‬‬ ‫‪1‬‬ ‫‪3x + 2‬‬
‫= )‪f ( x ) − ( x + 1‬‬ ‫‪−‬‬ ‫=‬ ‫‪ (3‬ﻨﺩﺭﺱ ﺇﺸﺎﺭﺓ )‪: f ( x ) − ( x + 1‬‬
‫)‪x + 1 ( x + 1‬‬ ‫‪2‬‬
‫)‪( x + 1‬‬
‫‪2‬‬

‫‪2‬‬ ‫‪2‬‬
‫‪، x<−‬‬ ‫‪ f ( x ) − ( x + 1) = 0‬ﺘﻜﺎﻓﺊ ‪ f ( x ) − ( x + 1) < 0 ، x = −‬ﺘﻜﺎﻓﺊ‬
‫‪3‬‬ ‫‪3‬‬
‫‪2‬‬
‫‪ f ( x ) − ( x + 1) > 0‬ﺘﻜﺎﻓﺊ ‪x > −‬‬
‫‪3‬‬
‫‪‬‬ ‫‪2‬‬ ‫‪ 2‬‬ ‫‪‬‬
‫) ‪ ( C‬ﺃﻋﻠﻰ ∆ ﻓﻲ ﺍﻝﻤﺠﺎل ‪  − ; +∞ ‬ﻭ ) ‪ ( C‬ﺃﺴﻔل ∆ ﻓﻲ ﺍﻝﻤﺠﺎﻝﻴﻥ [‪ ]−∞; −1‬ﻭ ‪.  −1; − ‬‬
‫‪‬‬ ‫‪3‬‬ ‫‪ 3‬‬ ‫‪‬‬
‫‪ lim f ( x ) = +∞ (1 73‬ﻭ ‪lim  f ( x ) − ( x + 2 )  = 0‬‬
‫∞‪x →+‬‬ ‫∞‪x →+‬‬

‫‪ ∆ : y = x + 2 (2‬ﻤﺴﺘﻘﻴﻡ ﻤﻘﺎﺭﺏ ﻤﺎﺌل ﻝﻠﻤﻨﺤﻨﻲ ) ‪ ( C‬ﻋﻨﺩ ∞‪. +‬‬


‫‪ (3‬ﺃ( ∞‪lim f ( x ) = +‬‬
‫∞‪x →−‬‬

‫‪ 4 5‬‬ ‫‪ 4 5 ‬‬


‫‪x 2 1 + + 2 ‬‬ ‫‪− x 1 + + 2 ‬‬
‫)‪f ( x‬‬ ‫‪x + 4x + 5‬‬
‫‪2‬‬
‫‪ x x ‬‬ ‫‪ x x ‬‬
‫‪lim‬‬ ‫‪= lim‬‬ ‫‪= lim‬‬ ‫‪= lim‬‬ ‫ﺏ( ‪= −1‬‬
‫∞‪x →−‬‬ ‫‪x‬‬ ‫∞‪x →−‬‬ ‫‪x‬‬ ‫∞‪x →−‬‬ ‫‪x‬‬ ‫∞‪x →−‬‬ ‫‪x‬‬
 5
x4+ 
lim  f ( x ) + x  = lim x + 4 x + 5 + x = lim
2 4x + 5  x
x →−∞ x →−∞
= lim = −2
x →−∞
x + 4x + 5 − x
2 x →−∞  4 5 
− x  1 + + 2 + 1
 x x 
. y = − x − 2 ‫ ﻤﻌﺎﺩﻝﺘﻪ‬−∞ ‫ ( ﻴﻘﺒل ﻤﺴﺘﻘﻴﻤﺎ ﻤﻘﺎﺭﺒﺎ ' ∆ ﻋﻨﺩ‬C ) ‫ﺝ( ﻨﺴﺘﻨﺘﺞ ﺃﻥ ﺍﻝﻤﻨﺤﻨﻲ‬
lim g ( x ) = +∞ ، lim f ( x ) = +∞ (1 74
x →+∞ x →+∞

  1  3   1 
lim  g ( x ) −  x +   = ، lim  f ( x ) −  x +   = 0 (2
x →+∞
  2  2 x →+∞
  2 
‫ ( ﻻ ﻴﻘﺘﺭﺏ ﻤﻥ ﺍﻝﻤﺴﺘﻘﻴﻡ ﻋﻨﺩ‬C g ) ‫ ﻭ ﻝﻜﻥ‬+∞ ‫ ﻋﻨﺩ‬y = x + ‫ ( ﻴﻘﺘﺭﺏ ﻤﻥ ﺍﻝﻤﺴﺘﻘﻴﻡ ∆ ﺍﻝﺫﻱ ﻤﻌﺎﺩﻝﺘﻪ‬C f ) :‫ﺍﻝﺘﺨﻤﻴﻥ‬
1
2
+∞
  1  3    1   3
lim  g ( x ) − ( x + 2 )  = 0 ‫ ﺃﻱ‬lim  g ( x ) −  x +  −  = 0 ‫ ﻤﻌﻨﺎﻩ‬lim  g ( x ) −  x +   =
x →+∞ x →+∞
  2  2 x →+∞
  2  2
. +∞ ‫ ( ﻋﻨﺩ‬C g ) ‫ ﻤﻘﺎﺭﺏ ﻝﻠﻤﻨﺤﻨﻲ‬y = x + 2 ‫ﻨﺴﺘﻨﺘﺞ ﺃﻥ ﺍﻝﻤﺴﺘﻘﻴﻡ ' ∆ ﺍﻝﺫﻱ ﻤﻌﺎﺩﻝﺘﻪ‬
‫ت‬-
‫ ا‬9:$ ‫ت‬2" ‫ ـ‬3
x +1
D f = ℝ − {−3;1} ، f ( x ) = (1 82
x + 2x − 3 2

x +1 x 1
lim f ( x ) = 0 ‫ ﻭ‬lim f ( x ) = lim 2 = lim 2 = lim = 0
x →+∞ x →−∞ x →−∞ x + 2 x − 3 x →−∞ x x →−∞ x

x + 1
> →1 ( )
lim
> →1
x 2 + 2 x − 3 = 0+ ‫ ﻭ‬lim > →1
x + 1 = 2 ‫ ﻷﻥ‬lim f x = lim > →1 2
= +∞
x  x  x  x  x + 2x − 3
lim
>
f ( x ) = +∞ ‫ ﻭ‬lim <
f ( x ) = −∞ ‫ ﻭ‬lim <
f ( x ) = −∞ : ‫ﻭ ﺒﺎﻝﻤﺜل‬
x →−3 x →−3 x →1

( x + 2)
3
−8
Df = ℝ ، f ( x)

= (6
x
lim f ( x ) = +∞ ‫ ﻭ ﻜﺫﻝﻙ‬lim f ( x ) = lim (
x + 2) − 8
3
x3
= lim = lim x 2 = +∞
x →+∞ x →−∞ x →−∞ x x →−∞ x x →−∞

( x + 2) ( x + 2 − 2) (( x + 2) + 2 ( x + 2) + 4 )
3 2
−8
lim f ( x ) = lim = lim
x →0 x →0 x x →0 x
(
x x 2 + 6 x + 12 ) = lim
= lim
x →0 x x →0
(x 2
+ 6 x + 12 = 12 )
g ( x)
f ( x) = ‫ ﻭ‬h ( x ) = 2 cos x − 1 ‫ ﻭ‬g ( x ) = sin 3x :‫ ﻨﻀﻊ‬x ‫ﻤﻥ ﺍﺠل ﻜل ﻋﺩﺩ ﺤﻘﻴﻘﻲ‬
h ( x) 88
π  π 
sin 3x − sin 3   g ( x) − g  
sin 3 x − 0 3 3
π π π
x− x− x−
sin 3 x
f ( x) = = 3 = 3 = 3
2 cos x − 1 2 cos x − 1 π  π 
π 2cos x − 2 cos   h ( x) − h  
x− 3 3
3 π π
x− x−
3 3
π 
g ' 
π  π
‫ ﻗﺎﺒﻠﺘﺎﻥ ﻝﻼﺸﺘﻘﺎﻕ ﻋﻨﺩ‬h ‫ ﻭ‬g ‫ ﻷﻥ ﺍﻝﺩﺍﻝﺘﺎﻥ‬lim f ( x ) =   ‫ﺇﺫﻥ‬
3
h '  ≠ 0 ‫ﻭ‬
3 3 x→
π π 
3 h ' 
3
h ' ( x ) = −2sin x ‫ ﻭ‬g ' ( x ) = 3cos 3 x : ‫ﻝﺩﻴﻨﺎ‬
π  π 
lim f ( x ) = 3 ‫ ﻓﺈﻥ‬h '   = − 3 ‫ ﻭ‬g '   = −3 ‫ﺒﻤﺎ ﺃﻥ‬
π
x→
3
3 3
0
. ‫ ﻭ ﻤﻨﻪ ﻝﺩﻴﻨﺎ ﺤﺎﻝﺔ ﻋﺩﻡ ﺘﻌﻴﻴﻥ ﻤﻥ ﺍﻝﺸﻜل‬lim 1 − cos x = 0 ‫ ﻭ‬lim sin 2 x = 0 (1 90
0 x →0 x →0

sin 2 x sin 2 x 1 + cos x sin 2 x 1 + cos x


lim = lim × = lim
x →0 1 − cos x x →0 1 − cos x 1 + cos x x → 0 sin x
sin 2 x
sin 2 x 1 + cos x 1 + cos x
l1 = lim+ = lim 2 × 2 x = 2 2 : ‫ ﻓﺈﻥ‬x > 0 ‫ﺇﺫﺍ ﻜﺎﻥ‬
x →0 sin x x →0 sin x +

x
. 0 × ∞ ‫ ﺤﺎﻝﺔ ﻋﺩﻡ ﺘﻌﻴﻴﻥ ﻤﻥ ﺍﻝﺸﻜل‬، l2 = lim (π − 2 x ) tan x (2
π
x→
2
π π π
X → 0 ‫ ﻓﺈﻥ‬x → ‫ ﺇﺫﺍ ﻜﺎﻥ‬. x = X +
:‫ﻨﻀﻊ‬ ‫ ﻭ ﻤﻨﻪ‬X = x −
2 2 2
π  −2 X
l2 = lim (π − 2 x ) tan x = lim (π − π − 2 X ) tan  + X  = lim
2
= lim =2
x→
π X →0
 2  X → 0 − tan X X → 0 tan X
2
X
%‫ر‬.2
 ‫ت‬-
‫ – ا‬%/‫آ‬+ %
‫ دا‬%- ‫ ـ‬4
:‫( ﺃﻭﻻ ﻨﻌﻴﻥ ﻤﺠﻤﻭﻋﺔ ﺍﻝﺘﻌﺭﻴﻑ‬1 101
x − x 2 + 1 < 0 : x ‫ ﻷﻥ ﻤﻥ ﺃﺠل ﻜل ﻋﺩﺩ ﺤﻘﻴﻘﻲ‬D f = ℝ

1
+ 2x =
(x + x2 + 1 ) (
+ 2x = − x + )
x 2 + 1 + 2 x = x − x 2 + 1 :‫ﻝﺩﻴﻨﺎ‬
x − x +12
(x − )(
x2 + 1 x + x2 + 1 )
1 1
< −2 x ‫ﺃﻱ‬ + 2 x < 0 ‫ ﺇﺫﻥ‬، x − x 2 + 1 < 0 : x ‫ﻭ ﻝﺩﻴﻨﺎ ﻤﻥ ﺃﺠل ﻜل ﻋﺩﺩ ﺤﻘﻴﻘﻲ‬
x − x +1
2
x − x +1 2

0 ≤ 1 + sin x ≤ 2 ‫ ﻭ ﻤﻨﻪ‬−1 ≤ sin x ≤ 1 ‫( ﻝﺩﻴﻨﺎ‬2


x > 0 : ‫ ﻷﻥ‬0 ≤ x (1 + sin x ) ≤ 2 x ‫ﻭ ﻤﻨﻪ‬
x (1 + sin x ) x (1 + sin x ) 1
‫ﺃﻱ‬ < −4 x 2 ‫ﺃﻱ‬ < −2 x  x (1 + sin x )  ‫ﻴﻨﺘﺞ‬ < −2 x : ‫ﻤﻥ‬
x − x +1 2
x − x +1 2
x − x2 + 1
f ( x ) < −4 x 2
. lim f ( x ) = −∞ : ‫ ﻨﺴﺘﻨﺘﺞ ﺃﻥ‬. lim − 4 x 2 = −∞ : ‫ﻝﺩﻴﻨﺎ‬
x →+∞ x →+∞
‫ﺍﻷﻧﺸﻄﺔ‬
‫ا
ط اول‬
‫ ‪/ :‬‬
‫ا
ف‪P3 :‬آ' ل ا ‪-‬ت‪.‬‬
‫ ت‪ :‬م ا ‪-‬ط آ*) ‪P‬ا ا ‪1‬ب و ج ‪' #‬ة " ا‪ ."
R29‬و ‪  #‬أاج‪.‬‬
‫ا
‪ )6 63 @ :‬ا ‪ S1<3 #T U4‬ا ‪'1‬ه ت ل ا ‪-‬ت‪.‬‬
‫ا
ط ا
‬
‫ ‪/ :‬‬
‫ا
ف‪ >'63 :‬ا ا
" ?) "‪.‬‬
‫ ت‪ :‬م ا ‪-‬ط آ*) ('ة " درا‪
7‬دا
‪. "
V(V‬‬
‫ا
‪./ :‬‬
‫ا
ط ا

‬
‫ ‪/ :‬‬
‫ا
ف‪ :‬ر
‪
-‬ا ا
ا 'آ‪.
1‬‬
‫ ت‪ :‬م ا ‪-‬ط آ*) ('ة " ا‪2‬ق دا
'آ‪ "
1‬و ‪  #‬أاج ‪ A‬ا‪:;7‬ل ‪ 4‬ز ا ا‪.23‬‬
‫ا
‪../ :‬‬

‫ﺍﻷﻋﻤﺎﻝ ﺍﻟﻤﻮﺟﻬﺔ‬
‫ا
‪.2‬ر‪  %‬دوال و درا‪ %D‬او‪L‬ع ا
‪  2
% /I‬‬
‫ ‪/ :‬‬
‫ا
ف‪ >?3 :‬درا‪
7‬ا‪ ';3 !3‬دا
‪.‬‬
‫ ت‪ #3 @ :‬ا ‪ )@2  )6‬أاج آ @ ا‪'R‬ا‪ U‬آا‪. X  Y4‬‬
‫ا
‪./ :‬‬

‫درا‪ %D‬دا
‪2N %‬ء‬
‫ ‪/ :‬‬
‫ا
ف‪ >?3 :‬ا‪ ';3 !3‬دا
‪.‬‬
‫ ت‪ #3 @ :‬ا ‪ )@2  )6‬أاج آ @ ا‪'R‬ا‪ U‬آا‪. X  Y4‬‬
‫ا
‪./ :‬‬

‫‪ O-.‬دا
‪ %‬ا‪,+ %0D‬ول أو &‪%/D‬‬
‫ ‪/ :‬‬
‫ا
ف‪
'Z >?3 :‬أو '‪.‬‬
‫ ت‪ :‬م ا ‪-‬ط ‪67‬ل ‪ 4‬ز ا ا‪ 23‬أو ‪67‬ل ‪.

17‬‬
‫ا
‪./ :‬‬
‫ﺍﻟﺘﻤﺎﺭﻳﻦ‬
‫‪2‬ر‪.% . /0 -‬‬
‫‪ 1‬ـ ا‪% P."QE‬‬
‫‪ f 2‬ﺍﻝﺩﺍﻝﺔ ﺍﻝﻤﻌﺭﻓﺔ ﻋﻠﻰ ‪ِ ℝ‬ـ ‪. f ( x ) = x‬‬
‫)‪f ( x ) − f (0‬‬ ‫‪x‬‬ ‫)‪f ( x ) − f ( 0‬‬ ‫‪−x‬‬
‫‪ lim‬ﺇﺫﻥ ‪ f‬ﻻ ﺘﻘﺒل ﺍﻻﺸﺘﻘﺎﻕ ﻋﻨﺩ ‪. 0‬‬ ‫‪= lim‬‬ ‫‪ lim‬ﻭ ‪= 1‬‬ ‫‪= lim‬‬ ‫‪= −1‬‬
‫>‬
‫‪x −0‬‬
‫‪x ‬‬
‫‪→0‬‬
‫>‬
‫‪x ‬‬
‫‪→0 x‬‬
‫<‬
‫‪x ‬‬ ‫‪→0‬‬ ‫‪x −0‬‬ ‫<‬
‫‪x ‬‬ ‫‪→0 x‬‬

‫‪3‬‬
‫‪ 6‬ﺍﻝﻤﻨﺤﻨﻲ ﻴﻘﺒل ﻤﻤﺎﺴﺎ ﻋﻨﺩ ‪ A‬ﺇﺫﻥ ﺍﻝﺩﺍﻝﺔ ﺘﻘﺒل ﺍﻻﺸﺘﻘﺎﻕ ﻋﻨﺩ ‪ −2‬ﻭﻤﻌﺎﻤل ﺘﻭﺠﻴﻪ ﺍﻝﻤﻤﺎﺱ ‪ T‬ﻫﻭ = ) ‪ f ' ( −2‬ﻭﻝﺩﻴﻨﺎ‬
‫‪2‬‬
‫‪3‬‬
‫‪ f ( −2 ) = 3‬ﻭﺒﺎﻝﺘﺎﻝﻲ ﻤﻌﺎﺩﻝﺔ ﺍﻝﻤﻤﺎﺱ ‪ T‬ﻫﻲ ‪. y = ( x + 2 ) + 3‬‬
‫‪2‬‬
‫‪ 2‬ـ ا
‪."2‬ت وا
!‪ :2‬ت ‪ :$‬‬
‫‪ 12‬ﻓﻲ ﻜل ﺤﺎﻝﺔ ﻤﻥ ﺍﻝﺤﺎﻻﺕ ﺍﻝﻤﻘﺘﺭﺤﺔ ﺍﻝﺩﺍﻝﺔ ‪ f‬ﺘﻌﺘﺒﺭ ﻜﺜﻴﺭ ﺤﺩﻭﺩ ﻭﺒﺎﻝﺘﺎﻝﻲ ﻫﻲ ﺘﻘﺒل ﺍﻻﺸﺘﻘﺎﻕ ﻋﻠﻰ ‪. ℝ‬‬
‫أ ـ ‪. f ' ( x ) = 5x 4 + 4x 3 − 9x 2 + 2x + 4‬‬
‫‪6x 2 + 2x − 4‬‬
‫= ) ‪. f '(x‬‬ ‫بـ‬
‫‪4‬‬
‫‪4‬ـ ـ ‪. f ' ( x ) = 6mx 2 + 6m 3 x − m 2‬‬
‫د ـ ‪. f ( m ) = 2x 3 + 9m 2 x 2 − 2mx + 1‬‬
‫‪ 14‬أ ـ ‪ f ( x ) = x + x cos x‬؛ ‪f ' ( x ) = 1 + cos x − x sin x . D = ℝ‬‬
‫ب ـ ‪ f ( x ) = sin x cos x‬؛ ‪. f ' ( x ) = cos 2 x − sin 2 x = cos 2x . D = ℝ‬‬
‫‪x cos x − sin x‬‬ ‫‪sin x‬‬
‫= ) ‪. f '(x‬‬ ‫‪2‬‬
‫= ) ‪ f ( x‬؛ ∗‪. D = ℝ‬‬ ‫‪4‬ـ ـ‬
‫‪x‬‬ ‫‪x‬‬
‫‪ 3‬ـ ا‪  ّS T,‬دا
‪%‬‬
‫‪ 25‬أ ـ ‪ f ( x ) = 2x 4 − 27x + 7‬؛ ) ‪f ' ( x ) = 8x 3 − 27 = ( 2x − 3) ( 4x 2 + 6x + 9‬‬
‫‪3‬‬
‫≥ ‪ x‬ﻓﺈﻥ ‪ f ' ( x ) ≥ 0‬ﻭﻤﻨﻪ ‪ f‬ﻤﺘﺯﺍﻴﺩﺓ ﺘﻤﺎﻤﺎ ﻋﻠﻰ‬‫ﻤﻥ ﺃﺠل ﻜل ‪ 4x 2 + 6x + 9 > 0 ، x ∈ ℝ‬ﻭﻤﻨﻪ ﺇﺫﺍ ﻜﺎﻥ‬
‫‪2‬‬
‫‪‬‬ ‫‪3‬‬ ‫‪3‬‬ ‫‪3‬‬ ‫‪‬‬
‫‪  ; + ∞ ‬؛ ﺇﺫﺍ ﻜﺎﻥ ≤ ‪ x‬ﻓﺈﻥ ‪ f ' ( x ) ≤ 0‬ﻭﻤﻨﻪ ‪ f‬ﻤﺘﻨﺎﻗﺼﺔ ﺘﻤﺎﻤﺎ ﻋﻠﻰ ‪.  −∞ ; ‬‬
‫‪‬‬ ‫‪2‬‬ ‫‪2‬‬ ‫‪2‬‬ ‫‪‬‬
‫‪4‬ـ ـ ‪ ، f ' ( x ) = 1 − sin x . f ( x ) = x + cos x‬ﻤﻥ ﺃﺠل ﻜل ﻋﺩﺩ ﺤﻘﻴﻘﻲ ‪ 1 ≥ sin x ، x‬ﻭﻤﻨﻪ ‪ f ' ( x ) ≥ 0‬ﺇﺫﻥ‬
‫ﺍﻝﺩﺍﻝﺔ ‪ f‬ﻤﺘﺯﺍﻴﺩﺓ ﺘﻤﺎﻤﺎ ﻋﻠﻰ ‪. ℝ‬‬
‫‪1 2 x‬‬ ‫‪1‬‬ ‫‪1‬‬
‫‪ f ' ( x ) = −‬ﻭﻤﻨﻪ‬ ‫‪2‬‬
‫‪+‬‬ ‫‪ f ( x ) = −‬؛ ﺍﻝﺩﺍﻝﺔ ‪ f‬ﻤﻌﺭﻓﺔ ﻋﻠﻰ ‪ ℝ +‬ﻭﻗﺎﺒﻠﺔ ﻝﻼﺸﺘﻘﺎﻕ ﻋﻠﻰ ∗ ‪ ℝ +‬ﻭﻝﺩﻴﻨﺎ‬ ‫هـ ـ‬
‫‪x‬‬ ‫‪x‬‬ ‫‪x‬‬ ‫‪x‬‬
‫‪ f ' ( x ) > 0‬ﺇﺫﻥ ﺍﻝﺩﺍﻝﺔ ‪ f‬ﻤﺘﺯﺍﻴﺩﺓ ﺘﻤﺎﻤﺎ ﻋﻠﻰ ‪. ℝ +‬‬
‫‪y‬‬
‫‪3‬‬ ‫‪2‬‬
‫‪y‬‬
‫‪4‬‬ ‫‪ 27‬ﺍﻝﺸﻜل ﺍﻝﻤﻘﺎﺒل ﻫﻭ ﺍﻝﻤﻨﺤﻨﻲ ‪ C f‬ﻝﺩﺍﻝﺔ ‪ f‬ﻗﺎﺒﻠﺔ ﻝﻼﺸﺘﻘﺎﻕ ﻋﻨﺩ ﻜل ﻗﻴﻤﺔ‬
‫‪3‬‬
‫ﻤﻥ ﺍﻝﻤﺠﻤﻭﻋﺔ }‪. ℝ − {−2; 2‬‬
‫‪2‬‬
‫‪1‬‬ ‫‪2‬‬
‫‪-5 -4 -3 -2 -1 0‬‬ ‫‪1 2 3 4 x‬‬ ‫‪1‬‬
‫‪-1‬‬
‫‪-2‬‬
‫‪-3‬‬
‫‪-4 -3 -2 -1 0‬‬
‫‪-1‬‬
‫‪1 2 3 4 x‬‬ ‫ﺍﻝﻤﻨﺤﻨﻲ ﺍﻝﺫﻱ ﻴﻤﺜل ' ‪ f‬ﻫﻭ‬
‫‪ 4‬ـ ا‪."Q‬ق دا
‪+ %‬آ‪%/‬‬

‫‪ 34‬ﺃ ( )‪. f ' ( x ) = 3 ( 2x + 2 ) ( x 2 + 2x − 3‬‬


‫‪2‬‬

‫ﺏ( )‪. g ' ( x ) = 4 ( 4x + 1) ( 2x 2 + x − 1‬‬


‫‪3‬‬

‫ﺝ( )‪. h ' (t ) = 5 ( 3t 2 − 1)(t 3 − t + 1‬‬


‫‪4‬‬

‫‪16u‬‬
‫‪. t ' (u ) = −‬‬ ‫ﺩ(‬
‫)‪(u 2 + 3‬‬
‫‪9‬‬

‫‪ 39‬ﺒﺎﺴﺘﻌﻤﺎل ﺤﺎﺴﺒﺔ ﺒﻴﺎﻨﻴﺔ ﻤﺜﹼﻠﻨﺎ ﺍﻝﻤﻨﺤﻨﻴﻴﻥ ﺍﻝﺫﻴﻥ ﻤﻌﺎﺩﻝﺘﻴﻬﻤﺎ ‪y = x 2 − x + 1‬‬


‫‪1‬‬ ‫‪1‬‬
‫ﻭ ‪. y = − x 2 +x +‬‬
‫‪4‬‬ ‫‪4‬‬
‫‪ (1‬ﻴﺒﺩﻭ ﺃﻥ ﻝﻠﻤﻨﺤﻨﻴﻴﻥ ﻤﻤﺎﺱ ﻤﺸﺘﺭﻙ ﻋﻨﺩ ﺍﻝﻨﻘﻁﺔ ﺫﺍﺕ ﺍﻝﻔﺎﺼﻠﺔ ‪. 1‬‬
‫‪ (2‬ﺃ ـ ﺍﻝﺩﺍﻝﺔ ‪ g‬ﻜﺜﻴﺭ ﺤﺩﻭﺩ ﺇﺫﻥ ﻫﻲ ﻗﺎﺒﻠﺔ ﻝﻼﺸﺘﻘﺎﻕ ﻋﻠﻰ ‪. ℝ‬‬
‫ﺍﻝﺩﺍﻝﺔ ‪ u : x ֏ x 2 − x + 1‬ﺘﻘﺒل ﺍﻻﺸﺘﻘﺎﻕ ﻭﻤﻭﺠﺒﺔ ﺘﻤﺎﻤﺎ ﻋﻠﻰ ‪ ℝ‬ﺇﺫﻥ ﺍﻝﺩﺍﻝﺔ ‪ f : x ֏ u‬ﺘﻘﺒل ﺍﻻﺸﺘﻘﺎﻕ ﻋﻠﻰ ‪. ℝ‬‬
‫‪1‬‬ ‫‪1‬‬
‫ﺏ ـ ‪ f ' (1) = ، g (1) = 1 ، f (1) = 1‬ﻭ = )‪. g ' (1‬‬
‫‪2‬‬ ‫‪2‬‬
‫‪1‬‬
‫ﺝ ـ ﻤﻌﺎﺩﻝﺔ ﺍﻝﻤﻤﺎﺱ ﻝﻤﻨﺤﻨﻲ ﺍﻝﺩﺍﻝﺔ ‪ f‬ﻫﻲ ‪ y = ( x − 1) + 1 :‬ﻭﻨﺠﺩ ﻨﻔﺱ ﺍﻝﻤﻌﺎﺩﻝﺔ ﻝﻤﻤﺎﺱ ﻤﻨﺤﻨﻲ ﺍﻝﺩﺍﻝﺔ ‪. g‬‬
‫‪2‬‬
‫‪ 5‬ـ ا
"‪ O-.‬ا
"‪V
W‬‬
‫‪ 41‬ﺒﺭ‪‬ﺭ ﺍﻝﺘﻘﺭﻴﺏ ﺍﻝﺘﺂﻝﻔﻲ ﺍﻝﻤﺤﻠﻲ ﻋﻨﺩ ‪ 0‬ﻓﻲ ﻜل ﺍﻝﺤﺎﻝﺔ ﻤﻥ ﺍﻝﺤﺎﻻﺕ ﺍﻝﺘﺎﻝﻴﺔ ‪:‬‬
‫‪ x‬ﻭ ‪3x‬‬ ‫ﺃ ( ‪ . (1 + x ) ≈ 1 + 3x‬ﻝﺩﻴﻨﺎ ‪ (1 + x ) = x 3 + 3x 2 + 3x + 1‬ﻭﻋﻨﺩﻤﺎ ﻴﻘﺘﺭﺏ ‪ x‬ﻤﻥ ‪ 0‬ﻓﻴﻜﻭﻥ‬
‫‪2‬‬ ‫‪3‬‬ ‫‪3‬‬ ‫‪3‬‬

‫ﻗﻴﻤﺘﻴﻥ ﻤﻬﻤﻠﺘﻴﻥ‪.‬‬
‫ﻴﻤﻜﻥ ﺍﻋﺘﺒﺎﺭ ﻤﻌﺎﺩﻝﺔ ﻤﻤﺎﺱ ﻤﻨﺤﻨﻲ ﺍﻝﺩﺍﻝﺔ ) ‪ x ֏ (1 + x‬ﻋﻨﺩ ﺍﻝﻨﻘﻁﺔ ﺫﺍﺕ ﺍﻝﻔﺎﺼﻠﺔ ‪ 0‬ﻫﻲ ‪. y = 1 + 3x‬‬
‫‪3‬‬

‫‪2‬ر‪.Jّ2!"ّ :
-‬‬
‫‪ 1‬ـ ا‪% P."QE‬‬
‫‪y‬‬ ‫‪46‬‬
‫‪A‬‬
‫ﺍﻝﻤﻨﺤﻨﻲ ﺍﻝﺒﻴﺎﻨﻲ ‪ C f‬ﺍﻝﺘﺎﻝﻲ ﻫﻭ ﻝﺩﺍﻝﺔ ‪ f‬ﻗﺎﺒﻠﺔ ﻝﻼﺸﺘﻘﺎﻕ ﻋﻠﻰ ﻤﺠﻤﻭﻋﺔ ﺘﻌﺭﻴﻔﻬﺎ‬
‫‪1‬‬
‫‪C‬‬
‫‪0‬‬ ‫‪1‬‬ ‫‪x‬‬
‫‪. D f = [ −5; 2] .1‬‬
‫‪B‬‬
‫) ‪0 − ( −4‬‬ ‫‪ 1‬‬
‫= ) ‪. f ' ( −2‬‬ ‫‪ f ' ( −3) = 0 ، f '  −  = 0 .2‬ﻭ ‪= −2‬‬
‫‪−2 − 0‬‬ ‫‪ 2‬‬
‫‪9‬‬
‫‪ .4‬ﻋﻨﺩ ‪ y = 1 ، A‬؛ ﻋﻨﺩ ‪ y = − ، B‬ﻭﻋﻨﺩ ‪. y = −2 ( x + 2 ) ، C‬‬
‫‪4‬‬
‫‪ .5‬ﻻ ﺘﻭﺠﺩ ﻤﻤﺎﺴﺎﺕ ﺃﺨﺭﻯ ﻝﻠﻤﻨﺤﻨﻲ ‪ C f‬ﻤﻭﺍﺯﻴﺔ ﻝﻤﻤﺎﺴﻪ ﻋﻨﺩ ﺍﻝﻨﻘﻁﺔ ‪ C‬ﻷﻨﻬﺎ ﻫﻲ ﻨﻘﻁﺔ ﺍﻨﻌﻁﺎﻑ ﻝﻠﻤﻨﺤﻨﻲ ‪. C f‬‬
‫‪y‬‬
‫‪1‬‬ ‫‪ f‬ﺍﻝﺩﺍﻝﺔ ﺍﻝﻤﻌﺭ‪‬ﻓﺔ ﻋﻠﻰ ﺍﻝﻤﺠﺎل ]‪ ، [ 0; 2‬ﺘﻤﺜﻴﻠﻬﺎ ﺍﻝﺒﻴﺎﻨﻲ ‪ C‬ﻫﻭ ﻋﺒﺎﺭﺓ ﻋﻥ ﻨﺼﻑ ﺩﺍﺌﺭﺓ‬ ‫‪53‬‬
‫ﻜﻤﺎ ﻫﻭ ﻤﺒﻴ‪‬ﻥ ﻓﻲ ﺍﻝﺸﻜل ‪.‬‬
‫‪0‬‬ ‫‪1‬‬ ‫‪2‬‬ ‫‪x‬‬ ‫‪ (1‬ﺍﻝﻤﻤﺎﺱ ﻤﻨﻁﺒﻕ ﻋﻠﻰ ﻤﺤﻭﺭ ﺍﻝﺘﺭﺍﺘﻴﺏ‪.‬‬
‫‪ (2‬ﻨﻀﻊ ) ‪ Ω (1;0‬؛ ) ‪ M ( x ; y‬ﺘﻨﺘﻤﻲ ﺇﻝﻰ ‪ C‬ﻤﻌﻨﺎﻩ ‪ ΩM = 1‬ﻭ ‪ y ≥ 0‬ﺃﻱ ‪ ( x − 1) + y 2 = 1‬ﻭ ‪ y ≥ 0‬ﻭﻫﺫﺍ‬
‫‪2‬‬

‫)‪f ( x ) = 1 − ( x − 1‬‬ ‫ﻴﻌﻨﻲ )‪ y = 1 − ( x − 1‬ﺃﻱ‬


‫‪2‬‬ ‫‪2‬‬

‫)‪f ( x ) − f (0‬‬
‫‪ lim‬ﻏﻴﺭ ﻤﻨﺘﻬﻴﺔ ‪.‬‬
‫>‬
‫‪ (3‬ﻨﺠﺩ ﺒﺎﻝﺤﺴﺎﺏ‬
‫‪x ‬‬
‫‪→0‬‬ ‫‪x‬‬
‫‪ 2‬ـ ا
‪."2‬ت وا
!‪ :2‬ت ‪ :$‬‬
‫‪f ( x ) = x3 + 3x 2 + 3x + 3‬‬ ‫‪ f 58‬ﻫﻲ ﺍﻝﺩﺍﻝﺔ ﺍﻝﻤﻌﺭﻓﺔ ﻋﻠﻰ ‪ ℝ‬ـ ‪:‬‬
‫ﻋﻠﻰ ﺸﺎﺸﺔ ﺍﻝﺤﺎﺴﺒﺔ ﺍﻝﺒﻴﺎﻨﻴﺔ ﻨﺭﺴﻡ ﺍﻝﻤﻨﺤﻨﻲ ‪ C‬ﺍﻝﻤﻤﺜل ﻝﻠﺩﺍﻝﺔ ‪ f‬ﻭﺍﻝﻤﻤﺎﺱ ‪ T‬ﻋﻨﺩ ﺍﻝﻨﻘﻁﺔ ‪ A‬ﺍﻝﺘﻲ ﻓﺎﺼﻠﺘﻬﺎ ‪. 0‬‬
‫‪. y = 3x + 3 .1‬‬
‫‪ 5‬‬ ‫‪‬‬
‫‪ .2‬ﻴﺒﺩﻭ ﺃﻨﻪ ﺇﺫﺍ ﻜﺎﻥ ‪ x ∈  − ; + ∞ ‬ﻓﺈﻥ ﺍﻝﻤﻨﺤﻨﻲ ﻴﻘﻊ ﻓﻭﻕ ﺍﻝﻤﻤﺎﺱ ‪.‬‬
‫‪ 2‬‬ ‫‪‬‬
‫‪.3‬ﺘﺤﻘﻕ ﺃﻥ ﻤﻥ ﺃﺠل ﻜل ﻋﺩﺩ ﺤﻘﻴﻘﻲ ‪. f ( x ) − ( 3x + 3) = x ( x + 3) : x‬‬
‫‪2‬‬

‫‪ .4‬ﺇﺫﺍ ﻜﺎﻥ [∞ ‪ x ∈ [ −3; +‬ﻓﺈﻥ ‪ f ( x ) − ( 3x + 3) ≥ 0‬ﻭﻴﻜﻭﻥ ﺍﻝﻤﻨﺤﻨﻲ ﻓﻭﻕ ﺍﻝﻤﻤﺎﺱ ‪ ،‬ﻭﺇﺫﺍ ﻜﺎﻥ ]‪x ∈ ]−∞ ; − 3‬‬
‫ﻓﺈﻥ ‪ f ( x ) − ( 3x + 3) ≤ 0‬ﻭﻴﻜﻭﻥ ﺍﻝﻤﻨﺤﻨﻲ ﺘﺤﺕ ﺍﻝﻤﻤﺎﺱ‪.‬‬
‫ﺍﻷﻧﺸﻄﺔ‬
‫ا
ط اول‬
‫ ‪/ :‬‬
‫ا
ف‪ >'63 :‬ا ا
ا^‪.
7‬‬
‫ ت‪ :‬م ا ‪-‬ط آ*) ‪P‬ا ا ‪1‬ب و ج ‪' #‬ة " ا ا
ا^‪ ."
7‬و ‪  #‬أاج ‪ A‬ا‪:;7‬ل ‪ 4‬ز ا ا‪.23‬‬
‫ا
‪ @ :‬إ‪13‬ع ‪ >(F‬ا 'ا) ‪(1‬غ ا ‪ CD‬ا *ة ‪.‬‬
‫ا
ط ا
‬
‫ ‪/ :‬‬
‫ا
ف‪ >'63 :‬ا ا
ا (_ر
ا ‪.
'1‬‬
‫ ت‪ :‬م ا ‪-‬ط آ*) ('ة " ا ا
ا (_ر
"‬
‫ا
‪./ :‬‬

‫ﺍﻷﻋﻤﺎﻝ ﺍﻟﻤﻮﺟﻬﺔ‬
‫‪x ֏ e − λx‬‬ ‫ا
وال‬
‫ ‪/ :‬‬
‫ا
ف‪ >?3 :‬ا ا
ا^‪.
7‬‬
‫ ت‪ #3 @ :‬ا ‪ )@2  )6‬أاج آ @ ا‪'R‬ا‪ U‬آا‪. X  Y4‬‬
‫ا
‪../ :‬‬

‫‪x ֏ e − λx‬‬
‫‪2‬‬
‫ا
وال‬
‫ ‪/ :‬‬
‫ا
ف‪ >?3 :‬ا ا
ا^‪
7‬و ر
 ‪ a‬ت _ص‪.‬‬
‫ ت‪ #3 @ :‬ا ‪ )@2  )6‬أاج آ @ ا‪'R‬ا‪ U‬آا‪. X  Y4‬‬
‫ا
‪./ :‬‬
‫ا
‪!2‬د
‪ %‬ا
"‪ + % :LV‬ا
‪y ′ = ay + b X‬‬
‫ ‪/ :‬‬
‫ا
ف‪6 ) :‬د‪9‬ت ‪.
(3‬‬
‫ ت‪ :‬م ا ‪-‬ط  أاج أو آا‪. X  Y4‬‬
‫ا
‪./ :‬‬
‫دا
" ‪ O,‬و  ‪ O‬ا
[ا‪"-Z‬ن‬
‫ ‪/ :‬‬
‫ا
ف‪ >?3 :‬ا ا
ا^‪.
7‬‬
‫ ت‪ :‬م ا ‪-‬ط  أاج أو آا‪. X  Y4‬‬
‫ا
‪./ :‬‬
‫ا
"‪  2‬ا
‪
 /‬وال ‪
 %.\+‬ا
‪ %‬ا
‪4:‬ر‪ % 2"-‬ا
‪%- /‬‬
‫ ‪/ :‬‬
‫ا
ف‪ >?3 :‬ا ا
ا^‪.
7‬‬
‫ ت‪ :‬م ا ‪-‬ط  أاج أو آا‪. X  Y4‬‬
‫ا
‪./ :‬‬
‫ﺍﻟﺘﻤﺎﺭﻳﻦ‬
‫ﺘﻤﺎﺭﻴﻥ ﺘﻁﺒﻴﻘﻴﺔ‬
‫‪ 1‬ـ ا ا ا‬
‫‪1 − e −2 x e 1 − e‬‬
‫=‬
‫‪2x‬‬
‫(‬ ‫=‬
‫‪−2 x‬‬
‫‪e2 x − 1‬‬‫)‬‫‪(1 3‬‬
‫(‬
‫‪1 + e −2 x e 2 x 1 + e−2 x‬‬ ‫‪e2 x + 1‬‬‫)‬
‫‪1‬‬ ‫‪1‬‬ ‫‪ex −1‬‬
‫= ‪e− x − e −2 x‬‬ ‫‪−‬‬ ‫=‬ ‫‪(2‬‬
‫‪e x e2 x‬‬ ‫‪e2 x‬‬
‫‪ (3‬ﺘﺼﻭﻴﺏ‪(e x + e − x )2 = (e x ) 2 + 2e x e− x + (e− x )2 :‬‬
‫‪(e x + e − x ) 2 = e 2 x + e −2 x + 2‬‬
‫‪1‬‬ ‫‪e4 x + 1‬‬
‫‪(e x + e − x ) 2 = e 2 x +‬‬ ‫‪+‬‬ ‫‪2‬‬ ‫=‬ ‫‪+2‬‬
‫‪e2 x‬‬ ‫‪e2 x‬‬
‫‪e x − e− x e e − e‬‬
‫=‬
‫‪x‬‬ ‫‪x‬‬
‫(‬ ‫‪−x‬‬

‫=‬
‫‪e2 x − 1‬‬
‫‪(4‬‬
‫)‬
‫(‬
‫‪e x + e− x e x e x + e− x‬‬ ‫‪e2 x + 1‬‬ ‫)‬
‫‪ 2‬ـ ا وال ا ‪x ֏ e kx‬‬
‫‪ 15‬ﺘﺼﻭﻴﺏ‪f ( x + y ) = f ( x ) × f ( y ) :‬‬
‫‪ (1‬ﺃ( ﺇﺫﺍ ﻜﺎﻥ ‪ x = y = 0‬ﻓﺈﻥ‪ f ( 0 ) = f ( 0 ) × f ( 0 ) :‬ﻭﻤﻨﻪ ‪f ( 0 ) −  f ( 0 )  = 0‬‬
‫‪2‬‬

‫ﻭﻤﻨﻪ ‪ f ( 0 ) 1 − f ( 0 )  = 0‬ﻭﻤﻨﻪ ‪ f ( 0 ) = 1‬ﻷﻥ ‪ f‬ﻏﻴﺭ ﻤﻌﺩﻭﻤﺔ‪.‬‬


‫ﺏ( ﻤﻥ ﺃﺠل ﻜل ﻋﺩﺩ ﺤﻘﻴﻘﻲ ‪f ( x ) × f ( − x ) = f ( x − x ) : x‬‬
‫ﻭﻤﻨﻪ ) ‪ f ( x ) × f ( − x ) = f ( 0‬ﺃﻱ ‪f ( x ) × f ( − x ) = 1‬‬
‫‪x‬‬ ‫‪x‬‬ ‫‪x x‬‬
‫×‪f  ‬‬ ‫‪ .2‬ﺃ‪ -‬ﻤﻥ ﺃﺠل ﻜل ﻋﺩﺩ ﺤﻘﻴﻘﻲ ‪f   = f  +  = f ( x ) ، x‬‬
‫‪2‬‬ ‫‪2‬‬ ‫‪2 2‬‬
‫ﺏ( ﺍﻝﺩﺍﻝﺔ ‪ f‬ﻤﻭﺠﺒﺔ ﺘﻤﺎﻤﺎ ﻋﻠﻰ ‪. ℝ‬‬
‫‪ 3‬ـ درا ا ا ا‬
‫‪ f‬ﺩﺍﻝﺔ ﻤﻌﺭﻓﺔ ﻋﻠﻰ [∞‪ [ 0; +‬ﻜﻤﺎ ﻴﻠﻲ‪:‬‬
‫‪1‬‬
‫= )‪f ( x‬‬ ‫‪x + 1 − e− x‬‬
‫‪2‬‬
‫‪1‬‬
‫‪y‬‬ ‫‪.1‬ﺃ( ‪ f ' ( x ) = + e− x‬ﺍﻝﺩﺍﻝﺔ ‪ f‬ﻤﺘﺯﺍﻴﺩﺓ ﺘﻤﺎﻤﺎ ﻋﻠﻰ [∞‪. [ 0; +‬‬
‫‪3‬‬ ‫‪2‬‬
‫‪2‬‬ ‫ﺏ( ∞‪. lim f ( x ) = +‬‬
‫∞‪x →+‬‬

‫‪‬‬ ‫‪1‬‬ ‫‪‬‬


‫‪1‬‬
‫∞‪x →+‬‬
‫‪2‬‬ ‫∞‪  x →+‬‬
‫(‬
‫‪.2‬ﺃ( ‪lim  f ( x ) −  x + 1  = lim −e− x = 0‬‬ ‫)‬
‫‪‬‬
‫‪0‬‬ ‫‪1‬‬ ‫‪2‬‬ ‫‪3‬‬ ‫‪4‬‬ ‫‪x‬‬ ‫‪1‬‬
‫ﻤﻌﺎﺩﻝﺔ ﺍﻝﻤﺴﺘﻘﻴﻡ ﺍﻝﻤﻘﺎﺭﺏ ‪ D‬ﻫﻲ‪. y = x + 1 :‬‬
‫‪2‬‬
‫ﺏ( ﺍﻝﻤﻨﺤﻨﻲ ) ‪ ( C‬ﺃﺴﻔل ﺍﻝﻤﺴﺘﻘﻴﻡ ‪. D‬‬
‫‪ .3‬ﺍﻝﺭﺴﻡ )ﺍﻨﻅﺭ ﺍﻝﺸﻜل(‬

‫) ‪g ( x) − f ( x) = e− x (1 − sin x‬‬ ‫‪51‬‬


‫‪.1‬‬
‫‪ π −π‬‬ ‫‪‬‬ ‫‪π‬‬
‫‪A ;e 2‬‬ ‫= ‪. x‬ﺇﺫﻥ ﺍﻝﻤﻨﺤﻨﻴﺎﻥ ﻴﺸﺘﺭﻜﺎﻥ ﻓﻲ ﺍﻝﻨﻘﻁﺔ ‪‬‬ ‫‪ sin x = 1‬ﻋﻠﻰ ] ‪ [ 0; π‬ﻤﻌﻨﺎﻩ‬
‫‪2‬‬ ‫‪‬‬ ‫‪2‬‬
‫‪‬‬ ‫‪‬‬
‫ﻭ ‪g '( x) = e − x‬‬ ‫‪f '( x) = e− x ( − sin x + cos x ) .2‬‬
‫‪π‬‬
‫‪π ‬‬ ‫‪π ‬‬ ‫‪−‬‬
‫‪ f '   = g '   = −e 2‬ﺇﺫﻥ ﺍﻝﻤﻨﺤﻨﻴﺎﻥ ﻴﻘﺒﻼﻥ ﻓﻲ ﺍﻝﻨﻘﻁﺔ ‪ A‬ﻤﻤﺎﺴﺎ ﻤﺸﺘﺭﻜﺎ‪.‬‬
‫‪2‬‬ ‫‪2‬‬

‫‪ 4‬ـ ا ا ار ا‬


‫‪P ( x ) = −2 x3 + 3 x 2 + 11x − 6 61‬‬
‫‪P ( x ) = ( 2 x − 1)( x + 2 )( 3 − x ) (1‬‬
‫‪ P ( x ) = 0‬ﻤﻌﻨﺎﻩ ﺃﻭ ) ‪ ( x = −2‬ﺃﻭ )‪( x = 3‬‬ ‫‪(2‬‬
‫‪ 1 −2 3 ‬‬
‫‪x ∈  e 2 ; e ; e  (3‬‬
‫‪‬‬ ‫‪‬‬
‫‪ 1‬‬ ‫‪‬‬
‫‪x ∈ ln ;ln 3 (4‬‬
‫‪ 2‬‬ ‫‪‬‬
‫‪ 5‬ـ ااص ا‬
‫‪ x + 2 y = 16‬‬
‫‪2‬‬
‫‪73‬‬
‫‪‬‬
‫}) ‪S = {( −8; −24 ) , ( 2; 6‬‬ ‫‪  x‬ﻤﺠﻭﻋﺔ ﺍﻝﺤﻠﻭل ﻫﻲ‬ ‫‪(2‬‬
‫‪ln y = − ln 3‬‬
‫‪‬‬
‫‪ x 2 + y 2 = 169‬‬
‫ﻤﺠﻭﻋﺔ ﺍﻝﺤﻠﻭل ﻫﻲ }) ‪S = {( 5;12 ) , (12;5‬‬ ‫‪‬‬ ‫‪(3‬‬
‫‪ln x + ln y = ln 60‬‬
‫‪1 ‬‬
‫‪ 2t 2 − 5t + 2 = 0 (1 74‬ﻤﻌﻨﺎﻩ ‪t ∈  ; 2 ‬‬
‫‪2 ‬‬
‫‪ (2‬ﻤﺠﻤﻭﻋﺔ ﺤﻠﻭل ﺍﻝﺠﻤﻠﺔ ﻫﻲ‪S = {( − ln 2; ln 2 ) , ( ln 2; − ln 2 )} :‬‬
‫‪ 6‬ـ درا ا ا ار ا‬
‫‪f ( x ) = 3ln ( 2 + x ) + x 2 − 3 x 91‬‬
‫‪3‬‬ ‫‪2 x2 + x − 3‬‬
‫= )‪f '( x‬‬ ‫= ‪+ 2x − 3‬‬
‫‪x+2‬‬ ‫‪x+2‬‬
‫‪ f ' ( x ) = 0‬ﺘﻜﺎﻓﺊ ‪2 x + x − 3 = 0‬‬
‫‪2‬‬

‫‪‬‬ ‫‪3‬‬
‫‪ f ' ( x ) = 0‬ﺘﻜﺎﻓﺊ )‪ ( x = 1‬ﺃﻭ ‪ x = − ‬‬
‫‪‬‬ ‫‪2‬‬
‫‪3‬‬
‫ﺇﺫﻥ ﺍﻝﻤﻨﺤﻨﻲ ‪ C‬ﺍﻝﻤﻤﺜل ﻝﻠﺩﺍﻝﺔ ‪ f‬ﻴﻘﺒل ﻤﻤﺎﺴﻴﻥ ﻤﻭﺍﺯﻴﻴﻥ ﻝﻤﺤﻭﺭ ﺍﻝﻔﻭﺍﺼل ﻋﻨﺩ ﺍﻝﻨﻘﻁﺘﻴﻥ ﺍﻝﺘﻴﻥ ﻓﺎﺼﻠﺘﺎﻫﻤﺎ ‪ 1‬ﻭ‬
‫‪2‬‬
‫‪7‬ـ دا ار‪ %‬ا‪#$‬ي‬
‫‪. E (1234 log 2 ) = 371 . log ( 21234 ) = 1234 log 2 .1 98‬‬
‫‪ .2‬ﻤﻥ ‪ E ( log n ) = 371‬ﻨﺴﺘﻨﺘﺞ ﺃﻥ‪371 ≤ log n < 372 :‬‬
‫ﻭﻤﻨﻪ ‪ log10371 ≤ log n < log10372‬ﻭ ﻤﻨﻪ ‪10371 ≤ n < 10372‬‬
‫‪ .3‬ﺍﻝﻜﺘﺎﺒﺔ ﺍﻝﻌﺸﺭﻴﺔ ﻝﻠﻌﺩﺩ ‪ n‬ﺘﺘﻜﻭﻥ ﻤﻥ ‪ 372‬ﺭﻗﻤﺎ‪.‬‬
‫‪8‬ـ ا‪$‬د*ت ا('‬
‫‪f ( x) = λ e −2 x (2 ، f ( x) = λ e3 x (1 102‬‬
‫‪5‬‬
‫‪− x‬‬
‫‪f ( x) = λ e‬‬ ‫‪8x‬‬
‫= ) ‪(4 ، f ( x‬‬ ‫‪λe 2‬‬ ‫‪(3‬‬
‫‪1‬‬
‫‪− x‬‬
‫‪f ( x) = λ e‬‬ ‫‪2‬‬ ‫‪(1‬‬ ‫‪103‬‬
‫‪ (2‬ﺍﻝﺤل ﺍﻝﺨﺎﺹ ‪ f‬ﺍﻝﺫﻱ ﻴﺤﻘﻕ ‪ f ( ln 4 ) = 1‬ﻫﻭ‬
‫‪1‬‬
‫‪− x‬‬
‫= )‪f ( x‬‬ ‫‪2e 2‬‬
‫ﺘﻤﺎﺭﻴﻥ ﻝﻠﺘﻌﻤﻕ‪.‬‬

‫‪ (1 108‬ﺘﺼﻭﻴﺏ‪ :‬ﺍﻝﻤﺴﺘﻘﻴﻡ ﺍﻝﺫﻱ ﻤﻌﺎﺩﻝﺘﻪ ‪ y = 1‬ﻤﺴﺘﻘﻴﻡ ﻤﻘﺎﺭﺏ ﻝﻠﻤﻨﺤﻨﻲ ) ‪ (C‬ﻋﻨﺩ ∞‪. −‬‬
‫)‪( a, b, c ) = ( 2, −3,1‬‬
‫‪ (2‬ﺃ( ‪. lim f ( x) = 1‬ﺍﻝﻤﻨﺤﻨﻲ ) ‪ (C‬ﻴﻘﺒل ﺍﻝﻤﺴﺘﻘﻴﻡ ﺍﻝﺫﻱ ﻤﻌﺎﺩﻝﺘﻪ ‪ y = 1‬ﻜﻤﻘﺎﺭﺏ ﻋﻨﺩ ∞‪. −‬‬
‫∞‪x →−‬‬
‫• ∞‪lim f ( x) = +‬‬
‫∞‪x →+‬‬

‫(‬
‫• ‪f '( x) = e x 4e x − 3‬‬ ‫)‬
‫‪3‬‬
‫‪x‬‬ ‫∞‪−‬‬ ‫‪ln‬‬ ‫∞‪+‬‬
‫‪4‬‬
‫)‪f '( x‬‬ ‫‪-‬‬ ‫‪0‬‬ ‫‪+‬‬
‫‪1‬‬ ‫∞‪+‬‬
‫‪f‬‬ ‫‪1‬‬
‫‪−‬‬
‫‪8‬‬
‫ﺏ( ﺍﻝﻤﻨﺤﻨﻲ ) ‪ (C‬ﻴﻘﻁﻊ ﻤﺤﻭﺭ ﺍﻝﻔﻭﺍﺼل ﻓﻲ ﺍﻝﻨﻘﻁﺘﻴﻥ ﺍﻝﻠﺘﻴﻥ ﻓﺎﺼﻠﺘﺎﻫﻤﺎ ‪ 0‬ﻭ ‪. − ln 2‬‬
‫• ﻤﻌﺎﺩﻝﺔ ﺍﻝﻤﻤﺎﺱ ﻝﻠﻤﻨﺤﻨﻲ ) ‪ (C‬ﻋﻨﺩ ﺍﻝﻨﻘﻁﺔ ﺍﻝﺘﻲ ﻓﺎﺼﻠﺘﻬﺎ‪ 0‬ﻫﻲ ‪. y = x‬‬
‫)‪f ( x‬‬
‫‪lim‬‬ ‫ﺠـ( ∞‪= +‬‬
‫‪x →+∞ x‬‬
‫‪y‬‬ ‫د( ا '‪#7‬‬
‫‪2‬‬

‫‪1‬‬

‫‪-4‬‬ ‫‪-3‬‬ ‫‪-2‬‬ ‫‪-1‬‬ ‫‪0‬‬ ‫‪1‬‬ ‫‪x‬‬


‫‪-1‬‬
‫‪ex −1‬‬ ‫‪2‬‬
‫‪f ( x ) − ( x − 1) = x −‬‬ ‫‪− x +1 = x‬‬ ‫‪ (1 116‬ﺃ(‬
‫‪e +1‬‬
‫‪x‬‬
‫‪e +1‬‬
‫‪ex −1‬‬ ‫‪− 2e x‬‬
‫‪f ( x ) − ( x + 1) = − x‬‬ ‫‪−1 = x‬‬
‫‪e +1‬‬ ‫‪e +1‬‬
‫‪lim f ( x) = −∞ ،‬‬ ‫ﺏ( ∞‪lim f ( x) = +‬‬
‫∞‪x →−‬‬ ‫∞‪x →+‬‬

‫ﺠـ( ‪ lim f ( x) − ( x − 1) = 0‬ﻭ ‪lim f ( x) − ( x + 1) = 0‬‬


‫∞‪x →−‬‬ ‫∞‪x →+‬‬

‫ﺇﺫﻥ ﺍﻝﻤﺴﺘﻘﻴﻤﺎﻥ ‪ ∆1‬ﻭ ‪ ∆ 2‬ﺍﻝﻠﺫﻴﻥ ﻤﻌﺎﺩﻝﺘﺎﻫﻤﺎ ﻋﻠﻰ ﻤﻘﺎﺭﺒﺎﻥ ﻝـ ) ‪ ( C‬ﻋﻨﺩ ∞‪ −‬ﻭﻋﻨﺩ ∞‪. +‬‬
‫ﺩ( ﺒﺠﻭﺍﺭ ∞‪ ( C ) +‬ﺃﻋﻠﻰ ‪ ، ∆ 2‬ﻭ ﺒﺠﻭﺍﺭ ∞‪ ( C ) −‬ﺃﺴﻔل ‪. ∆1‬‬
‫‪e− x − 1‬‬ ‫‪1 − ex‬‬
‫‪f (− x) = − x −‬‬ ‫=‬ ‫‪−‬‬ ‫‪x‬‬ ‫‪+‬‬ ‫‪ (2‬ﺃ( ) ‪= − f ( x‬‬
‫‪e− x + 1‬‬ ‫‪1+ ex‬‬
‫‪2e x‬‬ ‫‪2e x + 1‬‬
‫‪f '( x) = 1 −‬‬ ‫=‬ ‫ﺏ(‬
‫(‬ ‫( )‬ ‫)‬
‫‪2‬‬ ‫‪2‬‬
‫‪ex +1‬‬ ‫‪ex + 1‬‬

‫‪x‬‬ ‫‪0‬‬ ‫‪α‬‬ ‫∞‪+‬‬


‫)‪f '( x‬‬ ‫‪+‬‬
‫∞‪+‬‬
‫‪f‬‬ ‫‪1‬‬
‫‪0‬‬
‫‪y‬‬
‫‪ (3‬ﺍﻝﺭﺴﻡ‬

‫‪3‬‬

‫‪2‬‬

‫‪1‬‬

‫‪-4‬‬ ‫‪-3‬‬ ‫‪-2‬‬ ‫‪-1‬‬ ‫‪0‬‬ ‫‪1‬‬ ‫‪2‬‬ ‫‪3‬‬ ‫‪4 x‬‬

‫‪-1‬‬

‫‪-2‬‬

‫‪-3‬‬

‫‪(1 117‬‬

‫‪.2‬ﺃ( ﺍﻝﺩﺍﻝﺔ ‪ f‬ﻤﺘﺯﺍﻴﺩﺓ‪.‬‬


‫ﺏ( ﺍﻝﺩﺍﻝﺔ ‪ f‬ﺘﻨﻌﺩﻡ ﻋﻨﺩ ‪. x = 0‬‬
‫‪2, 2 2 x − 0, 2 x‬‬ ‫‪2‬‬
‫‪f '( x) = 2 x − 2, 2 +‬‬ ‫=‬ ‫‪.3‬ﺃ( ﻤﻥ ﺃﺠل ‪: x > −1‬‬
‫‪x +1‬‬ ‫‪x +1‬‬
‫ﺇﺸﺎﺭﺓ )‪ f '( x‬ﻫﻲ ﻤﻥ ﻨﻔﺱ ﻹﺸﺎﺭﺓ ) ‪x ( 2 x − 0, 2‬‬
‫‪ f '( x) > 0‬ﻤﻌﻨﺎﻩ [∞‪ f '( x) < 0 ، x ∈ ]−1;0[ ∪ ]0,1; +‬ﻤﻌﻨﺎﻩ [‪ f '( x) = 0 ، x ∈ ]0; 0,1‬ﻤﻌﻨﺎﻩ }‪x ∈ {0; 0,1‬‬

‫‪x‬‬ ‫‪1-‬‬ ‫‪0‬‬ ‫‪0,1‬‬ ‫∞‪+‬‬


‫)‪f '( x‬‬ ‫‪+‬‬ ‫‪0‬‬ ‫‪-‬‬ ‫‪0‬‬ ‫‪+‬‬
‫‪0‬‬ ‫∞‪+‬‬
‫‪f‬‬
‫∞‪−‬‬ ‫‪≈ −0, 0003‬‬
‫ﺏ( ∞‪lim f ( x) = +∞ ، lim f ( x) = −‬‬
‫∞‪x →+‬‬ ‫‪x →−1‬‬

‫[∞‪[ f (0,1); +‬‬ ‫ﺠـ( ﻝﺩﻴﻨﺎ ‪ f (0) = 0‬ﻭ ﻋﻠﻰ ﺍﻝﻤﺠﺎل [∞‪ [ 0,1; +‬ﺍﻝﺩﺍﻝﺔ ‪ f‬ﻤﺴﺘﻤﺭﺓ ﻭ ﻤﺘﺯﺍﻴﺩﺓ ﻭ ﺘﺄﺨﺫ ﻗﻴﻤﻬﺎ ﻓﻲ‬
‫ﻭ ‪. f (0,1) < 0‬ﺇﺫﻥ ﻴﻭﺠﺩ ﻋﺩﺩ ﺤﻘﻴﻘﻲ ﻭﺤﻴﺩ ‪ x0‬ﺤﻴﺙ ‪f ( x0 ) = 0‬‬
‫ﺨﻼﺼﺔ‪ :‬ﺍﻝﻤﻌﺎﺩﻝﺔ ‪ f ( x) = 0‬ﺘﻘﺒل ﺤﻠﻴﻥ‪ 0‬ﻭ ‪. x0‬‬
‫ﺩ( ﻨﺘﺎﺌﺞ ﺩﺭﺍﺴﺔ ﺍﻝﺩﺍﻝﺔ ﻻ ﺘﺘﻁﺎﺒﻕ ﻤﻊ ﺍﻝﺘﺨﻤﻴﻥ‪.‬‬
‫‪.4‬ﺃ( ﻴﻤﻜﻥ ﺃﺨﺫ ‪−0, 0018 ≤ y ≤ 0, 00111‬‬
‫ﺏ( ‪ f (0,15) < 0‬ﻭ ‪ f (0,16) > 0‬ﻭﻤﻨﻪ ‪. 0,15α < 0,16‬ﻗﻴﻤﺔ ﻤﻘﺭﺒﺔ ﺒﺎﻝﺯﻴﺎﺩﺓ ﺇﻝﻰ ‪ 10−2‬ﻝﻠﻌﺩﺩ ‪ α‬ﻫﻲ ‪0,16‬‬

‫‪ (1 121‬ﺃ( ﻤﻥ ﺃﺠل ﻜل ﻋﺩﺩ ﺤﻘﻴﻘﻲ ‪ −1 ≤ cos 4 x ≤ 1 : x‬ﻭ ‪ e− x > 0‬ﻭﻤﻨﻪ ‪.‬‬


‫‪−e − x ≤ f ( x ) ≤ e − x‬‬
‫ﺏ( ‪lim f ( x) = 0‬‬
‫∞‪x →+‬‬

‫‪ π −k π‬‬ ‫‪‬‬


‫‪Mk  k ;e 2‬‬ ‫‪ (2‬ﺍﻝﻨﻘﻁ ﺍﻝﻤﺸﺘﺭﻜﺔ ﻝﻠﻤﻨﺤﻨﻴﻴﻥ ‪ Γ‬ﻭ ‪ C‬ﻫﻲ ﺍﻝﻨﻘﻁ ‪‬‬
‫‪ 2‬‬ ‫‪‬‬
‫‪‬‬ ‫‪‬‬
‫‪π‬‬ ‫‪π‬‬ ‫‪π‬‬ ‫‪π‬‬
‫‪−‬‬ ‫‪−n‬‬ ‫‪−‬‬ ‫‪−‬‬
‫‪e‬‬ ‫‪2‬‬ ‫‪. un+1 = e‬ﺍﻝﻤﺘﺘﺎﻝﻴﺔ ) ‪ ( un‬ﻫﻨﺩﺴﻴﺔ ﺃﺴﺎﺴﻬﺎ‬ ‫‪2‬‬ ‫‪×e‬‬ ‫‪2‬‬ ‫‪ (3‬ﺃ( ‪= e 2 u n‬‬
‫‪π‬‬
‫‪−‬‬
‫‪ 0 < e‬ﻭ ‪. u0 = 1‬ﺇﺫﻥ ﺍﻝﻤﺘﺘﺎﻝﻴﺔ ) ‪ ( un‬ﻤﻭﺠﺒﺔ ﻭ ﻤﺘﺯﺍﻴﺩﺓ ﻭ ﺘﺘﻘﺎﺭﺏ ﻨﺤﻭ‪.0‬‬ ‫‪2‬‬ ‫ﺏ(ﺃﺴﺎﺱ ﺍﻝﻤﺘﺘﺎﻝﻴﺔ ) ‪< 1 ( un‬‬
‫‪ (4‬ﺃ( ﻤﻥ ﺃﺠل ﻜل ﻋﺩﺩ ﺤﻘﻴﻘﻲ ‪ x‬ﻤﻥ [∞‪: [ 0; +‬‬
‫ﺘﺼﻭﻴﺏ‪f ' ( x ) = −e − x cos ( 4 x ) + 4sin ( 4 x )  :‬‬
‫‪π‬‬
‫‪ x = k‬ﻓﺈﻥ ‪ cos 4 x = 1‬ﻭ ‪sin 4 x = 0‬‬ ‫ﺏ( ‪. g ' ( x ) = −e − x‬ﺇﺫﺍ ﻜﺎﻥ‬
‫‪2‬‬
‫‪π‬‬
‫‪ π‬‬ ‫‪ π‬‬ ‫‪−k‬‬
‫‪f '  k  = g '  k  = −e 2‬‬
‫‪ 2‬‬ ‫‪ 2‬‬
‫ﺇﺫﻥ ﺍﻝﻤﻨﺤﻨﻴﻴﻥ ‪ Γ‬ﻭ ‪ C‬ﻝﻬﻤﺎ ﻨﻔﺱ ﺍﻝﻤﻤﺎﺱ ﻋﻨﺩ ﻜل ﻨﻘﻁﺔ ﻤﻥ ﻨﻘﻁ ﻨﻘﺎﻁﻌﻬﻤﺎ‪.‬‬
‫‪π‬‬
‫‪−1‬‬ ‫‪π ‬‬ ‫‪−‬‬
‫‪ (5‬ﻝﺩﻴﻨﺎ‪ . f '   = −e 2 :‬ﻗﻴﻤﺔ ﻤﻘﺭﺒﺔ ﺇﻝﻰ ‪ 10‬ﻝﻤﻌﺎﻤل ﺘﻭﺠﻴﻪ ﺍﻝﻤﻤﺎﺱ ‪ T‬ﻝﻠﻤﻨﺤﻨﻲ ‪ Γ‬ﻋﻨﺩ ﺍﻝﻨﻘﻁﺔ ﺍﻝﺘﻲ ﻓﺎﺼﻠﺘﻬﺎ‬
‫‪2‬‬
‫‪y‬‬
‫‪π‬‬
‫ﻫﻲ ‪. −0, 2‬‬
‫‪1‬‬

‫‪0,8‬‬ ‫‪2‬‬
‫‪0,6‬‬

‫‪0,4‬‬

‫‪0,2‬‬

‫‪0‬‬ ‫‪1‬‬ ‫‪2‬‬ ‫‪3‬‬ ‫‪x‬‬


‫‪-0,2‬‬

‫‪-0,4‬‬

‫ﻤﺴﺎﺌل‬
‫‪1‬‬
‫‪ f ( − x ) = − x‬ﺇﺫﻥ ﺍﻝﺩﺍﻝﺔ ‪ f‬ﺯﻭﺠﻴﺔ‬ ‫‪ (1 123‬ﺍﻝﻤﺠﻤﻭﻋﺔ ‪ ℝ‬ﻤﺘﻨﺎﻅﺭﺓ ﺒﺎﻝﻨﺴﺒﺔ ﻝﻠﺼﻔﺭ‪= f ( x) ،‬‬
‫‪e + ex‬‬
‫‪ (2‬ﺍﻝﺩﺍﻝﺔ ‪ x ֏ e x‬ﺀ ﻤﺘﺯﺍﻴﺩﺓ ﻋﻠﻰ ‪. ℝ‬ﻤﻥ ﺃﺠل ﻜل ﻋﺩﺩ ﺤﻘﻴﻘﻲ ﻤﻭﺠﺏ ‪ − x ≤ x : x‬ﻭﻤﻨﻪ ‪. e− x ≤ e x‬‬
‫‪ (3‬ﺃ( ‪lim f ( x) = 0‬‬
‫∞‪x →+‬‬

‫‪e x − e− x‬‬
‫‪ e x ≥ e − x‬ﻭ ﻤﻨﻪ ‪f '( x) < 0‬‬ ‫‪. f '( x) = −‬ﻤﻥ ﺃﺠل ﻜل ‪: x ≥ 0‬‬ ‫ﺏ(‬
‫(‬ ‫)‬
‫‪2‬‬
‫‪e x + e− x‬‬

‫‪x‬‬ ‫‪0‬‬ ‫∞‪+‬‬

‫)‪f '( x‬‬ ‫‪0‬‬ ‫‪-‬‬


‫‪1‬‬
‫‪f‬‬ ‫‪2‬‬
‫‪0‬‬
‫‪1‬‬ ‫‪1‬‬ ‫‪1‬‬
‫‪≤ −x‬‬ ‫‪≤ x‬‬ ‫‪ (4‬ﺃ( ﻤﻥ ﺃﺠل ﻜل ‪ 0 < e− x ≤ e x : x ≥ 0‬ﻭﻤﻨﻪ ‪ 0 < e x < e− x + e x ≤ 2e x‬ﻭﻤﻨﻪ‬
‫‪2e‬‬ ‫‪x‬‬
‫‪e +e‬‬ ‫‪x‬‬
‫‪e‬‬
‫ﺇﺫﻥ ‪:‬ﻤﻥ ﺃﺠل ﻜل ﻋﺩﺩ ﺤﻘﻴﻘﻲ ﻤﻭﺠﺏ ‪. h ( x ) ≤ f ( x ) ≤ g ( x ) : x‬‬
‫ﺏ( ﻨﺴﺘﻨﺘﺞ ﺃﻨﻪ ﻋﻠﻰ ‪ Γ ، ℝ +‬ﻴﻜﻭﻥ ﺒﻴﻥ ‪ Γ1‬ﻭ ‪. Γ 2‬‬
‫‪y‬‬
‫‪1‬‬

‫‪0,8‬‬

‫‪0,6‬‬

‫‪0,4‬‬

‫‪0,2‬‬

‫‪0‬‬ ‫‪1‬‬ ‫‪2‬‬ ‫‪3‬‬ ‫‪x‬‬


‫ﺍﻷﻧﺸﻄﺔ‬
‫ا
ط اول‬
‫ ‪/ :‬‬
‫ا
ف‪R >'63 :‬ى د  ‪.Y4‬‬
‫ ت‪ :‬م ا ‪-‬ط آ*) ‪P‬ا ا ‪1‬ب و ج ‪' #‬ة " ‪R‬ى د  ‪." Y4‬‬
‫ا
‪./ :‬‬
‫ا
ط ا
‬
‫ ‪/ :‬‬
‫ا
ف‪ :‬ر
 م ا !‪P‬ر ا ‪.‬‬
‫"‬ ‫‪x ֏ x‬‬ ‫‪n‬‬
‫و‬ ‫‪x ֏a‬‬ ‫‪x‬‬
‫ ت‪ :‬م ا ‪-‬ط آ*) ('ة " درا‪
7‬ا وال‬
‫ا
‪./ :‬‬
‫ا
ط ا

‬
‫ ‪/ :‬‬
‫ا
ف‪ :‬ر
آ)  ‪ ln x‬و ‪" x A e‬‬
‫‪n‬‬ ‫‪x‬‬

‫ ت‪ :‬م ا ‪-‬ط آ*) ('ة " ا ‪X‬ا ا رن " و ‪  #‬أاج‪.‬‬
‫ا
‪../ :‬‬

‫ﺍﻷﻋﻤﺎﻝ ﺍﻟﻤﻮﺟﻬﺔ‬
‫درا‪ %D‬دا
‪4
%‬ر‪% 2"-‬‬
‫ ‪/ :‬‬
‫ا
ف‪ >?3 :‬دا
ا (_ر‪ #‬ا ‪'1‬ي و ا ‪X‬ا ا رن‪.‬‬
‫ ت‪ #3 @ :‬ا ‪ )@2  )6‬أاج آ @ ا‪'R‬ا‪ U‬آا‪. X  Y4‬‬
‫ا
‪./ :‬‬
‫‪ %
]I+‬ا‪2"D‬ل‬
‫ ‪/ :‬‬
‫ا
ف‪ >?3 :‬دا
ا (_ر‪ #‬ا ‪'1‬ي و ا ‪X‬ا ا رن‪.‬‬
‫ ت‪ #3 @ :‬ا ‪ )@2  )6‬أاج آ @ ا‪'R‬ا‪ U‬آا‪. X  Y4‬‬
‫ا
‪./ :‬‬
‫‪.+‬ر‪ %‬ا‪$‬اد ‪ n‬و )‪( n + 1‬‬
‫‪n +1‬‬
‫‪n‬‬

‫ ‪/ :‬‬
‫ا
ف‪ >?3 :‬دا
ا (_ر‪ #‬ا ‪'1‬ي و ا ‪X‬ا ا رن‪.‬‬
‫ ت‪ #3 @ :‬ا ‪ )@2  )6‬أاج آ @ ا‪'R‬ا‪ U‬آا‪. X  Y4‬‬

‫‪(α ∈ ℝ ) x‬‬ ‫‪֏xα‬‬ ‫ا


وال‬
‫ ‪/ :‬‬
‫ا
ف‪ >?3 :‬ا وال ا^‪
7‬و ا ‪X‬ا ا رن‪.‬‬
‫ ت‪ #3 @ :‬ا ‪ )@2  )6‬أاج آ @ ا‪'R‬ا‪ U‬آا‪. X  Y4‬‬
‫ا
‪./ :‬‬
‫‪2‬ذج د‪42-‬ا\‬
‫ ‪/ :‬‬
‫ا
ف‪R >?3 :‬ى د  ‪.3 Y4‬‬
‫ ت‪ #3 @ :‬ا ‪ )@2  )6‬أاج آ @ ا‪'R‬ا‪ U‬آا‪. X  Y4‬‬
‫ا
‪./ :‬‬

‫\رة ا
)‬
‫ ‪/ :‬‬
‫ا
ف‪ >?3 :‬ا ‪
9‬ا^‪
7‬و ا ‪X‬ا ا رن‪.‬‬
‫ ت‪ #3 @ :‬ا ‪ )@2  )6‬أاج آ @ ا‪'R‬ا‪ U‬آا‪. X  Y4‬‬
‫ا
‪./ :‬‬

‫ﺍﻟﺘﻤﺎﺭﻳﻦ‬
‫ﺘﻤﺎﺭﻴﻥ ﺘﻁﺒﻴﻘﻴﺔ‬
‫‪ 1‬ـ ‪4‬ى ‪ 2‬د ‪+, -.+ /001‬‬
‫‪3‬‬ ‫‪1‬‬ ‫‪3‬‬ ‫‪1‬‬
‫‪4‬‬
‫‪a = 9 2 × 27 4 = 32‬‬ ‫) ( ) (‬
‫‪2‬‬ ‫‪× 33‬‬ ‫‪4‬‬

‫‪3‬‬ ‫‪3‬‬ ‫‪15‬‬


‫‪3+‬‬
‫‪a = 3 ×3 = 3‬‬
‫‪3‬‬ ‫‪4‬‬ ‫‪4‬‬
‫‪=3‬‬ ‫‪4‬‬

‫‪5‬‬ ‫‪5‬‬ ‫‪65‬‬


‫‪−‬‬
‫‪b = 3 × 81 = 3‬‬
‫‪4‬‬ ‫‪3‬‬ ‫‪12‬‬

‫‪1‬‬ ‫‪1‬‬ ‫‪7‬‬

‫) (‬
‫‪−‬‬ ‫‪−‬‬
‫‪−4 3‬‬
‫‪c= 3‬‬ ‫‪× 27‬‬ ‫‪3‬‬
‫‪=3‬‬ ‫‪3‬‬

‫‪eln12 = eln 3‬‬ ‫‪ 12 x = 3 (1 7‬ﺘﻜﺎﻓﺊ‬


‫‪x‬‬

‫‪e x ln12 = eln 3‬‬ ‫ﺘﻜﺎﻓﺊ‬


‫‪x ln12 = ln 3‬‬ ‫ﺘﻜﺎﻓﺊ‬
‫‪ln 3‬‬
‫=‪x‬‬ ‫ﺘﻜﺎﻓﺊ‬
‫‪ln12‬‬
‫‪x‬‬
‫‪ln 8‬‬ ‫‪1 1‬‬
‫=‪x‬‬ ‫‪   = (2‬ﺘﻜﺎﻓﺊ‬
‫‪ln 4‬‬ ‫‪4 8‬‬
‫‪x‬‬
‫‪ln 3‬‬ ‫‪1‬‬
‫‪x=−‬‬ ‫‪   = 3 (3‬ﺘﻜﺎﻓﺊ‬
‫‪ln 2‬‬ ‫‪2‬‬
‫‪ln 5‬‬
‫=‪x‬‬ ‫‪ 5 x −1 = 2 x (4‬ﺘﻜﺎﻓﺊ‬
‫‪ln 5 − ln 2‬‬
‫‪ln 4‬‬
‫=‪x‬‬ ‫‪ 3x = 42 x +1 (5‬ﺘﻜﺎﻓﺊ‬
‫‪3‬‬
‫‪ln‬‬
‫‪16‬‬
‫‪4‬‬ ‫‪1‬‬
‫= ‪ 51−3 x‬ﺘﻜﺎﻓﺊ = ‪x‬‬ ‫‪(6‬‬
‫‪3‬‬ ‫‪125‬‬
‫‪ 5− x < 52 x (4 12‬ﺘﻜﺎﻓﺊ ‪ − x ln 5 < 2 x ln 5‬ﺘﻜﺎﻓﺊ [∞‪x ∈ ]0; +‬‬
2.2 x − 1 2x 1
x ∈ ]−∞; −1[ ‫ ﺘﻜﺎﻓﺊ‬2 x+1 < 1 ‫ﺘﻜﺎﻓﺊ‬ < 0 ‫ﺘﻜﺎﻓﺊ‬ < (5
(
3 2x + 1 ) 2 +1 3
x

x
1  1 
. x ∈ [ −2; +∞[ ‫ ﺘﻜﺎﻓﺊ‬− x ≤ 1 ‫ ﺘﻜﺎﻓﺊ‬− x ln 2 ≤ ln 2 ‫ ﺘﻜﺎﻓﺊ‬  ≤ 2 (6
2  2
x
x ֏ n x ‫ و‬x ֏ a :‫ ـ درا ا وال‬2
2 x 3x
+ = 1 ‫ ﺘﻜﺎﻓﺊ‬2 x + 3x = 5 x (1 38
5x 5x
x x
 2 3
  +   = 1 ‫ ﺘﻜﺎﻓﺊ‬2 + 3 = 5
x x x

 5 5
 2   x ln 5   3   x ln 5 
2 3 x x
 2 3
f ' ( x ) =  ln   e  +  ln   e  ، f ( x ) =   +   (2
 5    5    5 5
x x
 2  2   3  3 
lim f ( x) = 0 ، lim f ( x) = +∞ f ' ( x ) =  ln   +  ln  
y
x →+∞ x →−∞  5  5   5  5 
5
x −∞ +∞
4
f '( x) -
3

2
+∞
f
1

0
-1 0 1 2 3 x

‫رن‬0‫ا ا‬7‫ ـ ا‬3


lim f ( x ) = lim x 2 + 1 − e x (1 40
x →+∞ x →+∞

 1 ex 
lim f ( x ) = lim x 2  1 + 2 − 2  = +∞
x →+∞ x →+∞
 x x 
ex − x ex x
lim f ( x ) = lim = lim − 2 = +∞ (2
x →+∞ x →+∞ x2 x →+∞ x 2 x
lim f ( x) = lim 2 xe x − e x = 0 (1 47
x →−∞ x →−∞

x
lim g ( x) = lim xe −2 x = lim 2x
= 0 (2
x →+∞ x →+∞ x →+∞ e
3x
lim f ( x ) = lim 2 = 0 (‫( ﺃ‬1 52
x →−∞ x →−∞ x

3x
lim f ( x ) = lim 2 = +∞ (‫ﺏ‬
x →0 x →0 x

3x e x ln 3 e × [ ln 3]
x ln 3 2
e x ln 3
f ( x) = 2 = 2 = = × [ ln 3] (2
2

x [ ln 3] [ x ln 3]
2 2 2
x x
‫‪e x ln 3‬‬
‫ﺒﻭﻀﻊ ‪. X = x ln 3‬‬ ‫‪lim f ( x ) = lim‬‬ ‫∞‪× [ ln 3] = +‬‬
‫‪2‬‬

‫]‪[ x ln 3‬‬
‫∞‪x →+‬‬ ‫∞‪x →+‬‬ ‫‪2‬‬

‫ﺘﻤﺎﺭﻴﻥ ﻝﻠﺘﻌﻤﻕ‪.‬‬
‫‪ 61‬ﺍﻝﺠﺯﺀ‪ (1 :1‬ﺃ( ‪lim f ( x) = 0‬‬
‫∞‪x →+‬‬

‫ﺏ( ‪ f '( x) = (1 − x)e− x‬ﺠـ(‬


‫‪x‬‬ ‫‪0‬‬ ‫‪1‬‬ ‫∞‪+‬‬

‫)‪f '( x‬‬ ‫‪+‬‬ ‫‪0‬‬ ‫‪-‬‬


‫‪1‬‬
‫‪f‬‬ ‫‪e‬‬
‫‪0‬‬ ‫‪0‬‬
‫ﺠـ(‬
‫‪y‬‬

‫‪0,35‬‬

‫‪0,3‬‬

‫‪0,25‬‬

‫‪0,2‬‬

‫‪0,15‬‬

‫‪0,1‬‬

‫‪0,05‬‬

‫‪-1‬‬ ‫‪0‬‬ ‫‪1‬‬ ‫‪2‬‬ ‫‪3‬‬ ‫‪4‬‬ ‫‪5‬‬ ‫‪6‬‬ ‫‪7‬‬ ‫‪x‬‬

‫‪ (2‬ﺃ( ﺍﻝﻤﺴﺘﻘﻴﻡ ﺍﻝﺫﻱ ﻤﻌﺎﺩﻝﺘﻪ ‪ y = m‬ﻴﻘﻁﻊ ﺍﻝﻤﻨﺤﻨﻲ ) ‪ ( Γ‬ﻓﻲ ﻨﻘﻁﺘﻴﻥ‪.‬ﺇﺫﻥ ﺍﻝﻤﻌﺎﺩﻝﺔ ‪ f ( x ) = m‬ﺘﻘﺒل ﺤﻠﻴﻥ‪.‬‬
‫ﺏ( ‪ f (0, 3573) ≈ 0, 2499‬ﻭ ‪f (0, 3574) ≈ 0, 25001‬‬
‫‪1‬‬
‫ﺠـ( ‪ f ( x) = 0‬ﺘﻜﺎﻓﺊ ‪ x = 0‬ﻭ = )‪ f ( x‬ﺘﻜﺎﻓﺊ ‪x = 1‬‬
‫‪e‬‬
‫ﺍﻝﺠﺯﺀ‪ :2‬ﺘﺼﻭﻴﺏ‪ u0 = α :‬ﻭ ‪un +1 = un e− un‬‬
‫‪ (1‬ﺃ( ‪ u0 = α‬ﻭ ‪ α > 0‬ﻭ ﺇﺫﺍ ﻜﺎﻥ ‪ un > 0‬ﻓﺈﻥ ‪ un e− un > 0‬ﻭ ﻤﻨﻪ ‪. un +1 > 0‬ﺇﺫﻥ ‪. un > 0‬‬
‫(‬
‫ﺏ( ‪un +1 − un = un e− un − 1‬‬ ‫)‬
‫ﺒﻤﺎ ﺃﻥ ‪ un > 0‬ﻭ ‪ e− un < 1‬ﻓﺈﻥ ‪ un +1 − un < 0‬ﻭ ﺒﺎﻝﺘﺎﻝﻲ ) ‪ ( un‬ﻤﺘﻨﺎﻗﺼﺔ‬
‫ﺠـ( ) ‪ ( un‬ﻤﺘﻨﺎﻗﺼﺔ ﻭ ﻤﺤﺩﻭﺩﺓ ﻤﻥ ﺍﻷﺴﻔل ﺒـ‪ 0‬ﻓﻬﻲ ﻤﺘﻘﺎﺭﺒﺔ‪.‬ﻝﺘﻜﻥ ‪ ℓ‬ﻨﻬﺎﻴﺘﻬﺎ‪.‬‬
‫ﻝﺩﻴﻨﺎ ‪ ℓ = ℓe − ℓ‬ﺘﻜﺎﻓﺊ ‪ℓ = 0‬‬
‫‪ un +1 = un e− un . wn = ln un .2‬ﻭﻤﻨﻪ ‪ln un +1 = ln un + ln e − un = ln un − un‬‬
‫ﻭﻤﻨﻪ ‪ wn +1 = wn − un‬ﺃﻱ ‪un = wn − wn+1‬‬
‫ﺏ( ‪S n = u0 + u1 + ..... + un = (w0 − w1) + (w1 − w2 ) + (wn−1 − wn ) + (wn − wn+1) = w0 − wn+1‬‬
‫ﺠـ( ﺒﻤﺎ ﺃﻥ ‪ un‬ﻴﺅﻭل ﺇﻝﻰ‪ wn ، 0‬ﻴﺅﻭل ﺇﻝﻰ ∞‪ ، −‬ﺇﺫﻥ ‪ S n‬ﻴﺅﻭل ﺇﻝﻰ ∞‪. +‬‬
‫‪1‬‬ ‫‪1‬‬
‫= ) ‪ f ( β‬ﺇّﻥ ﺇﺫﺍ ﺃﺨﺫﻨﺎ ‪ ، v0 = β‬ﺍﺒﺘﺩﺍ ‪‬ﺀ ﻤﻥ ﺍﻝﺭﺘﺒﺔ‪ 1‬ﻴﻜﻭﻥ ‪. un = vn‬‬ ‫‪ u1 = f (α ) = (3‬ﻭ‬
‫‪4‬‬ ‫‪4‬‬
‫‪ (1 62‬ﺍﻝﻤﺴﺘﻘﻴﻡ ‪ D‬ﻴﻤﺭ ﺒﺎﻝﻨﻘﻁﺘﻴﻥ )‪ J (0;1‬ﻭ )‪ K (−1; 0‬ﻤﻌﺎﺩﻝﺘﻪ ‪y = x + 1‬‬
‫‪ lim ϕ ( x ) = 0 (2‬ﻭﻤﻨﻪ ‪lim f ( x ) − ( mx + p ) = 0‬‬
‫∞‪x →+‬‬ ‫∞‪x →+‬‬

‫ﺃﻱ ﺃﻥ ﺍﻝﻤﺴﺘﻘﻴﻡ ﺍﻝﺫﻱ ﻤﻌﺎﺩﻝﺘﻪ ‪ y = mx + p‬ﻤﻘﺎﺭﺏ ﻝﻠﻤﻨﺤﻨﻲ ﻋﻨﺩ ∞‪ +‬ﻭ ﻫﻭ ﺍﻝﻤﺴﺘﻘﻴﻡ ‪. D‬ﺇﺫﻥ ‪. m = p = 1‬‬
‫ﺏ( ﺍﻝﻨﻘﻁﺔ ‪ J‬ﻤﺭﻜﺯ ﺘﻨﺎﻅﺭ ﻝﻠﻤﻨﺤﻨﻲ‪.‬‬
‫ﺠـ( )‪f ( − x ) = − x + 1 + ϕ (− x) ، f ( x ) = x + 1 + ϕ ( x‬‬
‫ﻭﻤﻨﻪ )‪ f ( x ) + f (− x) = 2 + ϕ ( x) + ϕ (− x‬ﻭ ﻨﻌﻠﻡ ﺃﻥ ‪، f ( x ) + f ( − x ) = 2‬ﺇﺫﻥ ‪ϕ ( x) + ϕ (− x) = 0‬‬
‫)‪ ϕ (− x) = −ϕ ( x‬ﻭﻤﻨﻪ ﺍﻝﺩﺍﻝﺔ ‪ ϕ‬ﻓﺭﺩﻴﺔ‬
‫ﺩ( ‪ f ( x ) + f ( − x ) = 2‬ﻭ ﻤﻨﻪ ‪ f ' ( x ) − f ' ( − x ) = 0‬ﻭﻤﻨﻪ ) ‪. f ' ( x ) = f ' ( − x‬ﺇﺫﻥ ' ‪ f‬ﺯﻭﺠﻴﺔ‪.‬‬
‫‪ (3‬ﺃ( ‪ ϕ ( x ) = (ax + b)e − x‬ﻭﻤﻨﻪ ‪، ϕ ( − x ) = (− ax + b)e − x‬‬
‫‪2‬‬ ‫‪2‬‬

‫ﺒﻤﺎ ﺃﻥ ﺍﻝﺩﺍﻝﺔ ‪ ϕ‬ﻓﺭﺩﻴﺔ ﻴﻜﻭﻥ ‪ − ax + b = − ax − b‬ﻭ ﻤﻨﻪ ‪b = 0‬‬


‫(‬ ‫)‬
‫ﺏ( ‪ f ( x ) = x + 1 + ϕ ( x) = x + 1 + axe− x‬ﻭﻤﻨﻪ ‪f ' ( x ) = 1 + ϕ '( x) = 1 + a 1 − 2 x 2 e− x‬‬
‫‪2‬‬ ‫‪2‬‬

‫ﺠـ( ﻤﻌﺎﻤل ﺘﻭﺠﻴﻪ ﺍﻝﻤﻤﺎﺱ ‪ T‬ﻋﻨﺩ ﺍﻝﻨﻘﻁﺔ ﺍﻝﺘﻲ ﻓﺎﺼﻠﺘﻬﺎ‪) 0‬ﺍﻝﻨﻘﻁﺔ ‪ (J‬ﻫﻭ ‪f '(0) = 1 − e‬‬
‫‪ f '(0) = 1 + a‬ﻤﻌﻨﺎﻩ ‪ 1 − e = 1 + a‬ﺃﻱ ‪a = −e‬‬
‫ﺩ( ‪f ( x ) = x + 1 + axe − x = x + 1 − exe− x‬‬
‫‪2‬‬ ‫‪2‬‬

‫‪x‬‬
‫= )‪f ( x‬‬
‫‪ 64‬ﺍﻝﺠﺯﺀ‪:1‬‬
‫‪e −x‬‬ ‫‪x‬‬

‫‪ g '( x) . g '( x) = e x − 1 (1‬ﻤﻭﺠﺒﺔ ﺇﺫﺍ ﻜﺎﻥ ‪ x ≥ 0‬ﻭ ﺴﺎﻝﺒﺔ ﺇﺫﺍ ﻜﺎﻥ ‪x ≤ 0‬‬
‫ﺇﺫﻥ ﺍﻝﺩﺍﻝﺔ ‪ g‬ﻤﺘﺯﺍﻴﺩﺓ ﺇﺫﺍ ﻜﺎﻥ ‪ x ≥ 0‬ﻭ ﻤﺘﻨﺎﻗﺼﺔ ﺇﺫﺍ ﻜﺎﻥ ‪ x ≤ 0‬ﻭ ‪ g (0) = 0‬ﻭ ﺒﺎﻝﺘﺎﻝﻲ ‪. g ( x) ≥ 0‬‬
‫‪ g ( x) ≥ 0 (2‬ﻤﻌﻨﺎﻩ ‪ e x − x ≥ 1‬ﺃﻱ ‪e x − x > 0‬‬
‫ﺍﻝﺠﺯﺀ‪:2‬‬
‫‪ (1‬ﺃ( ‪lim f ( x) = −1 ، lim f ( x) = 0‬‬
‫∞‪x →−‬‬ ‫∞‪x →+‬‬

‫ﺏ( ﻋﻨﺩ ∞‪ −‬ﺍﻝﻤﻨﺤﻨﻲ ) ‪ ( C‬ﻴﻘﺒل ﻤﺴﺘﻘﻴﻤﺎ ﻤﻘﺎﺭﺒﺎ ﻤﻌﺎﺩﻝﺘﻪ ‪ y = −1‬ﻭ ﻋﻨﺩ ∞‪ +‬ﺍﻝﻤﻨﺤﻨﻲ ) ‪ ( C‬ﻴﻘﺒل ﻤﺴﺘﻘﻴﻤﺎ ﻤﻘﺎﺭﺒﺎ‬
‫ﻤﻌﺎﺩﻝﺘﻪ ‪. y = 0‬‬
‫‪(1 − x)e x‬‬
‫= )‪f '( x‬‬ ‫‪ (2‬ﺃ(‬
‫‪(e‬‬ ‫)‬
‫‪2‬‬
‫‪x‬‬
‫‪−x‬‬
‫ﺏ( ﺇﺸﺎﺭﺓ )‪ f '( x‬ﻫﻲ ﻤﻥ ﻨﻔﺱ ﺇﺸﺎﺭﺓ )‪(1 − x‬‬
‫‪x‬‬ ‫∞‪−‬‬ ‫‪1‬‬ ‫∞‪+‬‬

‫)‪f '( x‬‬ ‫‪+‬‬ ‫‪0‬‬ ‫‪-‬‬


‫‪1‬‬
‫‪f‬‬ ‫‪e −1‬‬
‫‪-1‬‬ ‫‪0‬‬
‫‪ (3‬ﺃ( ﻤﻌﺎﺩﻝﺔ ﺍﻝﻤﻤﺎﺱ ‪ T‬ﻝﻠﻤﻨﺤﻨﻲ ) ‪ ( C‬ﻋﻨﺩ ﺍﻝﻨﻘﻁﺔ ﺍﻝﺘﻲ ﻓﺎﺼﻠﺘﻬﺎ‪ 0‬ﻫﻲ ‪. y = x‬‬
‫)‪− xg ( x‬‬
‫= ‪. f ( x) − x‬‬ ‫ﺏ( ﻭﻀﻌﻴﺔ ﺍﻝﻤﻨﺤﻨﻲ ) ‪ ( C‬ﺒﺎﻝﻨﺴﺒﺔ ﻝﻠﻤﻤﺎﺱ‪: T‬‬
‫‪ex − x‬‬
‫ﺒﻤﺎ ﺃﻥ ‪ g ( x) ≥ 0‬ﻭ ‪ e x − x > 0‬ﻓﺈﻥ ﺇﺸﺎﺭﺓ ‪ f ( x ) − x‬ﻫﻲ ﻤﻥ ﺇﺸﺎﺭﺓ )‪(− x‬‬
‫ﻓﻲ ﺍﻝﻤﺠﺎل [‪ ( C ) ]−∞;0‬ﺃﻋﻠﻰ‪ T‬ﻭ ﻓﻲ ﺍﻝﻤﺠﺎل [∞‪ ( C ) ]0; +‬ﺃﺴﻔل ‪. T‬‬
‫‪y‬‬ ‫‪(4‬‬

‫‪-5 -4 -3 -2 -1 0‬‬ ‫‪1 2 3 4 5 6 7 x‬‬

‫‪-1‬‬

‫‪f ' ( x ) = ( x + 1)(1 − x)e − x .1 66‬‬


‫‪ f ' ( x ) > 0‬ﺇﺫﺍ ﻜﺎﻥ ‪−1 < x < 1‬‬
‫‪ f ' ( x ) < 0‬ﺇﺫﺍ ﻜﺎﻥ ‪ x < −1‬ﺃﻭ ‪x > 1‬‬
‫‪ f ' ( x ) = 0‬ﺇﺫﺍ ﻜﺎﻥ ‪ x = −1‬ﺃﻭ ‪x = 1‬‬
‫[∞‪[1; +‬‬ ‫ﺇﺫﻥ ﺍﻝﺩﺍﻝﺔ ‪ f‬ﻤﺘﺯﺍﻴﺩﺓ ﺘﻤﺎﻤﺎ ﻓﻲ ﺍﻝﻤﺠﺎل ]‪ [ −1;1‬ﻭ ﻤﺘﻨﺎﻗﺼﺔ ﺘﻤﺎﻤﺎ ﻓﻲ ﺍﻝﻤﺠﺎﻝﻴﻥ ]‪ ]−∞; −1‬ﻭ‬
‫‪x2‬‬
‫‪ lim f ( x) = lim x 2e − x = lim‬ﺍﻝﻤﻨﺤﻨﻲ ) ‪ ( C‬ﻴﻘﺒل‬
‫‪1‬‬ ‫‪2 −x‬‬
‫‪= lim x = 0 ، lim f ( x) = lim x e = +∞ .2‬‬
‫∞‪x →+‬‬ ‫∞‪x →+‬‬ ‫‪x →+∞ e‬‬ ‫‪x‬‬ ‫‪x →+∞ e‬‬ ‫‪x‬‬ ‫∞‪→−‬‬ ‫‪x‬‬ ‫∞‪→−‬‬

‫‪x2‬‬
‫ﻤﺴﺘﻘﻴﻤﺎ ﻤﻘﺎﺭﺒﺎ ﻤﻌﺎﺩﻝﺘﻪ ‪ y = 0‬ﻋﻨﺩ ∞‪+‬‬
‫‪5‬‬
‫‪y‬‬ ‫‪.3‬ﺍﻝﺘﻤﺜﻴل ﺍﻝﺒﻴﺎﻨﻲ‪:‬‬

‫‪4‬‬

‫‪3‬‬

‫‪2‬‬

‫‪1‬‬

‫‪-2‬‬ ‫‪-1‬‬ ‫‪0‬‬ ‫‪1‬‬ ‫‪2‬‬ ‫‪3‬‬ ‫‪4‬‬ ‫‪5 x‬‬

‫‪.4‬ﺃ( ﺇﺫﺍ ﻜﺎﻥ ‪ k < 0‬ﺍﻝﻤﻌﺎﺩﻝﺔ ﻻ ﺘﻘﺒل ﺤﻠﻭﻻ‪.‬‬


‫ﻜﺎﻥ ‪ k = 0‬ﺍﻝﻤﻌﺎﺩﻝﺔ ﺘﻘﺒل ﺤﻼ ﻭﺍﺤﺩﺍ ‪x = −1‬‬ ‫‪ -‬ﺇﺫﺍ‬
‫‪4‬‬
‫ﻜﺎﻥ < ‪ 0 < k‬ﺍﻝﻤﻌﺎﺩﻝﺔ ﺘﻘﺒل ‪ 3‬ﺤﻠﻭل‪.‬‬ ‫‪ -‬ﺇﺫﺍ‬
‫‪e‬‬
‫‪4‬‬
‫ﻜﺎﻥ = ‪ k‬ﺍﻝﻤﻌﺎﺩﻝﺔ ﺘﻘﺒل ﺤﻠﻴﻥ ﺃﺤﺩﻫﻤﺎ ‪x = 1‬‬ ‫‪ -‬ﺇﺫﺍ‬
‫‪e‬‬
‫‪4‬‬
‫ﻜﺎﻥ > ‪ k‬ﺍﻝﻤﻌﺎﺩﻝﺔ ﺘﻘﺒل ﺤﻼ ﻭﺍﺤﺩﺍ‬ ‫‪ -‬ﺇﺫﺍ‬
‫‪e‬‬
‫‪4‬‬
‫≤ )‪ f ( x‬ﻭ ﺒﺎﻝﺘﺎﻝﻲ ‪. f ( x) < 2‬ﺇﺫﻥ ﺍﻝﻤﻌﺎﺩﻝﺔ ‪ f ( x ) = 2‬ﻝﻴﺱ ﻝﻬﺎ ﺤل ﻋﻠﻰ ﺍﻝﻤﺠﺎل‬ ‫ﺏ( ‪ -‬ﺇﺫﺍ ﻜﺎﻥ ‪ x > −1‬ﻓﺈﻥ‬
‫‪e‬‬
‫[∞‪[ −1; +‬‬
‫‪ -‬ﺇﺫﺍ ﻜﺎﻥ ‪ x < −1‬ﻓﺈﻥ ﺍﻝﺩﺍﻝﺔ ‪ f‬ﻤﺴﺘﻤﺭﺓ ﻭ ﺭﺘﻴﺒﺔ ﺘﻤﺎﻤﺎ ﻭ ﺘﺄﺨﺫ ﻗﻴﻤﻬﺎ ﻓﻲ ﺍﻝﻤﺠﺎل [∞‪. ]0; +‬ﺒﻤﺎ ﺃﻥ ‪ 2‬ﻴﻨﺘﻤﻲ ﺇﻝﻰ ﺍﻝﻤﺠﺎل‬
‫[∞‪ ]0; +‬ﻓﺈﻨﻪ ﺘﻭﺠﺩ ﻗﻴﻤﺔ ﻭﺤﻴﺩﺓ ‪ x‬ﺘﺤﻘﻕ ‪f ( x ) = 2‬‬
‫‪ f (−2) ≈ 7,39‬ﻭ ‪f (−1) = 0‬‬
‫ﺒﻤﺎ ﺃﻥ ‪ 0 < 2 < 7,39‬ﻓﺈﻥ ‪−2 < α < −1‬‬
‫ﺠـ( ﻨﻌﻠﻡ ﺃﻥ ‪ f (α ) = 2‬ﻭ ﻤﻨﻪ ‪ (α + 1) 2 e −α = 2‬ﻭﻤﻨﻪ ‪(α + 1)2 = 2eα‬‬

‫‪(α + 1 = −‬‬ ‫( )‬
‫‪ α + 1 = 2eα‬ﺃﻭ ‪2eα‬‬ ‫ﻭﻤﻨﻪ )‬
‫‪α‬‬
‫ﺒﻤﺎ ﺃﻥ ‪ α < −1‬ﻓﺈﻥ ‪α = −1 − 2e 2‬‬
‫‪2 − ln x‬‬
‫= )‪f '( x‬‬ ‫‪ (1 68‬ﺃ(‬
‫‪2x x‬‬
‫ﺏ( ∞‪lim f ( x) = 0 ، lim f ( x) = −‬‬
‫∞‪x →+‬‬ ‫‪x →0‬‬

‫‪x‬‬ ‫‪0‬‬ ‫‪e‬‬ ‫‪2‬‬


‫∞‪+‬‬
‫)‪f '( x‬‬ ‫‪+‬‬ ‫‪0‬‬ ‫‪-‬‬
‫‪2‬‬
‫‪f‬‬ ‫‪e‬‬
‫∞‪−‬‬ ‫‪0‬‬
‫‪ f (1) = 0 (2‬ﻭ ‪ f '(1) = 1‬ﺇﺫﻥ ﻤﻌﺎﺩﻝﺔ ‪ T‬ﻫﻲ ‪y = x − 1‬‬
‫‪g ( x ) = x − 1 − f ( x ) (3‬‬

‫= )‪g '( x) = 1− f '( x‬‬


‫‪1 ‬‬
‫‪2x x ‬‬
‫(‬
‫ﺃ( ‪ln x + 2 x x − 1 ‬‬
‫‪‬‬ ‫)‬
‫(‬ ‫)‬
‫ﺏ( ‪، g '(1) = 0‬ﻭ ﺇﺸﺎﺭﺓ ) ‪ g ' ( x‬ﻫﻲ ﻤﻥ ﻨﻔﺱ ﺇﺸﺎﺭﺓ ‪ln x + 2 x x − 1‬‬

‫• ﻋﻠﻰ [‪ ln x < 0 : ]0;1‬ﻭ ‪ x x − 1 < 0‬ﻭﻤﻨﻪ ‪g ' ( x ) < 0‬‬


‫‪y‬‬ ‫• ﻋﻠﻰ [∞‪ ln x > 0 : ]1; +‬ﻭ ‪ x x − 1 > 0‬ﻭﻤﻨﻪ ‪. g ' ( x ) < 0‬‬
‫ﺠـ( ‪g (1) = 0‬‬
‫‪1‬‬

‫‪0‬‬ ‫‪1‬‬ ‫‪2‬‬ ‫‪3‬‬ ‫‪4‬‬ ‫‪5‬‬ ‫‪x‬‬


‫‪x‬‬ ‫‪0‬‬ ‫‪1‬‬ ‫∞‪+‬‬
‫‪-1‬‬
‫)‪g '( x‬‬ ‫‪-‬‬ ‫‪0‬‬ ‫‪+‬‬
‫‪-2‬‬

‫‪-3‬‬ ‫‪g‬‬
‫‪0‬‬
‫ﻨﺴﺘﻨﺘﺞ ﻤﻥ ﺠﺩﻭل ﺍﻝﺘﻐﻴﺭﺍﺕ ﺃﻨﻪ ﻤﻥ ﺃﺠل ﻜل ‪ x‬ﻤﻥ [∞‪g ( x ) ≥ 0 : ]0; +‬‬
‫‪-4‬‬

‫ﺩ( ﻤﻥ ﺃﺠل ﻜل ‪ x‬ﻤﻥ [∞‪ ( C ) ، ]0; +‬ﺃﺴﻔل ‪.T‬‬


‫‪ (4‬ﺍﻝﺭﺴﻡ)ﺍﻨﻅﺭ ﺍﻝﺸﻜل(‬
‫ﻤﺴﺎﺌل‬
‫‪ex −1‬‬
‫= )‪f ( x‬‬ ‫‪ 73‬ﺍﻝﺠﺯﺀ ﺍﻷﻭل‪:‬‬
‫‪xe x + 1‬‬
‫‪h '( x) = e x ( x + 1) .1‬‬

‫‪x‬‬ ‫∞‪−‬‬ ‫‪-1‬‬ ‫∞‪+‬‬


‫)‪h '( x‬‬ ‫‪-‬‬ ‫‪0‬‬ ‫‪+‬‬

‫‪h‬‬ ‫‪1‬‬
‫‪1−‬‬
‫‪e‬‬
‫‪1‬‬
‫ﺃﻱ ‪ h( x) > 0‬ﻤﻥ ﺃﺠل ‪ x‬ﻤﻥ ‪. ℝ‬‬ ‫‪h( x) ≥ 1 −‬‬ ‫ﻤﻥ ﺃﺠل ﻜل ﻋﺩﺩ ﺤﻘﻴﻘﻲ ‪: x‬‬
‫‪e‬‬
‫‪ .2‬ﺘﺼﻭﻴﺏ‪ g ( x) :‬ﺒﺩﻻ ﻤﻥ )‪h( x‬‬
‫‪ 2 ex‬‬ ‫‪‬‬
‫‪lim g ( x) = lim x  1 + −‬‬ ‫‪ = −∞ ، xlim‬‬ ‫ﺃ‪g ( x) = −∞ -‬‬
‫∞‪x →+‬‬ ‫∞‪x →+‬‬ ‫∞‪→−‬‬
‫‪ x x‬‬ ‫‪‬‬
‫ﺏ( ‪g '( x) = 1 − e x‬‬
‫‪x‬‬ ‫∞‪−‬‬ ‫‪β‬‬ ‫‪0‬‬ ‫‪α‬‬ ‫∞‪+‬‬
‫)‪g '( x‬‬ ‫‪+‬‬ ‫‪0‬‬ ‫‪-‬‬
‫‪0‬‬ ‫‪1‬‬ ‫‪0‬‬
‫‪g‬‬
‫∞‪−‬‬ ‫∞‪−‬‬
‫ﺠـ( ﻨﺴﺘﻌﻤل ﻤﺒﺭﻫﻨﺔ ﺍﻝﻘﻴﻡ ﺍﻝﻤﺘﻭﺴﻁﺔ‬
‫ﻭ ﺇﺫﺍ ﻜﺎﻥ [ ‪ x ∈ ]β ; α‬ﻓﺈﻥ ‪g ( x) > 0‬‬ ‫ﺩ( ﺇﺫﺍ ﻜﺎﻥ [∞‪ x ∈ ]−∞; β [ ∪ ]α ; +‬ﻓﺈﻥ ‪g ( x) < 0‬‬
‫ﺍﻝﺠﺯﺀ ﺍﻝﺜﺎﻨﻲ‪ :‬ﺩﺭﺍﺴﺔ ﺘﻐﻴﺭﺍﺕ ﺍﻝﺩﺍﻝﺔ ‪ f‬ﻭ ﺭﺴﻡ ﺍﻝﻤﻨﺤﻨﻲ ‪: C‬‬
‫‪lim f ( x) = 0 ، lim f ( x) = −1 .1‬‬
‫∞‪x →+‬‬ ‫∞‪x →−‬‬

‫= )‪f '( x‬‬


‫‪x‬‬
‫(‬ ‫‪x‬‬
‫( )‬
‫‪e xe + 1 − xe + e x e x − 1‬‬ ‫‪.2‬ﺃ( ) ()‬
‫‪x‬‬

‫)‪( xe + 1‬‬
‫‪2‬‬
‫‪x‬‬

‫‪2e x − e 2 x + xe x‬‬ ‫)‪e (2 − e + x‬‬


‫‪x‬‬ ‫‪x‬‬
‫)‪e g ( x‬‬ ‫‪x‬‬
‫= )‪f '( x‬‬ ‫=‬ ‫=‬
‫‪( xe‬‬ ‫)‬ ‫)‪( xe + 1‬‬ ‫)‪( xe + 1‬‬
‫‪2‬‬ ‫‪2‬‬
‫‪+1‬‬
‫‪x‬‬ ‫‪x‬‬ ‫‪2‬‬
‫‪x‬‬

‫ﺏ( ﺠﺩﻭل ﺍﻝﺘﻐﻴﺭﺍﺕ‬


‫‪x‬‬ ‫∞‪−‬‬ ‫‪β‬‬ ‫‪α‬‬ ‫∞‪+‬‬
‫)‪f '( x‬‬ ‫‪-‬‬ ‫‪0 +‬‬ ‫‪0‬‬ ‫‪-‬‬
‫‪-1‬‬ ‫) ‪f (α‬‬
‫‪f‬‬
‫) ‪f (β‬‬ ‫‪0‬‬
‫‪α‬‬
‫‪ (3‬ﺃ( ‪ g (α ) = 0‬ﻭﻤﻨﻪ ‪e = α + 2‬‬
‫‪eα − 1‬‬ ‫‪α + 2 −1‬‬ ‫‪α +1‬‬ ‫‪1‬‬
‫= ) ‪f (α‬‬ ‫=‬ ‫=‬ ‫=‬
‫‪α e + 1 α (α + 2 ) + 1 (α + 1) α + 1‬‬
‫‪α‬‬ ‫‪2‬‬

‫‪ 1,14 < α < 1,15‬ﻭ ﻤﻨﻪ ‪2,14 < α + 1 < 2,15‬‬ ‫ﺏ( ﺘﺼﻭﻴﺏ ‪ :‬ﻋﻴﻥ ﺤﺼﺭﺍ ﻝﻠﻌﺩﺩ ) ‪f (α‬‬
‫‪1‬‬ ‫‪1‬‬ ‫‪1‬‬
‫ﺃﻱ ‪) 0, 465 < f (α ) < 0, 467‬ﺍﻝﺼﺭ ﺴﻌﺘﻪ ‪( 2 ×10−3‬‬ ‫<‬ ‫<‬ ‫ﻭﻤﻨﻪ‬
‫‪2,15 α + 1 2,14‬‬
‫‪.4‬ﻤﻌﺎﺩﻝﺔ ﺍﻝﻤﻤﺎﺱ ‪ T‬ﻫﻲ ‪y = x‬‬
‫‪ex −1‬‬ ‫‪e x − 1 − x 2e x − x e x − 1 − x 2e x − x + xe x − xe x‬‬
‫= ‪f ( x) − x‬‬ ‫‪−‬‬ ‫‪x‬‬ ‫=‬ ‫=‬ ‫‪ .5‬ﺃ (‬
‫‪xe x + 1‬‬ ‫‪xe x + 1‬‬ ‫‪xe x + 1‬‬

‫= ‪f ( x) − x‬‬
‫(‬ ‫( )‬
‫‪x e x − xe x − 1 + e x − xe x − 1‬‬ ‫‪) = (e‬‬ ‫‪x‬‬
‫)‬
‫)‪− xe x − 1 ( x + 1‬‬
‫‪xe x + 1‬‬ ‫‪xe x + 1‬‬

‫ﺏ( ‪u '( x) = − xe x‬‬

‫ﻫﻲ ﻤﻥ ﻨﻔﺱ ﺇﺸﺎﺭﺓ ) ‪( − x‬‬ ‫ﺇﺸﺎﺭﺓ )‪u '( x‬‬

‫‪x‬‬ ‫∞‪−‬‬ ‫‪0‬‬ ‫∞‪+‬‬

‫)‪u '( x‬‬ ‫‪+‬‬ ‫‪0‬‬ ‫‪-‬‬


‫‪0‬‬
‫‪u‬‬

‫ﺇﺸﺎﺭﺓ )‪ : u ( x‬ﻤﻥ ﺃﺠل ﻜل ﻋﺩﺩ ﺤﻘﻴﻘﻲ ‪u ( x) ≤ 0 ،x‬‬

‫ﺠـ( ﺇﺸﺎﺭﺓ ‪ f ( x) − x‬ﻫﻲ ﻤﻥ ﻨﻔﺱ ﺇﺸﺎﺭﺓ )‪− ( x + 1‬‬

‫ﺃﺴﻔل ‪ T‬ﻓﻲ ﺍﻝﻤﺠﺎل [‪]−1;0‬‬ ‫) ‪ (C‬ﺃﻋﻠﻰ ‪ T‬ﻓﻲ ﺍﻝﻤﺠﺎﻝﻴﻥ [‪ ]−∞; −1‬ﻭ [∞‪ ]0; +‬ﻭ ) ‪(C‬‬

‫‪y‬‬
‫‪ (6‬ﺍﻝﺭﺴﻡ‬
‫‪1‬‬

‫‪-4‬‬ ‫‪-3‬‬ ‫‪-2‬‬ ‫‪-1‬‬ ‫‪0‬‬ ‫‪1‬‬ ‫‪2‬‬ ‫‪3‬‬ ‫‪4‬‬ ‫‪x‬‬

‫‪-1‬‬
‫ﺍﻷﻧﺸﻄﺔ‬
‫ا
ط اول‬
‫ ‪/ :‬‬
‫ا
ف‪ >'63 :‬ا ا
ا^‪.
(f‬‬
‫ ت‪ :‬م ا ‪-‬ط آ*) ‪P‬ا ا ‪1‬ب و ج ‪' #‬ة " ا وال ا^‪."
(f‬‬
‫ا
‪./ :‬‬
‫ا
ط ا
‬
‫ ‪/ :‬‬
‫ا
ف‪ :‬إ<ء د‪ 
9‬م ا ا
ا^‪.
(f‬‬
‫ ت‪ :‬م ا ‪-‬ط  أاج ‪'21‬ة ‪ 6‬ا ‪-‬ط ا^ول‪.‬‬
‫ا
‪./ :‬‬
‫ا
ط ا

‬
‫ ‪/ :‬‬
‫ا
ف‪ :‬إ'از وا
ا ا
ا^‪
(f‬ا  ‪.. 6 Z'2 Sa3‬‬
‫ ت‪ :‬م ا ‪-‬ط  أاج ‪'21‬ة ‪ 6‬ا ‪-‬ط ا ‪. V‬‬
‫ا
‪./ :‬‬

‫ﺍﻷﻋﻤﺎﻝ ﺍﻟﻤﻮﺟﻬﺔ‬
‫درا‪ %D‬دا
‪ %‬أ‪% :N‬‬
‫ ‪/ :‬‬
‫ا
ف‪ :‬إ'از إ@
)  ‪ i6‬ا ‪9a‬ت ( درا‪';3
7‬ات دا
أ‪
(f‬دون ‪1 63‬ر‪
9  3‬ا ! ل‪.‬‬
‫ ت‪ #3 @ :‬ا ‪ )@2  )6‬أاج آ @ ا‪'R‬ا‪ U‬آا‪. X  Y4‬‬
‫ا
‪./ :‬‬
‫! دوال أ‪
% :N‬ا
‪%‬‬
‫ ‪/ :‬‬
‫ا
ف‪* >?3 :‬اص ا وال ا^‪
(f‬و ا‪1 7‬ط ‪ i6‬ا <'ق ‪. 6‬‬
‫ ت‪ #3 @ :‬ا ‪ )@2  )6‬أاج آ @ ا‪'R‬ا‪ U‬آا‪. X  Y4‬‬
‫ا
‪./ :‬‬
‫‪x ֏ u ′ ( x )e‬‬ ‫) ‪u (x‬‬
‫ا
وال ا‪:
% :N‬وال‬
‫ ‪/ :‬‬
‫ا
ف‪* >?3 :‬اص ا وال ا^‪
(f‬و ا‪1 7‬ط ‪ i6‬ا <'ق ‪. 6‬‬
‫ ت‪ #3 @ :‬ا ‪ )@2  )6‬أاج‪.‬‬
‫ا
‪./ :‬‬
‫) ‪u ′( x‬‬
‫֏ ‪x‬‬ ‫ا
وال ا‪:
% :N‬وال‬
‫) ‪u (x‬‬
‫ ‪/ :‬‬
‫ا
ف‪* >?3 :‬اص ا وال ا^‪
(f‬و ا‪1 7‬ط ‪ i6‬ا <'ق ‪. 6‬‬
‫ ت‪ #3 @ :‬ا ‪ )@2  )6‬أاج ‪.‬‬
‫ا
‪./ :‬‬
‫ﺍﻟﺘﻤﺎﺭﻳﻦ‬
‫‪2‬ر‪% . /0 -‬‬
‫‪ 1‬ـ ا
وال ا‪% :N‬‬
‫ﻨﺒﻴﻥ ﺃﻥ ) ‪F ' ( x ) = f ( x‬‬ ‫‪1‬‬
‫ﺍﻝﺩﺍﻝﺔ ﺍﻷﺼﻠﻴﺔ ﻝﻠﺩﺍﻝﺔ ‪ f‬ﻫﻲ ‪H‬‬ ‫‪(2‬‬ ‫‪ (1‬ﺍﻝﺩﺍﻝﺔ ﺍﻷﺼﻠﻴﺔ ﻝﻠﺩﺍﻝﺔ ‪ f‬ﻫﻲ ‪H‬‬ ‫‪2‬‬
‫ﺍﻝﺩﺍﻝﺔ ﺍﻷﺼﻠﻴﺔ ﻝﻠﺩﺍﻝﺔ ‪ h‬ﻫﻲ ‪F‬‬ ‫‪(4‬‬ ‫‪ (3‬ﺍﻝﺩﺍﻝﺔ ﺍﻷﺼﻠﻴﺔ ﻝﻠﺩﺍﻝﺔ ‪ g‬ﻫﻲ ‪K‬‬
‫‪ (5‬ﺍﻝﺩﺍﻝﺔ ﺍﻷﺼﻠﻴﺔ ﻝﻠﺩﺍﻝﺔ ‪ k‬ﻫﻲ ‪G‬‬
‫‪ 2‬ـ ﺤﺴﺎﺏ ﺍﻝﺩﻭﺍل ﺍﻷﺼﻠﻴﺔ‬

‫‪f ( x ) = e−2 x ( e −2 x + 2 ) = −  −2e −2 x ( e−2 x + 2 )  (5 22‬‬


‫‪3‬‬ ‫‪1‬‬ ‫‪3‬‬

‫‪2‬‬‫‪‬‬ ‫‪‬‬

‫(‬ ‫ﻤﺠﻤﻭﻋﺔ ﺍﻝﺩﻭﺍل ﺍﻷﺼﻠﻴﺔ ﻝﻠﺩﺍﻝﺔ ‪ f‬ﻋﻠﻰ ‪ ℝ‬ﻫﻲ ﺍﻝﺩﻭﺍل ‪e + 2 ) + c‬‬


‫‪1 −2 x‬‬ ‫‪4‬‬
‫‪ F : x ֏ −‬ﺤﻴﺙ ‪ c‬ﺜﺎﺒﺕ ﺤﻘﻴﻘﻲ‪.‬‬
‫‪8‬‬
‫‪1‬‬ ‫‪1‬‬ ‫‪1‬‬
‫= )‪f ( x‬‬ ‫× =‬ ‫‪(5 23‬‬
‫)‪x(ln x + 2‬‬ ‫‪2‬‬
‫‪x (ln x + 2)2‬‬
‫‪1‬‬
‫‪ F : x ֏ −‬ﺤﻴﺙ ‪ c‬ﺜﺎﺒﺕ ﺤﻘﻴﻘﻲ‬ ‫ﻤﺠﻤﻭﻋﺔ ﺍﻝﺩﻭﺍل ﺍﻷﺼﻠﻴﺔ ﻝﻠﺩﺍﻝﺔ ‪ f‬ﻋﻠﻰ ﺍﻝﻤﺠﺎل [∞ ‪ [1; +‬ﻫﻲ ﺍﻝﺩﻭﺍل ‪+ c‬‬
‫‪ln x + 2‬‬
‫‪2e x‬‬ ‫‪ex‬‬
‫= ) ‪I = ]0; +∞[ ، f ( x‬‬ ‫×‪= 2‬‬ ‫‪(5‬‬ ‫‪25‬‬
‫‪ex −1‬‬ ‫‪ex −1‬‬
‫ﻤﺠﻤﻭﻋﺔ ﺍﻝﺩﻭﺍل ﺍﻷﺼﻠﻴﺔ ﻝﻠﺩﺍﻝﺔ ‪ f‬ﻋﻠﻰ ﺍﻝﻤﺠﺎل [∞ ‪ [1; +‬ﻫﻲ ﺍﻝﺩﻭﺍل ‪ F : x ֏ 4 e x − 1 + c‬ﺤﻴﺙ ‪ c‬ﺜﺎﺒﺕ ﺤﻘﻴﻘﻲ‬
‫‪6x + 3‬‬ ‫‪2x + 1‬‬
‫‪f ( x) = 2‬‬ ‫‪= 3× 2‬‬ ‫‪(4 27‬‬
‫‪x + x +1‬‬ ‫‪x + x +1‬‬
‫ﻤﺠﻤﻭﻋﺔ ﺍﻝﺩﻭﺍل ﺍﻷﺼﻠﻴﺔ ﻝﻠﺩﺍﻝﺔ ‪ f‬ﻋﻠﻰ ‪ ℝ‬ﻫﻲ ﺍﻝﺩﻭﺍل ‪ F : x ֏ 3ln ( x 2 + x + 1) + c‬ﺤﻴﺙ ‪ c‬ﺜﺎﺒﺕ ﺤﻘﻴﻘﻲ‪.‬‬
‫‪3 1x‬‬ ‫‪ 1  x‬‬
‫‪1‬‬
‫= )‪ f ( x‬؛‬ ‫‪e‬‬ ‫=‬ ‫‪−‬‬‫‪3‬‬ ‫×‬ ‫‪−‬‬
‫‪ 2‬‬ ‫‪e‬‬ ‫‪(4 28‬‬
‫‪x2‬‬ ‫‪ x ‬‬
‫‪1‬‬
‫ﻤﺠﻤﻭﻋﺔ ﺍﻝﺩﻭﺍل ﺍﻷﺼﻠﻴﺔ ﻝﻠﺩﺍﻝﺔ ‪ f‬ﻋﻠﻰ ﺍﻝﻤﺠﺎل [∞‪ ]0; +‬ﻫﻲ ﺍﻝﺩﻭﺍل ‪ F : x ֏ −3 e x + c‬ﺤﻴﺙ ‪ c‬ﺜﺎﺒﺕ ﺤﻘﻴﻘﻲ‪.‬‬
‫‪1‬‬ ‫‪x2‬‬ ‫‪1 1 3x 2‬‬
‫‪ f ( x) = × 3‬؛‬ ‫‪= × × 3‬‬ ‫‪(3‬‬ ‫‪29‬‬
‫‪2 x +1 2 3 x +1‬‬
‫ﻤﺠﻤﻭﻋﺔ ﺍﻝﺩﻭﺍل ﺍﻷﺼﻠﻴﺔ ﻝﻠﺩﺍﻝﺔ ‪ f‬ﻋﻠﻰ ﺍﻝﻤﺠﺎل [∞‪ ]−1; +‬ﻫﻲ ﺍﻝﺩﻭﺍل ‪ln ( x 3 + 1) + c‬‬
‫‪1‬‬
‫֏ ‪ F : x‬ﺤﻴﺙ ‪ c‬ﺜﺎﺒﺕ ﺤﻘﻴﻘﻲ‪.‬‬
‫‪6‬‬
‫‪f ( x ) = sin x cos x (3 30‬‬
‫‪1‬‬ ‫‪1‬‬
‫ﺩﺍﻝﺔ ﺃﺼﻠﻴﺔ ﻝﻠﺩﺍﻝﺔ ‪ f‬ﻋﻠﻰ ‪ ℝ‬ﻫﻲ ﺍﻝﺩﺍﻝﺔ ) ‪ F : x ֏ ( sin x‬ﺃﻭ ﺍﻝﺩﺍﻝﺔ ) ‪G : x ֏ − ( cos x‬‬
‫‪2‬‬ ‫‪2‬‬

‫‪2‬‬ ‫‪2‬‬
‫‪ 3‬ـ ﺍﻝﻤﻌﺎﺩﻻﺕ ﺍﻝﺘﻔﺎﻀﻠﻴﺔ‬
‫‪1‬‬ ‫‪2 3 1 2‬‬
‫‪y =x 2 +x +‬‬ ‫‪+ c (2‬‬ ‫‪x + x − x + c (1 31‬‬
‫= ‪y‬‬
‫‪x‬‬ ‫‪3‬‬ ‫‪2‬‬
‫‪3‬‬ ‫‪1‬‬ ‫‪x 2 +1‬‬ ‫‪1‬‬
‫‪ c ، y = − cs ( 2x ) + c (4‬ﺜﺎﺒﺕ ﺤﻘﻴﻘﻲ‬ ‫= ' ‪ y‬ﻭ ‪y = x − +c‬‬ ‫‪2‬‬
‫‪= 1 + 2 (3‬‬
‫‪2‬‬ ‫‪x‬‬ ‫‪x‬‬ ‫‪x‬‬
% . /0 -‫ر‬2
f ( x ) = sin x ( a cos2 x + b cos 4 x ) 44

f ( x ) = sin x ( sin 2 x cos 2x )


f ( x ) = sin x (1 − cos 2 x ) cos2 x
f ( x ) = sin x ( cos 2 x − cos 4 x )
1 1
F ( x ) = − cos3 x + cos5 x
3 5
cos x + 3sin 2 x cos 2 x
4
48
، u '(x ) = (1
cos6 x

u '(x ) =
(
cos2 x + 3 1 − cos 2 x sin 2 x ) =
3

2
4
cos x cos x cos 2 x
4

1 2 
v ( x ) = u ' ( x ) + (2
3 cos 2 x 
1  π
‫ ﺜﺎﺒﺕ ﺤﻘﻴﻘﻲ‬k ‫ ﺤﻴﺙ‬x ֏ u ( x ) + 2 tan x  + k ‫ ﻤﻌﺭﻓﺔ ﺒـ‬0;  ‫ ﻋﻠﻰ‬v ‫ﺍﻷﺼﻠﻴﺔ ﻝﻠﺩﺍﻝﺔ‬ ‫ﺍﻝﺩﻭﺍل‬
3  4
1  sin x 
.V ( x ) =  3 + 2 tan x  ‫ ﻭ‬k = 0 ‫ ﻓﺈﻥ‬V ( 0 ) = 0 ‫ﺒﻤﺎ ﺃﻥ‬
3  cos x 
1 1 1 1
f ( x) = × + .1 37
2 ( x − 1) 2 ( x + 1)3
3

:‫] ﻫﻲ ﺍﻝﺩﻭﺍل ﻤﻥ ﺍﻝﺸﻜل‬1; +∞[ ‫ ﻋﻠﻰ‬f ‫ ﻤﺠﻤﻭﻋﺔ ﺍﻝﺩﻭﺍل ﺍﻷﺼﻠﻴﺔ ﻝﻠﺩﺍﻝﺔ‬.2


1 1 1 1
‫ ﺜﺎﺒﺕ ﺤﻘﻴﻘﻲ‬k ‫ ﺤﻴﺙ‬x ֏ − × − × +k
4 ( x − 1) 4 ( x + 1)2
2

3 1 1 1 1
k= ‫ ﺃﻱ‬− × − × + k = 1 ‫ ﻤﻌﻨﺎﻩ‬F ( 0 ) = 1 .3
2 4 ( 0 − 1) 4 ( 0 + 1)2
2

1 1 1 1 3
F ( x) = − × − × + ‫ﻭ ﺒﺎﻝﺘﺎﻝﻲ‬
4 ( x − 1) 4 ( x + 1) 2
2 2

f ( x ) = sin x + sin 3 x = sin x (1 + sin 2 x ) = sin x ( 2 − cos 2 x ) = 2sin x − sin x cos 2 x .1 43


1
F ( x ) = −2 cos x + cos3 x .2
3
f ( x ) = sin x cos x = sin x ( sin x cos x ) .1 44
3 2 2 2

f ( x ) = sin x (1 − cos 2 x ) cos 2 x


f ( x ) = sin x ( cos 2 x − cos 4 x )
1 1
F ( x ) = − cos3 x + cos5 x .2
3 5
f ( x ) = sin x cos x = cos x ( sin x cos x ) .1 45
4 5 4 4

f ( x ) = cos x sin 4 x (1 − sin 2 x ) = cos x ( sin 4 x − 2sin 6 x + sin 8 x )


2
1 2 1
F ( x ) = sin 5 x − sin 7 x + sin 9 x .2
5 7 9
f '' ( x ) = −4sin 4 x + 12 cos 2 x sin 2 x ‫ ﻭ‬f ' ( x ) = 4 cos x sin 3 x .1 46
( )
f '' ( x ) = −4 f ( x ) + 12 1 − sin 2 x sin 2 x .2
f '' ( x ) = −4 f ( x ) + 12sin x − 12sin 4 x
2

 1 − cos 2 x 
f '' ( x ) = −16 f ( x ) + 12  
 2 
1 3 3
f ( x) = − f '' ( x ) − cos 2 x + ‫ ﻭﻤﻨﻪ‬f '' ( x ) = −16 f ( x ) − 6cos 2 x + 6
16 8 8
1 3 3
. ℝ‫ ﻋﻠﻰ‬f ‫ ﺃﺼﻠﻴﺔ ﻝﻠﺩﺍﻝﺔ‬F : x ֏ − f ' ( x ) − sin 2 x + x ‫ ﻨﺴﺘﻨﺘﺞ ﺃﻥ ﺍﻝﺩﺍﻝﺔ‬.3
16 16 8
1 3 3
F ( x ) = − cos x sin 3 x − sin 2 x + x ‫ﺃﻱ‬
4 16 8
f ( x ) = tan x + tan x : ‫ﺘﺼﻭﻴﺏ‬
2004 2006
47

u ( x ) = tan x ‫ ﺤﻴﺙ‬u ' u n ‫ ﻭ ﻫﻲ ﻤﻥ ﺍﻝﺸﻜل‬f ( x ) = (1 + tan 2 x ) tan 2004 x ‫ ﻋﻠﻰ ﺍﻝﺸﻜل‬f ( x ) ‫ﻴﻤﻜﻥ ﺃﻥ ﻨﻜﺘﺏ‬
1  π π  π π
F ( x ) = tan 2005 x ‫ ﻫﻲ‬ − ;  ‫ ﺇﺫﻥ ﺩﺍﻝﺘﻬﺎ ﺍﻷﺼﻠﻴﺔ ﻋﻠﻰ‬،  − ;  ‫ﻭ ﻫﻲ ﻤﺴﺘﻤﺭﺓ ﻋﻠﻰ ﺍﻝﻤﺠﺎل‬
5  2 2  2 2
f ( x) = e x cos x 57
f " ( x ) = e ( −2sin x ) ‫ﻭ‬ f ' ( x ) = e ( cos x − sin x ) .1
x x

( b = 1) ‫ ﻭ‬ a = −  ‫ ﻤﻌﻨﺎﻩ‬f ( x) = af "( x) + bf '( x) .2


1
 2
1
f ( x ) = − f " ( x ) + f ' ( x ) ‫ﺇﺫﻥ‬
2
1
ℝ ‫ ﻋﻠﻰ‬f ‫ ﺃﺼﻠﻴﺔ ﻝﻠﺩﺍﻝﺔ‬F : x ֏ − f ' ( x ) + f ( x ) ‫ ﻨﺴﺘﻨﺘﺞ ﺃﻥ ﺍﻝﺩﺍﻝﺔ‬.3
2
1 x 1 x 1
F ( x ) = e cos x + e sin x = e x ( cos x + sin x ) ‫ﺃﻱ‬
2 2 2
( )
F ( x) = ax 3 + bx 2 + cx + d e 2 x 58
(
F '( x) = 2ax 3
+ ( 2b + 3a ) x + ( 2c + 2b ) x + 2d + c ) e
2 2x

 3  3  3  1
 d = −  ‫ ﻭ‬ c =  ‫ ﻭ‬ b = −  ‫ ﻭ‬ a =  ‫ ﻤﻌﻨﺎﻩ‬F '( x) = f ( x ) : x ‫ﻤﻥ ﺍﺠل ﻜل ﻋﺩﺩ ﺤﻘﻴﻘﻲ‬
 8  4  4  2
1 3 3 3
F ( x ) =  x3 − x 2 + x −  e 2 x
2 4 4 8
‫ﺍﻷﻧﺸﻄﺔ‬
‫ا
ط اول‬
‫ ‪/ :‬‬
‫ا
ف‪ :‬ا '‪ a  ka3 X
/  .‬ا
‪! j(
14‬ل و ا وال ا^‪.
(f‬‬
‫ ت‪ :‬م ا ‪-‬ط آ*) ‪P‬ا ا ‪1‬ب و ج ‪' #‬ة " ‪ )@3‬دا
"‪.‬‬
‫ا
‪./ :‬‬
‫ا
ط ا
‬
‫ ‪/ :‬‬
‫ا
ف‪ :‬ا ‪ 
R:6‬دا
أ‪
(f‬و ‪.X
/‬‬
‫ ت‪ :‬م ا ‪-‬ط  أاج و ج ‪ #‬ا 'ة " ‪ >?3‬ا ‪/a‬ب ا @( ‪ 6‬دوال أ‪."
(f‬‬
‫ا
‪./ :‬‬

‫ﺍﻷﻋﻤﺎﻝ ﺍﻟﻤﻮﺟﻬﺔ‬
‫درا‪ %D‬دا
‪4
2a" %‬ر‪ /  F"-‬ي‬
‫ ‪/ :‬‬
‫ا
ف‪ :‬ا‪1 7‬ط ‪/a
'Z‬ب ‪a X
/‬د  ‪. a‬‬
‫ ت‪ #3 @ :‬ا ‪ )@2  )6‬أاج آ @ ا‪'R‬ا‪ U‬آا‪. X  Y4‬‬
‫ا
‪./ :‬‬
‫دا
‪+X" %\!+ %‬‬
‫ ‪/ :‬‬
‫ا
ف‪ >'63 >?3 :‬ا @) و ا ا
ا (_ر
ا ‪.
'1‬‬
‫ ت‪ #3 @ :‬ا ‪ )@2  )6‬أاج آ @ ا‪'R‬ا‪ U‬آا‪. X  Y4‬‬
‫ا
‪./ :‬‬

‫ﺍﻟﺘﻤﺎﺭﻳﻦ‬
‫ﺘﻤﺎﺭﻴﻥ ﺘﻁﺒﻴﻘﻴﺔ‬
‫‪ 1‬ـ ‪ +X‬دا
‪%‬‬
‫‪y‬‬ ‫‪y‬‬
‫‪1‬‬ ‫‪1‬‬ ‫‪3‬‬

‫‪0‬‬ ‫‪1‬‬ ‫‪2 x‬‬ ‫‪0‬‬ ‫‪1‬‬ ‫‪x‬‬


‫‪13‬‬
‫‪I =1‬‬ ‫=‪I‬‬
‫‪8‬‬
‫‪y‬‬
‫‪2‬‬ ‫‪.1 4‬ﺍﻨﺸﺎﺀ ﺍﻝﻤﻨﺤﻨﻲ ‪. C‬‬
‫‪1‬‬ ‫‪.2‬ﻨﻌﻡ ﺍﻝﺩﺍﻝﺔ ‪ f‬ﻤﺴﺘﻤﺭﺓ ﻋﻠﻰ ]‪[ −1;1‬‬
‫‪0‬‬ ‫‪1‬‬
‫‪1‬‬ ‫‪4‬‬
‫=‪I‬‬ ‫‪∫ −2 xdx + ∫ x dx I = 1 + 3 = 3‬‬ ‫‪.3‬‬
‫‪2‬‬
‫‪-1‬‬ ‫‪0‬‬ ‫‪1 x‬‬
‫‪−1‬‬ ‫‪0‬‬
‫‪y‬‬ ‫‪5‬‬
‫‪2‬‬
‫‪.1‬‬
‫‪1‬‬

‫‪-3‬‬ ‫‪-2‬‬ ‫‪-1‬‬ ‫‪0‬‬ ‫‪1‬‬ ‫‪2‬‬ ‫‪3 x‬‬

‫‪ .2‬ﻨﻌﻡ ‪ f‬ﻤﺴﺘﻤﺭﺓ ﻋﻠﻰ ]‪. [ −3;3‬‬


‫‪−1‬‬ ‫‪0‬‬ ‫‪3‬‬
‫‪2‬‬
‫=‪I‬‬ ‫‪∫ (0.5 x + 1.5)dx + ∫ ( x + 2)dx + ∫ (− 3 x + 2)dx .3‬‬
‫‪−3‬‬ ‫‪−1‬‬ ‫‪0‬‬

‫• ‪( ∗ ) ....... y = 1 + 2 − ( x − 1)2 .1‬‬ ‫‪6‬‬


‫) ∗ ( ﺘﻜﺎﻓﺊ‪ ( x − 1) 2 + ( y − 1) 2 = 2 :‬ﻭ ‪y − 1 ≥ 0‬‬
‫ﻭﺍﻗﻌﺔ ﻓﻲ ﻨﺼﻑ ﺍﻝﻤﺴﺘﻭﻱ ﺍﻝﺫﻱ ﻤﻌﺎﺩﻝﺘﻪ ‪. y ≥ 1‬‬ ‫‪ C‬ﻫﻭ ﻨﺼﻑ ﺩﺍﺌﺭﺓ ﻤﺭﻜﺯﻫﺎ )‪ ω (1;1‬ﻭ ﻨﺼﻑ ﻗﻁﺭﻫﺎ ‪2‬‬
‫‪ .2‬ﺘﻜﺎﻤل ﺍﻝﺩﺍﻝﺔ ‪ f‬ﻫﻭ ) ‪I = 2 (π + 2‬‬

‫• ‪ y = 4 − x 2 .1‬ﺘﻜﺎﻓﺊ‪ x 2 + y 2 = 4 :‬ﻭ ‪y ≥ 0‬‬


‫‪ C‬ﻫﻭ ﻨﺼﻑ ﺩﺍﺌﺭﺓ ﻤﺭﻜﺯﻫﺎ‪ O‬ﻭ ﻨﺼﻑ ﻗﻁﺭﻫﺎ ‪ 2‬ﻭﺍﻗﻌﺔ ﻓﻲ ﻨﺼﻑ ﺍﻝﻤﺴﺘﻭﻱ ﺍﻝﺫﻱ ﻤﻌﺎﺩﻝﺘﻪ ‪. y ≥ 0‬‬
‫‪.2‬ﺘﻜﺎﻤل ﺍﻝﺩﺍﻝﺔ ‪ f‬ﻫﻭ ‪. I = 2π‬‬
‫‪2‬‬ ‫‪2‬‬ ‫‪3‬‬
‫‪. ∫ f ( x) dx = 4 ،‬‬ ‫∫‬ ‫‪f ( x) dx = 7 ،‬‬ ‫‪∫ f ( x) dx = 8‬‬ ‫‪7‬‬
‫‪0‬‬ ‫‪−1‬‬ ‫‪−1‬‬
‫‪2‬‬ ‫‪2‬‬

‫(‬ ‫‪1 2‬‬


‫)‬‫‪‬‬ ‫‪3‬‬
‫‪∫1 2 x x − 1 dx =  2 x − 1  = 2‬‬
‫‪2‬‬
‫(‬ ‫)‬ ‫‪(1 10‬‬
‫‪1‬‬
‫‪ln 3‬‬ ‫‪4‬‬

‫(‬ ‫)‬
‫‪10‬‬
‫‪5x‬‬ ‫‪5‬‬ ‫‪dt‬‬
‫‪∫ln 2 e dx = 1 (4 ، ∫3 x 2 − 2 3 dx = 56 (3 ،‬‬ ‫∫‬ ‫‪=2‬‬ ‫‪10 − 1 (2‬‬
‫‪x‬‬

‫(‬ ‫)‬ ‫‪1‬‬ ‫‪t‬‬


‫‪1‬‬ ‫‪1‬‬
‫‪، ∫ (3 x − 6) ( x 2 − 4 x + 1 ) dx =  3 ( x 2 − 4 x + 1 )  11‬‬
‫‪3‬‬ ‫‪4‬‬

‫‪0‬‬ ‫‪8‬‬ ‫‪0‬‬


‫‪3‬‬
‫‪dt‬‬ ‫‪3‬‬
‫‪،‬‬ ‫∫‬ ‫‪=  2 1 + t  = 4 − 2 = 2‬‬
‫‪0‬‬ ‫‪1+ t‬‬ ‫‪0‬‬

‫‪2‬‬ ‫‪2‬‬
‫‪1‬‬ ‫‪‬‬ ‫‪1  17 ‬‬
‫‪3‬‬
‫‪dt =  ln ( t 4 + 1)  =  ln ‬‬
‫‪t‬‬
‫‪،∫ 4‬‬
‫‪1‬‬
‫‪t +1‬‬ ‫‪4‬‬ ‫‪1 4  2 ‬‬
‫‪1‬‬ ‫‪1‬‬

‫)‪∫0 e2 x + 1 dx =  2 ln ( e + 1) 0 = 2 ( e − 1‬‬


‫‪e2 x‬‬ ‫‪1‬‬ ‫‪2x‬‬ ‫‪‬‬ ‫‪1 2‬‬

‫‪ 2‬ـ ‪c‬اص ا
"‪+X‬‬
‫‪y‬‬ ‫‪32‬‬
‫‪1‬‬

‫‪0‬‬ ‫‪1‬‬ ‫‪2‬‬ ‫‪3‬‬ ‫‪4‬‬ ‫‪5‬‬ ‫‪x‬‬


‫‪3‬‬ ‫‪1‬‬ ‫‪3‬‬
‫‪1‬‬ ‫‪1‬‬
‫‪I = ∫ f ( x) dx = ∫ x dx + ∫ dx = + ln 3‬‬
‫‪0‬‬ ‫‪0‬‬ ‫‪1‬‬
‫‪x‬‬ ‫‪2‬‬
‫‪1‬‬ ‫‪1‬‬
‫‪2‬‬ ‫‪1‬‬ ‫‪2‬‬
‫‪1‬‬ ‫‪3‬‬
‫‪J = ∫ f ( x) dx = ∫ dx + ∫ x dx = − ln 2 −‬‬
‫‪2‬‬ ‫‪2‬‬
‫‪x‬‬ ‫‪1‬‬
‫‪8‬‬
‫‪ 3‬ـ ا
‪ %2 .‬ا
‪%0D"2‬‬
‫‪µ = 3 ، f ( x) = 2 x + 3‬‬ ‫‪36‬‬
‫‪µ = 0 ، f ( x) = x‬‬
‫‪1‬‬ ‫‪1‬‬ ‫‪1‬‬
‫‪1 ‬‬
‫‪ (1 37‬ﻤﻥ ﺃﺠل ﻜل ‪ x‬ﻤﻥ ‪  2 ;1‬ﻝﺩﻴﻨﺎ‪ ln x > ln 2 :‬ﻭ ﻤﻨﻪ ‪∫1 ln x dx ≥ − ln 2∫1 dx‬‬
‫‪ln 2‬‬ ‫‪1‬‬
‫‪∫ ln x dx ≥ −‬‬
‫‪1‬‬ ‫‪2‬‬
‫ﺃﻱ‬
‫‪2‬‬ ‫‪2‬‬ ‫‪2‬‬
‫‪2‬‬ ‫‪2‬‬ ‫‪2‬‬
‫‪1‬‬ ‫‪1‬‬ ‫‪1‬‬ ‫‪1‬‬ ‫‪1‬‬ ‫‪1‬‬
‫ﻭ ﻤﻨﻪ ‪ ∫1 1 + x3 dx ≤ 2 ∫1 dx‬ﺃﻱ ‪∫1 1 + x3 dx ≤ 2‬‬ ‫‪ (2‬ﻤﻥ ﺃﺠل ﻜل ‪ x‬ﻤﻥ ]‪ [1; 2‬ﻝﺩﻴﻨﺎ‪≤ :‬‬
‫‪1+ x‬‬ ‫‪3‬‬
‫‪2‬‬
‫‪π‬‬
‫‪π‬‬
‫‪π‬‬ ‫‪π ‬‬
‫‪−‬‬ ‫≤ ‪≤ ∫ sin( x 2 + 1)dx‬‬ ‫‪ (3‬ﻤﻥ ﺃﺠل ﻜل ‪ x‬ﻤﻥ ‪  ; π ‬ﻝﺩﻴﻨﺎ‪ −1 ≤ sin( x 2 + 1) ≤ 1 :‬ﻭﻤﻨﻪ‬
‫‪2‬‬ ‫‪π‬‬ ‫‪2‬‬ ‫‪2 ‬‬
‫‪2‬‬

‫‪ (1 44‬ﻤﻥ ﺃﺠل ﻜل ‪ x‬ﻤﻥ ]‪ [ 0;1‬ﻝﺩﻴﻨﺎ‪:‬‬


‫‪1‬‬
‫‪1‬‬ ‫‪1‬‬ ‫‪1‬‬ ‫‪1‬‬
‫∫≤‬ ‫ﻭﻤﻨﻪ ‪dx ≤ 1‬‬ ‫≤‬ ‫‪≤1‬‬
‫‪2 0 1+ x‬‬ ‫‪3‬‬
‫‪2 1 + x3‬‬
‫‪1‬‬
‫֏ ‪ f : x‬ﻤﺘﻨﺎﻗﺼﺔ ﺘﻤﺎﻤﺎ‪ ،‬ﺇﺫﻥ ﻤﻥ ﺃﺠل ﻜل ‪ x‬ﻤﻥ ]‪: [ 0;9‬‬ ‫‪ (2‬ﻋﻠﻰ ﺍﻝﻤﺠﺎل ]‪ [ 0;9‬ﺍﻝﺩﺍﻝﺔ‪:‬‬
‫‪1+ x‬‬
‫‪9‬‬
‫‪9‬‬ ‫‪1‬‬
‫∫≤‬ ‫)‪ f (9) ≤ f ( x ) ≤ f (0‬ﻭ ﻤﻨﻪ ‪ 1 ≤ f ( x ) ≤ 1‬ﺃﻱ‪dx ≤ 9 :‬‬
‫‪4 0 1+ x‬‬ ‫‪4‬‬

‫‪ (1 45‬ﻋﻠﻰ ﺍﻝﻤﺠﺎل ]‪ [1; 2‬ﺍﻝﺩﺍﻝﺔ‪ f : x ֏ x3 + 1 :‬ﻤﺘﺯﺍﻴﺩﺓ ﺘﻤﺎﻤﺎ‪،‬‬


‫‪2‬‬
‫ﺃﻱ‪2 ≤ ∫ x3 + 1 dx ≤ 3 :‬‬ ‫ﺇﺫﻥ ﻤﻥ ﺃﺠل ﻜل ‪ x‬ﻤﻥ ]‪ f (1) ≤ f ( x ) ≤ f (2) : [1; 2‬ﻭﻤﻨﻪ ‪2 ≤ f ( x ) ≤ 3‬‬
‫‪1‬‬
‫‪2‬‬
‫‪2e −4 ≤ ∫ e− x dx ≤ 2‬‬ ‫‪ (2‬ﻤﻥ ﺃﺠل ﻜل ‪ x‬ﻤﻥ ]‪ e−4 ≤ e x ≤ 1 : [ 0; 2‬ﻭﻤﻨﻪ‬
‫‪2‬‬ ‫‪2‬‬

‫‪0‬‬
‫‪4‬‬
‫‪ (3‬ﻤﻥ ﺃﺠل ﻜل ‪ x‬ﻤﻥ ]‪ ln 3 ≤ ln( x 2 − 1) ≤ ln15 : [ 2; 4‬ﻭﻤﻨﻪ ‪2 ln 3 ≤ ∫ ln( x 2 − 1) dx ≤ 2 ln 3 + 2 ln 5‬‬
‫‪2‬‬
‫‪1 2‬‬ ‫‪1‬‬ ‫‪46‬‬
‫‪−‬‬ ‫ﺘﺼﻭﻴﺏ‪ .1 :‬ﺒﺎﺴﺘﻌﻤﺎل ﺍﻝﺸﻜل ﺒﻴﻥ ﺃﻥ‪x + 2 x − 5 ≤ f ( x ) ≤ x + 2 :‬‬
‫‪10‬‬ ‫‪2‬‬
‫ﺒﻘﺭﺍﺀﺓ ﺒﻴﺎﻨﻴﺔ ﺍﻝﻤﻨﺤﻨﻲ ‪ C f‬ﻴﻘﻊ ﺃﺴﻔل ∆ ﻭ ﺃﻋﻠﻰ ‪ P‬ﻓﻲ ﺍﻝﻤﺠﺎل ]‪، [ 4;12‬‬
‫‪1‬‬ ‫‪1‬‬
‫ﻨﺴﺘﻨﺘﺞ ﺃﻥ ‪− x 2 + 2x − 5 ≤ f ( x ) ≤ x + 2 :‬‬
‫‪10‬‬ ‫‪2‬‬
‫‪12‬‬
‫‪.2‬ﻋﻠﻰ ﺍﻝﻤﺠﺎل ]‪ ، [ 4;12‬ﺍﻝﻤﻨﺤﻨﻲ ‪ C f‬ﺃﻋﻠﻰ ﻤﺤﻭﺭ ﺍﻝﻔﻭﺍﺼل‪ ،‬ﺇﺫﻥ‪A = ∫ f ( x ) dx :‬‬
‫‪4‬‬
‫‪12‬‬ ‫‪12‬‬ ‫‪12‬‬
‫‪ 1 2‬‬ ‫‪‬‬ ‫‪1‬‬ ‫‪‬‬ ‫‪1 2‬‬ ‫‪1‬‬
‫ﺒﻤﺎ ﺃﻥ ‪ − 10 x + 2x − 5 ≤ f ( x ) ≤ 2 x + 2‬ﻓﺈﻥ‪∫4  − 10 x + 2x − 5  dx ≤ ∫4 f ( x ) dx ≤ ∫4  2 x + 2  dx :‬‬

‫‪1 3‬‬ ‫‪1‬‬


‫‪G (x ) = −‬‬ ‫ﺩﺍﻝﺔ ﺃﺼﻠﻴﺔ ﻝﻠﺩﺍﻝﺔ ‪ g‬ﺍﻝﻤﻌﺭﻓﺔ ﺒـ ‪ g ( x ) = − x 2 + 2x − 5‬ﻫﻲ ﺍﻝﺩﺍﻝﺔ ‪ G‬ﺍﻝﻤﻌﺭﻓﺔ ﺒـ‪x + x 2 − 5x :‬‬
‫‪30‬‬ ‫‪10‬‬
‫‪1 2‬‬ ‫‪1‬‬
‫ﺩﺍﻝﺔ ﺃﺼﻠﻴﺔ ﻝﻠﺩﺍﻝﺔ ‪ h‬ﺍﻝﻤﻌﺭﻓﺔ ﺒـ ‪ h ( x ) = x + 2‬ﻫﻲ ﺍﻝﺩﺍﻝﺔ ‪ H‬ﺍﻝﻤﻌﺭﻓﺔ ﺒـ‪H ( x ) = x + 2x :‬‬
‫‪4‬‬ ‫‪2‬‬
‫‪12‬‬
‫‪ 1 2‬‬ ‫‪‬‬ ‫‪792  184  976‬‬
‫‪∫4  − 10 x + 2x − 5  dx = G (12 ) − G ( 4 ) = 30 −  − 30  = 30‬‬
‫‪12‬‬
‫‪1‬‬ ‫‪‬‬
‫‪∫  2 x + 2  dx = H (12 ) − H ( 4 ) = 60 − 12 = 48‬‬
‫‪4‬‬
‫‪976‬‬
‫ﺇﺫﻥ‪≤ A ≤ 48 :‬‬
‫‪30‬‬
‫‪3‬‬ ‫‪4‬‬
‫‪،‬‬ ‫‪∫ f ( x) dx = 2µ = 2 ln 2‬‬
‫‪1‬‬
‫‪(2 ،‬‬ ‫‪∫ f ( x) dx = 3µ = 6‬‬
‫‪1‬‬
‫‪(1 49‬‬
‫‪π‬‬ ‫‪π‬‬
‫‪4‬‬
‫‪1‬‬ ‫‪4‬‬
‫‪1 π‬‬ ‫‪1‬‬
‫∫‬ ‫= ‪f ( x) dx‬‬ ‫‪∫π‬‬ ‫‪f ( x) dx = × × µ = (3‬‬
‫‪0‬‬
‫‪2‬‬ ‫‪2 2‬‬ ‫‪2‬‬
‫‪−‬‬
‫‪74‬‬
‫‪1‬‬ ‫‪1‬‬ ‫‪1‬‬ ‫‪1 1‬‬
‫‪.‬‬ ‫ﻭﻤﻨﻪ ≤ ‪≤ I n‬‬ ‫‪ (1 51‬ﻤﻥ ﺃﺠل ﻜل ‪ x‬ﻤﻥ ]‪≤ ≤ : [ n ; n + 1‬‬
‫‪n +1‬‬ ‫‪n‬‬ ‫‪n +1 x n‬‬
‫‪ (2‬ﺤﺴﺏ ﻤﺒﺭﻫﻨﺔ ﺍﻝﺤﺼﺭ ) ‪ ( I n‬ﻤﺘﻘﺎﺭﺒﺔ ﻭ ﺘﺘﻘﺎﺭﺏ ﻨﺤﻭ‪.0‬‬
‫‪ 4‬ـ ا
"‪ -2‬إ
‪ 9‬دا
‪ %‬إ‪Q‬ر آ ‪% V‬‬
‫‪y‬‬ ‫‪59‬‬
‫‪A‬‬
‫‪-1‬‬ ‫‪0‬‬ ‫‪1‬‬ ‫‪2 x‬‬

‫‪-1‬‬
‫‪B‬‬
‫‪1‬‬
‫‪‬‬ ‫‪‬‬ ‫‪−x‬‬
‫‪1‬‬ ‫‪1‬‬
‫‪1‬‬ ‫‪1‬‬ ‫‪1‬‬ ‫‪2x‬‬
‫= ‪A1‬‬ ‫‪ A1 =  −‬ﺃﻱ‬ ‫ﻭﻤﻨﻪ ‪‬‬ ‫∫ = ‪A1‬‬ ‫‪ A1 = ∫ −‬ﻭﻤﻨﻪ ‪dx‬‬ ‫‪dx‬‬
‫‪ 2( x + 1)  0‬‬ ‫(‬ ‫)‬ ‫(‬ ‫)‬
‫‪2‬‬ ‫‪2‬‬ ‫‪2‬‬
‫‪4‬‬ ‫‪2 0 x2 +1‬‬ ‫‪0‬‬ ‫‪x2 + 1‬‬
‫‪ (2‬ﺃ‪ -‬ﻤﻌﺎﺩﻝﺔ ﺍﻝﻤﻤﺎﺱ ‪ T‬ﻝﻠﻤﻨﺤﻨﻲ ) ‪ ( C‬ﻋﻨﺩ ﺍﻝﻤﺒﺩﺃ ﻫﻲ‪y = − x :‬‬
‫ﺏ‪-‬ﺍﻝﻤﻨﺤﻨﻲ ) ‪ ( C‬ﺃﺴﻔل‪ T‬ﻓﻲ ﺍﻝﻤﺠﺎل [‪ ]−∞;0‬ﻭ ) ‪ ( C‬ﺃﻋﻠﻰ‪ T‬ﻓﻲ ﺍﻝﻤﺠﺎل [∞‪. ]0; +‬‬
‫‪1‬‬
‫ﺝ‪ -‬ﺍﻝﻤﺴﺎﺤﺔ ‪ A2‬ﻝﻠﻤﺜﻠﺙ ﺍﻝﻤﺤﺩﺩ ﺒـ ‪ ، T‬ﻤﺤﻭﺭ ﺍﻝﻔﻭﺍﺼل ﻭ ﺍﻝﻤﺴﺘﻘﻴﻡ ‪ D‬ﻫﻲ ‪. A2 = u.a‬‬
‫‪2‬‬
‫‪1 1 1‬‬
‫‪A = A2 − A1 = − = u.a (3‬‬
‫‪2 4 4‬‬
‫‪λ‬‬ ‫‪λ‬‬
‫‪1‬‬ ‫‪2x‬‬ ‫‪‬‬ ‫‪1‬‬ ‫‪‬‬
‫∫ ‪I (λ ) = −‬‬ ‫‪dx = ‬‬ ‫‪ (4‬ﺃ ‪ -‬‬
‫(‬ ‫)‬
‫‪ 2( x + 1)  0‬‬
‫‪2‬‬ ‫‪2‬‬
‫‪2 0 x2 + 1‬‬
λ
 1  1 1
I (λ ) =   = −
 2( x + 1)  0 2(λ + 1) 2
2 2

1
. lim I ( λ ) = − -‫ﺏ‬
λ →+∞ 2
‫ ﻤﺴﺎﺤﺔ‬A2 ‫ ( ﺤﻴﺙ‬− A2 ) ‫ ( ﻭ ﻤﺤﻭﺭ ﺍﻝﻔﻭﺍﺼل ﺘﻘﺘﺭﺏ ﻤﻥ‬C ) ‫ ﻤﺴﺎﺤﺔ ﺍﻝﻤﺴﺘﻭﻱ ﺍﻝﻤﺤﺩﺩ ﺒﺎﻝﻤﻨﺤﻨﻲ‬، +∞ ‫ ﺇﻝﻰ‬λ ‫ﻋﻨﺩﻤﺎ ﻴﺅﻭل‬
. OAB‫ﺍﻝﻤﺜﻠﺙ‬
% :N‫ب دوال أ‬I
:+X"
‫ب ا‬I
‫ ) ا‬e ‫ ـ‬5
π2
. I+J = .1 71
8
cos 2 x − sin 2 x = cos(2 x) -‫ﺃ‬.2
1
v( x) = sin 2 x ‫ ﻭ‬u '( x) = 1 ‫ ﻭﻤﻨﻪ‬v '( x) = cos 2 x ‫ ﻭ‬u ( x) = x :‫ﻨﻀﻊ‬-‫ﺏ‬
2
π π

x 
2
1 1 2
I−J = ‫ ﻭﻤﻨﻪ‬I − J =  sin 2 x  − ∫ sin 2 xdx
2 2 0 2 0
1π2 1 11 π2 
J=  −  ‫ ﻭ‬I=  +  .3
2 8 2 22 8 
:+X"
‫ب ا‬I
‫ت ا‬. /0 f! ‫ـ‬6
y
1
73
a = ∫ −( x − 1)e x dx
0
0 1 x 1
a = (2 − x)e x  = e − 2
0
1 1
v = ∫ π ( x − 1) e x  dx = π ∫ ( x 2 − 2 x + 1) e 2 x dx (2
-1 2

0 0
1
e2 − 5  1 3 5 
v =π u.v ، v = π  x 2 − x +  e 2 x 
4  2 2 4  0
‫ﺘﻤﺎﺭﻴﻥ ﻝﻠﺘﻌﻤﻕ‬
π
1 2 1
J =  ln (1 + 2sin x )  = ln 3 .1 86
2 0 2
π
2
sin 2 x + cos x
I+J =∫ dx
0
1 + 2 sin x
π
2
1
I = 1 − J = 1 − ln 3 ‫ ﻭﻤﻨﻪ‬I + J = ∫ cos xdx = 1 .2
2 0

.‫ ﻭﺍﻗﻌﺔ ﻓﻲ ﺍﻝﺭﺒﻊ ﺍﻷﻭل‬r ‫ ﻭ ﻨﺼﻑ ﻗﻁﺭﻫﺎ‬O ‫ ﻫﻲ ﺭﺒﻊ ﺩﺍﺌﺭﺓ ﻤﺭﻜﺯﻫﺎ‬M ‫( ﻤﺠﻤﻭﻋﺔ ﺍﻝﻨﻘﻁ‬1 87
r
π r
π

−r
r 2 − x 2 dx =
2
r2 ، ∫
0
r 2 − x 2 dx =
4
r 2 . (2
‫‪e nx‬‬ ‫‪e nx‬‬ ‫‪e nx‬‬ ‫‪1‬‬ ‫‪1‬‬ ‫‪1‬‬
‫≤‬ ‫≤‬ ‫ﻭ ‪ enx > 0‬ﻭ ﻤﻨﻪ‬ ‫≤‬ ‫‪ .1 97‬ﻤﻥ ﺍﺠل ﻜل ‪ x‬ﻤﻥ ]‪≤ : [ 0;1‬‬
‫‪1+ e 1 + ex‬‬ ‫‪2‬‬ ‫‪1+ e 1+ e‬‬ ‫‪x‬‬
‫‪2‬‬
‫‪ .2‬ﺒﺎﻝﻤﻜﺎﻤﻠﺔ ﻋﻠﻰ ﺍﻝﻤﺠﺎل ]‪ [ 0;1‬ﻨﺠﺩ ‪:‬‬
‫‪en − 1‬‬ ‫‪en − 1‬‬
‫ﻭﻤﻨﻪ ∞‪lim un = +‬‬ ‫≤ ‪≤ un‬‬
‫∞‪n →+‬‬ ‫‪(1 + e)n‬‬ ‫‪2n‬‬
‫‪−n‬‬
‫‪u‬‬ ‫‪1− e‬‬ ‫‪u 1 − e− n‬‬
‫‪،‬ﺤﺴﺏ ﻤﺒﺭﻫﻨﺔ ﺍﻝﺤﺼﺭ ﻴﻜﻭﻥ ‪. lim nn = 0‬‬ ‫≤ ‪≤ nn‬‬
‫‪n →+∞ e‬‬ ‫‪(1 + e)n e‬‬ ‫‪2n‬‬
‫ﻤﺴـﺎﺌل‬
‫‪112‬‬
‫ﺍﻝﺠﺯﺀ ‪: A‬‬
‫‪ f (0) = 1 .1‬ﻭ ‪ g (0) = 0‬ﺇﺫﻥ ‪ C g‬ﻫﻭ ﺍﻝﺫﻱ ﻴﻤﺭ ﺒﻤﺒﺩﺃ ﺍﻝﻤﻌﻠﻡ‪.‬‬
‫‪ .2‬ﺍﻝﺩﺍﻝﺘﺎﻥ ‪ f‬ﻭ‪ g‬ﺯﻭﺠﻴﺘﺎﻥ‪.‬‬
‫‪.3‬ﻨﻘﺘﺼﺭ ﺍﻝﺩﺭﺍﺴﺔ ﻋﻠﻰ ‪. ℝ +‬‬
‫‪2‬‬ ‫‪2‬‬
‫‪g '( x) = 2 x(1 − x 2 )e− x ، f '( x) = −2 xe − x‬‬
‫‪x‬‬ ‫‪0‬‬ ‫∞‪+‬‬

‫)‪f '( x‬‬ ‫‪-‬‬


‫‪1‬‬
‫‪f‬‬
‫‪0‬‬

‫‪x‬‬ ‫‪0‬‬ ‫‪1‬‬ ‫∞‪+‬‬

‫)‪g '( x‬‬ ‫‪+‬‬ ‫‪0‬‬ ‫‪-‬‬

‫‪g‬‬ ‫‪e −1‬‬


‫‪0‬‬ ‫‪0‬‬
‫‪) lim g ( x) = 0 ، lim f ( x) = 0‬ﺒﻭﻀﻊ ‪( X = − x‬‬
‫‪2‬‬
‫∞‪x →+‬‬ ‫∞‪x →+‬‬
‫‪2‬‬
‫‪f ( x) − g ( x) = (1 − x 2 )e− x .4‬‬
‫‪ C f‬ﺃﻋﻠﻰ ‪ C g‬ﺇﺫﺍ ﻜﺎﻥ ‪ −1 < x < 1‬ﻭ ‪ C f‬ﺃﺴﻔل ‪ C g‬ﺇﺫﺍ ﻜﺎﻥ ‪ x < −1‬ﺃﻭ ‪ C f ، x > 1‬ﻴﻘﻁﻊ ‪ C g‬ﻋﻨﺩ ﺍﻝﻨﻘﻁﺘﻴﻥ‬
‫ﺍﻝﻠﺘﻴﻨﻔﺎﺼﻠﺘﺎﻫﻤﺎ‪ -1‬ﻭ ‪.1‬‬
‫‪x‬‬
‫‪2‬‬
‫ﺍﻝﺠﺯﺀ‪G ( x) = ∫ t 2e − t dt : B‬‬
‫‪0‬‬

‫‪ G .1‬ﻫﻲ ﺍﻝﺩﺍﻝﺔ ﺍﻷﺼﻠﻴﺔ ﻝﻠﺩﺍﻝﺔ ‪ g‬ﺍﻝﺘﻲ ﺘﻨﻌﺩﻡ ﻋﻨﺩ‪.0‬‬


‫‪.2‬ﺍﻝﺩﺍﻝﺔ ‪ g‬ﻤﻭﺠﺒﺔ ﺘﻤﺎﻤﺎ ﻋﻠﻰ [∞‪. ]0; +‬ﻤﻥ ﺃﺠل ‪ G ( x) ، x > 0‬ﻫﻭ ﻤﺴﺎﺤﺔ ﺤﻴﺯ ﻤﺠﻤﻭﻋﺔ ﺍﻝﻨﻘﻁ )‪ M (a; b‬ﺤﻴﺙ ‪0 ≤ a ≤ x‬‬
‫ﻭ )‪. 0 ≤ b ≤ g ( x‬‬
‫‪ .3‬ﺍﻝﺩﺍﻝﺔ ‪ G‬ﻤﺘﺯﺍﻴﺩﺓ ﻋﻠﻰ ‪. ℝ‬‬
‫‪1‬‬
‫֏ ‪ x‬ﻫﻲ ‪:‬‬ ‫‪.4‬ﺍﻝﺩﺍﻝﺔ ‪ F‬ﻫﻲ ﺩﺍﻝﺔ ﺃﺼﻠﻴﺔ ﻝﻠﺩﺍﻝﺔ ‪ f‬ﻋﻠﻰ ‪ ، ℝ‬ﺇﺫﻥ ﻤﺸﺘﻘﺔ ﺍﻝﺩﺍﻝﺔ ‪F ( x) − xe − x ‬‬
‫‪2‬‬

‫‪2‬‬ ‫‪‬‬
‫‪1‬‬
‫‪ x ֏  f ( x) − e − x + 2 x 2e − x ‬ﺃﻱ ﺍﻝﺩﺍﻝﺔ ‪. g‬‬
‫‪2‬‬ ‫‪2‬‬

‫‪2‬‬ ‫‪‬‬ ‫‪‬‬


‫‪1‬‬ ‫‪1‬‬
‫‪ G‬ﻭ ‪ F‬ﻝﻬﻤﺎ ﻨﻔﺱ ﺍﻝﻤﺸﺘﻘﺔ ﻋﻠﻰ ‪ G (0) = 0 . ℝ‬ﻭ ‪ . [ F (0) − 0] = 0‬ﺇﺫﻥ ‪G ( x) =  F ( x) − xe − x ‬‬
‫‪2‬‬

‫‪2‬‬ ‫‪‬‬ ‫‪2‬‬


‫‪lim − xe− x = lim‬‬
‫∞‪x →+‬‬
‫‪2‬‬

‫‪x →+∞ x‬‬


‫‪1‬‬
‫‪ .5‬ﺃ ‪− x 2 e − x = 0 -‬‬
‫‪2‬‬

‫(‬ ‫)‬
‫‪ℓ‬‬
‫‪، lim F ( x) = ℓ‬ﺇﺫﻥ = )‪. lim G ( x‬‬
‫∞‪x →+‬‬ ‫‪2‬‬ ‫∞‪x →+‬‬
‫‪1‬‬
‫ﺏ‪ N = ∫ (e − t − t 2 e− t )dt -‬ﻭ ) ‪ f (t ) > g (t‬ﻋﻠﻰ ﺍﻝﻤﺠﺎل ]‪ N. [ 0;1‬ﻫﻭ ﻤﺴﺎﺤﺔ ﺍﻝﺤﻴﺯ ﺍﻝﻤﺤﺩﺩ ﺒﺎﻝﻤﻨﺤﻨﻴﻴﻥ ‪ C f‬ﻭ ‪C g‬‬
‫‪2‬‬ ‫‪2‬‬

‫‪0‬‬

‫ﻭ ﻤﺤﻭﺭ ﺍﻝﺘﺭﺍﺘﻴﺏ‪.‬‬
‫ﺠـ‪-‬ﻨﻀﻊ ﻤﻥ ﺃﺠل ﻜل ‪: x ≥ 1‬‬
‫‪ : D1‬ﻤﺴﺎﺤﺔ ﺍﻝﺤﻴﺯ ﺍﻝﻤﺤﺩﺩ ﺒﺎﻝﻤﻨﺤﻨﻲ ‪ ، C f‬ﻤﺤﻭﺭ ﺍﻝﻔﻭﺍﺼل ﻭ ﺍﻝﻤﺴﺘﻘﻴﻤﻴﻥ ﺍﻝﻠﺫﻴﻥ ﻤﻌﺎﺩﻝﺘﺎﻫﻤﺎ ‪ x = 0‬ﻭ ‪. x = 1‬‬
‫‪ : D2‬ﻤﺴﺎﺤﺔ ﺍﻝﺤﻴﺯ ﺍﻝﻤﺤﺩﺩ ﺒﺎﻝﻤﻨﺤﻨﻲ ‪ ، C g‬ﻤﺤﻭﺭ ﺍﻝﻔﻭﺍﺼل ﻭ ﺍﻝﻤﺴﺘﻘﻴﻤﻴﻥ ﺍﻝﻠﺫﻴﻥ ﻤﻌﺎﺩﻝﺘﺎﻫﻤﺎ ‪ x = 0‬ﻭ ‪. x = 1‬‬
‫) ‪ : D3 ( x‬ﻤﺴﺎﺤﺔ ﺤﻴﺯ ﻤﺠﻤﻭﻋﺔ ﺍﻝﻨﻘﻁ )‪ M (a; b‬ﺍﻝﺘﻲ ﺘﺤﻘﻕ ‪ 1 ≤ a ≤ x‬ﻭ )‪. 0 ≤ b ≤ f ( x‬‬
‫) ‪ : D4 ( x‬ﻤﺴﺎﺤﺔ ﺤﻴﺯ ﻤﺠﻤﻭﻋﺔ ﺍﻝﻨﻘﻁ )‪ M (a; b‬ﺍﻝﺘﻲ ﺘﺤﻘﻕ ‪ 1 ≤ a ≤ x‬ﻭ )‪. 0 ≤ b ≤ g ( x‬‬
‫ﺇﺫﺍ ﻜﺎﻨﺕ ‪ F‬ﻭ ‪ G‬ﺩﺍﻝﺘﺎﻥ ﺃﺼﻠﻴﺘﺎﻥ ﻝﻠﺩﺍﻝﺘﻴﻥ ‪ f‬ﻭ ‪ g‬ﻋﻠﻰ ‪ ℝ +‬ﻭ ‪F (0) = G (0) = 0‬‬

‫‪x‬‬ ‫‪1‬‬ ‫‪x‬‬ ‫‪x‬‬


‫‪−t 2‬‬ ‫‪2 −t2‬‬ ‫‪−t 2‬‬ ‫‪2 −t 2‬‬ ‫‪−t 2‬‬ ‫‪2 −t 2‬‬ ‫‪−t‬‬ ‫‪2‬‬
‫‪2 −t‬‬ ‫‪2‬‬

‫‪∫ (e‬‬
‫‪0‬‬
‫‪−t e‬‬ ‫‪)dt = ∫ (e‬‬
‫‪0‬‬
‫‪−t e‬‬ ‫‪)dt + ∫ (e‬‬
‫‪1‬‬
‫‪−t e‬‬ ‫ﻭ ‪)dt‬‬ ‫)‪∫ (e − t e )dt = F ( x) − G( x‬‬
‫‪0‬‬
‫‪x‬‬
‫ﺃﻱ ) )‪− t 2 e− t )dt = D2 − D1 − ( D4 ( x) − D3 ( x‬‬
‫‪2‬‬ ‫‪2‬‬
‫‪−t‬‬
‫‪∫ (e‬‬
‫‪0‬‬
‫‪x‬‬
‫‪−t 2‬‬
‫) )‪− t 2 e− t )dt = N − ( D4 ( x) − D3 ( x‬‬
‫‪2‬‬

‫‪∫ (e‬‬
‫‪0‬‬

‫ﺒﻤﺎ ﺃﻥ ﻤﻥ ﺃﺠل ﻜل ‪ D4 ( x) − D3 ( x) ≥ 0 : x ≥ 1‬ﻴﻜﻭﻥ ‪N ≥ F ( x) − G ( x) :‬‬


‫‪ℓ ℓ‬‬ ‫‪ℓ‬‬
‫‪ lim F ( x) = ℓ‬ﻭ = )‪ lim G ( x‬ﻭﻤﻨﻪ = ‪lim F ( x) − G ( x) = ℓ −‬‬
‫∞‪x →+‬‬ ‫‪2 2‬‬ ‫∞‪x →+‬‬ ‫∞‪2 x→+‬‬
‫‪ℓ‬‬
‫ﺇﺫﻥ )‪ N ≥ lim F ( x) − G ( x‬ﺃﻱ ≥ ‪. N‬‬
‫‪2‬‬ ‫∞‪x →+‬‬
‫‪x‬‬ ‫‪1‬‬ ‫‪x‬‬

‫ﻤﻼﺤﻅﺔ‪∫ [ f (t ) − g (t )]dt = ∫ [ f (t ) − g (t )]dt + + ∫ [ f (t ) − g (t )]dt :‬‬


‫‪0‬‬ ‫‪0‬‬ ‫‪1‬‬
‫‪x‬‬ ‫‪1‬‬ ‫‪x‬‬

‫ﻭﻤﻨﻪ ‪∫ [ f (t ) − g (t )]dt − ∫ [ f (t ) − g (t )]dt = ∫ [ f (t ) − g (t )]dt‬‬


‫‪0‬‬ ‫‪0‬‬ ‫‪1‬‬
‫‪x‬‬ ‫‪1‬‬ ‫‪x‬‬

‫ﻓﻴﻜﻭﻥ ‪ ∫ [ f (t ) − g (t )]dt < 0‬ﻭﻤﻨﻪ ‪∫ [ f (t ) − g (t )]dt < ∫ [ f (t ) − g (t )]dt‬‬ ‫ﻤﻥ ﺃﺠل ﻜل ‪f ( x) < g ( x) : x ≥ 1‬‬
‫‪0‬‬ ‫‪0‬‬ ‫‪1‬‬
‫‪1‬‬ ‫‪1‬‬ ‫‪1‬‬
‫‪ℓ‬‬ ‫‪ℓ‬‬
‫‪ F ( x) − G ( x) < ∫ [ f (t ) − g (t ) ] dt‬ﻭ ﺒﺎﻝﺘﺎﻝﻲ ‪ < ∫ [ f (t ) − g (t )] dt‬ﻭﻤﻨﻪ ‪< ∫ (1 − t 2 )e−t  dt‬‬ ‫ﻭﻤﻨﻪ‬
‫‪2‬‬

‫‪2 0‬‬ ‫‪‬‬ ‫‪2 0‬‬ ‫‪0‬‬


‫ﺍﻷﻧﺸﻄﺔ‬
‫ا
ط اول ‪:‬‬
‫‪
T:T " B :lam3‬أو‪ U4‬و ? ' أو ‪ ?
T:T‬ر و و‪" U4‬‬

‫‪  )D/ ) -‬ا‪99‬ت ‪p3‬ول إ ‪R >?3 j‬ن ا ز‪6‬ت ا  ‪.


o‬‬
‫‪ -‬ا ‪'6‬ف (‪. '
'!3 j‬‬
‫ا
ط ا
 ‪:‬‬
‫‪  )D/ ) -‬ا‪99‬ت ‪p3‬ول ا ‪R >?3 j‬ن ا ز‪6‬ت ا  ‪.
o‬‬
‫ا
ط ا

 ‪:‬‬
‫‪  )D/ ) -‬ا‪99‬ت ‪p3‬ول إ ‪R >?3 j‬ن ا ز‪6‬ت ا  ‪.
o‬‬
‫‪ >?3 -‬ا ;' ا ‪-6‬ا‪ D‬ا ‪  )D/ )a '/‬ا‪99‬ت‪.‬‬
‫ا
ط ا
ا ‪:‬‬
‫‪ -‬ا‪67‬ل ‪a CD‬آة  أ‪R )4‬س ‪:3‬ؤم ‪-
(/(7‬هة و ذج ا ‪.‬‬
‫ا
ط ا
‪: g+h‬‬
‫‪  )D/ ) -‬ا‪99‬ت ‪p3‬ول إ ‪R >?3 j‬ن ا ز‪6‬ت ا  ‪.
o‬‬
‫‪ -‬ا ‪'6‬ف (‪ j‬أن دا
‪! j(
'6‬ل ه آ‪
V‬ال‪.‬‬
‫‪/ -‬ب ‪R‬ن ال ;' ‪-‬ا‪ )1 D‬دا
‪ f‬آ@‪
V‬ال و ‪/‬ب ا^) ا '‪ ،‬ا ‪ 1‬و ا‪'a9‬اف‬
‫ا ‪6‬ري ‪.‬‬
‫‪-‬‬

‫ﺍﻷﻋﻤﺎﻝ ﺍﻟﻤﻮﺟﻬﺔ ) ‪( 1‬‬

‫‪a (I‬آة '‪13v :


64‬ع ا ‪<F‬ات ا ‪ j(
1‬ا !ول اآ‪/‬ل ‪ j( )ma3‬إ‪a 
4‬آة ( ‪-‬ط ر‪. 2 #R‬‬
‫‪ (II‬إ‪-‬ء ‪: w(V‬‬
‫‪
T:T ymF3 -‬أة !ورة   ا^اد ا ‪-6‬ا‪ z ، y ، x
D‬ا  ‪ )V3‬أ‪Z‬ال أ‪:‬ع ا ‪w(V‬‬
‫) ‪( A( )@
R 100 : :V‬‬
‫‪ -‬ا ‪6‬د ا 'ا‪'2  Sa( ymF A‬ط و‪4‬د ا ‪ : 1 ) w(V‬ا ‪4 w(V‬د ‪ : 0 ،‬ا ‪4 '_ w(V‬د (‬
‫و ذ ‪ SI (6  z‬و ‪. ET‬‬
‫‪  -‬ا ‪6‬د ا ‪ Y/a {F‬ال ‪  ) w(V Sa3‬ا ‪  23
(F‬ه‪P‬ا ا ‪6‬د ‪Y@ :V D‬‬
‫‪= SOMME( D1:D23)/23‬‬
‫‪ j( .;|  -‬ا ‪X‬ر ‪ }: F9‬ا ;'ات ا ‪
(fa‬و @ ‪ z‬ا‪'F7‬اج ‪
R‬ا‪'7‬ار ا ا‪'3‬ات ‪.‬‬

‫ﺍﻷﻋﻤﺎﻝ ﺍﻟﻤﻮﺟﻬﺔ ) ‪: ( 2‬‬

‫‪:63‬ت ‪  - :‬ا ‪<F‬ة ا ‪


V‬إد*ل ‪ #R‬ا ‪ t X‬وه ‪  ... ، 3 ،2 ، 1 ، 0‬ا ‪ A2:AG2 Xa‬ل ا ‪A2:A32 Xa‬‬
‫‪  -‬ا ‪<F‬ة ا 'ا‪
6‬ا ‪
(6‬ه )‪ = SI(ALEA()<$A$1;0;B2‬و ذ ‪  z‬ا ‪ j( #6 #T C2
(F‬ا ‪6‬د ‪C‬‬
‫‪  -‬ا ‪ Y@ D2
(F‬ا ‪ U^ = SI(C2=0;0;SI(ALEA()<$A$1;0;B2))
(6‬إذا ‪ k3‬ا ‪P‬رة  ‪
oa‬‬
‫ |'ورة ه 
 ا (‪ }a‬ا ا 
‪a #6 #T .‬ى ا ‪ j( D2
(F‬آ) ا ‪6‬د و ‪a‬ت ا ‪6‬د ‪
( D‬‬
‫ا^ة ا ‪ .
1‬و ا‪ 1/ )f‬ذآ'  ا @ب ‪:j( )ma 7‬‬
‫ﺍﻟﺘﻤﺎﺭﻳﻦ‬
2 4
1 1 3
p = C   ×   = (1 3
2
4
2 2 8
‫وي " ال أن‬/ " ‫ 'دي‬#R‫  د 'ات ? ر ر‬3 '1‫ أآ‬4‫ زو‬#R‫ أن " ال أن @ن د 'ات ? ر ر‬#(6 (2
" 4‫ زو‬#R‫  د 'ات ? ر ر‬3 '1‫ 'دي أآ‬#R‫@ن د 'ات ? ر ر‬
1− p 5
p'= = U  ‫ و‬p + p '+ p ' = 1 ‫   إذن‬
2 16
1
p'= ‫ و‬p = 0 (3
2
1
k = (1 15
2
π 3
p (X ≥ ) = (2
3 4
π
π
∫ xf (x )dx = 2
0
(3

" ‫ ا @'و‬Y  


4:V ‫
" ا‬T‫د‬a ‫ ا‬E ، " #a( ‫ ا‬Y  
4:V ‫
" ا‬T‫د‬a ‫ ا‬S A| (1 31
p (E ) = 0, 02 ، p (S ) = 0, 03   "
a f '_
4:V ‫
" ا‬T‫د‬a ‫ ا‬D
P (D ) = p (S ∪ E ) = p (S ) + p (E ) − p (S ∩ E )
= p (S ) + p (E ) − p (S ) × p (E ) = 0, 0494
"
a f '_
4:T " ‫
" و‬a f
4:T " 4'F ‫
ذات‬D‫ا‬-
'!3 ‫ر‬1‫ق @ ا‬/ ‫
 ا‬4:T 800 ‫(أ( 'ض‬2
0,0494 ‫ و‬800 <7‫ و‬a ‫ ا‬D T ‫ن‬R A1 X ‫إذن‬
C  800 ‫ و‬0  ‫ر‬ma k 61Z ‫) آ) د‬4‫إذن  أ‬
p (X = k ) = C 800
k
(0, 0494) k × (0,9506)800 − k
E (X ) = 800 × 0, 0494 = 39, 52 (‫ب‬
‫'اة‬- ‫ ا‬25 ‫ت‬4:V ‫
 ا‬a m ‫ت _' ا‬4:V ‫د ا‬6 ‫وي‬/ ‫ ا‬D‫ا‬-6 ‫ ا ;' ا‬Y '16 (‫( أ‬3
0,0494 ‫ و‬25 <7‫  و‬D T ‫ن‬R A1 Y ‫إذن‬
p (Y = 2) = C 25 (0, 0494) 2 × (0,9506) 23
2

‫'اة‬-
4:T n 
a m ‫ت _' ا‬4:V ‫د ا‬6 ‫وي‬/ ‫ ا‬D‫ا‬-6 ‫ ا ;' ا‬Z '16 (‫ب‬
1
p ( Z ≥ 1) ≤ w n  wa1 . 0,0494 ‫ و‬n <7‫  و‬D T ‫ن‬R A1 Z ‫إذن‬
2
1 1 1
p ( Z = 0) ≥ ‫ أي‬1 − p ( Z = 0) ≤ U  ‫ و‬1 − p ( Z ≺ 0) ≤ C 
2 2 2
− ln 2 1
n≤ ≃ 13, 68 U  ‫ و‬0,9506 n ≥    ‫و‬
ln(0, 9506) 2
. ‫ اآ‬9:$ %kl 13 ‫"ي‬- ‫ ا
" أن‬9:!\ ، j :$ ‫و‬
0,0007 j0 D ‫ و‬D‫ن أ‬P /"- \ O0$ ‫ دون أي‬%k
‫ ا‬% &kN ‫ة‬2
J\2
‫ ا‬Z‫  ا
!ا‬S"2
‫ ا‬W /"! (4
1000
p (700 ≤W ≤ 1000) = ∫
700
f (x )dx  

= e −0,49 − e −0,7 ≃ 0,116


( ' ‫ا ا‬P
63 '_ ‫ د‬/ ‫ ج‬/ ‫) ا 'وع ب‬
‫ﺍﻷﻧﺸﻄﺔ‬
‫ا
ط اول ‪:‬‬
‫‪  )D/ ) -‬ا‪99‬ت ‪ >?3‬ا ;'ات ا ‪-6‬ا‪R ،
D‬ن ا‪9‬ل ‪ ،‬ا^) ا '‪ ، 3‬ا ‪ 1‬و ا‪'a9‬اف‬
‫ا ‪6‬ري ‪.‬‬
‫‪<6 #o 3 -‬ت  أ‪ )4‬ه ‪67‬ل ا ‪1‬أ ا^‪. 6( 77‬‬
‫ا
ط ا
 ‪:‬‬
‫‪ -‬إ!د ‪R‬ن ال ;' ‪-‬ا‪D‬‬
‫‪  )D/ ) -‬ا‪99‬ت ‪ >?3‬ا ;'ات ا ‪-6‬ا‪R ،
D‬ن ا‪9‬ل ‪ ،‬ا^) ا '‪ ، 3‬ا ‪ 1‬و ا‪'a9‬اف‬
‫ا ‪6‬ري ‪.‬‬
‫‪<6 #o 3 -‬ت  أ‪ )4‬ه ‪67‬ل ا ‪1‬أ ا^‪. 6( 77‬‬
‫ا
ط ا

 ‪:‬‬
‫‪<6 #o 3 -‬ت  أ‪ )4‬ه ‪67‬ل ا ‪1‬أ ا^‪. 6( 77‬‬
‫‪
4P -‬و‪6‬ت ‪9‬د (‪ j‬ا !رب ا '‪ Ya/(
64‬أو ا ء ‪.‬‬
‫ا
ط ا
ا ‪:‬‬
‫‪<6 #o 3 - -‬ت  أ‪ )4‬ه ‪67‬ل ا ‪1‬أ ا^‪. 6( 77‬‬
‫ا
ط ا
‪: g+h‬‬
‫‪  )D/ ) -‬ا‪99‬ت ‪ >?3‬ا ;'ات ا ‪-6‬ا‪R ،
D‬ن ا‪9‬ل ‪ ،‬ا^) ا '‪ ، 3‬ا ‪ 1‬و ا‪'a9‬اف‬
‫ا ‪6‬ري ‪.‬‬
‫‪<6 #o 3 -‬ت  أ‪ )4‬ه ‪67‬ل ا ‪1‬أ ا^‪. 6( 77‬‬
‫ا
ط ا
‪I‬دس ‪:‬‬
‫) ‪  )D/‬ا‪99‬ت ‪ >?3‬ا ;'ات ا ‪-6‬ا‪R ،
D‬ن ا‪9‬ل ‪ ،‬ا^) ا '‪ ، 3‬ا ‪ 1‬و ا‪'a9‬اف‬ ‫‪-‬‬
‫ا ‪6‬ري ‪.‬‬
‫‪<6 #o 3‬ت  أ‪ )4‬ه ‪67‬ل ا ‪1‬أ ا^‪. 6( 77‬‬ ‫‪-‬‬
‫ا ‪'6‬ف (‪ j‬ا‪:7‬ل أو ار‪13‬ط د‪. T‬‬ ‫‪-‬‬
‫‪ >?3‬د‪7‬ر ا‪99‬ت ا @(
‪  )D/ )a‬ا‪99‬ت ‪  Ya/  S(63‬أآ‪  'V‬آ{ ‪.‬‬ ‫‪-‬‬

‫ﺍﻷﻋﻤﺎﻝ ﺍﻟﻤﻮﺟﻬﺔ ) ‪( 1‬‬


‫‪3 (I‬ر€ ا ‪:‬د‬
‫‪34‬‬
‫‪A‬‬
‫‪Pn (A ) = 1 − P (A ) = 1 −‬‬ ‫‪≃ 1 − 0, 205 ≃ 0, 795‬‬
‫‪365‬‬
‫‪365‬‬
‫‪365‬‬
‫‪& -‬ا
 ‪ + 80 %‬ا
‪V‬ص
د ‪ 9:$ F-<2:‬ا‪ gV 2
P‬ر‪ p-‬ا
‪k 2‬د‬
‫‪n‬‬ ‫‪15‬‬ ‫‪20‬‬ ‫‪23‬‬ ‫‪30‬‬ ‫‪50‬‬ ‫‪57‬‬
‫‪Pn‬‬ ‫‪25 %‬‬ ‫‪41 %‬‬ ‫‪50 %‬‬ ‫‪70 %‬‬ ‫‪97 %‬‬ ‫‪99 %‬‬
‫‪ " A @ : S1<3‬آ) ‪a3
6<R‬ي ‪ j( p p
1‬ا^‪" )R‬‬
‫* ا !ول ا   ‪ 1‬ا ‪9a‬ت ا ‪6
(F‬د ‪1‬ت ا (‪  p p‬آ) ‪
6<R‬‬
‫‪1‬‬ ‫‪2‬‬ ‫‪2‬‬ ‫‪1‬‬ ‫‪1‬‬ ‫‪0‬‬ ‫‪0‬‬ ‫‪3‬‬ ‫‪0‬‬ ‫‪0‬‬ ‫ا
‪1 %!0.‬‬
‫‪1‬‬ ‫‪1‬‬ ‫‪0‬‬ ‫‪2‬‬ ‫‪0‬‬ ‫‪2‬‬ ‫‪1‬‬ ‫‪0‬‬ ‫‪3‬‬ ‫‪0‬‬ ‫ا
‪2 %!0.‬‬
‫‪1‬‬ ‫‪0‬‬ ‫‪1‬‬ ‫‪0‬‬ ‫‪2‬‬ ‫‪1‬‬ ‫‪2‬‬ ‫‪0‬‬ ‫‪0‬‬ ‫‪3‬‬ ‫ا
‪3 %!0.‬‬
‫‪1‬‬ ‫‪U1‬‬ ‫‪2‬‬
‫‪6‬‬ ‫‪A‬د ا ‪9a‬ت ا ‪
D:‬ـ‬
‫‪2‬‬ ‫‪3‬‬ ‫= ) ‪P (A‬‬ ‫‪= 0, 6‬‬ ‫= و ‪j +‬‬
‫)‪P(A‬‬
‫‪10‬‬ ‫د ا ‪9a‬ت ا @(

‫‪B‬‬
‫‪1‬‬ ‫‪ (2‬ا‪67‬ل ا ‪'!-‬ة‬
‫‪2‬‬ ‫‪U2‬‬ ‫‪4‬‬ ‫‪1 2 1 4 5‬‬
‫‪9‬‬ ‫= × ‪P (B ) = × +‬‬
‫‪2 3 2 9 9‬‬
1 2 1
P (U 1 ∩ B ) = × =
2 3 3
P (U 1 ∩ B ) 3
P (U 1 / B ) = =
P (B ) 5
M : ‫>  'ض‬-@ ‫ر ا‬1*‫ا‬: S1<3
0,99 P
" i' yF- ‫ " ا‬M ، " Y4 ‫ر‬1*9‫ " ا‬T : A|
B P (T ) = 0,99 × p + 0, 01× (1 − p )
0, 01 = 0,98 p + 0, 01
M 1− P
P (M ∩T )
P (M /T ) = PT (M ) =
P (T )
0,99 p 99 p
= =
0,98 p + 0, 01 98 p + 1

( 2 ) ‫ﺍﻷﻋﻤﺎﻝ ﺍﻟﻤﻮﺟﻬﺔ‬
P0
‫ء‬9‫ا‬
q0 AA
p0
q0 :
T‫ ا را‬#(
 
( ‫ ( ا‬.<F ‫!'ة ) ا‬- ‫ ا‬Y/
AA
Aa Aa 1  
p0
1
AA 1
q0 1 2
2
AA 2  
1
Aa 2
4 1 1 1  q 
P1 = p + p 0q 0 + p 0q 0 + q 02 =  p 0 + 0 
2
0
2 2 4  2
2
 q  ! )V 
r1 =  r0 + 0 
 2
2 2
 q0   q0 
q1 = 1 − p1 − r1 =  p 0 +  −  r0 +  ‫ أن‬C / ‫و‬
 2  2
q 0 = 1 − p 0 − r0 C  α = p 0 − r0 ‫( و  أن‬2
 1−α  α   1+ α 
2 2 2
 1
r1 =   ! )V  p1 =  p 0 + − p 0 +  =   U  ‫و‬
 2   2 2  2 
p1 − r1 = α = p 0 − r0 :
o:
1−α 2
q1 = 1 − p1 − r1 =   ‫و‬
2
‫ا‬P@‫ و ه‬..... p1 + q1 + r1 = 1 ، p 0 + q 0 + r0 = 1 : ‫} أن‬9
α = p1 − r1 A α
9 r2 ; p 2 ; q 2  '16 ‫ @ ا‬V ‫
(!) ا‬1/ 1 α = p1 − r1 = p 0 − r0 ‫و دام‬
:
f:F ‫ا‬
 1+α  1−α  1−α 
2 2 2

pn =   ، qn = ، rn =   ‫) أي‬4 ‫) أس‬4‫


 أ‬T CD ‫ ا‬j13
 2  2  2 
: USB l3
3 R " USB ‫ ح‬i'3 ‫ " وة ا 'ز‬R ، " l f '_ USB ‫ " ح‬D A|
100 24 4 1
96 p (D ) = 1 − p (D ) = U  ‫ و‬p (D ) = =  
25 100 25
100 D
97 97 3
p D (R ) = 1 − p D (R ) = U  ‫ و‬p D (R ) = |‫و أ‬
100 R 100 100
1 95 19
95 R p D (R )1 − p D (R ) = U  ‫ و‬p D (R ) = = |‫و أ‬
4 20 100 20
100
100 D ‫!'ة‬- ‫ ا‬j( ( ‫ آ‬CD ‫ ا‬P‫ ه‬yF( ‫و‬
1
5 p1 = p (D ∩ R ) = p (D ) × p D (R ) = = 0, 002 (‫( أ‬1
R 500
100
18
p 2 = p (R ∩ D ) = p (D ) × p D (R ) = = 0, 0288
625
77
p 3 = p (D ∩ R ) + p (R ∩ D ) = p 2 + p1 = ≃ 0, 031
2500
2333
p 4 = p (R ) = p (D ) × p D (R ) + p (D ) × p D (R ) = ≃ 0, 933 (‫ب‬
2500
 
‫و‬a n   ‫ 'ة‬k ، R
T‫د‬a ‫ ا‬Sa3 )4‫ن '  و  أ‬R Y/ (2
n −k
p (R ) = C nk [ p (R )] ×  p (R ) 
k

4
p 7 = 1 − p 5 − p 6 ≃ 0, 044....;.... p 6 ≃ 0, 708 C  )V  ‫ و‬p 5 = C 51 p (R ) ×  p (R )  ≃ 0, 249 U  ‫و‬

‫ﺍﻟﺘﻤﺎﺭﻳﻦ‬
6
P (F ) P (F ) PF (B ) PF (B ) PF (B ) PF (B )
1 1 4 3 2 5
2 2 7 7 7 7

p (X ≤ 10) = 1 − e −0,08(10) = 1 − e −0,8 ≃ 0,55 (‫( أ‬1 9


p (X ≥ 30) = 1 − p (X ≺ 30) = 1 − (1 − e −0,08(30) ) = e −2,4 ≃ 0, 09 (‫ب‬
1
E (X ) = = 12,5 (2
λ
 
Y3' ‫ و‬j !3‫ ن  ا‬T‫ و ا‬i !3‫ ن  ا‬T‫ *<ات ا‬4 '7 ‫ت‬1@ 6 ‫ ا‬j( ;1  A j ‫ ا‬O  ‫ل‬: - 25
C 42 = 6 ‫رات ا <(
و ا  ده‬/ ‫ ا‬C 3
6‫<ات ا^ر‬F ‫ا‬

6 ‫ و‬i !3‫ *<ة  ا‬5 - 4 = 1 ، ‫ *<ات‬4 '7 ‫ت‬1@ 6 ‫ ا‬j( ;1  C( 5 ; 6 ) j ‫ ا‬B( 4 ; 3 )  ‫ل‬: -

C 41 = 4 ‫ ه‬C j ‫ ا‬B  ‫رات‬/ ‫ د ا‬U( ‫ و‬j !3‫ – *<ة  ا‬3 = 3
(‫ آ‬CD ‫@ن ا‬3 U( ‫و‬
‫ال‬qI
‫ا‬ 1 2 3 4 5
‫رات‬I2
‫د ا‬$ C4 = 6
2
C 7 = 35 C 11 = 462 C 4 ×C 7 = 210 C 4 ×C 3 × C 41 = 72
3 5 2 3 2 1

P (X ≺ 1) = p (X = −1) =
1
(3
5
P (X ≥ ) = p (X ≥ 3) =
5
(2
13
a=
(1 37
3 2 12 60
2
P (X 2 − 6X + 8 ≺ 0) = p [ (X − 4)(X − 2) ≺ 0] = p (2 ≤ X ≤ 4) = (4
3
12 88
p F (L ) = 0,08 ، p F (L ) = 0, 2 ، p (F ) = 1 − = = 0,88 (1 65
100 100
p (F ∩ L ) = p (F ) × p (L ) = 0.2 × 0.12 = 0.024 (‫( أ‬2
p (F ∩ L ) = p (F ) × p (L ) = 0.88 × 0.08 = 0.0704 (‫ب‬
p(F)= p (F ∩ L ) + p (F ∩ L ) = 0, 024 + 0, 0704 = 0.0944 (‫ج‬
p (F ∩ L ) 0, 024
p F (L ) = = ≃ 0, 25 (3
p (F ) 0, 0944
p (F ∩ L ) = 1 − (0, 024 + 0, 0704 + 0, 0964) = 0,8096 (‫( ﺃ‬4
p = 1 − (0,8096) 20 ≃ 0, 985 (‫ﺏ‬
‫ﺍﻝﺸﻌﺏ‪ • :‬ﺭﻴﺎﻀﻴﺎﺕ‬
‫ﺘﻘﻨﻲ ﺭﻴﺎﻀﻲ‬ ‫•‬

‫ﻋﻠﻭﻡ ﺘﺠﺭﻴﺒﻴﺔ‬ ‫•‬

‫ﺍﻝﺠﺯﺀ ﺍﻝﺜﺎﻨﻲ‬
‫ﺍﻷﻧﺸﻄﺔ‬
‫ا
ط اول‬
‫ ‪/ :‬‬
‫ا
ف‪ :‬ر
‪1‬أ ا‪979‬ل  'ا‪.A4‬‬
‫ ت‪ :‬م ا ‪-‬ط آ*) ('ة " ‪1‬أ ا‪979‬ل  'ا‪ " A4‬و ‪ #‬إ!ز  أاج ‪ A‬ا‪67‬ل ‪ 4‬ز ا ا‪.23‬‬
‫ا
‪:‬‬

‫‪(1‬‬

‫‪. 1 + 3 + ... + 55 = 784 = 282 (2‬‬


‫‪. 1 + 3 + ... + 87 = 1936 = 442 (3‬‬
‫‪ 2n − 1 + 1 ‬‬
‫‪2‬‬

‫‪. 1 + 3 + ... + ( 2n − 1) = ‬‬ ‫‪ = n (4‬‬


‫‪2‬‬

‫‪‬‬ ‫‪2‬‬ ‫‪‬‬


‫‪ (5‬ﻨﻀﻊ )‪A = 1 + 3 + ... + ( 2n − 1) + ( 2n + 1‬‬
‫ﺇﺫﻥ )‪. A = n 2 + ( 2n + 1) = ( n + 1‬‬
‫‪2‬‬

‫ا
ط ا
‬
‫ ‪/ :‬‬
‫ا
ف‪ :‬ا ‪P‬آ'   
ا ‪
/a‬و ا  
ا ‪.
7‬‬
‫ ت‪ :‬م ا ‪-‬ط آ*) ‪P‬ا ا ‪1‬ب و ج ‪ #‬ا 'ة " ‪P3‬آ' ل ا  ت " و ‪ #‬إ!ز  أاج‪.‬‬
‫ا
‪:‬‬
‫‪(1 .A‬‬
‫‪ (2‬ﻝﻴﻜﻥ ‪ n‬ﻋﺩﺩﺍ ﻁﺒﻴﻌﻴﺎ ‪ u n +1 = u n + 0,15u n = 1,15u n ،‬؛ ﺇﺫﻥ ) ‪ (u n‬ﻤﺘﺘﺎﻝﻴﺔ ﻫﻨﺩﺴﻴﺔ ﺃﺴﺎﺴﻬﺎ ‪ 1,15‬ﻭﺤﺩﻫﺎ ﺍﻷﻭ‪‬ل‬
‫‪. u 1 = 15000‬‬
‫‪ (3‬ﻤﻥ ﺃﺠل ﻜل ‪. u n = 15000 × 1,15n ، n ∈ ℕ‬‬
‫‪ (4‬ﺍﺒﺘﺩﺍﺀ ﻤﻥ ‪ n = 5‬ﻴﻜﻭﻥ ‪. u n > 25000‬‬
‫‪(1 .B‬‬
‫‪ (2‬ﻝﻴﻜﻥ ‪ n‬ﻋﺩﺩﺍ ﻁﺒﻴﻌﻴﺎ ‪ v n +1 = v n + 1500 ،‬؛ ﺇﺫﻥ ) ‪ (v n‬ﻤﺘﺘﺎﻝﻴﺔ ﺤﺴﺎﺒﻴﺔ ﺃﺴﺎﺴﻬﺎ ‪ 1500‬ﻭﺤﺩﻫﺎ ﺍﻷﻭ‪‬ل ‪.v 1 = 15000‬‬
‫‪ (3‬ﻤﻥ ﺃﺠل ﻜل ‪.v n = 1500n + 15000 ، n ∈ ℕ‬‬
‫‪ (4‬ﺍﺒﺘﺩﺍﺀ ﻤﻥ ‪ n = 8‬ﻴﻜﻭﻥ ‪.v n > 25000‬‬
‫ا
ط ا

‬
‫ ‪/ :‬‬
‫ا
ف‪ :‬ر
 م  
‪a‬ودة  ا^(‪.j‬‬
‫ ت‪ :‬م ا ‪-‬ط آ*) ('ة "  
‪a‬ودة ‪ " ...‬و ‪  #‬أاج آ ‪ #‬ا‪:;7‬ل ‪ 4‬ز ا ا‪
o: 23‬‬
‫ا رب‪.‬‬
‫ا
‪:‬‬
‫‪D f = [ −6; + ∞[ (1 .A‬‬
‫) ‪f ( x ) − f ( −6‬‬ ‫‪x +6‬‬
‫‪lim‬‬ ‫‪= lim‬‬ ‫‪= +∞ (2‬‬
‫‪x ‬‬
‫‪→ −6‬‬ ‫‪x +6‬‬
‫>‬
‫‪x‬‬
‫>‬
‫‪‬‬ ‫‪→−‬‬ ‫‪6‬‬ ‫‪x +6‬‬
‫‬
‫ﺍﻝﺩﺍﻝﺔ ‪ f‬ﻻ ﺘﻘﺒل ﺍﻻﺸﺘﻘﺎﻕ ﻋﻠﻰ ﻴﻤﻴﻥ ‪ −6‬؛ ﻭ ) ‪ (C f‬ﻴﻘﺒل ﻤﻤﺎﺴﺎ ﻤﻭﺍﺯﻴﺎ ﻝﻤﻨﺤﻰ ‪ j‬ﻋﻨﺩ ﺍﻝﻨﻘﻁﺔ ﺫﺍﺕ ﺍﻝﻔﺎﺼﻠﺔ ‪. −6‬‬
‫‪. lim f ( x ) = +∞ (3‬‬
‫∞‪x →+‬‬

‫‪1‬‬
‫= ) ‪ f ' ( x‬ﻭﻤﻥ ﺃﺠل ﻜل ‪ x‬ﻤﻥ ﺍﻝﻤﺠﺎل [∞ ‪ f ' ( x ) > 0 ، ]−6; +‬ﺇﺫﻥ ﺍﻝﺩﺍﻝﺔ ‪ f‬ﻤﺘﺯﺍﻴﺩﺓ ﺘﻤﺎﻤﺎ‬ ‫‪ (4‬ﻝﺩﻴﻨﺎ‬
‫‪2 x +6‬‬
‫ﻋﻠﻰ [∞ ‪. [ −6; +‬‬
‫‪−6‬‬ ‫∞‪+‬‬ ‫‪(5‬‬
‫‪x‬‬
‫) ‪f '(x‬‬ ‫‪+‬‬
‫) ‪f (x‬‬ ‫∞‪+‬‬
‫‪0‬‬
‫ﻤﻌﻨﺎﻩ ‪ ∆ = 25 x − x − 6 = 0‬؛ ‪ x ' = −2‬؛ ‪x ' = 3‬‬
‫‪2‬‬
‫‪x + 6 = x (6‬‬
‫‪ (C f‬ﻭ ) ∆ ( ﻫﻭ ﺍﻝﻨﻘﻁﺔ ﺫﺍﺕ ﺍﻹﺤﺩﺍﺜﻴﺘﻴﻥ )‪. ( 3;3‬‬ ‫)‬ ‫ﺒﻤﺎ ﺃﻥ ‪ x ∈ D f‬ﻓﺈﻥ ﺘﻘﺎﻁﻊ‬
‫‪y‬‬
‫‪3‬‬ ‫‪(7‬‬
‫‪2‬‬

‫‪1‬‬

‫‪-6‬‬ ‫‪-5‬‬ ‫‪-4‬‬ ‫‪-3‬‬ ‫‪-2‬‬ ‫‪-1‬‬ ‫‪0‬‬ ‫‪1‬‬ ‫‪2‬‬ ‫‪3‬‬ ‫‪4x‬‬

‫‪ ، u n +1 = 6 + u n = f (u n ) (1 .B‬ﻝﺩﻴﻨﺎ ‪ u 1 > 0‬ﻭﻤﻥ ﺃﺠل ﻜل ∗‪ ، n ∈ ℕ‬ﺇﺫﺍ ﻜﺎﻥ [∞ ‪ u n ∈ ]0; +‬ﻓﺈﻥ‬


‫[∞ ‪f (u n ) ∈ ]0; +‬‬
‫‪ (2‬ﺍﺴﺘﻌﻤﺎل ‪.TI 83 plus‬‬

‫‪(3‬‬
‫‪y‬‬
‫‪3‬‬

‫‪2‬‬

‫‪1‬‬

‫‪ (4‬ﺘﺒﺩﻭ ﺍﻝﻤﺘﺘﺎﻝﻴﺔ ) ‪ (u n‬ﻤﺘﺯﺍﻴﺩﺓ‪.‬‬


‫‪-5‬‬
‫‪u0‬‬ ‫‪-4‬‬ ‫‪-3‬‬ ‫‪-2‬‬ ‫‪-1‬‬ ‫‪0‬‬ ‫‪1‬‬
‫‪u1‬‬
‫‪2‬‬
‫‪uu‬‬
‫‪2‬‬
‫‪3‬‬
‫‪3‬‬
‫‪x‬‬
‫‪ (5‬ﻤﻥ ﺍﻝﺠﻭﺍﺏ ‪ (2‬ﺘﺒﺩﻭ ﺍﻝﻤﺘﺘﺎﻝﻴﺔ ) ‪ (u n‬ﺃﻨﻬﺎ ﺘﻘﺘﺭﺏ ﻤﻥ ‪. 3‬‬
‫‪−u n 2 + u n + 6‬‬
‫‪.‬‬ ‫= ‪ u n +1 − u n = 6 + u n − u n‬؛ ﻤﻥ ﺃﺠل ﻜل ∗‪6 + u n + u n > 0 ، n ∈ ℕ‬‬ ‫‪(6‬‬
‫‪6 +un +un‬‬
‫ﻭ ‪ 0 < u n < 3‬ﺇﺫﻥ ‪ −2 < u n < 3‬ﻭﻤﻨﻪ ‪. −u n 2 + u n + 6 > 0‬‬
‫ا
ط ا
ا‬
‫ ‪/ :‬‬
‫ا
ف‪ :‬ر
 م   !ور‪.3‬‬
‫ ت‪ :‬م ا ‪-‬ط آ*) ('ة "  ن !ور‪3‬ن " و ‪  #‬أاج ‪ A‬ا‪:;7‬ل ‪ 4‬ز ا ا‪}: mR 23‬‬
‫ا‪ ';3 !3‬آ)  ا   و ‪3‬ر ‪.‬‬
‫ا
‪:‬‬
‫‪3x + 10‬‬ ‫‪3x + 2‬‬
‫= ) ‪. g (x‬‬ ‫= ) ‪ f (x‬؛‬ ‫ﻴﻤﻜﻥ ﺍﻋﺘﺒﺎﺭ ﺍﻝﺩﺍﻝﺘﻴﻥ ‪ f‬ﻭ ‪ g‬ﺍﻝﻤﻌﺭﻓﺘﻴﻥ ﻋﻠﻰ [∞‪ِ [ 0; +‬ـ‪:‬‬
‫‪x +2‬‬ ‫‪x +1‬‬
‫‪1‬‬
‫= ) ‪ f ' ( x‬ﻭﻤﻨﻪ ﺍﻝﺩﺍﻝﺔ ‪ f‬ﻤﺘﺯﺍﻴﺩﺓ ﺘﻤﺎﻤﺎ ﻋﻠﻰ [∞‪ [ 0; +‬ﺇﺫﻥ ﺍﻝﻤﺘﺘﺎﻝﻴﺔ ) ‪ (u n‬ﻤﺘﺯﺍﻴﺩﺓ ﺘﻤﺎﻤﺎ‪.‬‬ ‫‪ (1‬ﻝﺩﻴﻨﺎ‬
‫)‪( x + 1‬‬
‫‪2‬‬

‫‪−4‬‬
‫= ) ‪ g ' ( x‬ﻭﻤﻨﻪ ﺍﻝﺩﺍﻝﺔ ‪ g‬ﻤﺘﻨﺎﻗﺼﺔ ﺘﻤﺎﻤﺎ ﻋﻠﻰ [∞‪ [ 0; +‬ﺇﺫﻥ ﺍﻝﻤﺘﺘﺎﻝﻴﺔ ) ‪ (v n‬ﻤﺘﻨﺎﻗﺼﺔ ﺘﻤﺎﻤﺎ‪.‬‬ ‫‪ (2‬ﻝﺩﻴﻨﺎ‬
‫)‪( x + 2‬‬
‫‪2‬‬

‫‪3n + 2 3n + 10‬‬
‫‪lim (u n − v n ) = lim‬‬ ‫‪−‬‬ ‫‪ (3‬ﻝﺩﻴﻨﺎ‬
‫∞‪n →+‬‬ ‫‪n →+∞ n + 1‬‬ ‫‪n +2‬‬
‫ﻭﻤﻨﻪ ‪. lim (u n −v n ) = 3 − 3 = 0‬‬
‫∞‪n →+‬‬

‫‪ (4‬ﻨﻼﺤﻅ ﺃﻥ ‪lim u n = lim v n = 3‬‬


‫∞‪n →+‬‬ ‫∞‪n →+‬‬

‫ﺍﻷﻋﻤﺎﻝ ﺍﻟﻤﻮﺟﻬﺔ‬
‫ا
‪u n +1 = f (u n ) X‬‬ ‫درا‪ %
""+ %D‬ا! ‪+ %‬‬
‫ا
ف‪ :‬درا‪
7‬ا‪ ';3 !3‬و ‪3‬رب  
 ا ‪. u n +1 = au n + b )@-‬‬
‫ ت‪ #3 @ :‬ا ‪ )@2  )6‬أاج آ @ ا‪'R‬ا‪ U‬آا‪. X  Y4‬‬
‫ا
‪:‬‬
‫‪ (1 .1‬ﺍﻝﻤﺘﺘﺎﻝﻴﺔ ) ‪ (u n‬ﻝﻴﺴﺕ ﺭﺘﻴﺒﺔ ﻭﺘﺒﺩﻭ ﺃﻨﹼﻬﺎ ﺘﺘﻘﺎﺭﺏ ﻨﺤﻭ ‪. 2‬‬
‫‪y‬‬
‫‪3‬‬

‫‪(2‬‬
‫‪2‬‬

‫‪1‬‬

‫‪1‬‬
‫‪0‬‬ ‫‪1‬‬ ‫‪2‬‬ ‫‪3‬‬ ‫‪x‬‬ ‫‪ − x + 3 = x (3‬ﻤﻌﻨﺎﻩ ‪ . x = 2‬ﺇﺫﻥ ‪. α = 2‬‬
‫‪u0‬‬ ‫‪u2 u4 u3‬‬ ‫‪u1‬‬ ‫‪2‬‬
‫‪1‬‬
‫ﻝﻴﻜﻥ ‪ v n +1 = u n +1 − 2 = − u n + 1 ، n ∈ ℕ‬ﺃﻱ ‪:‬‬
‫‪2‬‬
‫‪ v n +1 = −‬ﻭﺒﺎﻝﺘﺎﻝﻲ ) ‪ (v n‬ﻫﻨﺩﺴﻴﺔ‪.‬‬
‫‪1‬‬ ‫‪1‬‬
‫‪(v n + 2 ) + 1 = − v n‬‬
‫‪2‬‬ ‫‪2‬‬
‫‪1‬‬
‫ﻝﺩﻴﻨﺎ ‪ −1 < − < 1‬ﺇﺫﻥ ‪ lim v n = 0‬ﻭ ‪. lim u n = lim v n + 2 = 2‬‬
‫∞‪n →+‬‬ ‫∞‪n →+‬‬ ‫∞‪n →+‬‬ ‫‪2‬‬
‫‪ (1 .2‬ﺇﺫﺍ ﻜﺎﻥ ‪ a = 0‬ﻓﺈﻨﻪ ﻤﻥ ﺃﺠل ﻜل ﻋﺩﺩ ﻁﺒﻴﻌﻲ ‪. u n +1 = b ، n‬‬
‫ﻭﻤﻨﻪ ﺇﺫﺍ ﻜﺎﻥ ‪ u 0 = b‬ﻓﺈﻥ ) ‪ (u n‬ﺜﺎﺒﺘﺔ ﻭﺇﺫﺍ ﻜﺎﻥ ‪ u 0 ≠ b‬ﻓﺈﻥ ) ‪ (u n‬ﺘﻜﻭﻥ ﺜﺎﺒﺘﺔ ﺍﺒﺘﺩﺍﺀ ﻤﻥ ﺍﻝﺤﺩ ﺍﻝﺜﺎﻨﻲ‪.‬‬
‫‪ (2‬ﺇﺫﺍ ﻜﺎﻥ ‪ a = 1‬ﻓﺈﻨﻪ ﻤﻥ ﺃﺠل ﻜل ﻋﺩﺩ ﻁﺒﻴﻌﻲ ‪ . u n +1 = u n + b ، n‬ﺇﺫﻥ ﺍﻝﻤﺘﺘﺎﻝﻴﺔ ) ‪ (u n‬ﺤﺴﺎﺒﻴﺔ ﺃﺴﺎﺴﻬﺎ ‪. b‬‬
‫‪ a ≠ 0 (3‬ﻭ ‪. a ≠ 1‬‬
‫• ﺍﻝﻭﻀﻌﻴﺔ ﺍﻝﻨﺴﺒﻴﺔ ﻝﻠﻤﺴﺘﻘﻴﻤﻴﻥ ) ‪ ( D‬ﻭ ) ∆ ( ﺘﺴﺘﻨﺘﺞ ﻤﻥ ﺇﺸﺎﺭﺓ ﺍﻝﻌﺒﺎﺭﺓ ‪. ( a − 1) x + b‬‬
‫ﺇﺫﺍ ﻜﺎﻥ [‪ a ∈ ]−∞;0[ ∪ ]0;1‬ﻓﺈﻥ ‪:‬‬
‫‪‬‬ ‫‪−b ‬‬
‫;∞‪ x ∈  −‬؛‬ ‫) ∆ ( ﻴﻘﻊ ﻓﻭﻕ ) ‪ ( D‬ﻝﻤﺎ ﻴﻜﻭﻥ‬
‫‪‬‬ ‫‪a − 1 ‬‬
‫‪ −b‬‬ ‫‪‬‬
‫‪.x ∈‬‬ ‫ﻭ ) ∆ ( ﻴﻘﻊ ﺃﺴﻔل ) ‪ ( D‬ﻝﻤﺎ ﻴﻜﻭﻥ ‪; +∞ ‬‬
‫‪ a −1‬‬ ‫‪‬‬
‫ﺇﺫﺍ ﻜﺎﻥ [∞‪ a ∈ ]1; +‬ﻓﺈﻥ ‪:‬‬
‫‪‬‬ ‫‪−b ‬‬
‫;∞‪ x ∈  −‬؛‬ ‫) ∆ ( ﻴﻘﻊ ﺃﺴﻔل ) ‪ ( D‬ﻝﻤﺎ ﻴﻜﻭﻥ‬
‫‪‬‬ ‫‪a − 1 ‬‬
‫‪ −b‬‬ ‫‪‬‬
‫‪.x ∈‬‬ ‫ﻭ ) ∆ ( ﻴﻘﻊ ﻓﻭﻕ ) ‪ ( D‬ﻝﻤﺎ ﻴﻜﻭﻥ ‪; +∞ ‬‬
‫‪ a −1‬‬ ‫‪‬‬
‫‪−b‬‬
‫= ‪.α‬‬ ‫•‬
‫‪a −1‬‬
‫‪b‬‬ ‫‪‬‬ ‫‪b  ba‬‬ ‫‪b‬‬
‫‪v n +1 = av n‬‬ ‫‪ v n +1 = au n + b +‬ﻤﻌﻨﺎﻩ‬ ‫‪= a v n −‬‬ ‫‪+‬‬ ‫‪ v n +1 = u n +1 +‬ﺃﻱ‬ ‫• ﻝﻴﻜﻥ ‪، n ∈ ℕ‬‬
‫‪a −1‬‬ ‫‪‬‬ ‫‪a −1  a −1‬‬ ‫‪a −1‬‬
‫ﺇﺫﻥ ) ‪ (v n‬ﻫﻨﺩﺴﻴﺔ ﺃﺴﺎﺴﻬﺎ ‪. a‬‬
‫‪ u n +1 = −u n + b (4‬؛ ‪ u 0 = u 0‬؛ ‪ u 1 = −u 0 + b‬؛ ‪ u 2 = u 0‬؛ ‪ u 3 = −u 0 + b‬؛ ‪ u 4 = u 0‬؛ ‪ u 5 = −u 0 + b‬؛ ‪. u 6 = u 0‬‬
‫ﻨﻼﺤﻅ ﺃﻨﹼﻪ ﻤﻥ ﺃﺠل ﻜل ‪ u 2 p = u 0 : p ∈ ℕ‬ﻭ ‪u 2 p +1 = −u 0 + b‬‬
‫‪n ‬‬ ‫‪b b‬‬
‫ﺒﺼﻴﻐﺔ ﺃﺨﺭﻯ ﻤﻥ ﺃﺠل ﻜل ‪. u n = ( −1)  u 0 −  + : n ∈ ℕ‬‬
‫‪‬‬ ‫‪2 2‬‬
‫‪b‬‬ ‫‪b‬‬
‫ﻝﻠﺒﺭﻫﺎﻥ ﻋﻠﻰ ﺍﻝﺘﺨﻤﻴﻥ ﻴﻤﻜﻥ ﺍﺴﺘﻌﻤﺎل ﺍﻝﺴﺅﺍل ‪ (3‬ﻝﺩﻴﻨﺎ ‪ u n = v n + α = v n + = v 0 ( −1) +‬؛ ﺃﻱ‬
‫‪n‬‬

‫‪2‬‬ ‫‪2‬‬
‫‪‬‬ ‫‪b‬‬ ‫‪b‬‬
‫‪. u n =  u 0 −  ( −1) +‬‬
‫‪n‬‬

‫‪‬‬ ‫‪2‬‬ ‫‪2‬‬


‫‪."+ %
""+‬ر‪  %‬ا
!د ‪e‬‬
‫ ‪/ :‬‬
‫ا
ف‪ >?3 :‬ا ا
ا^‪
7‬و ا  ت‪.‬‬
‫ ت‪ #3 @ :‬ا ‪ )@2  )6‬أاج آ @ ا‪'R‬ا‪ U‬آا‪. X  Y4‬‬
‫ا
‪:‬‬
‫‪e‬‬ ‫‪1+ x ‬‬
‫‪x‬‬
‫‪. lim f ( x ) = lim x  −‬‬ ‫‪ xlim‬؛ ∞‪ = +‬‬ ‫‪f ( x ) = lim e x − (1 + x ) = +∞ (1 .1‬‬
‫∞‪x →+‬‬ ‫∞‪x →+‬‬ ‫∞‪→−‬‬ ‫∞‪x →−‬‬
‫‪‬‬ ‫‪x‬‬ ‫‪x‬‬ ‫‪‬‬
‫‪ f ' ( x ) = e x − 1‬ﻭﻤﻨﻪ ﺠﺩﻭل ﺍﻝﺘﻐﻴﺭﺍﺕ ‪:‬‬
‫∞‪−‬‬ ‫‪0‬‬ ‫∞‪+‬‬
‫‪x‬‬
‫) ‪f '(x‬‬ ‫‪-‬‬ ‫‪0‬‬ ‫‪+‬‬
‫) ‪f (x‬‬ ‫∞‪+‬‬ ‫∞‪+‬‬
‫‪0‬‬
‫‪(1) ... 1 + x‬‬ ‫‪≤ex‬‬ ‫‪ (2 .1‬ﻤﻥ ﺃﺠل ﻜل ﻋﺩﺩ ﺤﻘﻴﻘﻲ ‪ f ( x ) ≥ 0 ، x‬ﻤﻌﻨﺎﻩ ﻤﻥ ﺃﺠل ﻜل ﻋﺩﺩ ﺤﻘﻴﻘﻲ ‪، x‬‬
‫‪1‬‬ ‫‪1‬‬
‫‪−X‬‬
‫≤‬ ‫‪ (3 .1‬ﺒﻭﻀﻊ ‪ (1) x = − X‬ﺘﺼﺒﺢ ‪ 1 − X ≤ e − X‬ﻭﺒﻔﺭﺽ ‪ X < 1‬ﺃﻱ ‪ 1 − X > 0‬ﻴﻜﻭﻥ‬
‫‪e‬‬ ‫‪1− X‬‬
‫‪1‬‬ ‫‪1‬‬
‫≤ ‪( 2 ) ... e x‬‬ ‫≤ ‪ . e X‬ﻭﺒﺎﻝﺘﺎﻝﻲ ﺇﺫﺍ ﻜﺎﻥ ‪ x < 1‬ﻓﺈﻥ‬ ‫ﺃﻱ‬
‫‪1− x‬‬ ‫‪1− X‬‬
‫‪n‬‬ ‫‪1‬‬
‫‪ 1‬‬ ‫‪1‬‬ ‫‪1‬‬
‫‪ (1 .2‬ﺒﻭﻀﻊ = ‪ (1) x‬ﺘﺼﺒﺢ ‪ 1 + ≤ e n‬ﺃﻱ ‪. 1 +  ≤ e‬‬
‫‪ n‬‬ ‫‪n‬‬ ‫‪n‬‬
‫‪1‬‬
‫‪ (2 .2‬ﺒﻭﻀﻊ )‪ x = − ( n + 1‬ﻴﻜﻭﻥ ‪ − ( n + 1) < 1‬ﻭﺍﻝﻤﺘﺒﺎﻴﻨﺔ ) ‪ ( 2‬ﺘﺼﺒﺢ‬
‫)‪−( n +1‬‬
‫‪ e‬ﺇﺫﻥ‬ ‫≤‬
‫‪n +2‬‬
‫‪n +1‬‬
‫‪1 ‬‬ ‫)‪−( n +1‬‬ ‫‪1‬‬ ‫‪1 1‬‬
‫‪e ≤  + 1‬‬ ‫‪ e‬ﻭﻤﻨﻪ‬ ‫≤‬ ‫‪≤ ≤ +1‬‬
‫‪n ‬‬ ‫‪n +2 n n‬‬
‫‪n‬‬ ‫‪n +1‬‬ ‫‪n‬‬ ‫‪n +1‬‬
‫‪ 1‬‬ ‫‪ 1‬‬ ‫‪ 1‬‬ ‫‪ 1‬‬
‫‪ 1 +  − u n ≤ e − u n ≤ 1 + ‬ﺃﻱ‬ ‫‪ 1 +  ≤ e ≤ 1 + ‬ﻤﻌﻨﺎﻩ ‪− u n‬‬ ‫‪ (1 .3‬ﻝﺩﻴﻨﺎ‬
‫‪ n‬‬ ‫‪ n‬‬ ‫‪ n‬‬ ‫‪ n‬‬
‫‪n‬‬ ‫‪n‬‬ ‫‪n‬‬
‫‪3‬‬ ‫‪ 1 1 3‬‬ ‫‪ 1‬‬ ‫‪ 1 1‬‬
‫≤ ‪. 0 ≤ e −un‬‬ ‫‪ 1 + ‬ﻭﺒﺎﻝﺘﺎﻝﻲ‬ ‫≤‬ ‫‪ 0 ≤ e − u n ≤ 1 + ‬ﻭﻝﺩﻴﻨﺎ ‪ 1 +  ≤ e ≤ 3‬ﺇﺫﻥ‬
‫‪n‬‬ ‫‪ n n n‬‬ ‫‪ n‬‬ ‫‪ n n‬‬
‫‪3‬‬
‫‪ (2 .3‬ﻝﺩﻴﻨﺎ ‪ lim = 0‬ﺇﺫﻥ ‪ lim e − u n = 0‬ﻭﺒﺎﻝﺘﺎﻝﻲ ‪. lim u n = e‬‬
‫∞‪n →+‬‬ ‫∞‪n →+‬‬ ‫‪n →+∞ n‬‬

‫‪(3 . 3‬‬

‫ﺘﻁﺒﻴﻕ‬
‫‪1‬‬
‫= ‪ u n +1 − u n‬ﻭﻤﻨﻪ ‪u n +1 − u n > 0‬‬ ‫‪(1‬‬
‫! )‪( n + 1‬‬
‫‪1‬‬ ‫‪1‬‬ ‫‪1‬‬
‫= ‪v n +1 −v n‬‬ ‫‪−‬‬ ‫‪+‬‬ ‫‪(2‬‬
‫!)‪( n + 1) ( n + 1)! n ( n !) ( n + 1‬‬
‫‪1  −1 ‬‬
‫= ‪ v n +1 −v n‬ﺇﺫﻥ ‪.v n +1 −v n < 0‬‬ ‫‪‬‬ ‫‪‬‬
‫‪( n !)  n ( n + 1)2 ‬‬
‫‪1‬‬
‫‪. lim (v n − u n ) = lim‬‬ ‫‪= 0 (3‬‬
‫∞‪n →+‬‬ ‫)! ‪n →+∞ n ( n‬‬

‫ﺍﻝﻤﺘﺘﺎﻝﻴﺘﺎﻥ ) ‪ (u n‬ﻭ ) ‪ (v n‬ﻤﺘﺠﺎﻭﺭﺘﺎﻥ ﻭﺒﺎﻝﺘﺎﻝﻲ ﻝﻬﻤﺎ ﻨﻔﺱ ﺍﻝﻨﻬﺎﻴﺔ‪.‬‬


‫&‪I‬ب ‪%&I+‬‬
‫ ‪/ :‬‬
‫ا
ف‪ >?3 :‬ا   ا !ور‪.3‬‬
‫ ت‪ :‬م ا ‪-‬ط  أاج ‪.‬‬
‫ا
‪./ :‬‬
‫‪."+ %
""+‬ر‪  %‬ا
!د )‪ln(2‬‬
‫ ‪/ :‬‬
‫ا
ف‪ >?3 :‬ا ا
ا (_ر
و ا  ت‪.‬‬
‫ ت‪ #3 @ :‬ا ‪ )@2  )6‬أاج آ @ ا‪'R‬ا‪ U‬آا‪. X  Y4‬‬
:
‫ا‬
‫ل‬‫ﺍﻝﺠﺯﺀ ﺍﻷﻭ‬
، n ∈ ℕ∗ ‫( ﻝﻴﻜﻥ‬1
(2
‫ﺍﻝﺠﺯﺀ ﺍﻝﺜﺎﻨﻲ‬
1
. h (x ) = 1− − ln x ‫ ﻭ‬g ( x ) = 1 − x + ln x ‫ ﺤﻴﺙ‬h ‫ ﻭ‬g ‫( ﻨﻌﺘﺒﺭ ﺍﻝﺩﺍﻝﺘﻴﻥ‬1
x
 1 − x ln x 
. lim g ( x ) = lim x  +  = −∞ ‫ ؛‬x lim >
g ( x ) = −∞
x →+∞ x →+∞
 x x   →0

1 1− x
g ' ( x ) = −1 + =
x x
0 1 +∞
x
g '( x ) + 0 -
g (x ) 0
−∞ −∞
. ln x ≤ x − 1 ‫ ﺃﻱ‬1 − x + ln x ≤ 0 ‫ ﻤﻌﻨﺎﻩ‬g ( x ) ≤ 0 ، x ∈ ]0; +∞[ ‫ﺇﺫﻥ ﻤﻥ ﺃﺠل ﻜل‬
1
‫ ؛‬lim
>
h ( x ) = lim
>
1− (1 + x ln x ) = −∞
x 
→0 x 
→0 x
1 1 1− x
h '(x ) = − = 2 . lim h ( x ) = −∞
x2 x x x →+∞

0 1 +∞
x
h '(x ) + 0 -
h (x ) 0
−∞ −∞
1 1
. 1 − ≤ ln x ‫ ﺃﻱ‬1 − − ln x ≤ 0 ‫ ﻤﻌﻨﺎﻩ‬h ( x ) ≤ 0 ، x ∈ ]0; +∞[ ‫ﺇﺫﻥ ﻤﻥ ﺃﺠل ﻜل‬
x x
1
. 1 − ≤ ln x ≤ x − 1 ‫ﺨﻼﺼﺔ‬
x
.‫ ﻋﺩﺩ ﻁﺒﻴﻌﻲ ﻏﻴﺭ ﻤﻌﺩﻭﻡ‬p ‫( ﻝﻴﻜﻥ‬2
p p +1 p +1 p +1
‫ ﻭﻫﺫﺍ ﻴﻌﻨﻲ‬1 − ≤ ln ≤ − 1 : ‫ ﻭﺒﺎﻝﺘﻌﻭﻴﺽ ﻓﻲ ﺍﻝﻤﺘﺒﺎﻴﻨﺔ ﺍﻝﺴﺎﺒﻘﺔ ﻨﺠﺩ‬x = ‫ﻨﻀﻊ‬
p +1 p p p
1 p +1 1
. ≤ ln ≤
p +1 p p
1 2n 1 1 n +2 1 1 n +1 1
. ≤ ln ≤ ‫ ؛‬... ‫؛‬ ≤ ln ≤ ‫؛‬ ≤ ln ≤ (a (3
2n 2n − 1 2 n − 1 n +2 n +1 n +1 n +1 n n
 n +1 n + 2 2n  1 1 1
. ln  × × ... ×  = ln 2 : 2 ‫ ﺍﻝﻁﺭﻑ‬. + + = u n : 1 ‫( ﺍﻝﻁﺭﻑ‬b
 n n +1 2n − 1  n + 1 n + 2 2n
1 1 1 1 1 1 1 1 1 1 1
+ + + = un + ‫ ﺃﻱ‬+ + + = +un − :3‫ﺍﻝﻁﺭﻑ‬
n n + 1 n + 2 2n − 1 2n n n + 1 n + 2 2n − 1 n 2n
‫‪1‬‬
‫‪. u n ≤ ln 2 ≤ u n +‬‬ ‫ﺇﺫﻥ‬
‫‪2n‬‬
‫‪1‬‬ ‫‪1‬‬
‫‪ lim‬ﺇﺫﻥ ‪ lim ( ln 2 − u n ) = 0‬ﺃﻱ ‪. lim u n = ln 2‬‬ ‫≤ ‪ 0 ≤ ln 2 − u n‬ﻭﻝﺩﻴﻨﺎ ‪= 0‬‬ ‫‪ (4‬ﺍﻝﻤﺘﺒﺎﻴﻨﺔ ﺍﻝﺴﺎﺒﻘﺔ ﺘﻜﺎﻓﺊ‬
‫∞‪n →+‬‬ ‫∞‪n →+‬‬ ‫‪n →+∞ 2n‬‬ ‫‪2n‬‬

‫ﺍﻟﺘﻤﺎﺭﻳﻦ‬
‫ﺍﻝﺘﻤﺎﺭﻴﻥ ﺍﻝﺘﻁﺒﻴﻘﻴﺔ‬
‫‪. u 17 = u 3 + 14r = 97‬‬ ‫‪ 1‬ـ ‪r‬آ  
‪
""2‬ت ا
!د‪.%-‬‬
‫‪y‬‬
‫‪‬‬ ‫‪u0 = 2‬‬
‫‪. u n = u1 + 3 ( n − 1) = 3n − 5 (1 6‬‬ ‫‪‬‬
‫‪ 1‬أـ‬
‫‪1‬‬

‫‪‬‬ ‫‪1‬‬
‫‪20‬‬
‫‪ u 1 + u 2 + ... + u 20 = (u 1 + u 20 ) (2‬و  ‬
‫‪-6‬‬
‫‪u4 u3‬‬
‫‪-5‬‬ ‫‪-4‬‬
‫‪u2‬‬
‫‪-3‬‬ ‫‪-2‬‬
‫‪u1‬‬
‫‪-1‬‬ ‫‪0‬‬

‫‪-1‬‬
‫‪1‬‬ ‫‪2‬‬
‫‪u0‬‬ ‫‪u n +1 = 2 u n − 3‬‬
‫‪x‬‬

‫‪2‬‬
‫‪ u 20 = 55‬إذن ‪. u 1 + u 2 + ... + u 20 = 530‬‬
‫‪-2‬‬
‫ﻤﺘﻨﺎﻗﺼﺔ ﺘﻤﺎﻤﺎ‪.‬‬
‫‪-3‬‬

‫‪1‬‬ ‫‪3‬‬ ‫‪5‬‬


‫‪. S = + 1 + + 2 + + ... + 10 7‬‬
‫‪-4‬‬

‫‪-5‬‬
‫‪2‬‬ ‫‪2‬‬ ‫‪2‬‬
‫‪-6‬‬
‫‪1‬‬ ‫‪y‬‬
‫‪ u0 = 1‬‬
‫‪ S‬ﻫﻭ ﻤﺠﻤﻭﻉ ﺤﺩﻭﺩ ﻤﺘﺘﺎﺒﻌﺔ ﻤﻥ ﻤﺘﺘﺎﻝﻴﺔ ﻫﻨﺩﺴﻴﺔ ﺃﺴﺎﺴﻬﺎ‬ ‫‪1‬‬
‫‪‬‬
‫‪2‬‬ ‫‪. ‬‬ ‫‪1‬‬ ‫بـ‬
‫‪1 1‬‬ ‫‪1‬‬ ‫‪u‬‬
‫‪ n +1‬‬ ‫=‬ ‫‪−‬‬ ‫‪u‬‬
‫ﻭﺤﺩﻫﺎ ﺍﻷﻭل = ‪ a1‬؛ ‪an = a1 + ( n − 1)   = n‬‬
‫‪n‬‬
‫‪2‬‬
‫‪2 2‬‬ ‫‪2‬‬ ‫‪-1‬‬ ‫‪0‬‬ ‫‪1‬‬ ‫‪x‬‬ ‫ﻝﻴﺴﺕ ﺭﺘﻴﺒﺔ ‪.‬‬
‫‪20  1‬‬ ‫‪‬‬ ‫‪u1 u3 u2‬‬ ‫‪u0‬‬
‫ﺇﺫﻥ ‪ an = 10‬ﻤﻌﻨﺎﻩ ‪ n = 20‬ﻭﺒﺎﻝﺘﺎﻝﻲ ‪S =  + 10 ‬‬
‫‪2 2‬‬ ‫‪‬‬ ‫‪-1‬‬

‫ﺃﻱ ‪. S = 105‬‬ ‫‪y‬‬


‫‪n +1‬‬
‫‪5‬‬ ‫‪5 ×5‬‬
‫‪n‬‬
‫‪5‬‬ ‫‪5‬‬ ‫‪n‬‬
‫‪5‬‬ ‫‪6‬‬
‫= ‪. u n +1‬‬ ‫‪n +2‬‬
‫‪= n +1‬‬ ‫× ‪= n +1 × = u n‬‬ ‫‪8‬‬
‫‪7‬‬ ‫‪7 ×7 7‬‬ ‫‪7‬‬ ‫‪7‬‬ ‫‪5‬‬

‫‪3‬‬
‫‪ 18 ‬‬
‫‪4‬‬
‫‪18‬‬ ‫‪u‬‬
‫= ‪ q‬؛ ‪u 30 = u10 × q 20‬‬ ‫‪ q 3 = 10 =   9‬؛‬
‫‪ u1 = 1‬‬
‫‪3‬‬
‫‪11‬‬ ‫‪u 7  11 ‬‬ ‫‪. ‬‬ ‫ـ ـ‬
‫‪u n +1 = u n + 2‬‬
‫‪2‬‬

‫‪27 × 1820‬‬ ‫‪‬‬


‫= ‪u 30‬‬ ‫ﺃﻱ‬ ‫‪1‬‬

‫‪1123‬‬ ‫‪0‬‬ ‫‪1‬‬ ‫‪2‬‬ ‫‪3‬‬ ‫‪4‬‬ ‫‪5‬‬ ‫‪6‬‬ ‫‪7x‬‬


‫ﻤﺘﺯﺍﻴﺩﺓ ﺘﻤﺎﻤﺎ‪.‬‬
‫‪. u n = −2 × 3n −1 (1 10‬‬ ‫‪-1‬‬
‫‪u‬‬ ‫‪u‬‬ ‫‪0‬‬ ‫‪u‬‬ ‫‪1‬‬ ‫‪2‬‬
‫‪3‬‬
‫‪v n +1 − v n = r 2‬‬
‫‪37 − 1‬‬ ‫‪5‬‬
‫) ‪. u 1 + u 2 + ... + u 7 = ( −2‬‬ ‫‪= −2186 (2‬‬
‫‪3 −1‬‬ ‫ﻭ ‪.w n +1 − w n = u 3n +3 − u 3 n = u 3n + 3r − u 3n = 3r‬‬
‫‪.v n +1 = u 2 n + 2 = u 2 n × 32 = 9v n (3‬‬ ‫‪ 3‬ﻝﺩﻴﻨﺎ ‪ ( 90 − 2r ) + ( 90 − r ) + 90 = 180‬ﻭﻤﻨﻪ‬
‫ﺇﺫﻥ ) ‪ (v n‬ﻫﻨﺩﺴﻴﺔ ﺃﺴﺎﺴﻬﺎ ‪ 9‬ﻭﺤﺩﻫﺎ ﺍﻷﻭل ‪v 1 = u 2 = −6‬‬ ‫‪ 3r = 90‬ﺃﻱ ‪ r = 30‬ﺇﺫﻥ ﺍﻷﻗﻴﺎﺱ ﻫﻲ ‪ 60° ، 30°‬ﻭ ‪. 90°‬‬
‫‪9n − 1‬‬ ‫‪ (1 4‬ﺍﺴﺘﻌﻤﺎل ﺍﻝﺘﺭﺍﺠﻊ‪.‬‬
‫)‪= − ( 9n − 1‬‬
‫‪3‬‬
‫) ‪.v 1 + v 2 + ... + v n = ( −6‬‬
‫‪9 −1‬‬ ‫‪4‬‬ ‫‪1‬‬ ‫‪v +1‬‬ ‫‪1‬‬
‫= ‪ u n +1‬ﺇﺫﻥ‬ ‫‪= n‬‬ ‫‪= 1+‬‬ ‫‪ (2‬ﻝﺩﻴﻨﺎ ‪= 1 + u n‬‬
‫‪ u 3 = 9 u 1 (1 11‬ﻤﻌﻨﺎﻩ ‪ u 1 × q = 9u 1‬ﺃﻱ ‪q = 9‬‬
‫‪2‬‬ ‫‪2‬‬
‫‪v n +1‬‬ ‫‪vn‬‬ ‫‪vn‬‬
‫ﻷﻥ ‪ u 1 > 0‬ﻭﻋﻠﻴﻪ ‪ q = 3‬ﺃﻭ ‪ q = −3‬ﻭﺒﻤﺎ ﺃﻥ ﻜل ﺍﻝﺤﺩﻭﺩ‬ ‫) ‪ (u n‬ﺤﺴﺎﺒﻴﺔ ﺃﺴﺎﺴﻬﺎ ‪. 1‬‬
‫ﻤﻭﺠﺒﺔ ﺘﻤﺎﻤﺎ ﻓﺈﻥ ‪. q = 3‬‬ ‫‪u7 −u3‬‬
‫= ‪ r‬ﺃﻱ ‪ r = 6‬؛‬ ‫‪ u 7 = u 3 + 4r‬ﻤﻌﻨﺎﻩ‬ ‫‪5‬‬
‫‪4‬‬
‫‪9‬‬ ‫‪9‬‬ ‫‪3‬‬ ‫‪. u n = u 0 × q n = 2 × 3n (2‬‬
‫≤ ‪u k +1‬‬ ‫ﺇﺫﺍ ﻜﺎﻥ ≤ ‪ u k +1‬ﻓﺈﻥ ≤ ‪ u k +1‬ﻭﻤﻨﻪ‬
‫‪4‬‬ ‫‪4‬‬ ‫‪2‬‬ ‫‪q n +1 − 1 n +1‬‬
‫‪3‬‬ ‫‪. sn = u0‬‬ ‫‪= 3 − 1 (3‬‬
‫ﻤﻊ ﺍﻝﻤﻼﺤﻅﺔ ﺃﻥ ‪ u k +1 > 1‬ﺇﺫﻥ ≤ ‪. u k + 2‬‬ ‫‪q −1‬‬
‫‪2‬‬
‫‪ 2‬ـ ا‪E"DE‬ل 
"ا ‪.‬‬
‫‪ p ( n ) 16‬ﻫﻲ ﺍﻝﺨﺎﺼﻴﺔ " ‪" u n = 3‬‬
‫)‪0 ( 0 + 1‬‬
‫) ‪ p ( 0‬ﺘﻌﻨﻲ ‪. u 0 = 3‬‬ ‫= ‪.0‬‬ ‫‪ p ( 0 ) 12‬ﺘﻌﻨﻲ‬
‫‪2‬‬
‫ﺇﺫﺍ ﻜﺎﻨﺕ ‪ u k = 3‬ﻓﺈﻥ ‪. u k +1 = 6 + u k = 9 = 3‬‬ ‫)‪k ( k + 1‬‬
‫= ‪ 1 + 2 + ... + k‬ﻓﺈﻥ‬ ‫ﺇﺫﺍ ﻜﺎﻨﺕ‬
‫‪ p ( 0 ) (1 17‬ﺘﻌﻨﻲ ‪0 < u 0 < 1‬‬ ‫‪2‬‬
‫‪ 0 < u k < 1‬ﻤﻌﻨﺎﻩ ‪ 02 < u k 2 < 12‬ﺃﻱ ‪. 0 < u k +1 < 1‬‬ ‫)‪k ( k + 1‬‬ ‫)‪2 ( k + 1‬‬
‫= )‪1 + 2 + ... + k + ( k + 1‬‬ ‫‪+‬‬
‫‪2‬‬ ‫‪2‬‬
‫‪ u n +1 − u n = u n (u n − 1) (2‬ﺒﻤﺎ ﺃﻥ ‪ 0 < u n < 1‬ﻓﺈﻥ‬
‫= )‪. 1 + 2 + ... + k + ( k + 1‬‬
‫) ‪( k + 1)( k + 2‬‬ ‫ﺃﻱ‬
‫‪ u n > 0‬ﻭ ‪ u n − 1 < 0‬ﺇﺫﻥ ‪. u n +1 − u n < 0‬‬ ‫‪2‬‬
‫‪ p ( 0 ) 18‬ﺘﻌﻨﻲ ‪ 20 − 1‬ﻤﻀﺎﻋﻑ ِـ ‪. 7‬‬ ‫)‪0 ( 0 + 1)( 2 × 0 + 1‬‬
‫= ‪. 02‬‬ ‫‪ p ( 0 ) 13‬ﺘﻌﻨﻲ‬
‫ﻝﺩﻴﻨﺎ ‪ 23k − 1 = 7α‬ﻤﻌﻨﺎﻩ ‪. 23k = 7α + 1‬‬ ‫‪6‬‬
‫)‪k ( k + 1)( 2k + 1‬‬
‫‪ 23( k +1) − 1 = 8 × 23k − 1 = 8 × ( 7α + 1) − 1‬ﺃﻱ‬ ‫= ‪ 12 + 22 + ... + k 2‬ﻓﺈﻥ‬ ‫ﺇﺫﺍ ﻜﺎﻨﺕ‬
‫‪6‬‬
‫)‪. 23( k +1) − 1 = 7 × ( 8α + 1‬‬ ‫= )‪12 + 22 + ... + k 2 + ( k + 1‬‬
‫‪2‬‬

‫‪ p ( 0 ) 19‬ﺘﻌﻨﻲ ‪ 30 − 1‬ﻤﻀﺎﻋﻑ ِـ ‪. 8‬‬


‫)‪k ( k + 1)( 2k + 1) + 6 ( k + 1‬‬
‫‪2‬‬

‫ﻝﺩﻴﻨﺎ ‪ 32 k − 1 = 8α‬ﻤﻌﻨﺎﻩ ‪. 32 k = 8α + 1‬‬ ‫‪6‬‬


‫‪ 32( k +1) − 1 = 9 × 32 k − 1 = 9 × ( 8α + 1) − 1‬ﺃﻱ‬ ‫) ‪( k + 1) ( 2k 2 + 7k + 6‬‬
‫=‬
‫( ‪.3‬‬ ‫)‪− 1 = 8 × ( 9α + 1‬‬
‫)‪2 k +1‬‬
‫‪6‬‬
‫‪ p ( n ) 20‬ﻫﻲ " ‪ 3 2n − 2n‬ﻤﻀﺎﻋﻑ ﻝﻠﻌﺩﺩ ‪" 7‬‬ ‫=‬
‫)‪( k + 1)( k + 2 )( 2k + 3‬‬
‫‪6‬‬
‫) ‪ p ( 0‬ﻫﻲ ﺍﻝﺨﺎﺼﻴﺔ ‪ 30 − 20‬ﻤﻀﺎﻋﻑ ِـ ‪. 7‬‬
‫)‪02 ( 0 + 1‬‬
‫‪2‬‬

‫‪ 32 n − 2n = 7α‬ﻤﻌﻨﺎﻩ ‪. 32 n − 2 n = 7α + 2n‬‬ ‫= ‪.0‬‬ ‫‪3‬‬


‫‪ p ( 0 ) 14‬ﺘﻌﻨﻲ‬
‫‪4‬‬
‫‪ 32( n +1) − 2n +1 = 9 × 32 n − 2 × 2n‬ﺃﻱ‬ ‫)‪k 2 ( k + 1‬‬
‫‪2‬‬

‫= ‪ 1 + 2 + ... + k‬ﻓﺈﻥ‬
‫‪3‬‬ ‫‪3‬‬ ‫‪3‬‬
‫ﺇﺫﺍ ﻜﺎﻨﺕ‬
‫(‪3‬‬ ‫) ‪− 2n +1 = 9 ( 7α + 2n ) − 2 × 2n = 7 ( 9α + 2n‬‬
‫)‪2 n +1‬‬
‫‪4‬‬
‫= )‪13 + 23 + ... + k 3 + ( k + 1‬‬
‫‪3‬‬
‫ﻴﻤﻜﻥ ﺍﻋﺘﺒﺎﺭ ) ‪ p ( n‬ﻫﻲ " ‪ 32 n +1 + 2n + 2‬ﻤﻀﺎﻋﻑ ﻝﻠﻌﺩﺩ‬
‫)‪k 2 ( k + 1) + 4 ( k + 1‬‬
‫‪2‬‬ ‫‪3‬‬
‫‪." 7‬‬
‫‪ p ( 0 ) 21‬ﺘﻌﻨﻲ ‪ 03 − 2 × 0‬ﻴﻘﺒل ﺍﻝﻘﺴﻤﺔ ﻋﻠﻰ ‪. 3‬‬ ‫‪4‬‬
‫) ‪( k + 1) ( k 2 + 4k + 4‬‬
‫‪2‬‬

‫ﻨﻔﺭﺽ ﺃﻥ ‪n 3 + 2n = 3α‬‬ ‫=‬


‫‪4‬‬
‫ﻝﺩﻴﻨﺎ ‪( n + 1) + 2 ( n + 1) = n 3 + 3n 2 + 3n + 2n + 3‬‬
‫‪3‬‬

‫‪ 15‬ﺃ( )‪ p (1‬ﺘﻌﻨﻲ ‪ u 1 > 1‬ﺃﻱ ‪. 2 > 1‬‬


‫ﺃﻱ ‪. ( n + 1) + 2 ( n + 1) = 3α + 3n 2 + 3n + 3‬‬
‫‪3‬‬
‫ﻭﻤﻌﻨﺎﻩ ‪. u k +1 > 1‬‬ ‫‪ u k > 1‬ﻤﻌﻨﺎﻩ ‪u k > 1‬‬
‫‪ (1 22‬ﻝﺩﻴﻨﺎ ‪ 10n + 1 = 9α‬ﻤﻌﻨﺎﻩ ‪ 10 n = 9α − 1‬ﺃﻱ‬ ‫‪3‬‬ ‫‪3‬‬
‫≤‪. 2‬‬ ‫ﺏ( )‪ p ' (1‬ﺘﻌﻨﻲ ≤ ‪ u 2‬ﺃﻱ‬
‫‪ 10 n +1 = 90α − 10‬ﻭﻤﻌﻨﺎﻩ ‪. 10n +1 = 9 (10α − 1) − 1‬‬ ‫‪2‬‬ ‫‪2‬‬
‫‪ (2‬ﻤﻥ ﺃﺠل ‪ 100 + 1 = 2 ، n = 0‬ﻭ ‪ 2‬ﻝﻴﺱ ﻤﻀﺎﻋﻑ ِـ ‪. 9‬‬
1 1 1 nπ . %-‫د‬$ %
""+ ‫رب‬. ‫ ـ‬3
، −
≤ u n ≤ ‫ ﻝﺩﻴﻨﺎ‬. u n = cos (3
n n n 17 1
0< < 10 −3 ‫ ﻤﻌﻨﺎﻩ‬−10−3 < u n < 10−3 ‫ ﻝﺩﻴﻨﺎ‬23
1 1
. lim u n = 0 ‫ ﻓﺈﻥ‬lim − = lim = 0 ‫ﺒﻤﺎ ﺃﻥ‬ n n
n →+∞ n →+∞ n n →+∞ n
1
. n > 10 2 ‫ ﺃﻱ‬n 3 > 106 ‫ ﻭﻴﻜﺎﻓﺊ‬n n > ‫ﻭﻤﻌﻨﺎﻩ‬
. A (1;1) ‫ ﻭ‬O ( 0;0 ) ‫ ﻭ ∆ ﻫﻤﺎ ﺍﻝﻨﻘﻁﺘﺎﻥ‬C ‫( ﺘﻘﺎﻁﻊ‬1 31 10−3
.‫( ﻤﺘﺯﺍﻴﺩﺓ ﺘﻤﺎﻤﺎ‬u n ) ‫ [ ﺇﺫﻥ‬0; +∞[ ‫ ﻤﺘﺯﺍﻴﺩﺓ ﺘﻤﺎﻤﺎ ﻋﻠﻰ‬f (2 n 3 > 1012 ‫ ﻭﻤﻌﻨﺎﻩ‬n n > 106 ‫ ﻤﻌﻨﺎﻩ‬u n > 106 24
.‫( ﻤﺘﺒﺎﻋﺩﺓ‬u n ) ‫ ﻭﻤﻨﻪ‬lim u n = lim f ( x ) = +∞ . n > 10 4 ‫ﺃﻱ‬
n →+∞ x →+∞
3
y
‫} أن‬: v 0 = 0,8
  (3 2n > 3 × 105 ‫ ﻤﻌﻨﺎﻩ‬u n < 10−5 ‫ ؛‬u n = n ‫ ﻝﺩﻴﻨﺎ‬25
1 2
3
mR  (v n )
  ‫ا‬ 3 × 105
. 432809 ‫ ﺇﺫﻥ ﺍﺒﺘﺩﺍﺀ ﻤﻥ ﺍﻝﺩﻝﻴل‬n > ‫ﺃﻱ‬
0 a ‫رب‬3‫و‬ ln 2
‫ ﺃﻱ‬3n > 1012 ‫ ﻤﻌﻨﺎﻩ‬u n > 1012 ‫ ؛‬u n = 3n ‫ ﻝﺩﻴﻨﺎ‬26
‫} أن‬: v 0 = −1,1
 
1012
0
u3 u2 u1 u0
1 x 3 ‫اة‬X (v n )
  ‫ا‬ . 910239226627 ‫ ﺇﺫﻥ ﺍﺒﺘﺩﺍﺀ ﻤﻥ ﺍﻝﺩﻝﻴل‬n >
ln 3
lim v n = +∞ ‫و‬ 5n − 2 5 3n + 2 3
n →+∞
. lim = (2 . lim = (1 27
3 ‫اة‬X (v n )
  ‫} أن ا‬: v 0 = 1,1
  n →+∞ 4n − 3 4 n →+∞ 2n − 1 2
2
lim v n = +∞ ‫و‬ . lim 2n − = +∞ (3
n →+∞ n →+∞ n +1
n +2
y y
n
4 4 . lim − 4 + 2 = +∞ (4
n →+∞ 3 n +1
7 n 2 − 3n + 2
3 3

. lim = 7 (1 28
n →+∞ n 2 − n + 1
2 2

− n 2 + 4n + 2
1 1

. lim = −1 (2
( n + 2)
n →+∞ 2
-1 0 1 2 3 4 x 0 1 2 3 4 x
u0 u1u2 u3 uu01u2 u3

v 0 = 0 ‫ إذن *ر‬f (1) = 1 ‫ و‬f ( 0 ) = 0   (4 n 2 + 2n −3n + 12


. lim = +∞ (4 . lim = 0 (3
.
T (v n ) ‫ @ن‬v 0 = 1 ‫أو‬ n →+∞ 4 n + 3 n →+∞ n 2 + 1

3n + 2 3
. ‫ودة‬2
‫""
ت ا‬2
‫ ـ ا‬4 . lim = (1 29
n →+∞ 2n + 1 2
10
= 5 − 10−7 = 4, 9999999   32
u 104 = 5 − n2 + 2
. lim = lim n = +∞ (2
8
10
{ 4,99999 ‫ و‬0 ‫د‬6 ‫ إذن ا‬u 104 > 4,99999 U ‫و‬ n →+∞ n + 3 n →+∞
. (u n )
 ( j(^‫'ان ان  ا‬m  n +2
. lim = 0 (3
10
n →+∞ 2n + 1
− < 0 , n ‫وم‬6 '_ 61Z ‫) آ) د‬4‫    أ‬
n n +n
n2
 ‫'ان دان‬m  ‫ ه‬6 ‫ و‬5   ‫ و‬u n < 5 U ‫و‬
. lim
n →+∞ n +1
= lim
n
n →+∞ n + 1
(
n + 1 = +∞ (4 )
.
 ( j(^‫ا‬  3π n + 2  3π
. lim sin   = sin = −1 (1 30
 nπ  n →+∞
 2n + π  2
−1 ≤ sin   ≤ 1 ، n ∈ ℕ ‫ أ ـ ﻝﺩﻴﻨﺎ ﻤﻥ ﺃﺠل ﻜل‬33
 7   −3π n + 2 
. 1 ‫ ﻭ‬−1 ‫( ﻤﺤﺩﻭﺩﺓ ـ‬u n ) ‫ﺇﺫﻥ‬
. lim cos   = cos ( −3π ) = −1 (2
n →+∞
 n + 2π 
‫‪−3 2‬‬ ‫‪ 1 < 1 +‬ﺇﺫﻥ‬
‫‪1‬‬
‫ب ـ ﻝﺩﻴﻨﺎ ﻤﻥ ﺃﺠل ﻜل ∗‪≤ 2 ، n ∈ ℕ‬‬
‫‪.‬‬ ‫ﻤﻥ ﺃﺠل ﻜل ‪≤ u n < 0 ، n ∈ ℕ‬‬ ‫‪2‬‬
‫‪2‬‬ ‫‪n‬‬
‫‪ 35‬أ ـ ‪ u n = 2 n‬؛ ) ‪ (u n‬ﻤﺘﺘﺎﻝﻴﺔ ﻤﺘﺯﺍﻴﺩﺓ ﺇﺫﻥ ﻤﺤﺩﻭﺩﺓ ﻤﻥ‬ ‫) ‪ (u n‬ﻤﺤﺩﻭﺩﺓ ـ ‪ 1‬ﻭ ‪. 2‬‬

‫ﺍﻷﺴﻔل ـ ‪ u 0 = 1‬ﻭﺒﻤﺎ ﺃﻨﻬﺎ ﻏﻴﺭ ﻤﺘﻘﺎﺭﺒﺔ ﺃﻱ‬ ‫‪1‬‬


‫‪ 1 < 1 +‬ﺇﺫﻥ‬ ‫ـ ـ ﻝﺩﻴﻨﺎ ﻤﻥ ﺃﺠل ﻜل ‪< 2 ، n ∈ ℕ‬‬
‫‪n +2‬‬
‫∞‪ lim u n = +‬ﻓﺈﻨﻬﺎ ﻏﻴﺭ ﻤﺤﺩﻭﺩﺓ ﻤﻥ ﺍﻷﻋﻠﻰ‪.‬‬
‫∞‪n →+‬‬ ‫) ‪ (u n‬ﻤﺤﺩﻭﺩﺓ ـ ‪ 1‬ﻭ ‪. 2‬‬
‫ب ـ ‪ u n = n 3 − 2‬؛ ) ‪ (u n‬ﻤﺘﺘﺎﻝﻴﺔ ﺤﺴﺎﺒﻴﺔ ﻤﺘﺯﺍﻴﺩﺓ ﺇﺫﻥ‬ ‫‪1‬‬
‫< ‪ 0‬ﺇﺫﻥ‬ ‫د ـ ﻝﺩﻴﻨﺎ ﻤﻥ ﺃﺠل ﻜل ‪≤ 1 ، n ∈ ℕ‬‬
‫ﻤﺤﺩﻭﺩﺓ ﻤﻥ ﺍﻷﺴﻔل ـ ‪ u 0 = −2‬ﻭﺒﻤﺎ ﺃﻨﻬﺎ ﻏﻴﺭ ﻤﺘﻘﺎﺭﺒﺔ ﺃﻱ‬ ‫‪1+ n 2‬‬
‫∞‪ lim u n = +‬ﻓﺈﻨﻬﺎ ﻏﻴﺭ ﻤﺤﺩﻭﺩﺓ ﻤﻥ ﺍﻷﻋﻠﻰ‪.‬‬ ‫) ‪ (u n‬ﻤﺤﺩﻭﺩﺓ ـ ‪ 0‬ﻭ ‪. 1‬‬
‫∞‪n →+‬‬
‫‪x‬‬ ‫‪n‬‬
‫ـ ـ ‪ u n = n 2 + n − 1‬؛ ‪ f ( x ) = x + x − 1‬؛‬
‫‪2‬‬ ‫= ) ‪ f (x‬؛‬ ‫= ‪ un‬؛‬ ‫‪ 34‬أ ـ‬
‫‪x +1‬‬‫‪2‬‬
‫‪n 2 +1‬‬
‫‪f ' ( x ) = 2x + 1‬‬ ‫‪1‬‬
‫= ) ‪f '(x‬‬
‫‪x‬‬
‫‪0‬‬ ‫∞‪+‬‬ ‫‪(x‬‬ ‫‪2‬‬
‫‪+ 1) x 2 + 1‬‬
‫) ‪f '(x‬‬ ‫‪+‬‬ ‫‪x‬‬
‫‪0‬‬ ‫∞‪+‬‬
‫) ‪f (x‬‬ ‫∞‪+‬‬ ‫) ‪f '(x‬‬ ‫‪+‬‬
‫‪−1‬‬
‫) ‪f (x‬‬ ‫‪1‬‬
‫) ‪ (u n‬ﻤﺤﺩﻭﺩﺓ ﻤﻥ ﺍﻷﺴﻔل ـ ‪ u 0 = −1‬ﻓﻘﻁ‪.‬‬ ‫‪0‬‬
‫‪1‬‬ ‫ﻤﻥ ﺃﺠل ﻜل ‪. 0 ≤ u n < 1 ، n ∈ ℕ‬‬
‫= ‪ u n‬؛ ﻤﻥ ﺃﺠل ﻜل ﻋﺩﺩ ﻁﺒﻴﻌﻲ ‪، n‬‬ ‫‪ 36‬أ ـ ‪+ n 2‬‬
‫‪n +1‬‬ ‫‪x 2 −1‬‬ ‫‪n 2 −1‬‬
‫‪ u n > n 2 ≥ 0‬ﺇﺫﻥ ) ‪ (u n‬ﻤﺤﺩﻭﺩﺓ ﻤﻥ ﺍﻷﺴﻔل ﻭﻏﻴﺭ‬ ‫= ) ‪ f (x‬؛‬ ‫؛‬ ‫‪u‬‬ ‫=‬ ‫بـ‬
‫‪x 2 +1‬‬ ‫‪n 2 +1‬‬
‫‪n‬‬

‫ﻤﺤﺩﻭﺩﺓ ﻤﻥ ﺍﻷﻋﻠﻰ‪.‬‬ ‫‪4x‬‬ ‫‪x 2 +1‬‬


‫= ) ‪. f '(x‬‬
‫ب ـ ‪ u n = n + cos n‬؛ ‪ f ( x ) = x + cos x‬؛‬ ‫‪2 ( x 2 + 1) x − 1‬‬
‫‪2‬‬

‫‪ f ' ( x ) = 1 − sin x‬ﻭﻝﺩﻴﻨﺎ ‪. 1 − sin x ≥ 0‬‬ ‫‪1‬‬ ‫∞‪+‬‬


‫‪x‬‬
‫ﺒﻤﺎ ﺃﻥ ‪ x − 1 ≤ x + cos x‬ﻓﺈﻥ ∞‪. lim f ( x ) = +‬‬ ‫) ‪f '(x‬‬ ‫‪+‬‬
‫∞‪x →+‬‬
‫) ‪f (x‬‬ ‫‪1‬‬
‫‪0‬‬ ‫∞‪+‬‬ ‫‪0‬‬
‫‪x‬‬
‫) ‪f '(x‬‬ ‫‪0‬‬ ‫‪+‬‬ ‫ﻤﻥ ﺃﺠل ﻜل ∗‪. 0 ≤ u n < 1 ، n ∈ ℕ‬‬
‫) ‪f (x‬‬ ‫∞‪+‬‬ ‫‪−3‬‬ ‫‪−3‬‬
‫= ) ‪f (x‬‬ ‫= ‪ un‬؛‬ ‫ـ ـ‬
‫‪1‬‬ ‫‪3x + 2‬‬ ‫‪3n + 2‬‬
‫) ‪ (u n‬ﻤﺤﺩﻭﺩﺓ ﻤﻥ ﺍﻷﺴﻔل ـ ‪ u 0 = 1‬ﻓﻘﻁ‪.‬‬ ‫‪9‬‬
‫= ) ‪f '(x‬‬
‫ـ ـ ‪ u n = ( −1) × n 2‬؛ ﺇﺫﺍ ﻜﺎﻥ ‪ n‬ﺯﻭﺠﻴﺎ ﻓﺈﻥ‬
‫‪n‬‬ ‫‪2 ( 3x + 2 ) 3x + 2‬‬

‫‪ u n = n 2‬ﻭﺒﺎﻝﺘﺎﻝﻲ ) ‪ (u n‬ﻝﻴﺴﺕ ﻤﺤﺩﻭﺩﺓ ﻤﻥ ﺍﻷﻋﻠﻰ؛‬ ‫‪0‬‬ ‫∞‪+‬‬


‫‪x‬‬
‫ﻭﺇﺫﺍ ﻜﺎﻥ ‪ n‬ﻓﺭﺩﻴﺎ ﻓﺈﻥ ‪ u n = − n 2‬ﻭﺒﺎﻝﺘﺎﻝﻲ ) ‪ (u n‬ﻝﻴﺴﺕ‬ ‫) ‪f '(x‬‬ ‫‪+‬‬

‫ﻤﺤﺩﻭﺩﺓ ﻤﻥ ﺍﻷﺴﻔل‪.‬‬ ‫) ‪f (x‬‬ ‫‪0‬‬


‫‪−3 2‬‬
‫‪2‬‬
lim (u n − v n ) = lim
−6
= 0 .‫( ﻤﺘﻨﺎﻗﺼﺔ‬v n ) . f ' ( x ) = 2x − 5 ‫ ؛‬f ( x ) = x 2 − 5x + 6 (1 37
n →+∞ n →+∞ n + 1
5
. ‫( ﻤﺘﺠﺎﻭﺭﺘﺎﻥ‬v n ) ‫( ﻭ‬u n ) ‫ﺇﺫﻥ‬ x −∞ +∞
2
( −1) ( 2n + 1) ‫ ؛‬u
n +1
( −1)
n
f '(x ) - 0 +
u n +1 − u n = = 3+ ‫بـ‬
n +1 f (x ) +∞ +∞
n
n
‫( ﻏﻴﺭ‬v n ) ‫( ﻭ‬u n ) ‫( ﻝﻴﺴﺕ ﺭﺘﻴﺒﺔ ﻭﺒﺎﻝﺘﺎﻝﻲ‬u n ) ‫ﻭﻤﻨﻪ‬ −
1
4
. ‫ﻤﺘﺠﺎﻭﺭﺘﻴﻥ‬
‫ [ ﺇﺫﻥ ﻤﻥ ﺃﺠل ﻜل‬4; +∞[ ‫ ﻤﺘﺯﺍﻴﺩﺓ ﺘﻤﺎﻤﺎ ﻋﻠﻰ‬f ‫( ﺍﻝﺩﺍﻝﺔ‬2
1 1 1
‫ ؛‬un = + + ... + 41
n +1 n + 2 2n . x 2 − 5x + 6 ≥ 2 ‫ ﺃﻱ‬f ( x ) ≥ f ( 4 ) ، x ≥ 4

u n +1 − u n =
1
+
1

1 ، 4 ‫ ﺃﻜﺒﺭ ﻤﻥ ﺃﻭ ﻴﺴﺎﻭﻱ‬n ‫ﻭﺒﺎﻝﺘﺎﻝﻲ ﻤﻥ ﺃﺠل ﻜل ﻋﺩﺩ ﻁﺒﻴﻌﻲ‬
2 n + 1 2n + 2 n + 1 1 1
1 . 2 ≤ ‫ ﻭﻫﺫﺍ ﻴﻌﻨﻲ‬n 2 − 5n + 6 ≥ 2
.‫( ﻤﺘﺯﺍﻴﺩﺓ‬u n ) ‫ ﺇﺫﻥ‬u n +1 − u n = n − 5n + 6 2
2 ( n + 1)( 2n + 1)
. ‫ورن‬,"2
‫""
"ن ا‬2
‫ ـ ا‬5
−3n − 2 1
v n +1 − v n = ‫ ؛‬v n = un + −1
2n ( n + 1)( 2n + 1) n ‫ ؛ ﻤﻥ ﺃﺠل ﻜل‬v n =
1
‫ ﻭ‬un = 38
n +1 2n + 4
1
. lim (v n − u n ) = lim = 0 .‫( ﻤﺘﻨﺎﻗﺼﺔ‬v n ) ‫ﺇﺫﻥ‬ 1
n →+∞ n →+∞ n u n +1 − u n = : ‫ ﻝﺩﻴﻨﺎ‬n ∈ ℕ∗
1 1 1
2 ( n + 2 )( n + 3)
‫ ؛‬un = + + ... + 42 −1
n +1 n + 2 2n ‫( ﻤﺘﺯﺍﻴﺩﺓ‬u n ) ‫ ﺇﺫﻥ‬v n +1 − v n = ‫ﻭ‬
.‫( ﻤﺘﺯﺍﻴﺩﺓ‬u n ) ‫ ﺇﺫﻥ‬u n +1 − u n =
1 ( n + 1)( n + 2 )
2 ( n + 1)( 2n + 1) −3n − 5
‫ ﺇﺫﻥ‬u n − v n = ‫ ﻭﻝﺩﻴﻨﺎ‬، ‫( ﻤﺘﻨﺎﻗﺼﺔ‬v n ) ‫ﻭ‬
1 1 1 2 n 2 + 6n + 4
‫ ؛‬vn = + + ... +
n n +1 2n − 1 .‫( ﻤﺘﺠﺎﻭﺭﺘﻴﻥ‬v n ) ‫( ﻭ‬u n ) ‫ ﻭﺒﺎﻝﺘﺎﻝﻲ‬lim (u n − v n ) = 0
n →+∞
−1
.‫( ﻤﺘﻨﺎﻗﺼﺔ‬v n ) ‫ ﺇﺫﻥ‬v n +1 −v n = lim u n = lim v n = 0
2n ( 2n + 1) n →+∞ n →+∞

1 n −1 1
. lim (u n − v n ) = lim − = 0
1 ‫ ؛‬v n = 1+ 2
‫ ﻭ‬un = = 1 − 39
n →+∞ n →+∞ n n n n
1
1 1
‫ ؛‬u n = 1 + 2 + 2 + ... + 2 43
1 .‫( ﻤﺘﺯﺍﻴﺩﺓ‬u n ) ‫ ﺇﺫﻥ‬u n +1 − u n =
2 3 n n ( n + 1)
1 − ( 2n + 1)
.‫( ﻤﺘﺯﺍﻴﺩﺓ‬u n ) ‫ ﺇﺫﻥ‬u n +1 − u n = .‫( ﻤﺘﻨﺎﻗﺼﺔ‬v n ) ‫ ﺇﺫﻥ‬v n +1 − v n =
( n + 1) n 2 ( n + 1)
2 2

−1 1
(v n ) ‫ ﺇﺫﻥ‬v n +1 − v n = ‫ ؛‬v n = un +  1 1 
. lim (u n − v n ) = lim  − − 2  = 0
n ( n + 1)
2
n n →+∞ n →+∞
 n n 
. lim (u n − v n ) = lim −
1
= 0 .‫ﻤﺘﻨﺎﻗﺼﺔ‬ . lim u n = lim v n = 1
n →+∞ n →+∞
n →+∞ n →+∞ n
5 2n − 3
1 1 u n +1 − u n = ‫ ؛‬un = ‫ أ ـ‬40
‫ ؛‬un = 1+ + ... + − 2 n + 1 44 ( n + 1)( n + 2 ) n +1
2 n
1 .‫( ﻤﺘﺯﺍﻴﺩﺓ‬u n ) ‫ﻭﻤﻨﻪ‬
u n +1 − u n =
(
n + 1 2n + 3 + 2 ( n + 2 )( n + 1) ) ‫ ﻭﻤﻨﻪ‬v n +1 − v n =
−1
‫ ؛‬vn =
2n + 3
( n + 2 )( n + 1) n +1
‫‪−2 n 2 − 2 n − 1‬‬ ‫ﻭﻤﻨﻪ ) ‪ (u n‬ﻤﺘﺯﺍﻴﺩﺓ‪.‬‬
‫= ‪ v n +1 − v n‬ﻭﻤﻨﻪ‬
‫(‬
‫)‪n + 1 2 n ( n + 1) + ( 2n + 1‬‬ ‫)‬ ‫‪ v n = 1+‬؛‬
‫‪1‬‬
‫‪+ ... +‬‬
‫‪1‬‬
‫‪−2 n‬‬
‫) ‪ (v n‬ﻤﺘﻨﺎﻗﺼﺔ‪.‬‬
‫‪2‬‬ ‫‪n‬‬

‫‪−2‬‬
‫‪lim (u n − v n ) = lim‬‬ ‫‪=0‬‬
‫∞‪n →+‬‬ ‫∞‪n →+‬‬ ‫‪n + n +1‬‬

‫ﺘﻤﺎﺭﻴﻥ ﻝﻠﺘﻌﻤ‪‬ﻕ‬
‫ﻭﺒﺎﻝﺘﺎﻝﻲ ﺃﺼﻐﺭ ﻋﺩﺩ ﻁﺒﻴﻌﻲ ‪ n‬ﻫﻭ ‪. 2008‬‬ ‫‪ 1‬ـ ‪r‬آ  
‪
""2‬ت ا
!د‪. %-‬‬

‫‪ 2 S n = n ( 3 n + 7 ) (2‬ﻤﻌﻨﺎﻩ ) ‪n (u 1 + u n ) = n ( 3n + 7‬‬ ‫= ‪ u n‬؛ ﻝﻴﻜﻥ ‪ n‬ﻋﺩﺩ ﻁﺒﻴﻌﻲ ﻏﻴﺭ ﻤﻌﺩﻭﻡ ؛‬


‫‪ln n 45‬‬
‫‪n‬‬
‫ﺃﻱ ‪ (d − 3) n − d + 2u1 − 7 = 0‬ﺇﺫﻥ ‪ d = 3‬ﻭ ‪. u 1 = 5‬‬ ‫‪1 − ln x‬‬ ‫‪ln x‬‬
‫= ) ‪f '(x‬‬ ‫= ) ‪ f (x‬؛‬
‫‪ (u n ) 50‬ﻤﺘﺘﺎﻝﻴﺔ ﺤﺴﺎﺒﻴﺔ ﺃﺴﺎﺴﻬﺎ ‪ −5‬ﻭ ‪. u 0 = −4‬‬
‫‪2‬‬
‫‪x‬‬ ‫‪x‬‬
‫‪ f ' ( x ) ≤ 0‬ﻤﻌﻨﺎﻩ ‪ x ≥ e‬؛ ﺇﺫﻥ ‪ f‬ﻤﺘﻨﺎﻗﺼﺔ ﺘﻤﺎﻤﺎ ﻋﻠﻰ‬
‫‪. u n = − 5n − 4 ( 1‬‬
‫[∞‪ [e ; +‬ﻭﺒﺎﻝﺘﺎﻝﻲ ) ‪ (u n‬ﻤﺘﻨﺎﻗﺼﺔ ﺍﺒﺘﺩﺍﺀ ﻤﻥ ﺍﻝﺭﺘﺒﺔ ‪. 3‬‬
‫‪ S = u 26 + u 27 + ... + u125 = 50 (u 26 + u125 ) (2‬ﺃﻱ‬
‫‪5n‬‬
‫‪S = 50 ( −5 × 26 − 4 − 5 × 125 − 4 ) = −38150‬‬ ‫֏ ‪ u : n‬؛ ﻜل ﺤﺩﻭﺩ ‪ u‬ﻤﻭﺠﺒﺔ ﺘﻤﺎﻤﺎ ؛ ﻭﻝﺩﻴﻨﺎ‬ ‫‪46‬‬
‫!‪n‬‬
‫‪v n = 2 u n − 9 ، 4 u n +1 − 2 u n = 9 ، u 0 = 2 51‬‬ ‫‪5‬‬ ‫‪u‬‬ ‫‪u‬‬ ‫‪5‬‬
‫ﺃﻱ ‪n > 4‬‬ ‫‪ n +1‬؛ ‪ n +1 < 1‬ﻤﻌﻨﺎﻩ ‪< 1‬‬ ‫=‬
‫‪n +1‬‬ ‫‪un‬‬ ‫‪un‬‬ ‫‪n +1‬‬
‫أ ـ ‪ u 2 = 3,875 ، u 1 = 3, 25‬ﻭ ‪. u 3 = 4,1875‬‬
‫ﺇﺫﻥ ﺍﻝﻤﺘﺘﺎﻝﻴﺔ ‪ u‬ﺘﻜﻭﻥ ﻤﺘﻨﺎﻗﺼﺔ ﺍﺒﺘﺩﺍﺀ ﻤﻥ ﺍﻝﺩﻝﻴل ‪ 5‬ﺃﻱ‬
‫‪ v 2 = −1, 25 ،v 1 = −2, 5 ، v 0 = −5‬ﻭ ‪.v 3 = −0, 625‬‬
‫ﺍﻝﺭﺘﺒﺔ ﺍﻝﺴﺎﺩﺴﺔ‪.‬‬
‫‪9 1‬‬
‫‪v n +1 = 2 u n +1 − 9 = u n −‬‬ ‫ب ـ ‪= vn‬‬ ‫‪u n +1‬‬ ‫‪u‬‬ ‫‪n +1‬‬ ‫!‪n‬‬
‫‪2 2‬‬ ‫ﻤﻌﻨﺎﻩ‬ ‫= ‪ n +1‬؛ ‪> 1‬‬ ‫‪ u : n ֏ n 47‬؛‬
‫‪n +1‬‬ ‫‪n‬‬ ‫‪un‬‬ ‫‪un‬‬ ‫‪7‬‬ ‫‪7‬‬
‫‪1‬‬ ‫‪9‬‬ ‫‪1‬‬
‫‪. u n = −5  ‬‬ ‫‪−‬‬ ‫‪4‬ـ ـ ‪ v n = −5  ‬؛‬ ‫‪ n > 6‬ﺇﺫﻥ ‪ u‬ﺘﻜﻭﻥ ﻤﺘﺯﺍﻴﺩﺓ ﺍﺒﺘﺩﺍﺀ ﻤﻥ ﺍﻝﺩﻝﻴل ‪ 7‬ﺃﻱ ﺍﻝﺭﺘﺒﺔ‬
‫‪2‬‬ ‫‪2‬‬ ‫‪2‬‬
‫‪ 1  n +1 ‬‬ ‫ﺍﻝﺜﺎﻤﻨﺔ‪.‬‬
‫د ـ ‪ v 0 + v 1 + ... + v n = 10   − 1‬؛‬ ‫‪1 1‬‬ ‫‪1‬‬
‫‪ 2 ‬‬ ‫‪‬‬ ‫‪ un = 1+‬؛‬ ‫‪+ + ... +‬‬ ‫‪48‬‬
‫!‪1! 2‬‬ ‫!‪n‬‬
‫‪1‬‬
‫= ‪ u n +1 − u n‬ﺇﺫﻥ ‪. u n +1 − u n > 0‬‬
‫‪ 1 n +1  9‬‬ ‫! )‪( n + 1‬‬
‫)‪. u 0 + u 1 + ... + u n = 5   − 1 − ( n + 1‬‬
‫‪ 2 ‬‬ ‫‪ 2‬‬ ‫‪−1‬‬
‫= ‪ v n +1 − v n‬ﺇﺫﻥ ‪.v n +1 − v n < 0‬‬
‫)‪( n !) n ( n + 1‬‬
‫‪2‬‬
‫‪.v n = u n + 1 ، u n +1 = 4 u n + 3 ، u 0 = 14 52‬‬
‫‪v + v + v = 24‬‬
‫) ‪(v n‬‬ ‫‪ v n +1 = u n +1 + 1 = 4 (u n + 1) = 4v n (1‬ﺇﺫﻥ‬ ‫‪  1 2 3‬ﻤﻌﻨﺎﻩ‬ ‫‪ (1 49‬ﺃ ـ‬
‫‪v 4 + v 5 + v 6 + v 7 = 74‬‬
‫ﻤﺘﺘﺎﻝﻴﺔ ﻫﻨﺩﺴﻴﺔ ﺃﺴﺎﺴﻬﺎ ‪ 4‬ﻭﺤﺩﻫﺎ ﺍﻷﻭل ‪.v 0 = u 0 + 1 = 15‬‬
‫‪v = 5‬‬ ‫‪v + r = 8‬‬
‫‪ v n = 15 × 4n (2‬؛ ‪. u n = 15 × 4n − 1‬‬ ‫‪.  1‬‬ ‫‪  1‬ﺃﻱ‬
‫‪r =3‬‬ ‫‪ 2v 1 + 9r = 37‬‬
‫ﺏ ـ ‪ v n = 3n + 2‬؛ ‪ v n > 6023‬ﻤﻌﻨﺎﻩ ‪n > 2007‬‬
5 1
(v n ) ‫ ﺇﺫﻥ‬v n +1 = 2 u n +1 +
5 1
= u n + = v n (2 ‫ ؛ ﻝﺩﻴﻨﺎ‬S n = u 0 2 + u 12 + ... + u n 2 (3
3 2 12 4 ‫ ﻭﻤﻨﻪ‬u n 2 = 225 × 42 n − 30 × 4n + 1
1 5 2 1
. u n = n − ‫ ؛‬v n = n ‫ ـ‬. ‫ﻫﻨﺩﺴﻴﺔ ﺃﺴﺎﺴﻬﺎ‬
4 6 4 4 S n = 225 ( 40 + 42 + ... + 4 2 n )
: ‫ ﺤﻴﺙ‬t n ‫ ﻭ‬s n ‫ ﻜﻼ ﻤﻥ‬n ‫( ﺃﺤﺴﺏ ﺒﺩﻻﻝﺔ‬3 − 30 ( 40 + 41 + ... + 4n ) + n + 1
n +1
1 2( n +1)
  −1
. S n = 15 × 4 − 10 × 4n +1 + n − 4
، s n = v 0 + v 1 + ... + v n = 2  4 
2 8
=− + 2
1
−1 3× 4 n
3 u0 = ‫ ﻭ‬3 ‫( ﻤﺘﺘﺎﻝﻴﺔ ﻫﻨﺩﺴﻴﺔ ﺃﺴﺎﺴﻬﺎ‬u n ) 53
4 9
1 5 1 1 5 38 − 1 2 3 38 − 1
.t n = − − n + ‫ ﻭﻤﻨﻪ‬u n = v n − ‫ﻝﺩﻴﻨﺎ‬ S = u 3 + u 4 + ... + u 10 = u 3 = ×3 ×
3× 4 n
6 2 2 6 2 9 2
u = 2 , u1 = 4 . S = 19680 ‫ﺃﻱ‬
.  0 58
u n +1 = 4u n − u n −1 : n ≥ 1 )„„‫) آ‬4‫ أ‬ ‫ ؛‬S = 0, 02 − 0,1 + 0, 5 − 2, 5 + ... + 312,5 54
a + b = 4 ‫ ؛‬−5 ‫ﻤﺠﻤﻭﻉ ﺤﺩﻭﺩ ﻤﺘﺘﺎﺒﻌﺔ ﻤﻥ ﻤﺘﺘﺎﻝﻴﺔ ﻫﻨﺩﺴﻴﺔ ﺃﺴﺎﺴﻬﺎ‬
‫ ﻫﻤﺎ ﺤﻼ ﺍﻝﻤﻌﺎﺩﻝﺔ‬b ‫ ﻭ‬a ‫ ؛‬ (1
 ab = 1
u n = 0, 02 ( −5 )
n −1
‫ ﻴﻜﻭﻥ‬u 1 = 0, 02 ‫ﺒﻭﻀﻊ‬
( )
‫ ( أو‬a ; b ) = 2 − 3 ; 2 + 3 ‫ إذن‬x − 4x + 1 = 0 2

. n = 7 ‫ ( ﺃﻱ‬−5 )
n −1
= 15625 ‫ ﻤﻌﻨﺎﻩ‬u n = 312, 5
(
. (a ;b ) = 2 + 3 ; 2 − 3 )
( −5 ) −1
7

، 61Z ‫ دا‬n @ ‫ ؛‬v n = u n +1 − au n (2 .S = 0, 02 = −52,08


−6
‫ أي‬v n +1 = u n + 2 − au n +1 = 4u n +1 − u n − au n +1
s n = u 0 + u 1 + ... + u n . u n = 2 × 3n + 3 × 4n 55
‫ أي‬v n +1 = ( 4 − a ) u n +1 − u n = bu n +1 − abu n
3n +1 − 1 4 n +1 − 1 n +1 n +1
. b  77‫
أ‬7 ‫(  
ه‬v n ) ‫ إذن‬v n +1 = bv n . sn = 2 +3 =3 +4 −2
2 3
، 61Z ‫ دا‬n @ ‫ ؛‬w n = u n +1 − bu n (3
v n = u n + 3 ، u n +1 = 2 u n + 3 ، u 1 = 1 56
w n +1 = u n + 2 − bu n +1 = 4u n +1 − u n − bu n +1
‫( ﻫﻨﺩﺴﻴﺔ‬v n ) ‫ ﺇﺫﻥ‬v n +1 = u n +1 + 3 = 2u n + 6 = 2v n (1
w n +1 = ( 4 − b ) u n +1 − u n = au n +1 − abu n ‫أي‬
. a  77‫
أ‬7 ‫(  
ه‬w n ) ‫ إذن‬w n +1 = aw n . 2 ‫ﺃﺴﺎﺴﻬﺎ‬

v 0 = u 1 − au 0 = 4 − 2a = b − a (4 . u n = 2 n +1 − 3 ‫ ؛‬v n = 4 × 2n −1 = 2 n +1 ‫ـ‬
w 0 = u 1 − bu 0 = 4 − 2b = a − b ‫و‬ 2n − 1
. sn =v1 = 2 n + 2 − 4 (2
w n = w 0a n = ( a − b ) a n ‫ و‬v n = v 0b n = (b − a ) b n 2 −1
‫ إذن‬w n = u n +1 − bu n ‫ و‬v n = u n +1 − au n :   ‫ ﺃﻱ‬u 1 + u 2 + ... + u n + 3n = v 1 + v 2 + ... + v n ‫ـ‬
U ‫ و‬w n − v n = −bu n + au n = ( a − b ) u n . u 1 + u 2 + ... + u n + 3n = 2n + 2 − 4 = 4 ( 2n − 4 )
w n −v n
( ) ( )
n n
.u n = = an + b n = 2 + 3 + 2 − 3 5 1 5 1
a −b .v n = 2 u n + ، u n +1 = u n − ، u 0 = 57
3 4 8 6
. ‫ ﺃﻋﺩﺍﺩ ﺤﻘﻴﻘﻴﺔ ﻏﻴﺭ ﻤﻌﺩﻭﻤﺔ‬c ‫ ﻭ‬b ، a 59
157 37 7
،v 0 = 2 . u 3 = − ‫ ﻭ‬u 2 = − ، u1 = − (1
( a + b + c )( a − b + c ) = a 2 + 2ca − b 2 + c 2 (1 192 48 12
U ‫ و‬، 2b 2 = 2ac ‫ن‬v b 2 = ac ‫ أن‬ 1
. v 2 = ‫ ﻭ‬v1 =
1
( a + b + c )( a − b + c ) = a 2 + b 2 + c 2 8 2
v 1 q n − 1 4n a + b + c = 78
‫ ؛‬sn =× +  6  2   (2
13 q − 1 13  a + b + c = 3276
2 2

4   3   4n a + b + c = 78
n
10 
s n = −  a −   −  − 1 + ‫ أي‬
13  13   10   13  ( a + b + c )( a − b + c ) = 3276
15 a + b + c = 78
. α3 + α5 = ‫ ؛‬α1 = 3 63 . b = 18 ‫ أي‬2b = 78 − 42 = 36 ‫ إذن‬
16 a − b + c = 42
1
. q = (1 a + c = 60
‫د
ذات‬6 ‫ ا‬: ‫ ه‬c ‫ و‬a ‫ ؛‬
ac = 18 = 324
2 2

q n −1
n
1 ‫ إذن‬. x 2 − 60x + 324 = 0 :
  ‫ ا‬x ‫ا ! ل‬
s n = α1 = −6   + 6 (2
q −1 2 . ( a ; b ; c ) = ( 54;18;6 ) ‫ ( أو‬a ; b ; c ) = ( 6;18;54 )
، ‫وم‬6 '_ 61Z ‫ د‬n @
1 . ‫ ﺜﻼﺙ ﺤﺩﻭﺩ ﻤﺘﺘﺎﺒﻌﺔ ﻤﻥ ﻤﺘﺘﺎﻝﻴﺔ ﻫﻨﺩﺴﻴﺔ‬c ‫ ﻭ‬b ، a 60
β n +1 − β n = ln (α n +1 ) − ln (α n ) = ln = − ln 2
2 . b = 7 ‫ ﺃﻱ‬b 3 = 343
− ln 2  77‫
أ‬/
  ‫ ( ه‬β n )   ‫و‬ a + c = 29, 75
x 2 − 29, 75x + 49 = 0 . 
( β1 + β n ) = ( 2β1 − ( n − 1) ln 2 ) ‫ب ـ‬
n n
tn =  ac = 49
2 2
7   7
. ( a; b ; c ) =  ; 7; 28  ‫ ( ﺃﻭ‬a; b ; c ) =  28; 7; 
t n = ( 2 ln 3 − ln 2n −1 ) = ln n −1
n n 9
2 2 2 4   4
، n ‫وم‬6 '_ 61Z ‫) آ) د‬4‫    أ‬64 ‫ ﺇﺫﻥ‬3a + c = 4b ‫ ؛‬c = q 2a ‫ ﻭ‬b = qa ‫ ﻝﺩﻴﻨﺎ‬61
U ‫ و‬9A n = 99...9

‫ ﻭﻤﻨﻪ‬A n = 11...1
q 2 − 4q + 3 = 0 ‫ ﻓﺈﻥ‬a ≠ 0 ‫ ﺒﻤﺎ ﺃﻥ‬3a + q 2a = 4qa
#„„„„„„R‫ر‬n #„„„„„„R‫ر‬n

9A n + 1 = 100...0 . q = 3 ‫ ﺃﻭ‬q = 1 ‫ﺃﻱ‬



= 10
n

#„„„„„„R‫ر‬n
 6 v n +1 = 13u n +1 − 4 ، n ∈ ℕ∗ @ (1 62

7 ‫ ه ه‬u n = 10 ‫ ِـ‬ℕ j(


'6 ‫( ا‬u n )
  ‫ا‬
n
 4 3  12 39 v + 4
v n +1 = 13  − u n  − 4 = − × n
An =
9
(
1 n
10 − 1) = (u n − 1) :   ‫ و‬. 10  77‫أ‬
1
9
 10 10  10 10 13
12 3 12 3
1 1 1   ‫ و‬v n +1 = − v n + = − v n : ‫أي‬
s n = (u 1 − 1) + (u 2 − 1) + ... + (u n − 1) U ‫و‬ 10 10 13 10
9 9 9 3
. q =−  77‫
أ‬7 ‫(  
ه‬v n )
. s n = (10n − 1) − n
10 1
10
81 9 n −1
 3
1 v n = (13a − 4 )  −  U ‫ و‬v n = v 1 × q n −1 (2
A n = (u n − 1) ‫ و‬u n = 10n 65  10 
3
n −1
1 v +4 1   3 
‫ ؛‬s n = (u1 + u 2 + ... + u n ) − n  U(‫و‬ un = n = (13a − 4 )  −  + 4 
3 13 13   10  
1  10n − 1  10 (13a − 4 )  − 3 n −1 + 4 ‫أي‬
s n = u1
3
−n = (
10n − 1 − n
1
) . un =  
9  27 3 13  10  13
‫ ؛‬u n +1 = 5u n − 7 n ، u 0 = 5 66  6 s n = u 1 + u 2 + ... + u n (3
1
7
. v n = un − n −
7  6‫ و‬s n = (v 1 + 4 + v 2 + 4 + ... + v n + 4 )
4 16 13
1
‫ أي‬s n = (v 1 + v 2 + ... + v n + 4n )
13
β v n +1

a γ =
R:6 ‫ ا‬γ = 1 ‫ و‬β = 2 , α = 3 ‫ب ـ‬ : ‫ب‬/ A  ( )ma ‫ ا‬CD ‫( ا‬1
α −1 vn
‫ ه ا^ول‬α = 3  77‫
أ‬7 ‫( ه‬v n )   ‫و‬ n 0 1 2 3 4
un 5 25 118 576 2859
3 −1
n
3 1 n
. sn =v 0 = − + U ‫ ؛ و‬v 0 = −1 vn 4.5625 22.81 114.06 570.31 2851.56
3 −1 2 2 5 5 5 5 5
U ‫ و‬u n = v n − 1  6 v n = u n + 1  
t n = (v 0 − 1) + (v 1 − 1) + ... + (v n − 1) = s n − ( n + 1) 5 6 7 8
14267 71300 356458 1782241
3n 1 14257,81 71289,1 356445,313 1782226,56
. tn = − −n − 5 5 5 5
2 2
. 5 ‫س‬7^‫
ذات ا‬7 ‫( ه‬v n )
  ‫و أن ا‬1
. ‫ل 
"ا‬E"DE‫ ـ ا‬2
7 7
. s 4 = 30 ‫ ﻭ‬s 3 = 14 ، s 2 = 5 ، s1 = 1 ‫( أ ـ‬1 68 ( n + 1) − ، 61Z ‫ د‬n @ (2
v n +1 = u n +1 −
4 16
. s n +1 = s n + ( n + 1) ‫ب ـ‬
2
. 5 ‫س‬7^‫
ذات ا‬7 ‫( ه‬v n )
  ‫ إذن ا‬v n +1 = 5v n
1(1 + 1)( 2 × 1 + 1) ‫ ؛‬vn =
73 n
× 5 U ‫ و‬v n = v 0 × 5n
s1 = ‫ ﻫﻲ ﺍﻝﺨﺎﺼﻴﺔ‬p (1) (2 16
6
k ( k + 1)( 2k + 1)
73 7 7
. u n = × 5n + n +
‫ ﻓﺈﻥ‬s k = ‫ﺇﺫﺍ ﻜﺎﻥ‬ 16 4 16
6
  ‫ و‬، s n = u 0 + u 1 + ... + u n (3
k ( k + 1)( 2k + 1)
‫ ﺃﻱ‬s k +1 = + ( k + 1)
2
7 7
6 ، un =v n + n +

. s k +1 =
( k + 1)( k + 2 )( 2k + 3)  7  7 7 
4
7
16
7
6 s n = v 0 +  + v 1 + +  + ... + v n + n + 
 16   4 16   4 16 
، t 2 = t 1 + 2 × 3 = 8 ، t 1 = 1× 2 = 2 (1 69 7 7
s n = (v 0 + v 1 + ... + v n ) + (1 + 2 + ... + n ) + ( n + 1)
. t 4 = t 3 + 4 × 5 = 40 ‫ ﻭ‬t 3 = t 2 + 3 × 4 = 20 4 16
5n +1 − 1 7 n ( n + 1) 7
. t n +1 = t n + ( n + 1)( n + 2 ) ‫ ؛‬sn =v 0 + + ( n + 1)
4 8 16
1
.‫ ﻭﻫﻲ ﺼﺤﻴﺤﺔ‬t 1 = × 1(1 + 1)(1 + 2 ) ‫ ﺘﻌﻨﻲ‬p (1) (2
. s n = ( 5n +1 − 1) + ( 2n 2 + 3n + 1)
73 7
3
64 16
1
‫ ﻓﺈﻥ‬t k = k ( k + 1)( k + 2 ) ‫ﺇﺫﺍ ﻜﺎﻨﺕ‬ ‫ ﻋﺩﺩﺍﻥ‬β ‫ ﻭ‬α ، u n +1 = αu n + β ، u 0 = −2 67
3
1 . 1 ‫ﺤﻘﻴﻘﻴﺎﻥ ﻏﻴﺭ ﻤﻌﺩﻭﻤﻴﻥ ﻭﻴﺨﺘﻠﻔﺎﻥ ﻋﻥ‬
‫ ﻤﻌﻨﺎﻩ‬t k +1 = k ( k + 1)( k + 2 ) + ( k + 1)( k + 2 )
3 U ‫ و‬u n = u 0 = −2 ، n 61Z ‫) آ) د‬4‫(  أ‬1
1  : ‫ أي‬−2 = −2α + β l1m3 u n +1 = αu n + β
R:6 ‫ا‬
‫ ﺃﻱ‬t k +1 =  k + 1 ( k + 1)( k + 2 )
3  . β = 2α − 2
1
. t k +1 = ( k + 1)( k + 2 )( k + 3) . v n +1 = u n +1 + γ ، 61Z ‫ دا‬n @ ‫( أ ـ‬2
3
v n +1 = αu n + β + γ = α (v n − γ ) + β + γ
1 
. s 2 = 1 +  × 2 − 1 22 .‫ ﺘﻌﻨﻲ‬p ( 2 ) ‫ ﻭ‬s 2 = 1 70 v n +1 = αv n − αγ + β + γ = αv n − γ (α − 1) + β
2 
‫ أن @ن‬Y!
7 ‫( ه‬v n )
  ‫@ن ا‬3 @ ‫إذن‬
β
.γ = : ‫ أي‬−γ (α − 1) + β = 0
α −1
‫ﻭﺒﺎﻝﺘﺎﻝﻲ ‪ k 2 > 2k + 1‬ﻭﺤﺴﺏ ﻓﺭﻀﻴﺔ ﺍﻝﺘﺭﺍﺠﻊ ﻝﺩﻴﻨﺎ‬ ‫‪1‬‬ ‫‪‬‬
‫ﺇﺫﺍ ﻜﺎﻨﺕ ‪ s k = 1 +  k − 1 2k‬ﻓﺈﻥ‬
‫‪ 2k ≥ k 2‬ﺇﺫﻥ ‪2k > 2k + 1‬‬ ‫‪2‬‬ ‫‪‬‬
‫‪1‬‬ ‫‪‬‬
‫ﻝﺩﻴﻨﺎ ﺇﺫﻥ ‪ 2k ≥ k 2‬ﻭ ‪ 2k > 2k + 1‬ﺒﺠﻤﻊ ﻁﺭﻑ ﺇﻝﻰ‬ ‫‪ s k +1 = 1 +  k − 1 2 k + k × 2k −1‬ﻤﻌﻨﺎﻩ‬
‫‪2‬‬ ‫‪‬‬
‫ﻁﺭﻑ ﻨﺠﺩ ‪ 2k + 2 k > k 2 + 2k + 1‬ﺃﻱ‬
‫‪ k + 1  k +1‬‬
‫‪. s k +1 = 1 + ‬‬ ‫‪− 1 2‬‬
‫)‪ 2 × 2 k > ( k + 1‬ﻤﻌﻨﺎﻩ )‪2 k +1 > ( k + 1‬‬
‫‪2‬‬ ‫‪2‬‬
‫‪ 2‬‬ ‫‪‬‬
‫ﻭﻤﻨﻪ )‪. 2 k +1 ≥ ( k + 1‬‬
‫‪2‬‬
‫‪1‬‬
‫‪( n − 1) 2n − n × 2n −1 + 1 = n × 2n − 2n − n × 2n + 1‬‬
‫‪2‬‬
‫‪ P ( 2 ) 75‬ﺘﻌﻨﻲ ‪. 52 ≥ 4 2 + 32‬‬
‫‪1‬‬ ‫‪1‬‬ ‫‪‬‬
‫‪= −2 n + n × 2 n + 1 = 1 +  n − 1  2 n = s n‬‬
‫ﻨﻔﺭﺽ ‪ 5k ≥ 4k + 3k‬ﻭﻤﻨﻪ ‪، 5k +1 ≥ 5 × 4 k + 5 × 3k‬‬ ‫‪2‬‬ ‫‪2‬‬ ‫‪‬‬
‫ﺒﻤﺎ ﺃﻥ ‪ 5 × 4k ≥ 4 × 4k‬ﻭ ‪ 5 × 3k ≥ 3 × 3k‬ﻓﺈﻥ‬ ‫)‪1(1 + 1)(1 + 2 )(1 + 3‬‬
‫= ‪. 1× 2 × 3‬‬ ‫‪ p (1) 71‬ﺘﻌﻨﻲ‬
‫‪ 5 × 4k + 5 × 3k ≥ 4k +1 + 3k +1‬ﻭﻤﻨﻪ ‪. 5k +1 ≥ 4k +1 + 3k +1‬‬ ‫‪4‬‬
‫‪ 76‬ﻤﺘﺒﺎﻴﻨﺔ ﺒﺭﻨﻭﻝﻲ ) ‪( Bernoulli‬‬ ‫ﻨﻀﻊ ) ‪α n = 1× 2 × 3 + 2 × 3 × 4 + ... + n ( n + 1)( n + 2‬‬
‫)‪k ( k + 1)( k + 2 )( k + 3‬‬
‫‪ p (1) (1‬ﺘﻌﻨﻲ ‪. 1 + a ≥ 1 + a‬‬ ‫= ‪αk‬؛‬ ‫ﻨﻔﺭﺽ‬
‫‪4‬‬
‫ﻨﻔﺭﺽ ‪ (1 + a ) ≥ 1 + k a‬ﻭﻤﻨﻪ‬
‫‪k‬‬

‫)‪ α k +1 = α k + ( k + 1)( k + 2 )( k + 3‬ﻤﻌﻨﺎﻩ‬


‫) ‪. (1 + a‬‬ ‫‪≥ 1 + ( k + 1) a + ka 2 ≥ 1 + ( k + 1) a‬‬
‫‪k +1‬‬
‫)‪k ( k + 1)( k + 2 )( k + 3‬‬
‫= ‪α k +1‬‬ ‫‪+‬‬
‫‪ (2‬ﺇﺫﺍ ﻜﺎﻥ ‪ q > 1‬ﻓﺈﻥ ‪ q = 1 + a‬ﻤﻊ ‪a > 0‬‬ ‫‪4‬‬
‫ﻭﻤﻨﻪ ﻤﻥ ﺃﺠل ﻜل ∗‪ q n ≥ 1 + an ، n ∈ ℕ‬ﻭﻝﺩﻴﻨﺎ‬ ‫)‪4 ( k + 1)( k + 2 )( k + 3‬‬
‫∞‪ lim 1 + an = +‬ﻷﻥ ‪ a > 0‬ﺇﺫﻥ ∞‪lim q n = +‬‬ ‫‪4‬‬
‫∞‪n →+‬‬ ‫∞‪n →+‬‬
‫= ‪α k +1‬‬
‫(‬ ‫‪k‬‬ ‫‪+‬‬ ‫‪1‬‬‫()‬ ‫‪k‬‬ ‫‪+‬‬ ‫‪2‬‬ ‫()‬ ‫‪k‬‬ ‫‪+‬‬ ‫‪3‬‬ ‫) ‪)( k + 4‬‬‫ﺃﻱ‬
‫‪ p ( 2 ) (1 77‬ﺘﻌﻨﻲ ‪ . 12 ≥ 9‬ﻨﻔﺭﺽ )‪3k 2 ≥ ( k + 1‬‬
‫‪2‬‬
‫‪4‬‬
‫ﻭﻤﻨﻪ ‪3k 2 + 6k + 3 ≥ ( k + 1) + 6k + 3‬‬ ‫‪ p (1) 72‬ﺘﻌﻨﻲ ‪. 1 = (1 + 1) ! − 1‬‬
‫‪2‬‬

‫ﺃﻱ ‪ 3 ( k + 1) ≥ k 2 + 8k + 4‬ﺒﻤﺎ ﺃﻥ ‪ 8k ≥ 4k‬ﻓﺈﻥ‬


‫‪2‬‬
‫ﻨﻀﻊ )! ‪α n = 1 + 2 × 2! + 3 × 3! + ... + n ( n‬‬
‫‪ 3 ( k + 1) ≥ k 2 + 4k + 4‬ﺃﻱ ) ‪. 3 ( k + 1) ≥ ( k + 2‬‬
‫‪2‬‬ ‫‪2‬‬ ‫‪2‬‬
‫ﻨﻔﺭﺽ ‪. α k = ( k + 1) ! − 1‬‬
‫‪ (2‬ﻨﺴﻤﻲ ‪ Pn‬ﺍﻝﺨﺎﺼﻴﺔ ‪. " 3n ≥ 2n + 5n 2 " :‬‬ ‫‪ α k +1 = α k + ( k + 1) ( k + 1) !‬ﻤﻌﻨﺎﻩ‬
‫ﺃ ـ ‪ P1‬ﺘﻌﻨﻲ ‪ 3 ≥ 7‬؛ ‪ P2‬ﺘﻌﻨﻲ ‪ 9 ≥ 24‬؛ ‪ P3‬ﺘﻌﻨﻲ‬ ‫‪ α k +1 = ( k + 1) ! − 1 + ( k + 1) ( k + 1) !‬ﺃﻱ‬
‫‪ 27 ≥ 53‬؛ ‪ P4‬ﺘﻌﻨﻲ ‪ 81 ≥ 96‬؛ ‪ P5‬ﺘﻌﻨﻲ ‪243 ≥ 157‬‬ ‫‪α k +1 = ( k + 1)! ( k + 2 ) − 1 = ( k + 2 )! − 1‬‬
‫ﺇﺫﻥ ‪ P5‬ﻫﻲ ﺍﻝﺨﺎﺼﻴﺔ ﺍﻷﻭﻝﻰ ﺍﻝﺼﺤﻴﺤﺔ ‪.‬‬ ‫‪ p (1) 73‬ﺘﻌﻨﻲ ‪. 1! ≥ 21−1‬‬
‫ﺏ ـ ﻨﻔﺭﺽ ‪ 3k ≥ 2 k + 5k 2‬ﻤﻊ ‪ ، k ≥ 5‬ﻭﻤﻨﻪ‬ ‫ﻤﻥ ﺃﺠل ﻜل ∗‪k + 1 ≥ 2 ، k ∈ ℕ‬‬
‫)‪3k +1 ≥ 3 × 2 k + 5 × 3k 2 ≥ 2 × 2k + 5 × ( k + 1‬‬ ‫‪( k !)( k‬‬ ‫ﺇﺫﺍ ﻜﺎﻥ ‪ k ! ≥ 2k −1‬ﻓﺈﻥ ‪+ 1) ≥ 2k −1 × 2‬‬
‫‪2‬‬

‫ﻷﻥ ‪ 3 ≥ 2‬ﻭﻤﻥ ‪ (1‬ﻝﺩﻴﻨﺎ )‪. 3k 2 ≥ ( k + 1‬‬ ‫ﺃﻱ ‪. ( k + 1) ! ≥ 2k‬‬


‫‪2‬‬

‫‪ 78‬ﻤﻥ ﺃﺠل ﻜل ‪ Pn ، n ∈ ℕ‬ﺘﻌﻨﻲ " ) ‪." 3n ≥ ( n + 2‬‬ ‫‪ P ( 4 ) 74‬ﺘﻌﻨﻲ ‪. 24 ≥ 42‬‬


‫‪2‬‬

‫‪ P0 (1‬ﺘﻌﻨﻲ ‪ P1 ، 1 ≥ 4‬ﺘﻌﻨﻲ ‪ P2 ، 3 ≥ 9‬ﺘﻌﻨﻲ ‪9 ≥ 16‬‬ ‫ﻝﻴﻜﻥ ‪ k‬ﻋﺩﺩﺍ ﻁﺒﻴﻌﻴﺎ ﻜﻴﻔﻴﺎ ﺤﻴﺙ ‪ k ≥ 4‬ﻭﻨﻔﺭﺽ ‪. 2k ≥ k 2‬‬
‫ﻭ ‪ P3‬ﺘﻌﻨﻲ ‪ 27 ≥ 25‬ﻭﻫﻲ ﺍﻝﺨﺼﻴﺔ ﺍﻝﺼﺤﻴﺤﺔ‪.‬‬ ‫ﺇﺫﺍ ﻜﺎﻥ ‪ k ≥ 4‬ﻓﺈﻥ ‪ k 2 ≥ 4k‬ﻭﻜﺫﻝﻙ ﺇﺫﺍ ﻜﺎﻥ ‪k ≥ 4‬‬
‫ﻓﺈﻥ ‪ 2k ≥ 8‬ﻭﻤﻨﻪ ‪ 2k > 1‬ﺇﺫﻥ ‪2k + 2k > 1 + 2k‬‬
‫ ؛‬s k = k 2 ‫ ﻨﻔﺭﺽ‬. s 1 = 12 ‫ ﺘﻌﻨﻲ‬p (1) (2 ‫ ﺇﺫﻥ‬k ≥ 3 ‫ ﻤﻥ ﺃﺠل‬3k ≥ ( k + 2 ) ‫( ﻨﻔﺭﺽ‬2
2

. s k +1 = s k + ( 2k + 1) = k 2 + 2k + 1 = ( k + 1) 3k +1 ≥ 3k 2 + 12k + 12 ‫ ﻤﻌﻨﺎﻩ‬3k +1 ≥ 3 ( k + 2 )
2 2

‫ ﺤﺩﺍ ﺍﻷﻭﻝﻰ ﻤﻥ ﻤﺘﺘﺎﻝﻴﺔ ﺤﺴﺎﺒﻴﺔ‬n ‫ ﻫﻭ ﻤﺠﻤﻭﻉ‬s n (3 . 3k +1 ≥ ( k + 3) ‫ ﺃﻱ‬3k +1 ≥ k 2 + 6k + 9 ‫ﻭﻤﻨﻪ‬


2

n
. s n = (1 + 2n − 1) = n 2 ‫ ﺇﺫﻥ‬1 ‫ ﻭﺤﺩﻫﺎ ﺍﻷﻭل‬2 ‫ﺃﺴﺎﺴﻬﺎ‬ . u n +1 =
un
‫ ﻭ‬u 0 = 1 79
2
un 2 +1
. u n +1 = n + u n ، u 0 = 1 84
1 1 3 2
. u 5 = 11 ‫ ؛‬u 4 = 7 ‫ ؛‬u 3 = 4 ‫ ؛‬u 2 = 2 ‫ ؛‬u 1 = 1 (1 ‫ ؛ ﻴﺒﺩﻭ ﺃﻥ‬u 3 = = , u2 = ، u1 = (1
4 2 3 2
n ( n − 1)
. un = + 1 ‫ﻴﺒﺩﻭ ﺃﻥ‬ n +1
2 .u n =
n +1
0 ( 0 − 1)
.u 0 = + 1 ‫ ﺘﻌﻨﻲ‬p ( 0 ) (2 . u 0 = 1 ‫ ﺘﻌﻨﻲ‬p ( 0 ) (2
2
k ( k − 1)
uk 1 1
. u k +1 = = ‫ ؛‬uk = ‫ﻨﻔﺭﺽ‬
‫ ؛‬uk = + 1 ‫ﻨﻔﺭﺽ‬ uk 2 +1 k +2 k +1
2
k ( k − 1) k ( k + 1) . u n +1 = 10u n − 18 ، u 0 = 7 80
. u k +1 = k + +1 = +1
2 2 u 4 50002 ، u 3 = 5002 ، u 2 = 502 ، u 1 = 52 (1
un
. u n +1 = ، u 0 = 1 85 ‫ ( ﺼﻔﺭﺍ‬n − 1) ‫ ﻴﻭﺠﺩ‬u n ‫ ﻓﻲ ﺍﻝﺤﺩ‬. u 5 = 500002 ‫ﻭ‬
un + 2
1 1 1 1 1 . u n = 5 × 10n + 2 (2
. u5 = ، u 4 = ، u 3 = ، u 2 = ، u 1 = (1
63 31 15 7 3 . u 0 = 5 × 100 + 2 = 7 ‫ ﺘﻌﻨﻲ‬p ( 0 )
1 ‫ ﻤﻌﻨﺎﻩ‬u k +1 = 10 u k − 18 ‫ ؛‬u k = 5 × 10 k + 2 ‫ﻨﻔﺭﺽ‬
. u n = n +1
2 −1
1 1 . u k +1 = 10 ( 5 × 10n + 2 ) − 18 = 5 × 10n +1 + 2
‫ ؛‬u k = k +1 ‫ ﻨﻔﺭﺽ‬. u 0 = ‫ ﺘﻌﻨﻲ‬p ( 0 ) (2
2 −1 2 −1 . u n +1 = 2u n − 3 ، u 0 = 2 81
uk 1  1  u 4 = −13 ، u 3 = −5 ، u 2 = −1 ، u 1 = 1 (1
‫ ﺃﻱ‬u k +1 = = k +1 /  k +1 + 2 
u k + 2 2 −1  2 −1 
. 3 − u n = 2 n . u 5 = −29 ‫ﻭ‬
k +2
1  2 −1  1
. u k +1 = k +1
/  k +1  = k + 2 ‫ ؛‬3 − u k = 2k ‫ ﻨﻔﺭﺽ‬. 3 − u 0 = 20 ‫ ﺘﻌﻨﻲ‬p ( 0 ) (2
2 −1  2 −1  2 −1
. 3 − u k +1 = 6 − 2u k = 6 − 2 ( 3 − 2 k ) = 2k +1
. u n +1 = u n + 2 ، u 0 = 1 86
. v n +1 = v n + u n ، v 0 = 1 . u n +1 = 4 − u n ، u 0 = 3 82

‫ ﻭﻤﻨﻪ‬2 ‫ ﻫﻭ ﺍﻝﺤﺩ ﺍﻝﻌﺎﻡ ﻝﻤﺘﺘﺎﻝﻴﺔ ﺤﺴﺎﺒﻴﺔ ﺃﺴﺎﺴﻬﺎ‬u n (1 . u 5 = 1 ‫ ﻭ‬u 4 = 3 ، u 3 = 1 ، u 2 = 3 ، u 1 = 1 (1

. u n = 2n + 1 . u 2 n = 3 ‫ ﻭ‬u 2 n +1 = 1

‫ ؛‬v k = 1 + k 2 ‫ ﻨﻔﺭﺽ‬.v 0 = 1 + 02 ‫ ﺘﻌﻨﻲ‬p ( 0 ) (2 . u 1 = 1 ‫ ﻭ‬u 0 = 3 ‫ ﺘﻌﻨﻲ‬p ( 0 ) (2


‫ ﻝﺩﻴﻨﺎ‬. u 2 k +1 = 1 ‫ ﻭ‬u 2 k = 3 ‫ﻨﻔﺭﺽ‬
.v k +1 = v k + u k = 1 + k 2 + 2k + 1 = 1 + ( k + 1)
2

. u 2( k +1) +1 = 4 − u 2( k +1) = 1 ‫ ﻭ‬u 2( k +1) = 4 − u 2 k +1 = 3


. u n +1 = u n + 2n + 3 ، u 0 = 1 87
، n ∈ ℕ ‫ ﻭﻤﻨﻪ ﻤﻥ ﺃﺠل ﻜل‬u n +1 − u n = 2n + 3 (1 . s n = 1 + 3 + 5 + ... + ( 2n − 1) 83

.‫( ﻤﺘﺯﺍﻴﺩﺓ ﺘﻤﺎﻤﺎ‬u n ) ‫ ﺃﻱ‬u n +1 − u n > 0 . s n = n 2 . s 4 = 16 ‫ ﻭ‬s 3 = 9 ، s 2 = 4 ، s 1 = 1 (1

‫ ؛‬u k > k 2 ‫ ﻨﻔﺭﺽ‬u 0 > 0 ‫ ﺘﻌﻨﻲ‬p ( 0 ) (2


‫‪ u k +1 > u k‬ﻭﻤﻨﻪ ‪ 12 + u k +1 > 12 + u k‬ﻭﻤﻤﺎ ﺴﺒﻕ ﻜل‬ ‫‪ u k +1 = u k + 2k + 3‬ﻭﻤﻨﻪ ‪u k +1 > k 2 + 2k + 3‬‬
‫ﻭﻤﻨﻪ‬ ‫ﺍﻝﺤﺩﻭﺩ ﻤﻭﺠﺒﺔ ﺇﺫﻥ ‪12 + u k +1 > 12 + u k‬‬ ‫ﻭﻋﻠﻴﻪ ‪. u k +1 > k 2 + 2k + 1‬‬
‫‪. u k + 2 > u k +1‬‬ ‫‪ ، u 0 ∈ ]0 ;1[ 88‬ﺘﺼﺤﻴﺢ ‪. u n +1 = −u n 2 + 2u n‬‬
‫‪ u 0 = 2 91‬ﻭ ‪. u n +1 = 0, 6 u n − 1, 2‬‬ ‫) ‪ p ( 0‬ﺘﻌﻨﻲ ‪ . 0 < u 0 < 1‬ﻨﻔﺭﺽ ‪. 0 < u k < 1‬‬
‫‪ (1‬ﻤﻥ ﺃﺠل ﻜل ‪ P ( n ) ، n ∈ ℕ‬ﻫﻲ ﺍﻝﺨﺎﺼﻴﺔ ‪. u n +1 < u n‬‬ ‫ﻨﻌﺘﺒﺭ ﺍﻝﺩﺍﻝﺔ ‪ f : x ֏ − x 2 + 2x‬؛ ‪f ' ( x ) = −2x + 2‬‬
‫ﺍﻝﺨﺎﺼﻴﺔ ) ‪ P ( 0‬ﻫﻲ ‪ u 1 < u 0‬ﻭﻝﺩﻴﻨﺎ ﻤﻥ ﺘﻌﺭﻑ ﺍﻝﻤﺘﺘﺎﻝﻴﺔ‬ ‫ﻭﻤﻨﻪ ' ‪ f‬ﻤﻭﺠﺒﺔ ﺘﻤﺎﻤﺎ ﻋﻠﻰ [‪ ]0;1‬ﺃﻱ ‪ f‬ﻤﺘﺯﺍﻴﺩﺓ ﺘﻤﺎﻤﺎ‬
‫‪ u 0 = 2‬ﻭ ‪ u 1 = 0‬ﻭﻤﻨﻪ ‪ u 1 < u 0‬ﺇﺫﻥ ) ‪ P ( 0‬ﺼﺤﻴﺤﺔ ‪.‬‬ ‫ﻋﻠﻰ [‪ ]0;1‬ﻭﺒﺎﻝﺘﺎﻝﻲ )‪ f ( 0 ) < f (u k ) < f (1‬ﺃﻱ‬
‫ﻝﻴﻜﻥ ‪ k‬ﻋﺩﺩﺍ ﻁﺒﻴﻌﻴﺎ ﻜﻴﻔﻴﺎ ﻭﻨﻔﺭﺽ ‪ u k +1 < u k‬ﻭﻤﻨﻪ‬ ‫‪. 0 < u k +1 < 1‬‬
‫‪ 0, 6 u k +1 < 0, 6 u k‬ﺃﻱ ‪0, 6 u k +1 − 1, 2 < 0, 6 u k − 1, 2‬‬ ‫‪. u n +1 = 2 + u n ، u 0 = 1 89‬‬
‫ﻭﻤﻌﻨﺎﻩ ‪. u k + 2 < u k +1‬‬ ‫* ) ‪ p ( 0‬ﺘﻌﻨﻲ ‪ . 0 ≤ u 0 ≤ 2‬ﻨﻌﺘﺒﺭ ﺍﻝﺩﺍﻝﺔ ‪ f‬ﺍﻝﻤﻌﺭﻓﺔ ﻋﻠﻰ‬
‫‪ (2‬ﻤﻥ ﺃﺠل ﻜل ‪ P ' ( n ) ، n ∈ ℕ‬ﻫﻲ ﺍﻝﺨﺎﺼﻴﺔ ‪. u n > −3‬‬ ‫‪1‬‬
‫= ) ‪f '(x‬‬ ‫]‪ [ 0; 2‬ـ ‪ f ( x ) = 2 + x :‬؛‬
‫) ‪ P ' ( 0‬ﺘﻌﻨﻲ ‪ u 0 > −3‬ﻭﻫﺫﺍ ﺼﺤﻴﺢ ﻷﻥ ‪. u 0 = 2‬‬ ‫‪2 2+x‬‬

‫ﻝﻴﻜﻥ ‪ k‬ﻋﺩﺩﺍ ﻁﺒﻴﻌﻴﺎ ﻜﻴﻔﻴﺎ ﻭﻨﻔﺭﺽ ‪ u k > −3‬ﻤﻌﻨﺎﻩ‬ ‫' ‪ f‬ﻤﻭﺠﺒﺔ ﻋﻠﻰ ]‪ [ 0; 2‬ﻭﻤﻨﻪ ‪ f‬ﻤﺘﺯﺍﻴﺩﺓ ﺘﻤﺎﻤﺎ ﻋﻠﻰ ]‪[ 0; 2‬‬
‫‪ 0, 6 u k > −1,8‬ﺃﻱ ‪0, 6 u k − 1, 2 > −1,8 − 1, 2‬‬ ‫ﻭﺒﺎﻝﺘﺎﻝﻲ ﺇﺫﺍ ﻜﺎﻥ ‪ 0 ≤ u k ≤ 2‬ﻓﺈﻥ ) ‪f ( 0 ) ≤ f (u k ) ≤ f ( 2‬‬

‫ﻭﺒﺎﺴﺘﻌﻤﺎل ﺘﻌﺭﻴﻑ ﺍﻝﻤﺘﺘﺎﻝﻴﺔ ﻴﻜﻭﻥ ‪. u k +1 > −3‬‬ ‫ﻭﻤﻨﻪ ‪0 ≤ u k +1 ≤ 2‬‬ ‫ﺃﻱ ‪2 ≤ u k +1 ≤ 2‬‬
‫‪u +1‬‬ ‫* ) ‪ p ( n‬ﻫﻲ ﺍﻝﺨﺎﺼﻴﺔ ‪. u n +1 > u n‬‬
‫‪. u n +1 = n‬‬ ‫‪، u 0 = 1 92‬‬
‫‪un + 3‬‬ ‫) ‪ p ( 0‬ﺘﻌﻨﻲ ‪ u 1 > u 0‬ﺃﻱ ‪. 3 > 1‬ﻨﻔﺭﺽ ‪ u k +1 > u k‬ﻤﻌﻨﺎﻩ‬
‫‪ p ( 0 ) (1‬ﺘﻌﻨﻲ ‪ 0 ≤ u 0 ≤ 1‬ﻭﻫﻲ ﺼﺤﻴﺤﺔ‪.‬‬ ‫‪ u k +1 + 2 > u k + 2‬ﻭﺒﻤﺎ ﺃﻥ ‪ u n ≥ 0‬ﻓﺈﻥ ‪u k + 2 > u k +1‬‬
‫‪u +1‬‬
‫‪ u k +1 = k‬ﻭﻤﻨﻪ ‪. u k +1 ≥ 0‬‬ ‫ﻨﻔﺭﺽ ‪ 0 ≤ u k ≤ 1‬؛ ﻝﺩﻴﻨﺎ‬ ‫‪. u n +1 = 12 + u n ، u 0 = 0 90‬‬
‫‪uk + 3‬‬
‫‪−2‬‬
‫‪ (2‬ﺒﺭﻫﻥ ﺃﻨﻪ ﻤﻥ ﺃﺠل ﻜل ﻋﺩﺩ ﻁﺒﻴﻌﻲ ‪. 0 ≤ u n < 4 , n‬‬
‫= ‪ u k +1 − 1‬ﻭﻤﻨﻪ ‪ u k +1 − 1 ≤ 0‬ﺃﻱ ‪. 0 ≤ u k +1 ≤ 1‬‬
‫‪uk + 3‬‬ ‫‪ (3‬ﺃﺩﺭﺱ ﺍﺘﺠﺎﻩ ﺘﻐﻴﺭ ﺍﻝﻤﺘﺘﺎﻝﻴﺔ ) ‪. (u n‬‬
‫‪ p ' ( n ) (2‬ﻫﻲ ﺍﻝﺨﺎﺼﻴﺔ ‪. u n +1 < u n‬‬ ‫‪, u 2 = 12 + 12 ≃ 3.93 ، u1 = 12 ≃ 3.464 (1‬‬
‫‪1‬‬
‫‪ .‬ﻨﻌﺘﺒﺭ ﺍﻝﺩﺍﻝﺔ‬ ‫) ‪ p ' ( 0‬ﺘﻌﻨﻲ ‪ u 1 < u 0‬ﺃﻱ ‪< 1‬‬ ‫‪ ، u 3 = 12 + 12 + 12 ≃ 3,991‬ﻫﻲ ﺘﻨﺘﻤﻲ ﺇﻝﻰ‬
‫‪2‬‬
‫‪2‬‬ ‫‪x +1‬‬ ‫ﺍﻝﻤﺠﺎل [‪. [ 0; 4‬‬
‫= ) ‪ f ' ( x‬ﻭﻤﻨﻪ ‪f ' ( x ) > 0‬‬ ‫֏ ‪f :x‬؛‬
‫)‪( x + 3‬‬ ‫‪x +3‬‬ ‫‪ (2‬ﺍﻝﺨﺎﺼﻴﺔ ) ‪ P ( 0‬ﻫﻲ ‪ 0 ≤ u 0 < 4‬ﻭﻫﺫﺍ ﺼﺤﻴﺢ‬
‫ﺇﺫﻥ ‪ f‬ﻤﺘﺯﺍﻴﺩﺓ ﺘﻤﺎﻤﺎ ﻋﻠﻰ ]‪ [ 0;1‬ﻭﺒﺎﻝﺘﺎﻝﻲ ﺇﺫﺍ ﻜﺎﻥ‬ ‫ﻝﻴﻜﻥ ‪ k‬ﻋﺩﺩﺍ ﻁﺒﻴﻌﻴﺎ ﻜﻴﻔﻴﺎ ﻭﻨﻔﺘﺭﺽ ‪ 0 ≤ u k < 4‬ﻭﻤﻨﻪ‬
‫‪ u k +1 < u k‬ﻓﺈﻥ ) ‪ f (u k +1 ) < f (u k‬ﺃﻱ ‪. u k + 2 < u k +1‬‬ ‫‪ 12 ≤ 12 + u k < 16‬ﺃﻱ ‪12 ≤ 12 + u k < 4‬‬
‫‪ π‬‬ ‫ﻭﻤﻨﻪ ‪ 0 ≤ 12 + u k < 4‬ﺇﺫﻥ ‪. 0 ≤ u k +1 < 4‬‬
‫‪ θ 93‬ﻋﺩﺩ ﺤﻘﻴﻘﻲ ﻤﻥ ﺍﻝﻤﺠﺎل ‪.  0; ‬‬
‫‪ 2‬‬
‫‪ (3‬ﻨﻼﺤﻅ ﺃﻥ ‪ u 0 < u 1 < u 2 < u 3‬ﻝﻨﺒﺭﻫﻥ ﺃﻨﻪ ﻤﻥ ﺃﺠل ﻜل‬
‫‪. u n +1 = 2 + u n ، u 0 = 2 cos θ‬‬
‫‪ u n +1 > u n ، n ∈ ℕ‬ﻭﻫﺫﺍ ﺒﺎﺴﺘﻌﻤﺎل ﺍﻻﺴﺘﺩﻻل ﺒﺎﻝﺭﺍﺠﻊ‬
‫‪θ‬‬ ‫‪θ‬‬
‫‪. u1 = 2 (1 + cos θ ) = 4 cos‬‬ ‫‪2‬‬
‫‪= 2 cos‬‬ ‫‪ (1‬أ ـ‬ ‫ﺍﻝﺨﺎﺼﻴﺔ ) ‪ P ( 0‬ﻫﻲ ‪ u 1 > u 0‬ﻭﻫﺫﺍ ﺼﺤﻴﺢ ﻷﻥ ‪u 0 = 0‬‬
‫‪2‬‬ ‫‪2‬‬
‫ﻭ ‪ . u1 = 12‬ﻝﻴﻜﻥ ‪ k‬ﻋﺩﺩﺍ ﻁﺒﻴﻌﻴﺎ ﻜﻴﻔﻴﺎ ﻭﻨﻔﺭﺽ‬
‫ﻝﻴﻜﻥ ‪ k‬ﻋﺩﺩﺍ ﻁﺒﻴﻌﻴﺎ ﻜﻴﻔﻴﺎ ﻭﻨﻔﺭﺽ ‪ ، u k > 2‬ﻝﺩﻴﻨﺎ‬ ‫‪‬‬ ‫‪θ‬‬ ‫‪θ‬‬
‫‪. u 2 = 2 1 + cos  = 2 cos‬‬
‫‪ f‬ﻭ ‪ f (u k ) = u k +1‬ﻤﻥ ﻓﺭﻀﻴﺔ ﺍﻝﺘﺭﺍﺠﻊ‬ ‫= )‪( 2‬‬ ‫‪2‬‬ ‫‪‬‬ ‫‪2‬‬ ‫‪4‬‬

‫‪ u k > 2‬ﻭﺒﻤﺎ ﺃﻥ ‪ f‬ﻤﺘﺯﺍﻴﺩﺓ ﺘﻤﺎﻤﺎ ﻋﻠﻰ ‪ 2 ; + ∞ ‬‬ ‫ب ـ ) ‪ p ( 0‬ﻫﻲ ‪ u 0 > 0‬ﺃﻱ ‪ 2 cos θ > 0‬ﻭﻫﺫﺍ ﺼﺤﻴﺢ‬
‫‪‬‬ ‫‪‬‬
‫‪ π‬‬
‫‪ f (u k ) > f‬ﺃﻱ ‪. u k +1 > 2‬‬ ‫ﻴﻨﺘﺞ ) ‪( 2‬‬ ‫ﻷﻥ ‪ . θ ∈  0; ‬ﻨﻔﺭﺽ ‪ u k > 0‬ﺇﺫﻥ ‪2 + u k > 2‬‬
‫‪ 2‬‬
‫‪2 −un 2‬‬ ‫ﻭﻤﻨﻪ ‪ 2 + u k > 2‬ﺃﻱ ‪ u k +1 > 2‬ﻭﻤﻨﻪ ‪. u k +1 > 0‬‬
‫ﻝﺩﻴﻨﺎ ‪ 2 − u n 2 < 0‬ﻭ ‪ u n > 0‬ﺇﺫﻥ ‪< 0‬‬
‫‪2u n‬‬ ‫‪θ‬‬
‫‪ u n = 2 cos‬؛‬ ‫‪ p ' ( n ) (2‬ﻫﻲ ﺍﻝﺨﺎﺼﻴﺔ‬
‫ﺃﻱ ‪ u n +1 − u n < 0‬ﻭﺒﺎﻝﺘﺎﻝﻲ ﺍﻝﻤﺘﺘﺎﻝﻴﺔ ) ‪ (u n‬ﻤﺘﻨﺎﻗﺼﺔ ﺘﻤﺎﻤﺎ‪.‬‬ ‫‪2n‬‬
‫‪θ‬‬
‫‪ 95‬ﻤﻥ ﺃﺠل ﻜل ∗‪. u n = n × 2n −1 : n ∈ ℕ‬‬ ‫‪. u 0 = 2 cos‬‬ ‫) ‪ p ' ( 0‬ﺘﻌﻨﻲ ‪= 2 cos θ‬‬
‫‪20‬‬
‫)‪ p (1‬ﺘﻌﻨﻲ ‪. u1 = 1 + (1 − 1) 2‬‬
‫‪1‬‬

‫‪‬‬ ‫‪θ ‬‬ ‫‪θ‬‬


‫ﻨﻔﺭﺽ ﺃﻥ ‪ u1 + u 2 + ... + u k = 1 + ( k − 1) 2k‬؛‬ ‫ﻨﻔﺭﺽ ‪ u k +1 = 2 1 + cos k  ، u k = 2 cos k‬ﺃﻱ‬
‫‪‬‬ ‫‪2 ‬‬ ‫‪2‬‬
‫‪u1 + u 2 + ... + u k + u k +1 = 1 + ( k − 1) 2k + ( k + 1) 2k‬‬ ‫‪θ‬‬ ‫‪θ‬‬
‫‪k +1‬‬ ‫‪u k +1 = 4 cos 2‬‬ ‫‪= 2 cos‬‬
‫ﺃﻱ ‪. u1 + u 2 + ... + u k + u k +1 = 1 + k 2‬‬ ‫‪2 ×2‬‬
‫‪k‬‬
‫‪2k +1‬‬
‫‪1‬‬
‫= ‪. un‬‬ ‫‪ 96‬ﻤﻥ ﺃﺠل ﻜل ∗‪: n ∈ ℕ‬‬
‫)‪n ( n + 1‬‬ ‫‪1‬‬ ‫‪2 ‬‬
‫‪ u 0 = 5 94‬ﻭ ‪. u n +1 =  u n + ‬‬
‫‪1‬‬ ‫‪1‬‬
‫= ‪. u1‬‬ ‫‪ (1‬ﻝﺩﻴﻨﺎ = ‪ u 1‬ﻭ )‪ p (1‬ﺘﻌﻨﻲ‬ ‫‪2‬‬ ‫‪un ‬‬
‫‪1+1‬‬ ‫‪2‬‬
‫‪k‬‬
‫= ‪ u 1 + u 2 + ... + u k‬؛‬ ‫ﻨﻔﺭﺽ‬ ‫=‪Y‬‬ ‫‪Mode‬‬
‫‪k +1‬‬
‫‪k‬‬ ‫‪1‬‬
‫= ‪u1 + u 2 + ... + u k + u k +1‬‬ ‫‪+‬‬
‫) ‪k + 1 ( k + 1)( k + 2‬‬
‫‪GRAPH‬‬ ‫‪2nd‬‬
‫‪k 2 + 2k + 1‬‬ ‫‪k +1‬‬
‫= ‪. u 1 + u 2 + ... + u k +1‬‬ ‫=‬ ‫ﺃﻱ‬
‫‪( k + 1)( k + 2 ) k + 2‬‬
‫‪ (2‬ﻨﺴﻤﻲ ‪ S‬ﺍﻝﻤﺠﻤﻭﻉ‬ ‫‪ (2‬ﻴﺒﺩﻭ ﺃﻥ ﺍﻝﻤﺘﺘﺎﻝﻴﺔ ) ‪ (u n‬ﻤﺘﻨﺎﻗﺼﺔ ‪.‬‬
‫‪1‬‬ ‫‪1‬‬ ‫‪1‬‬
‫‪.‬‬ ‫‪+‬‬ ‫‪+ ... +‬‬ ‫‪1‬‬ ‫‪2‬‬
‫‪1427 × 1428 1428 × 1429‬‬ ‫‪2007 × 2008‬‬ ‫ﻨﻌﺘﺒﺭ ﺍﻝﺩﺍﻝﺔ ‪ f : x ֏  x + ‬ﻝﺩﻴﻨﺎ ﻤﻥ ﺃﺠل ﻜل‬
‫‪2‬‬ ‫‪x ‬‬
‫‪1‬‬ ‫‪1‬‬ ‫‪1‬‬
‫= ‪T‬‬ ‫‪+‬‬ ‫‪+ ... +‬‬ ‫ﻭﻨﻀﻊ‬ ‫‪1x −2‬‬
‫‪2‬‬
‫‪1× 2 2 × 3‬‬ ‫‪2007 × 2008‬‬ ‫‪f '(x ) = ‬‬ ‫ﻋﺩﺩ [∞ ‪ ، x ∈ ]0; +‬‬
‫‪1‬‬ ‫‪1‬‬ ‫‪1‬‬ ‫‪2 x 2 ‬‬
‫=‪t‬‬ ‫‪+‬‬ ‫‪+ ... +‬‬ ‫ﻭ‬
‫‪1× 2 2 × 3‬‬ ‫‪1426 × 1427‬‬ ‫ﻭﻤﻨﻪ ﻤﻥ ﺃﺠل ﻜل ‪ x ∈  2 ; + ∞  f ' ( x ) ≥ 0‬ﻭﺒﺎﻝﺘﺎﻝﻲ‬
‫‪2007 1426‬‬ ‫‪581‬‬
‫= ‪S =T −t‬‬ ‫‪−‬‬ ‫=‬ ‫ﺇﺫﻥ‬
‫‪2008 1427 2865416‬‬ ‫ﺍﻝﺩﺍﻝﺔ ‪ f‬ﻤﺘﺯﺍﻴﺩﺓ ﺘﻤﺎﻤﺎ ﻋﻠﻰ ﺍﻝﻤﺠﺎل ‪.  2 ; + ∞ ‬‬

‫)‪(2 + 3‬‬
‫‪0‬‬
‫‪ 97‬ﺍﻝﺨﺎﺼﻴﺔ ﺍﻻﺒﺘﺩﺍﺌﻴﺔ ﻫﻲ ‪= p 0 + q 0 3‬‬ ‫‪1‬‬ ‫‪2 ‬‬ ‫‪2 −un2‬‬
‫= ‪u n +1 − u n =  u n +  − u n‬‬
‫‪2‬‬ ‫‪un ‬‬ ‫‪2u n‬‬
‫ﻭﻫﻲ ﺼﺤﻴﺤﺔ ﺒﺄﺨﺫ ‪ p 0 = 1‬ﻭ ‪. q 0 = 0‬‬
‫ﻤﻥ ﺃﺠل ﻜل ‪ P ( n ) ، n ∈ ℕ‬ﻫﻲ ﺍﻝﺨﺎﺼﻴﺔ " ‪." u n > 2‬‬
‫ﻨﻔﺭﺽ ﺃﻨﻪ ﻤﻥ ﺃﺠل ‪ ، k ∈ ℕ‬ﻴﻭﺠﺩ ﻋﺩﺩﺍﻥ ﻁﺒﻴﻌﻴﺎﻥ ‪p k‬‬
‫) ‪ P ( 0‬ﻫﻲ ‪ u 0 > 2‬ﻭﻫﺫﺍ ﺼﺤﻴﺢ ﻷﻥ ‪. u 0 = 5‬‬
‫(‬ ‫)‬
‫‪k‬‬
‫‪. 2+ 3‬‬ ‫ﻭ ‪ q k‬ﺤﻴﺙ ‪= p k + q k 3‬‬
( ) ( ) ( )( p )
2 k +1
1 4+3− 2 3 2− 3 ‫ ﻤﻌﻨﺎﻩ‬2 + 3 = 2+ 3 k + qk 3
4−2 3− = =
(2 + 3)
k +1
= ( 2 p k + 3q k ) + ( p k + 2q k )
2 2 2
3
( ) ( )
+ +
. u k +1 =  2 + 3  ‫ﻭﻤﻨﻪ‬
1 k 1 k 1
+ 2− 3
2  ‫ ﻭﻫﻤﺎ‬q k +1 = p k + 2q k ‫ ﻭ‬p k +1 = 2 p k + 3q k ‫ﺒﻭﻀﻊ‬
. u n +1 = u n + 1 ‫ ﻭ‬u 0 = 2 100
( )
k +1
. 2+ 3 = p k +1 + q k +1 3 ‫ﻋﺩﺩﺍﻥ ﻁﺒﻴﻌﻴﺎﻥ ﻓﻴﻜﻭﻥ‬
. u 0 > −1 ‫أ ـ ﺍﻝﺨﺎﺼﻴﺔ ﺍﻻﺒﺘﺩﺍﺌﻴﺔ‬
. u n + 2 = 2u n +1 − u n ، u 2 = 3 ، u 1 = 1 (1 98
u k + 1 > 0 ‫ ﻭﻤﻨﻪ‬u k + 1 > 0 ‫ ﻤﻌﻨﺎﻩ‬u k > −1 ‫ﻨﻔﺭﺽ‬
. u n = 2n − 1 . u 5 = 9 ‫ ﻭ‬u 4 = 7 , u 3 = 5 ‫ﺃ ـ‬
. u k +1 > −1 ‫ ﻭﺒﺎﻝﺘﺎﻝﻲ‬u k +1 > 0 ‫ﺃﻱ‬
. u 1 = 2 × 1 − 1 = 1 ‫ﺏ ـ ﺍﻝﺨﺎﺼﻴﺔ ﺍﻻﺒﺘﺩﺍﺌﻴﺔ ﺼﺤﻴﺤﺔ ﻷﻥ‬
. u 1 < u 0 ‫ ﻭﻤﻨﻪ‬u1 = 3 ‫ ﻭ‬u 0 = 2 ‫ب ـ‬
‫ ؛‬u k = 2k − 1 ‫ﻨﻔﺭﺽ‬
‫ ﺇﺫﻥ‬0 < u k +1 + 1 < u k + 1 ‫ ﻭﻤﻨﻪ‬u k +1 < u k ‫ﻨﻔﺭﺽ‬ u k +1 = 2u k − u k −1 = 4k − 2 − 2k + 3 = 2k + 1
. u k + 2 < u k +1 ‫ ﺃﻱ‬u k +1 + 1 < u k + 1 2
. v n + 2 = 5 v n +1 − 6 v n ‫ ﻭ‬v 1 = 1 ، v 0 = (2
1+ 5 1+ 5 5
.2 ≥ ‫ ﺃﻱ‬u 0 ≥ ‫ـ ـ ﺍﻝﺨﺎﺼﻴﺔ ﺍﻻﺒﺘﺩﺍﺌﻴﺔ‬4
2 2 2 +3
0 0
2
.v 0 = = ‫ﺍﻝﺨﺎﺼﻴﺔ ﺍﻻﺒﺘﺩﺍﺌﻴﺔ ﺼﺤﻴﺤﺔ ﻷﻥ‬
3+ 5 1+ 5 5 5
‫ ﺇﺫﻥ‬u k + 1 ≥ ‫ ﻭﻤﻨﻪ‬u k ≥ ‫ﻨﻔﺭﺽ‬
2 2 2k + 3k
v k +1 = 5 v k − 6 v k −1 ‫ ؛‬v k = ‫ﻨﻔﺭﺽ‬
1+ 5 3+ 5 5
. u k +1 ≥ ‫ﺃﻱ‬ uk +1 ≥
‫ ؛‬v k +1 = 2 k + 3k − ( 2k −1 + 3k −1 )
6
2 2
5
( )
2

3+ 5 6+2 5 1+ 5 1+ 5 3 2 1 1
= = = ‫ﻷﻥ‬ .v k +1 = 2 k + 3k − 2k − 3k = 2 k +1 + 3k +1
2 4 2 2 5 5 5 5
u +1 u +4 . u n +1 + u n −1 = 4 u n ، u 1 = 2 ، u 0 = 1 99
. vn = n ، u n +1 = n ، u 0 = 1 101
un − 4 un − 2
( ) ( )
. u 0 =  2 + 3 + 2 − 3  ‫ﺍﻝﺨﺎﺼﻴﺔ ﺍﻻﺒﺘﺩﺍﺌﻴﺔ‬
1 0 0

u k ≠ 4 ‫ ﻨﻔﺭﺽ‬. u 0 ≠ 4 ‫ ﻭﻤﻨﻪ‬u 0 = 1 (1 2  

( ) (
‫ ؛‬u k =  2 + 3 + 2 − 3  ‫ﻨﻔﺭﺽ‬
1
)
k k
u +4
u k = 4 ‫ ﻤﻌﻨﺎﻩ‬k = 4 ‫ ﺃﻱ‬u k +1 = 4 ‫ﻭﻨﻔﺘﺭﺽ‬ 2  
uk − 2
‫ ﻭﻤﻌﻨﺎﻩ‬u k +1 = 4 u k − u k −1
. u n ≠ 4 ، n ∈ ℕ ‫ ﺃﻱ ﻜل‬u k +1 ≠ 4 ‫ﻭﻫﺫﺍ ﺘﻨﺎﻗﺽ ﺇﺫﻥ‬
(
u k +1 = 2  2 + 3 ) + ( 2 − 3 ) 
k k
u + 1 2u n + 2 u −2 
‫؛‬v n +1 = n +1 = × n
u n +1 − 4 u n − 2 −3u n + 12
(2 + 3) + (2 − 3)
k −1 k −1

2 u +1 2
. v n +1 = − × n = − vn −
3 un − 4 3 2

( )  1
n n +1 k −1
 2  2 u k +1 = 2 + 3 4+ 2 3 − +
. v n = v 0  −  =  −  (2  2
 3  3
(2 − 3)  1
k −1
u +1
u n (v n − 1) = 4v n + 1 ‫ ﻤﻌﻨﺎﻩ‬v n = n 4−2 3 − 
un − 4  2

( )
2
v n ≠ 1 ، n ∈ ℕ ‫ﻝﻨﺒﺭﻫﻥ ﺃﻨﻪ ﻤﻥ ﺃﺠل ﻜل‬ 1 4+3+ 2 3 2+ 3
4+2 3 − = =
2 2 2
: ‫ ﻭﻝﺩﻴﻨﺎ‬a ≥ 4 ‫ ﻭﻋﻠﻴﻪ‬k = 5a + 7 ‫ ﻓﺈﻥ‬b = 1 ‫• ﺇﺫﺍ ﻜﺎﻥ‬ ‫ ﺇﺫﺍ ﻜﺎﻥ‬. v 0 = −
2
‫ ﻷﻥ‬v 0 ≠ 1 ‫ﻝﺩﻴﻨﺎ ﺍﻝﺨﺎﺼﻴﺔ ﺍﻻﺒﺘﺩﺍﺌﻴﺔ‬
. k + 1 = 5a − 20 + 28 = 5 ( a − 4 ) + 7 × 4 3
n +2 n +1
v k +1 ≠ 1 ‫ ﻭﻤﻨﻪ‬ − 2   2
k + 1 = 5a + 7b + 15 − 14 ‫ ﻓﺈﻥ‬b ≥ 2 ‫• ﺇﺫﺍ ﻜﺎﻥ‬ ≠ 1 ‫ ﻓﺈﻥ‬ −  ≠ 1 ‫ ﺃﻱ‬v k ≠ 1
 3  3
. k + 1 = 5 ( a + 3) + 7 (b − 2 ) ‫ﺃﻱ‬ 4v n + 1   2 
n +1
  2 n +1 
.u n = =  4  −  + 1 /  −  − 1
 n +1 1 v n − 1   3    3  
. u n +1 =   u n ، u 1 = (1 105
 2n  2
x3 x2 x
5 4 1 3 1 . p (x ) =
− + 102
. u5 = ، u4 = = ، u3 = ، u2 = ‫ﺃ ـ‬ 3 2 6
32 16 4 8 2
x3 1 x2
k 1 p ( x + 1) − p ( x ) =  +x 2 +x + − −
‫ ؛‬u k = k ‫ ﻨﻔﺭﺽ‬. u 1 = 1 ‫ﺏ ـ ﺍﻝﺨﺎﺼﻴﺔ ﺍﻻﺒﺘﺩﺍﺌﻴﺔ ﻫﻲ‬  3 3 2
2 2 (1
 k + 1   k +1  k k +1 1 x +1  x 3 x 2 x 
. u k +1 =  u k =   k = k +1 x− + − − + =x2
 2k   2k  2 2 2 6   3 2 6

. v n +1 = 
 n +1  1 ∗
، k ∈ ℕ (2 . p ( 0 ) ∈ ℕ ‫ ﺇﺫﻥ‬p ( 0 ) = 0 ‫( ﺍﻝﺨﺎﺼﻴﺔ ﺍﻻﺒﺘﺩﺍﺌﻴﺔ ﻫﻲ‬2
v n ، v 1 =
 kn  k
‫ ﻭﻤﻨﻪ‬p ( k + 1) = p ( k ) + k 2 ‫ ﻝﺩﻴﻨﺎ‬p ( k ) ∈ ℕ ‫ﻨﻔﺭﺽ‬
n 3 2 1
.v n = n ، ... v 3 = 3 ، v 2 = 2 ، v 1 = . p ( k + 1) ∈ ℕ
k k k k
r 1
، v r = r ‫ ﻨﻔﺭﺽ‬. v 1 = 1 ‫ﺍﻝﺨﺎﺼﻴﺔ ﺍﻻﺒﺘﺩﺍﺌﻴﺔ ﻫﻲ‬ ‫ ﻭﻫﺫﺍ ﺼﺤﻴﺢ ﻷﻥ‬p (1) = 02 ‫( ﺍﻝﺨﺎﺼﻴﺔ ﺍﻻﺒﺘﺩﺍﺌﻴﺔ ﻫﻲ‬3
k k 13 12 1 2 − 3 + 1
 r +1   r +1  r r +1 . p (1) = − + = =0
.v r +1 =  v n =   r = r +1 3 2 6 6
 kr   kr  k k
‫ ؛‬p ( k + 1) = 12 + 22 + ... + k 2 ‫ﻨﻔﺭﺽ‬
. %-‫د‬$ %
""+ ‫رب‬. ‫ ـ‬3
‫ ﻤﻌﻨﺎﻩ‬p ( n + 2 ) = p ( n + 1) + ( n + 1) ‫ﻝﺩﻴﻨﺎ‬
2

. lim ln 1 +
1 
 = 0 (2. nlim e 1− n = 0 (1 106 . p ( n + 2 ) = 12 + 2 2 + ... + n 2 + ( n + 1)
2
n →+∞
 2n + 3  →+∞

−1
n . u n +1 = ، u 1 = 0 103
. lim ( n + 2 ) e − n = lim n + 2e − n = 0 (3 un − 2
n →+∞ n →+∞ e
n −1
. lim ln ( 3 + e 2− n ) = ln 3 (4
3 2 1
un = ‫ ﻴﺒﺩﻭ ﺃﻥ‬u 4 = ، u 3 = ، u 2 = ‫ﻝﺩﻴﻨﺎ‬
n →+∞ n 4 3 2
−n
e −1 en −6 1 . ‫ﻝﻨﺒﺭﻫﻥ ﺒﺎﺴﺘﻌﻤﺎل ﺍﻝﺘﺭﺍﺠﻊ‬
. lim − n = −1 (6 . lim n = (5
n →+∞ 2e +1 n →+∞ 2e + 1 2 1 −1
.‫ ﻭﻫﻲ ﺼﺤﻴﺤﺔ‬u 1 = = 0 ‫ﺍﻝﺨﺎﺼﻴﺔ ﺍﻻﺒﺘﺩﺍﺌﻴﺔ ﻫﻲ‬
e −3
n
1
. lim ln  n  = ln1 = 0 (7 −1 −1 k −1
 e +1 
k
n →+∞
u k +1 = = = ‫؛‬u k = ‫ﻨﻔﺭﺽ‬
u k − 2 k −1 − 2 k +1 k
 en +2  k
. lim ln  2 n  = 0 (8
n →+∞
 e +1  . u 2006 =
2005
2006
 2 
. u n = n 2  3 + − 3  (1 107 n = 5 × 2 + 7 × 2 ‫ ﻓﺈﻥ‬n = 24 ‫ ﺇﺫﺍ ﻜﺎﻥ‬104
 n +1 
k = 5a + 7b ‫ ﻓﻴﻜﻭﻥ‬k ≥ 24 ‫ﻨﻔﺭﺽ ﻤﻥ ﺃﺠل‬
n 2
lim u n = lim = +∞ ‫ ؛‬u n = 2n 1
: ‫ ﻭﻝﺩﻴﻨﺎ‬a ≥ 5 ‫ ﻭﻋﻠﻴﻪ‬k = 5a ‫ ﻓﺈﻥ‬b = 0 ‫• ﺇﺫﺍ ﻜﺎﻥ‬
n →+∞ n →+∞ 3 n +1 2
3+ + 3
n +1 . k + 1 = 5a − 20 + 21 = 5 ( a − 4 ) + 7 × 3
1 1 −1
. ‫( ﻫﻨﺩﺴﻴﺔ‬v n ) ‫ ﻭﻤﻨﻪ‬v n +1 = (v n − 3) + 1 = v n . u n = 3n 2 − 1 − 3n = (2
3 3 3n − 1 + 3n
2

n n
1 1
‫ ﺒﻤﺎ‬u n = v n − 3 = 5   − 3 ‫ ﻭﻤﻨﻪ‬v n = 5   (2 . lim u n = 0
n →+∞
3  3
n n
1
n . un = − (3
. lim u n = −3 ‫ ﻓﺈﻥ‬lim   = 0 ‫ﺃﻥ‬ n +2 n +1
n →+∞
 
n →+∞ 3

3 15 3  1 
n un =
n
n 2 + 3n + 2
( n +1 − n + 2 )
. lim s n = v 0 = ‫ ﻭﻤﻨﻪ‬s n = v 0 1 −   
n →+∞ 2 2 2   3   −1
lim u n = 0 ، u n =
lim t n = −∞ ‫ ﻭﻤﻨﻪ‬t n = s n − 3 ( n + 1) ( )
n →+∞
3 2
n →+∞
1+ + n +1 + n + 2
n n2
n +1
2
. lim u n = lim n = +∞ ‫ ؛‬u n = 111 ، u n = 3n − 9n + 1 =
2
−1
(4
n +1
( )
n →+∞ n →+∞
2
n +5 n 2 + 5 3n + 9n 2 + 1
n2 un n 2 +1
. lim v n = lim 2 = 1 ‫ ؛‬v n = =
n →+∞ n →+∞ n n n2 + n . lim u n = 0
n →+∞
−n 1− n
. lim w n = lim = −1 ‫ ؛‬w n = u n − n = 2 2n 2
n
n →+∞ n →+∞ n n +1 . −1 < < 1 ‫ ﻷﻥ‬lim n = lim   = 0 ‫أ ـ‬ 108
5 n →+∞ 5 n →+∞
5
1 v −1 n −1
. lim t n = lim = 0 ‫ ؛‬tn = n = n
w n − 1 2n 2 . 3,01 > 1 ‫ ﻷﻥ‬lim n = lim 
3, 01 
n
n →+∞ n →+∞ 2 n 3, 01
 = +∞ ‫ب ـ‬
3 n →+∞ 3 n →+∞
 3 
3n 2 − 4
. lim u n = lim = +∞ 112 1 1 1
n →+∞ n + 1
n →+∞
‫ ؛‬u n = 1 + + 2 + ... + n ‫ـ ـ‬
u 5 5 5
. lim v n = lim n = 3 5 5  1 
n +1

n →+∞ n →+∞ n
. lim u n = ‫ ﻭﻤﻨﻪ‬u n = −   − 1
−3n − 4
n →+∞ 4 4  5  
. lim w n = lim (u n − 3n ) = lim = −3
n →+∞ n →+∞ n →+∞ n + 1 1 un
. v n = ‫ ﻭ‬u n +1 = ، u 0 = 2 109
1 un 3u n + 1
. u n = ، n ‫ ﻤﻥ ﺃﺠل ﻜل ﻋﺩﺩ ﻁﺒﻴﻌﻲ ﻏﻴﺭ ﻤﻌﺩﻭﻡ‬113
. u 0 > 0 ‫ ﺘﻌﻨﻲ‬p ( 0 ) ‫( ﺍﺴﺘﻌﻤﺎل ﺍﻝﺘﺭﺍﺠﻊ ؛‬1
n!
1 1 1
‫ ؛‬u4 = ‫ ؛‬u 3 = ‫ ؛‬u 2 = ‫ ؛‬u 1 = 1 (1 . u k +1 > 0 ‫ ﻭﻤﻨﻪ‬3u k + 1 > 0 ‫ ﻓﺈﻥ‬u k > 0 ‫ﺇﺫﺍ ﻜﺎﻨﺕ‬
24 6 2
. u6 =
1
‫ ؛‬u5 =
1 1 3u n + 1
. v n +1 = = = 3 + v n ، n ∈ ℕ ‫( ﻝﻴﻜﻥ‬2
720 120 u n +1 un
n ! ≥ n ‫ ؛‬n ‫( ﻤﻥ ﺃﺠل ﻜل ﻋﺩﺩ ﻁﺒﻴﻌﻲ ﻏﻴﺭ ﻤﻌﺩﻭﻡ‬2 2 1
. lim u n = 0 ‫ ﺇﺫﻥ‬u n = ‫ ﻭﻤﻨﻪ‬v n = 3n + (3
1 1 n →+∞ 6n + 1 2
. lim u n = 0 ‫ ﻓﺈﻥ‬lim = 0 ‫ ؛ ﺒﻤﺎ ﺃﻥ‬0 < u n ≤ ‫ﻭﻤﻨﻪ‬
n →+∞ n →+∞ n n 1
. v n = u n + 3 ‫ ﻭ‬u n +1 = u n − 2 ، u 0 = 2 110
cos ( 3n − π ) 3
. un = ، n ∈ ℕ∗ ‫ ﻤﻥ ﺃﺠل ﻜل‬114
n . t n = u 0 + u 1 + ... + u n ‫ ﻭ‬s n = v 0 + v 1 + ... + v n
1
‫ ﻤﻌﻨﺎﻩ‬v n +1 = u n +1 + 3 = u n + 1 ، n ∈ ℕ ‫( ﻝﻴﻜﻥ‬1
3
‫‪ lim u n +1 = lim 1 +‬ﺇﺫﻥ‬
‫‪1‬‬ ‫‪1‬‬
‫ﻭﻤﻥ ﺠﻬﺔ ﺃﺨﺭﻯ ‪= 1 +‬‬ ‫ﻤﻥ ﺃﺠل ﻜل ﻋﺩﺩ ﻁﺒﻴﻌﻲ ‪−1 ≤ cos ( 3n − π ) ≤ 1 ، n‬‬
‫∞‪n →+‬‬ ‫∞‪n →+‬‬ ‫‪un‬‬ ‫‪l‬‬ ‫‪1‬‬ ‫‪1‬‬
‫‪1‬‬ ‫‪ −‬؛ ﺒﻤﺎ ﺃﻥ‬ ‫≤ ‪≤ un‬‬ ‫ﻭﻤﻨﻪ ﻤﻥ ﺃﺠل ‪، n ≠ 0‬‬
‫‪ 1 + = l‬ﻭﻤﻌﻨﺎﻩ ‪ l 2 − l − 1 = 0‬ﻭ ‪. l ≠ 0‬‬ ‫‪n‬‬ ‫‪n‬‬
‫‪l‬‬ ‫‪1‬‬ ‫‪1‬‬
‫ﻤﻤﻴﺯ ﺍﻝﻤﻌﺎﺩﻝﺔ ‪ l − l − 1 = 0‬ﻫﻭ ‪ 5‬ﻭﻤﻨﻪ ﺍﻝﻤﻌﺎﺩﻝﺔ ﺘﻘﺒل‬
‫‪2‬‬ ‫‪ lim −‬ﻓﺈﻥ ‪. lim u n = 0‬‬ ‫‪= lim‬‬ ‫‪=0‬‬
‫∞‪n →+‬‬ ‫∞‪n →+‬‬ ‫‪n n →+∞ n‬‬
‫‪1− 5‬‬ ‫‪1+ 5‬‬ ‫‪nπ‬‬
‫=" ‪l‬‬ ‫=' ‪ l‬ﻭ‬ ‫ﺤﻠﻴﻥ ﻤﺘﻤﺎﻴﺯﻴﻥ ﻫﻤﺎ‬ ‫‪. u n = n + 1 − cos‬‬ ‫‪ 115‬ﻤﻥ ﺃﺠل ﻜل ‪، n ∈ ℕ‬‬
‫‪2‬‬ ‫‪2‬‬ ‫‪5‬‬
‫ﻝﺩﻴﻨﺎ ‪ u 0 = 1‬ﻭﻤﻨﻪ ‪ u 0 > 0‬ﻭﺇﺫﺍ ﻜﺎﻥ ‪ u k > 0‬ﻤﻥ ﺃﺠل ‪k‬‬ ‫‪nπ‬‬
‫‪ −1 ≤ − cos‬ﻭﻤﻨﻪ ‪ n ≤ u n ≤ n + 2‬؛ ﺒﻤﺎ ﺃﻥ‬ ‫‪≤1‬‬
‫‪1‬‬ ‫‪5‬‬
‫‪ 1 +‬ﺃﻱ ‪u k +1 > 0‬‬ ‫ﻋﺩﺩ ﻁﺒﻴﻌﻲ ﻜﻴﻔﻲ‪ ,‬ﻓﺈﻥ ‪> 0‬‬
‫‪uk‬‬ ‫∞‪ lim n = lim n + 2 = +‬ﻓﺈﻥ ∞‪lim u n = +‬‬
‫∞‪n →+‬‬ ‫∞‪n →+‬‬ ‫∞‪n →+‬‬
‫ﻭﺤﺴﺏ ﻤﺒﺩﺃ ﺍﻝﺘﺭﺍﺠﻊ ﻴﻨﺘﺞ ﺃﻨﻪ ﻤﻥ ﺃﺠل‬ ‫‪n‬‬
‫‪n‬‬ ‫‪‬‬
‫ﻜل ﻋﺩﺩ ﻁﺒﻴﻌﻲ ‪ u n > 0 ، n‬ﺇﺫﻥ‬ ‫‪ 116‬ﻤﻥ ﺃﺠل ﻜل ∗‪. u n =  − 1 ، n ∈ ℕ‬‬
‫‪ 10 ‬‬
‫‪1+ 5‬‬
‫=‪l‬‬ ‫‪ lim u n > 0‬ﻭﺒﺎﻝﺘﺎﻝﻲ‬
‫∞‪n →+‬‬
‫‪ (1‬ﺍﻝﻘﻴﻡ ﺍﻝﻤﻘﺭﺒﺔ ﻷﺤﺩ ﻋﺸﺭ ﺍﻝﺤﺩﻭﺩ ﺍﻷﻭﻝﻰ ﻤﻥ ) ‪. (u n‬‬
‫‪2‬‬
‫‪1‬‬
‫× ‪، u 1 = 0, 57 = 57‬‬ ‫‪ 118‬ﻝﺩﻴﻨﺎ‬
‫‪100‬‬
‫‪1‬‬
‫× ‪u 2 = 0,57 + 0, 0057 = 0, 57 + 0, 57‬‬
‫‪100‬‬
‫‪ 1‬‬ ‫‪1 ‬‬ ‫‪n‬‬
‫‪. u 2 = 57 ‬‬ ‫‪+‬‬ ‫‪2 ‬‬ ‫‪ (2‬ﻝﻴﻜﻥ ‪ n‬ﻋﺩﺩ ﻁﺒﻴﻌﻲ‪ n ≥ 30 ،‬ﻤﻌﻨﺎﻩ ‪− 1 ≥ 2‬‬
‫‪ 100 100 ‬‬ ‫‪10‬‬
‫‪ 1‬‬ ‫‪1‬‬ ‫‪1 ‬‬ ‫‪n‬‬
‫‪ u k = 57 ‬ﻤﻥ‬ ‫‪+‬‬ ‫‪+ ... +‬‬ ‫ﻨﻔﺘﺭﺽ ﺃﻥ ‪‬‬ ‫‪n‬‬ ‫‪‬‬
‫‪ 100 100‬‬
‫‪2‬‬
‫‪100 k ‬‬ ‫ﻭﻤﻌﻨﺎﻩ ‪  − 1  ≥ 2n‬ﺃﻱ ‪u n ≥ 2‬‬
‫‪n‬‬

‫‪ 10 ‬‬
‫‪u k +1 = 0, 57...57‬‬
‫ﺃﺠل ‪ k‬ﻋﺩﺩ ﻁﺒﻴﻌﻲ ﻜﻴﻔﻲ ﻏﻴﺭ ﻤﻌﺩﻭﻡ‪ .‬‬ ‫‪ 2 > 1‬ﻭﻤﻨﻪ ∞‪ lim 2n = +‬ﺇﺫﻥ ∞‪. lim u n = +‬‬
‫ﺎ‬
‫ﻤ‬
‫ﻗ‬
‫ر‬

‫‪2k +2‬‬
‫‪0‬‬
‫‪,‬‬
‫‪5‬‬
‫‪7‬‬
‫‪.‬‬
‫‪.‬‬
‫‪.‬‬
‫‪5‬‬
‫‪7‬‬

‫∞‪n →+‬‬ ‫∞‪n →+‬‬

‫= ‪u k +1‬‬ ‫ ‬ ‫‪ + 0, 00...0057‬‬
‫ ‬ ‫  ‬ ‫ﻭﻤﻨﻪ ‬ ‫‪ (1 117‬ﺍﺴﺘﻌﻤﺎل ﺍﻝﺤﺎﺴﺒﺔ ﺍﻝﺒﻴﺎﻨﻴﺔ ‪. TI 83‬‬
‫ﺎ‬
‫ـ‬
‫ـ‬
‫ـ‬
‫ـ‬
‫ـ‬
‫ـ‬
‫ـ‬
‫ـ‬
‫ـ‬
‫ـ‬
‫ﻤ‬
‫ﻗ‬
‫ر‬

‫ﺎ‬
‫ـ‬
‫ـ‬
‫ـ‬
‫ـ‬
‫ـ‬
‫ـ‬
‫ـ‬
‫ـ‬
‫ـ‬
‫ـ‬
‫ﻤ‬
‫ﻗ‬
‫ر‬

‫‪2k‬‬ ‫‪2k +2‬‬

‫‪57‬‬
‫‪ u k +1 = u k +‬؛‬ ‫ﺇﺫﻥ‬
‫‪102 k + 2‬‬
‫‪ 1‬‬ ‫‪1‬‬ ‫‪1 ‬‬ ‫‪1‬‬
‫‪u k +1 = 57 ‬‬ ‫‪+‬‬ ‫‪+ ... +‬‬ ‫‪k ‬‬
‫× ‪+ 57‬‬
‫‪ 100 100‬‬
‫‪2‬‬
‫‪100 ‬‬ ‫‪100k +1‬‬
‫‪ 1‬‬ ‫‪1‬‬ ‫‪1‬‬ ‫‪1 ‬‬
‫‪. u k +1 = 57 ‬‬ ‫‪+‬‬ ‫‪+ ... +‬‬ ‫‪+‬‬ ‫‪‬‬
‫‪ 100 100‬‬
‫‪2‬‬
‫‪100 100k +1 ‬‬
‫‪k‬‬

‫ﻭﺤﺴﺏ ﻤﺒﺩﺃ ﺍﻝﺘﺭﺍﺠﻊ ﻴﻨﺘﺞ ﺃﻨﻪ ﻤﻥ ﺃﺠل ﻜل ﻋﺩﺩ ﻁﺒﻴﻌﻲ ﻏﻴﺭ‬


‫‪ 1‬‬ ‫‪1‬‬ ‫‪1 ‬‬ ‫ﺍﺒﺘﺩﺍﺀ ﻤﻥ ‪ u 23‬ﺃﻱ ﺍﻝﺩﻝﻴل ‪ 23‬ﺘﺴﺘﻘﺭ ﻗﻴﻡ ﺍﻝﺤﺩﻭﺩ ﻋﻠﻰ‬
‫‪u n = 57 ‬‬ ‫‪+‬‬ ‫‪2‬‬
‫‪+ ... +‬‬ ‫ﻤﻌﺩﻭﻡ ‪ ، n‬‬
‫‪ 100 100‬‬ ‫‪100n ‬‬ ‫‪. 1, 618033989‬‬
‫‪ 1‬‬ ‫‪1‬‬ ‫‪1 ‬‬ ‫ﻴﺒﺩﻭ ﺃﻥ ﺍﻝﻤﺘﺘﺎﻝﻴﺔ ﻤﺘﻘﺎﺭﺒﺔ ﻭﻨﻬﺎﻴﺘﻬﺎ ﺍﻝﻌﺩﺩ ﺍﻝﺤﻘﻴﻘﻲ‬
‫‪ ‬ﻫﻭ ﻤﺠﻤﻊ ﺤﺩﻭﺩ ﻤﺘﺘﺎﺒﻌﺔ‬ ‫‪+‬‬ ‫‪2‬‬
‫‪+ ... +‬‬ ‫‪‬‬
‫‪ 100 100‬‬ ‫‪100 n ‬‬
‫‪. 1, 618033989‬‬
‫‪1‬‬
‫‪.‬‬ ‫ﻝﻤﺘﺘﺎﻝﻴﺔ ﻫﻨﺩﺴﻴﺔ ﺃﺴﺎﺴﻬﺎ ﻭﺤﺩﻫﺎ ﺍﻷﻭل ﻤﺴﺎﻭﻴﻴﻥ ﻝﻠﻌﺩﺩ‬ ‫ﺇﺫﺍ ﻜﺎﻨﺕ ﺍﻝﻤﺘﺘﺎﻝﻴﺔ ‪ u‬ﻤﺘﻘﺎﺭﺒﺔ ﻓﺈﻨﻪ ﻴﻭﺠﺩ ﻋﺩﺩ ﺤﻘﻴﻘﻲ ‪l‬‬
‫‪100‬‬
‫ﺤﻴﺙ ‪ lim u n = l‬ﻭﻜﺫﻝﻙ ‪ lim u n +1 = l‬ﻫﺫﺍ ﻤﻥ ﺠﻬﺔ ‪،‬‬
‫∞‪n →+‬‬ ‫∞‪n →+‬‬
‫ﻨﻔﺘﺭﺽ ﺃﻥ ‪ u k +1 ≥ u k‬ﻭﻫﺫﺍ ﻤﻥ ﺃﺠل ‪ k‬ﻋﺩﺩ ﻁﺒﻴﻌﻲ‬
‫‪n‬‬
‫‪ 1 ‬‬
‫‪1− ‬‬ ‫‪‬‬ ‫‪57 100   1  ‬‬
‫‪n‬‬

‫‪× ‬‬
‫‪1‬‬ ‫‪14‬‬ ‫‪1‬‬ ‫‪100 ‬‬
‫‪ f : x ֏ x +‬ﻤﺘﺯﺍﻴﺩﺓ ﺘﻤﺎﻤﺎ‬ ‫ﻜﻴﻔﻲ‪ .‬ﺍﻝﺩﺍﻝﺔ ﺍﻝﺘﺂﻝﻔﻴﺔ‬ ‫× ‪u n = 57‬‬ ‫=‬ ‫×‬ ‫‪ ‬‬
‫‪1‬‬ ‫‪−‬‬ ‫‪ ‬‬
‫‪100‬‬ ‫‪1‬‬ ‫‪100 99   100  ‬‬
‫‪3‬‬ ‫‪3‬‬ ‫‪1−‬‬
‫‪100‬‬
‫ﻋﻠﻰ ‪ ℝ‬ﺇﺫﻥ ) ‪ f (u k +1 ) > f (u k‬ﺃﻱ ‪u k + 2 > u k +1‬‬
‫‪57   1  ‬‬
‫‪n‬‬
‫‪1‬‬
‫ﻭﺤﺴﺏ ﻤﺒﺩﺃ ﺍﻝﺘﺭﺍﺠﻊ ﻴﻨﺘﺞ ﺃﻨﻪ ﻤﻥ ﺃﺠل ﻜل ﻋﺩﺩ ﻁﺒﻴﻌﻲ ‪، n‬‬ ‫< ‪−1‬‬ ‫= ‪ u n‬ﺒﻤﺎ ﺃﻥ ‪< 1‬‬ ‫‪1 − ‬‬ ‫ﻭﻤﻨﻪ ‪ ‬‬
‫‪100‬‬ ‫‪99   100  ‬‬
‫‪ u n +1 > u n‬ﺃﻱ ﺍﻝﻤﺘﺘﺎﻝﻴﺔ ) ‪ (u n‬ﻤﺘﺯﺍﻴﺩﺓ ﺘﻤﺎﻤﺎ ‪.‬‬ ‫‪n‬‬
‫‪57‬‬ ‫‪ 1 ‬‬
‫‪1‬‬ ‫‪14‬‬ ‫= ‪. lim u n‬‬ ‫‪ lim ‬ﻭﺒﺎﻝﺘﺎﻝﻲ‬ ‫ﻓﺈﻥ ‪ = 0‬‬
‫‪ x = x +‬ﻤﻌﻨﺎﻩ ‪ 3x = x + 14‬ﻭﻤﻌﻨﺎﻩ‬ ‫‪(2‬‬ ‫∞‪n →+‬‬ ‫‪99‬‬ ‫‪‬‬
‫‪n →+∞ 100‬‬
‫‪‬‬
‫‪3‬‬ ‫‪3‬‬
‫‪ (u n ) 119‬ﻤﻌﺭﻓﺔ ﻋﻠﻰ ‪ِ ℕ‬ـ ‪. u n = n 2 + n − n :‬‬
‫‪ 2x = 14‬ﺃﻱ ‪. x = 7‬‬ ‫‪u10 n‬‬
‫‪n‬‬
‫‪ (3‬ﺇﺫﺍ ﻜﺎﻨﺕ ﺍﻝﻤﺘﺘﺎﻝﻴﺔ ) ‪ (u n‬ﻤﺘﻘﺎﺭﺒﺔ ﻓﺈﻨﻬﺎ ﺘﻘﺒل ﻨﻬﺎﻴﺔ ﻋﺩﺩﺍ‬ ‫‪1‬‬ ‫‪0.4880884817015‬‬ ‫‪ (1‬ﺤﺴﺎﺏ ﺍﻝﺤﺩﻭﺩ ﺒﺎﺴﺘﻌﻤﺎل‬

‫ﺤﻘﻴﻘﻴﺎ ‪ l‬ﻭﻤﻨﻪ ‪ lim u n = l‬ﻭﻜﺫﻝﻙ ‪lim u n +1 = l‬‬


‫‪2‬‬ ‫‪0.4987562112089‬‬
‫ﻤﺠﺩﻭل ﺇﻜﺴﻴل ‪.‬‬
‫∞‪n →+‬‬ ‫∞‪n →+‬‬
‫‪3‬‬ ‫‪0.4998750624610‬‬
‫‪4‬‬ ‫‪0.4999875006251‬‬
‫‪n‬‬ ‫‪u‬‬ ‫‪n‬‬
‫‪1‬‬ ‫‪14‬‬ ‫‪1‬‬ ‫‪0.4142135623731‬‬
‫‪ u n +1 = u n +‬ﻭﻤﻨﻪ‬ ‫ﻝﺩﻴﻨﺎ‬ ‫‪5‬‬ ‫‪0.4999987500050‬‬
‫‪2‬‬ ‫‪0.4494897427832‬‬
‫‪3‬‬ ‫‪3‬‬ ‫‪6‬‬ ‫‪0.4999998749699‬‬
‫‪3‬‬ ‫‪0.4641016151378‬‬
‫‪1‬‬ ‫‪14  1 14‬‬ ‫‪7‬‬ ‫‪0.4999999869615‬‬
‫‪ lim u n +1 = lim  u n +  = l +‬ﻭﺒﺎﻝﺘﺎﻝﻲ‬ ‫‪8‬‬ ‫‪0.5000000000000‬‬ ‫‪4‬‬ ‫‪0.4721359549996‬‬
‫∞‪n →+‬‬
‫‪‬‬
‫‪n →+∞ 3‬‬ ‫‪3 3‬‬ ‫‪3‬‬ ‫‪9‬‬ ‫‪0.5000000000000‬‬ ‫‪n‬‬ ‫‪u‬‬
‫‪1 14‬‬ ‫‪10‬‬ ‫‪0.5000000000000‬‬
‫‪ l + = l‬ﻭﺤﺴﺏ ﺍﻝﺴﺅﺍل ﺍﻝﺴﺎﺒﻕ ﻴﻜﻭﻥ ‪. l = 7‬‬
‫‪3‬‬ ‫‪3‬‬ ‫‪11‬‬ ‫‪0.5000000000000‬‬
‫‪12‬‬ ‫‪0.5000000000000‬‬
‫‪ (4‬ﻨﻀﻊ ﻤﻥ ﺃﺠل ﻜل ﻋﺩﺩ ﻁﺒﻴﻌﻲ ‪ v n = u n − 7 ، n‬ﻤﻌﻨﺎﻩ‬ ‫‪13‬‬ ‫‪0.5000000000000‬‬
‫‪1‬‬ ‫‪14‬‬ ‫‪1‬‬ ‫‪7‬‬ ‫‪14‬‬ ‫‪0.5000000000000‬‬ ‫‪ (2‬ﻝﻴﻜﻥ ∗‪، n ∈ ℕ‬‬
‫‪ v n +1 = u n +1 − 7‬ﺃﻱ ‪v n +1 = u n + − 7 = u n −‬‬ ‫‪15‬‬ ‫‪0.0000000000000‬‬
‫‪3‬‬ ‫‪3‬‬ ‫‪3‬‬ ‫‪3‬‬ ‫‪un = n 2 + n − n‬‬

‫(‬ ‫()‬ ‫)‬


‫‪1‬‬ ‫‪7 1‬‬
‫ﺒﻤﺎ ﺃﻥ ‪ u n = v n + 7‬ﻓﺈﻥ ‪v n +1 = (v n + 7 ) − = v n‬‬ ‫‪n2 + n − n‬‬ ‫‪n2 + n + n‬‬
‫‪3‬‬ ‫‪3 3‬‬
‫=‬
‫(‬ ‫)‬
‫‪un‬‬
‫‪1‬‬
‫ﻭﺤﺩﻫﺎ ﺍﻷﻭل‬ ‫ﻭﺒﺎﻝﺘﺎﻝﻲ ﺍﻝﻤﺘﺘﺎﻝﻴﺔ ) ‪ (v n‬ﻫﻨﺩﺴﻴﺔ ﺃﺴﺎﺴﻬﺎ‬ ‫‪n2 + n + n‬‬
‫‪3‬‬
‫‪n‬‬ ‫‪n‬‬ ‫‪1‬‬
‫‪1‬‬
‫‪n‬‬
‫‪1‬‬
‫‪n‬‬
‫= ‪un‬‬ ‫=‬ ‫=‬
‫‪v n = v 0   = −6   . v 0 = −6‬‬ ‫‪1‬‬ ‫‪‬‬ ‫‪1‬‬ ‫‪‬‬ ‫‪1‬‬
‫‪3‬‬ ‫‪ 3‬‬ ‫‪n 1+‬‬ ‫‪+n‬‬ ‫‪n  1 + + 1‬‬ ‫‪1+ +1‬‬
‫‪n‬‬ ‫‪‬‬ ‫‪n‬‬ ‫‪‬‬ ‫‪n‬‬
‫‪ v n = u n − 7‬ﻤﻌﻨﺎﻩ ‪u n = v n + 7‬‬
‫‪1‬‬ ‫‪1‬‬
‫‪n‬‬ ‫ﻝﺩﻴﻨﺎ ‪ lim = 0‬ﻭﻤﻨﻪ = ‪lim u n‬‬
‫‪1‬‬ ‫‪1‬‬ ‫∞‪n →+‬‬ ‫‪2‬‬ ‫‪n →+∞ n‬‬
‫< ‪ −1‬ﻓﺈﻥ‬ ‫ﺃﻱ ‪ . u n = −6   + 7‬ﺒﻤﺎ ﺃﻥ ‪< 1‬‬
‫‪3‬‬ ‫‪3‬‬ ‫ﻤﻥ ﺃﺠل ﺍﻝﻘﻴﻡ ﺍﻝﻜﺒﻴﺭﺓ ﻝﻠﻌﺩﺩ ‪ ، n‬ﻓﻲ ﺍﻝﻤﺠﺩﻭﻻﺕ ﺃﻭ ﺍﻝﺤﺎﺴﺒﺎﺕ‬
‫‪n‬‬
‫‪1‬‬ ‫‪ n‬ﻴﻬﻤل ﺒﺎﻝﻨﺴﺒﺔ ِـ ‪ n 2‬ﻭﻻ ﻴﻤﻜﻥ ﺍﻝﺘﻤﻴﻴﺯ ﺒﻴﻥ ‪ n 2‬ﻭ ‪n 2 + n‬‬
‫) ‪ (u n‬ﻤﺘﻘﺎﺭﺒﺔ‪.‬‬ ‫‪ lim   = 0‬ﻭﻤﻨﻪ ‪ lim u n = 7‬ﺇﺫﻥ‬
‫∞‪n →+‬‬
‫‪ ‬‬
‫‪n →+∞ 3‬‬
‫ﻭﻝﻬﺫﺍ ﻨﺘﺎﺌﺞ ﺍﻝﺤﺴﺎﺏ ِـ ‪ n 2 + n − n‬ﺘﺴﺠل ‪.0‬‬
‫‪121‬‬ ‫‪1‬‬ ‫‪14‬‬
‫‪. u n +1 = u n +‬‬ ‫‪ (u n ) 120‬ﻤﻌﺭﻓﺔ ِـ ‪ u 0 = 1‬ﻭ‬
‫‪3‬‬ ‫‪3‬‬
‫‪ (1‬ﻝﻨﺒﺭﻫﻥ ﺃﻨﻪ ﻤﻥ ﺃﺠل ﻜل ﻋﺩﺩ ﻁﺒﻴﻌﻲ ‪. u n +1 > u n ، n‬‬
‫‪1‬‬ ‫‪14‬‬
‫ﻝﺩﻴﻨﺎ ‪ u 0 = 1‬ﻭ ‪ u 1 = u 0 + = 5‬ﺇﺫﻥ ‪. u 1 > u 0‬‬
‫‪ (1‬ﺍﻝﺨﺎﺼﻴﺔ !)‪ 2n ≤ ( n − 1‬ﺘﻜﻭﻥ ﺼﺤﻴﺤﺔ ﻤﻥ ﺃﺠل ‪. n = 6‬‬ ‫‪3‬‬ ‫‪3‬‬
‫‪ (3‬ﺤﺴﺏ ﺍﻝﺴﺅﺍل ﺍﻝﺴﺎﺒﻕ ﻤﻥ ﺃﺠل ﻜل ‪ n‬ﻤﻥ ‪، ℕ‬‬ ‫ﻨﻔﺘﺭﺽ ﺃﻥ ! )‪ 2k ≤ ( k − 1‬ﻤﻥ ﺃﺠل ‪ k‬ﻋﺩﺩ ﻁﺒﻴﻌﻲ ﻜﻴﻔﻲ‬

‫‪ −‬ﺃﻱ‬
‫‪a‬‬
‫≤ ‪≤ un‬‬
‫‪a‬‬
‫≤ ‪ u n‬ﻤﻌﻨﺎﻩ‬
‫‪a‬‬ ‫ﺃﻜﺒﺭ ﻤﻥ ﺃﻭ ﻴﺴﺎﻭﻱ ‪ . 6‬ﺇﺫﻥ ! )‪ 2k ≤ ( k − 1‬ﻭ ‪2 ≤ k‬‬
‫‪2n‬‬ ‫‪2n‬‬ ‫‪2n‬‬
‫‪n‬‬ ‫‪n‬‬
‫ﻭﻤﻨﻪ ! )‪ 2 × 2k ≤ k ( k − 1‬ﻭﺒﺎﻝﺘﺎﻝﻲ ! ‪2k +1 ≤ k‬‬
‫‪1‬‬ ‫‪1‬‬
‫‪− a   ≤ un ≤ a  ‬‬ ‫ﺇﺫﻥ ﺤﺴﺏ ﻤﺒﺩﺃ ﺍﻝﺘﺭﺍﺠﻊ ﻴﻨﺘﺞ ﺃﻨﻪ ﻤﻥ ﻜل ﻋﺩﺩ ﻁﺒﻴﻌﻲ‬
‫‪2‬‬ ‫‪2‬‬
‫‪n‬‬ ‫‪2n ≤ ( n − 1) ! ، n ≥ 6‬‬
‫‪1‬‬ ‫‪1‬‬
‫ﺒﻤﺎ ﺃﻥ ‪ −1 < < 1‬ﻓﺈﻥ ‪ lim   = 0‬ﻭﻤﻨﻪ‬ ‫‪ (2‬ﻝﻴﻜﻥ ‪ 2n ≤ ( n − 1) ! ، n ≥ 6‬ﻤﻌﻨﺎﻩ‬
‫‪ ‬‬
‫‪n →+∞ 2‬‬ ‫‪2‬‬
‫‪1‬‬
‫‪n‬‬
‫‪1‬‬
‫‪n‬‬
‫‪2n 1‬‬ ‫!‪n‬‬ ‫! )‪n ( n − 1‬‬
‫‪lim − a   = lim a   = 0‬‬ ‫‪.‬‬ ‫≤ ‪ 2n‬ﻭ ﺒﺎﻝﺘﺎﻝﻲ ≤‬ ‫≤ ‪ 2n‬ﺃﻱ‬
‫∞‪n →+‬‬
‫‪2‬‬ ‫∞‪n →+‬‬
‫‪2‬‬ ‫‪n! n‬‬ ‫‪n‬‬ ‫‪n‬‬
‫ﻭﺒﺎﻝﺘﺎﻝﻲ ‪. lim u n = 0‬‬ ‫ﺇﺫﻥ ﻤﻥ ﺃﺠل ﻜل ﻋﺩﺩ ﻁﺒﻴﻌﻲ ‪ n‬ﺤﻴﺙ ‪، n ≥ 6‬‬
‫∞‪n →+‬‬
‫‪2n‬‬ ‫‪1‬‬ ‫‪2n 1‬‬
‫‪un + 2‬‬ ‫‪، lim‬‬ ‫≤ ‪ 0‬ﺒﻤﺎ ﺃﻥ ‪ lim = 0‬ﻓﺈﻥ ‪= 0‬‬ ‫≤‬
‫= ‪. u n +1‬‬ ‫‪ (u n ) 123‬ﻤﻌﺭﻓﺔ ِـ ‪ u 0 = 2‬ﻭ‬ ‫! ‪n →+∞ n‬‬ ‫‪n →+∞ n‬‬ ‫‪n! n‬‬
‫‪2u n + 1‬‬
‫‪2n‬‬
‫‪ ،‬ﻤﺘﻘﺎﺭﺒﺔ ‪.‬‬ ‫ﺇﺫﻥ ﺍﻝﻤﺘﺘﺎﻝﻴﺔ ﺫﺍﺕ ﺍﻝﺤﺩ ﺍﻝﻌﺎﻡ‬
‫‪ (1‬أ ـ ﻝﺩﻴﻨﺎ ‪ u 0 > 0 :‬ﻨﻔﺘﺭﺽ ﺃﻥ ‪ u k > 0‬ﻤﻥ ﺃﺠل ‪k‬‬ ‫!‪n‬‬
‫ﻋﺩﺩ ﻁﺒﻴﻌﻲ ﻜﻴﻔﻲ ‪ ،‬ﻭﻤﻨﻪ ‪ u k + 2 > 0‬ﻭ ‪ 2u k + 1 > 0‬ﺇﺫﻥ‬ ‫‪ a 122‬ﻋﺩﺩ ﺤﻘﻴﻘﻲ ﻭ ) ‪ (u n‬ﻤﺘﺘﺎﻝﻴﺔ ﻤﻌﺭﻓﺔ ﻋﻠﻰ ‪ِ ℕ‬ـ ‪:‬‬
‫‪u +2‬‬ ‫‪un‬‬
‫‪ k‬ﻭﺒﺎﻝﺘﺎﻝﻲ ‪ u k +1 > 0‬ﻭﺤﺴﺏ ﻤﺒﺩﺃ ﺍﻝﺘﺭﺍﺠﻊ‬ ‫‪>0‬‬ ‫= ‪. u n +1‬‬ ‫‪ u 0 = a‬ﻭﺍﻝﻌﻼﻗﺔ ﺍﻝﺘﺭﺍﺠﻌﻴﺔ ‪:‬‬
‫‪2u k + 1‬‬ ‫‪2 +un2‬‬
‫ﻴﻨﺘﺞ ﺃﻨﻪ ﻤﻥ ﺃﺠل ﻜل ‪ n‬ﻤﻥ ‪. u n > 0 ، ℕ‬‬ ‫‪ (1‬ﻝﻴﻜﻥ ‪ n‬ﻋﺩﺩﺍ ﻁﺒﻴﻌﻴﺎ‪ 2 + u n 2 ≥ 2 ،‬ﻤﻌﻨﺎﻩ‬
‫ب ـ ﺇﺫﺍ ﻜﺎﻨﺕ ﺍﻝﻤﺘﺘﺎﻝﻴﺔ ) ‪ (u n‬ﻤﺘﻘﺎﺭﺒﺔ ﻓﺈﻨﻬﺎ ﺘﻘﺒل ﻨﻬﺎﻴﺔ ﺃﻱ‬ ‫‪un‬‬ ‫‪un‬‬ ‫‪1‬‬ ‫‪1‬‬
‫≤‬ ‫ﻭﻤﻌﻨﺎﻩ‬ ‫≤‬
‫∗‬
‫‪ lim u n = l‬ﺤﻴﺙ ‪ ، l ∈ ℝ‬ﻭﻤﻨﻪ ‪ lim u n +1 = l‬ﻫﺫﺍ‬
‫‪+‬‬
‫‪2 +un‬‬ ‫‪2‬‬ ‫‪2‬‬
‫‪2 +un‬‬ ‫‪2‬‬
‫‪2‬‬
‫∞‪n →+‬‬ ‫∞‪n →+‬‬
‫‪un‬‬ ‫‪un‬‬ ‫‪un‬‬
‫ﻤﻥ ﺠﻬﺔ ‪ .‬ﻭﻤﻥ ﺠﻬﺔ ﺃﺨﺭﻯ ﻝﺩﻴﻨﺎ‬ ‫‪u n +1‬‬ ‫=‬ ‫=‬ ‫ﺇﺫﻥ‬ ‫‪u‬‬ ‫‪+‬‬ ‫=‬ ‫ﻭﻝﺩﻴﻨﺎ‬
‫‪2 +un2 2 +un‬‬ ‫‪2 +un2‬‬
‫‪2‬‬ ‫‪n‬‬ ‫‪1‬‬

‫‪l +2‬‬ ‫‪u +2 l +2‬‬


‫‪ lim u n +1 = lim n‬ﻭﻤﻨﻪ ‪= l‬‬ ‫=‬
‫‪2l + 1‬‬ ‫‪n‬‬ ‫∞‪→+‬‬ ‫‪n‬‬ ‫∞‪→+‬‬ ‫‪2u n + 1 2l + 1‬‬ ‫≤ ‪. u n +1‬‬
‫‪un‬‬
‫ﻭﺒﺎﻝﺘﺎﻝﻲ ‪ :‬ﻤﻥ ﺃﺠل ﻜل ‪ n‬ﻤﻥ ‪، ℕ‬‬
‫ﻭﻤﻌﻨﺎﻩ ‪ 2l = 2‬ﺃﻱ ‪ l = 1‬ﻭﻤﻨﻪ ‪. l = 1‬‬
‫‪2‬‬ ‫‪2‬‬ ‫‪2‬‬
‫‪a‬‬ ‫‪a‬‬
‫‪x +2‬‬ ‫≤ ‪u0‬‬ ‫ﺇﺫﻥ ﺍﻝﺨﺎﺼﻴﺔ‬ ‫‪ (2‬ﻝﺩﻴﻨﺎ ‪ u 0 = a‬ﻭ ‪= a‬‬
‫֏ ‪ ، f : x‬ﺍﻝﺩﺍﻝﺔ‬ ‫‪ C (2‬ﺍﻝﻤﻨﺤﻨﻲ ﺍﻝﻤﻤﺜل ﻝﻠﺩﺍﻝﺔ‬ ‫‪2‬‬ ‫‪0‬‬
‫‪20‬‬
‫‪2x + 1‬‬
‫‪ 1‬‬ ‫‪ ‬‬ ‫‪1‬‬ ‫ﺼﺤﻴﺤﺔ ‪.‬‬
‫‪ f‬ﺘﻘﺒل ﺍﻻﺸﺘﻘﺎﻕ ﻋﻠﻰ ‪  −∞ ; − 2 ‬ﻭ ‪ − 2 ; + ∞ ‬‬ ‫‪a‬‬
‫≤ ‪ u k‬ﻤﻥ ﺃﺠل ﻋﺩﺩ ﻁﺒﻴﻌﻲ ‪k‬‬ ‫ﻨﻔﺘﺭﺽ ﺃﻥ ﺍﻝﺨﺎﺼﻴﺔ‬
‫= ) ‪f '(x‬‬
‫ﻭﻝﺩﻴﻨﺎ ‪( 2x + 1) − 2 ( x + 2 ) = −3 :‬‬ ‫‪2k‬‬
‫‪uk‬‬ ‫‪a‬‬ ‫‪1 a‬‬
‫)‪( 2x + 1‬‬ ‫)‪( 2x + 1‬‬ ‫‪1‬‬
‫‪2‬‬ ‫‪2‬‬
‫ﺒﻤﺎ ﺃﻥ‬ ‫≤‬ ‫‪k +1‬‬
‫ﻜﻴﻔﻲ ﺇﺫﻥ ‪u k ≤ × k‬‬
‫‪ ،‬ﻭﻤﻨﻪ‬
‫‪2‬‬ ‫‪2‬‬ ‫‪2‬‬ ‫‪2 2‬‬
‫ﻭﻤﻨﻪ ﻤﻥ ﺃﺠل ﻜل ﻋﺩﺩ ‪ x‬ﻤﻥ ]‪f ' ( x ) < 0 [ 0 ; 2, 2‬‬ ‫‪a‬‬ ‫‪u‬‬
‫‪ u k +1 ≤ k‬ﻤﻥ ﺍﻝﺴﺅﺍل ﺍﻝﺴﺎﺒﻕ ﻓﺈﻥ ‪ u k +1 ≤ k +1‬ﺇﺫﻥ‬
‫]‪[ 0 ; 2, 2‬‬ ‫ﺇﺫﻥ ﺍﻝﺩﺍﻝﺔ ‪ f‬ﻤﺘﻨﺎﻗﺼﺔ ﺘﻤﺎﻤﺎ ﻋﻠﻰ‬ ‫‪2‬‬ ‫‪2‬‬
‫‪0‬‬ ‫‪1‬‬ ‫‪2, 2‬‬ ‫ﺤﺴﺏ ﻤﺒﺩﺃ ﺍﻝﺘﺭﺍﺠﻊ ﻴﻨﺘﺞ ﻤﻥ ﺃﺠل ﻜل ‪ n‬ﻤﻥ ‪، ℕ‬‬
‫‪x‬‬
‫) ‪f (x‬‬ ‫‪2‬‬ ‫≤ ‪. un‬‬
‫‪a‬‬
‫‪1‬‬ ‫‪2n‬‬
‫‪0, 77‬‬
‫‪y‬‬ ‫ﻴﺒﺩﻭ ﺃﻥ ) ‪ (u n‬ﻤﺘﻘﺎﺭﺒﺔ‬

‫‪1‬‬

‫‪0‬‬ ‫‪1‬‬ ‫‪2x‬‬


‫ﻨﻔﺘﺭﺽ ‪ 2 ≤ u k +1 ≤ u k‬ﻴﻌﻨﻲ ‪4 ≤ 2 + u k +1 ≤ 2 + u k‬‬ ‫ﻭﻨﻬﺎﻴﺘﻬﺎ ‪. 1‬‬
‫ﺒﻤﺎ ﺃﻥ ﺍﻝﺩﺍﻝﺔ ﺍﻝﺠﺩﺭ ﺍﻝﺘﺭﺒﻴﻌﻲ ﻤﺘﺯﺍﻴﺩﺓ ﺘﻤﺎﻤﺎ ﻋﻠﻰ [∞ ‪[ 0; +‬‬
‫ﻓﺈﻥ ‪ 4 ≤ 2 + u k +1 ≤ 2 + u k‬ﺃﻱ ‪2 ≤ u k + 2 ≤ u k +1‬‬
‫ﻭﺤﺴﺏ ﻤﺒﺩﺃ ﺍﻝﺘﺭﺍﺠﻊ ﻨﺴﺘﻨﺘﺞ ﺃﻨﻪ ﻤﻥ ﺃﺠل ﻜل ﻋﺩﺩ ﻁﺒﻴﻌﻲ‬
‫‪. 2 ≤ u n +1 ≤ u n ، n‬‬
‫‪ (2‬ﻤﻥ ﺍﻝﺴﺅﺍل ﺍﻝﺴﺎﺒﻕ ﻴﻨﺘﺞ ﺃﻥ ﺍﻝﻤﺘﺘﺎﻝﻴﺔ ) ‪ (u n‬ﻤﺘﻨﺎﻗﺼﺔ‬
‫‪u n −1‬‬
‫ﻭﻤﺤﺩﻭﺩﺓ ﻤﻥ ﺍﻷﺴﻔل ﺒﺎﻝﻌﺩﺩ ‪ 2‬ﺇﺫﻥ ﻫﻲ ﻤﺘﻘﺎﺭﺒﺔ‬ ‫= ‪. vn‬‬ ‫‪ (3‬ﻨﻀﻊ ﻤﻥ ﺃﺠل ﻜل ‪ n‬ﻤﻥ ‪، ℕ‬‬
‫‪un +1‬‬
‫ﻭﻨﻬﺎﻴﺘﻬﺎ ‪. l ≥ 2‬‬
‫‪u −1‬‬
‫‪ lim u n = l (3‬ﺇﺫﻥ ‪ lim u n +1 = l‬ﻫﺫﺍ ﻤﻥ ﺠﻬﺔ ‪ ،‬ﻭﻤﻥ‬ ‫‪v n +1 = n +1‬‬ ‫أ ـ ﻝﻴﻜﻥ ‪ n‬ﻋﺩﺩﺍ ﻁﺒﻴﻌﻴﺎ ‪،‬‬
‫∞‪n →+‬‬ ‫∞‪n →+‬‬ ‫‪u n +1 + 1‬‬
‫‪ lim u n +1 = lim‬ﺇﺫﻥ‬ ‫ﺠﻬﺔ ﺃﺨﺭﻯ ‪2 + u n = 2 + l‬‬ ‫‪un + 2‬‬
‫∞‪n →+‬‬ ‫∞‪n →+‬‬ ‫‪−1‬‬
‫‪2u n + 1‬‬ ‫‪−u + 1‬‬ ‫‪1 u −1‬‬ ‫‪1‬‬
‫‪.l = 2+l‬‬ ‫= ‪v n +1‬‬ ‫‪= n‬‬ ‫‪=− × n‬‬ ‫‪=− vn‬‬
‫‪un + 2‬‬
‫‪+ 1 3u n + 3‬‬ ‫‪3 un +1‬‬ ‫‪3‬‬
‫ﻝﺩﻴﻨﺎ ‪ l ≥ 2‬ﻭ ‪ l = 2 + l‬ﺇﺫﻥ ‪ l 2 = 2 + l‬ﻤﻌﻨﺎﻩ‬ ‫‪2u n + 1‬‬
‫‪ l 2 − l − 2 = 0‬ﻭﻤﻌﻨﺎﻩ ‪ ( l + 1)( l − 2 ) = 0‬ﻴﻜﺎﻓﺊ‬ ‫‪u −1 1‬‬
‫ﺇﺫﻥ ) ‪ (v n‬ﻫﻨﺩﺴﻴﺔ ﺃﺴﺎﺴﻬﺎ ‪ −‬ﻭﺤﺩﻫﺎ ﺍﻷﻭل =‬
‫‪1‬‬
‫‪v0 = 0‬‬
‫‪u0 +1 3‬‬ ‫‪3‬‬
‫‪ l = 2‬ﺃﻭ ‪ . l = −1‬ﺒﻤﺎ ﺃﻥ ‪ l ≥ 2‬ﻓﺈﻥ ‪. l = 2‬‬
‫‪1‬‬
‫ﺒﻤﺎ ﺃﻥ ‪ −1 < − < 1‬ﻓﺈﻥ ) ‪ (v n‬ﻤﺘﻘﺎﺭﺒﺔ ‪.‬‬
‫‪ 125‬ﻨﻌﺘﺒﺭ ﺍﻝﻤﺘﺘﺎﻝﻴﺔ ‪ u‬ﺍﻝﻤﻌﺭﻓﺔ ﻋﻠﻰ ∗‪ ℕ‬ﺒﹻ‬ ‫‪3‬‬
‫‪n‬‬ ‫‪n‬‬ ‫‪n +1‬‬
‫‪1 1‬‬ ‫‪1‬‬ ‫‪1‬‬ ‫‪ 1‬‬ ‫‪1‬‬
‫∑ = ‪. u n = 1 + + + ... +‬‬ ‫‪. v n = v 0 ×  −  = ( −1) ×  ‬‬
‫‪n‬‬
‫‪2 3‬‬ ‫‪n k =1 k‬‬ ‫‪ 3‬‬ ‫‪3‬‬
‫‪ (1‬ﺍﻝﺩﺍﻝﺔ ‪ f : x ֏ ln ( x + 1) − x‬ﺘﻘﺒل ﺍﻻﺸﺘﻘﺎﻕ ﻋﻠﻰ‬ ‫‪u −1‬‬
‫‪vn = n‬‬ ‫ب ـ ﻝﻴﻜﻥ ‪ n‬ﻋﺩﺩﺍ ﻁﺒﻴﻌﻴﺎ ‪،‬‬
‫‪1‬‬ ‫‪−x‬‬ ‫‪un +1‬‬
‫= ) ‪. f '(x‬‬ ‫= ‪−1‬‬ ‫[∞ ‪ ]−1; +‬ﻭ‬
‫‪x +1‬‬ ‫‪x +1‬‬ ‫‪u −1‬‬
‫‪ n‬ﻭﻤﻌﻨﺎﻩ ‪u n − 1 = u n + 1‬‬ ‫ﻝﺩﻴﻨﺎ ‪ v n = 1‬ﻤﻌﻨﺎﻩ ‪= 1‬‬
‫‪ lim‬؛‬‫>‬
‫∞‪f ( x ) = −‬‬ ‫‪un +1‬‬
‫‪x ‬‬
‫‪→ −1‬‬

‫)‪ ln ( x + 1‬‬ ‫‪x ‬‬


‫ﺃﻱ ‪ −1 = 1‬ﻭﻫﺫﺍ ﺘﻨﺎﻗﺽ ﺇﺫﻥ ‪.v n ≠ 1‬‬
‫‪lim f ( x ) = lim ( x + 1) ‬‬ ‫‪−‬‬ ‫∞‪ = −‬‬ ‫‪u −1‬‬
‫∞‪x →+‬‬ ‫∞‪x →+‬‬
‫‪ x +1‬‬ ‫‪x + 1‬‬ ‫‪ v n = n‬ﻤﻌﻨﺎﻩ ‪ v nu n + v n = u n − 1‬ﻭﻤﻌﻨﺎﻩ‬
‫‪−1‬‬ ‫‪0‬‬ ‫∞‪+‬‬ ‫‪un +1‬‬
‫‪x‬‬
‫) ‪f '(x‬‬ ‫‪+‬‬ ‫‪0‬‬ ‫‪−‬‬ ‫‪(v n − 1)u n = −v n − 1‬‬ ‫‪ v nu n − u n = −v n − 1‬ﻴﻜﺎﻓﺊ‬
‫‪−v n − 1‬‬
‫) ‪f (x‬‬ ‫‪0‬‬ ‫= ‪. un‬‬ ‫ﺃﻱ‬
‫∞‪−‬‬ ‫∞‪−‬‬ ‫‪v n −1‬‬
‫‪n‬‬
‫‪ 1‬‬
‫ﻝﺩﻴﻨﺎ ‪ lim v n = lim v 0 ×  −  = 0‬ﺇﺫﻥ ‪lim u n = 1‬‬
‫‪ (2‬ﻤﻥ ﺃﺠل ﻜل [∞ ‪ f ( x ) ≤ 0 ، x ∈ ]−1; +‬ﻤﻌﻨﺎﻩ‬
‫∞‪n →+‬‬ ‫∞‪n →+‬‬ ‫∞‪n →+‬‬
‫‪ 3‬‬
‫‪1‬‬ ‫ﻭﺒﺎﻝﺘﺎﻝﻲ ) ‪ (u n‬ﻤﺘﻘﺎﺭﺒﺔ‪.‬‬
‫= ‪ x‬ﻤﻊ ∗‪ k ∈ ℕ‬ﻴﻜﻭﻥ‬ ‫‪ ، ln ( x + 1) ≤ x‬ﺒﻭﻀﻊ‬
‫‪k‬‬ ‫‪ (u n ) 124‬ﻤﻌﺭﻓﺔ ِـ ‪ u 0 = 5‬ﻭ ‪. u n +1 = 2 + u n‬‬
‫‪1  1‬‬
‫≤ ‪ ln  + 1‬ﻭﻝﺩﻴﻨﺎ‬ ‫[∞ ‪ x ∈ ]−1; +‬ﻭﻤﻨﻪ‬ ‫‪ (1‬ﻝﺩﻴﻨﺎ ‪ u1 = 7 ، u 0 = 5‬ﻭ ‪ 2 ≤ 7 ≤ 5‬ﺇﺫﻥ‬
‫‪k‬‬ ‫‪ k‬‬
‫‪. 2 ≤ u1 ≤ u 0‬‬
‫‪1 ‬‬ ‫‪ 1+ k ‬‬
‫‪ln  + 1 = ln ‬‬ ‫‪ = ln (1 + k ) − ln k‬‬
‫‪k‬‬ ‫‪‬‬ ‫‪ k ‬‬
‫‪‬‬ ‫‪1 ‬‬ ‫‪1‬‬
‫≤ ‪. ln ( k + 1) − ln k‬‬
‫‪1‬‬
‫ﺃﻱ ) ‪(u n‬‬ ‫‪ln 1 +‬‬ ‫‪ 1 +‬ﻤﻌﻨﺎﻩ ‪ > 0‬‬ ‫‪>1‬‬ ‫ﻭﻤﻨﻪ ﻤﻥ ﺃﺠل ﻜل ‪ k‬ﻤﻥ ∗‪، ℕ‬‬
‫‪ n +1 ‬‬ ‫‪n +1‬‬ ‫‪k‬‬
‫ﻤﺘﺯﺍﻴﺩﺓ ‪.‬‬ ‫ﺒﺘﻁﺒﻴﻕ ﻫﺫﻩ ﺍﻝﻤﺘﺒﺎﻴﻨﺔ ‪ n‬ﻤﺭﺓ ﻨﺤﺼل ﻋﻠﻰ ‪:‬‬
‫‪1‬‬ ‫‪1‬‬
‫‪3‬‬ ‫‪ n +1 ‬‬ ‫‪، ln 4 − ln 3 ≤ ، ln 3 − ln 2 ≤ ، ln 2 − ln1 ≤ 1‬‬
‫‪u n = ln ( 2 ) + ln   + ... + ln ‬‬ ‫‪ (2‬‬ ‫‪3‬‬ ‫‪2‬‬
‫‪2‬‬ ‫‪ n ‬‬
‫‪1‬‬
‫‪ 3‬‬ ‫‪n +1 ‬‬ ‫≤ ‪ ln ( n + 1) − ln n‬؛ ﻭﺒﺠﻤﻊ ﻁﺭﻓﺎ ﺇﻝﻰ ﻁﺭﻑ‬ ‫‪، ...‬‬
‫× ‪u n = ln  2 × × ...‬‬ ‫)‪ = ln ( n + 1‬‬ ‫‪n‬‬
‫‪ 2‬‬ ‫‪n ‬‬ ‫ﻨﺠﺩ ﻤﻥ ﺃﺠل ﻜل ‪ n‬ﻤﻥ ∗‪. ln ( n + 1) ≤ u n ، ℕ‬‬
‫‪ (u n ) (3‬ﻤﺘﺯﺍﻴﺩﺓ ﻭﻨﻬﺎﻴﺘﻬﺎ ∞‪ +‬ﺇﺫﻥ ﻫﻲ ﻤﺤﺩﻭﺩﺓ ﻤﻥ‬ ‫ﻝﺩﻴﻨﺎ ∞‪ lim ln ( n + 1) = +‬ﻭ ‪ ln ( n + 1) ≤ u n‬ﺇﺫﻥ‬
‫∞‪n →+‬‬
‫ﺍﻷﺴﻔل ﺒﹻ ‪ u 0 = ln 2‬ﻭﻝﻴﺴﺕ ﻤﺤﺩﻭﺩﺓ ﻤﻥ ﺍﻷﻋﻠﻰ‪.‬‬
‫∞‪. lim u n = +‬‬
‫∞‪n →+‬‬
‫‪1 1‬‬ ‫‪1‬‬
‫‪ (u n ) 128‬ﻤﻌﺭﻓﺔ ﺒِـ ‪u n = 1 + + 2 + ... + n :‬‬ ‫‪ (3‬ﺍﻝﺒﺭﻨﺎﻤﺞ ﺍﻝﺫﻱ ﻴﺤﺩ‪‬ﺩ ﺃﺼﻐﺭ ﻋﺩﺩ ﻁﺒﻴﻌﻲ ‪ n‬ﻴﺤﻘﻕ‪:‬‬
‫‪3 3‬‬ ‫‪3‬‬
‫‪.‬‬ ‫‪. u n ≥ 10‬‬
‫‪1‬‬
‫‪1−‬‬
‫= ‪ u n‬ﻭﻤﻨﻪ‬ ‫‪3n = 3 − 1‬‬ ‫‪(1‬‬
‫‪1 2 2 × 3n −1‬‬
‫‪1−‬‬
‫‪3‬‬
‫‪3‬‬ ‫‪3‬‬ ‫‪3‬‬ ‫‪1‬‬ ‫ﺃﺼﻐﺭ ﻋﺩﺩ ﻁﺒﻴﻌﻲ ‪ n‬ﻴﺤﻘﻕ ‪ u n ≥ 10‬ﻫﻭ ‪. n = 12367‬‬
‫‪ u n − = −‬ﺃﻱ ‪ u n − < 0‬ﻤﻌﻨﺎﻩ < ‪. u n‬‬
‫‪2‬‬ ‫‪2‬‬ ‫‪2‬‬ ‫‪2 × 3n −1‬‬ ‫‪ 4‬ـ ا
‪
""2‬ت ا
‪2‬ودة ‪.‬‬
‫‪3‬‬
‫ﻫﻭ ﻋﻨﺼﺭ ﺤﺎﺩ ﻤﻥ ﺍﻷﻋﻠﻰ ﻝﻠﻤﺘﺘﺎﻝﻴﺔ ) ‪. (u n‬‬ ‫ﺇﺫﻥ ﺍﻝﻌﺩﺩ‬ ‫‪ 126‬ﺍﻝﻤﺘﺘﺎﻝﻴﺘﺎﻥ ) ‪ (u n‬ﻭ ) ‪ (v n‬ﻤﻌﺭﻓﺘﺎﻥ ﻤﻥ ﺃﺠل ﻜل‬
‫‪2‬‬
‫‪1‬‬ ‫‪1‬‬ ‫‪1‬‬
‫‪ u n +1 − u n = n +1 (2‬ﺇﺫﻥ ‪u n +1 − u n > 0‬‬ ‫= ‪vn‬‬ ‫= ‪ un‬ﻭ‬ ‫ﻋﺩﺩ ﻁﺒﻴﻌﻲ ﻏﻴﺭ ﻤﻌﺩﻭﻡ ﺒِـ ‪:‬‬
‫‪3‬‬ ‫‪n‬‬ ‫‪n 2 +1‬‬
‫ﻭﻤﻨﻪ ﺍﻝﻤﺘﺘﺎﻝﻴﺔ ) ‪ (u n‬ﻤﺘﺯﺍﻴﺩﺓ ‪.‬‬ ‫‪.‬‬

‫) ‪ (u n‬ﻤﺘﺯﺍﻴﺩﺓ ﻭﻤﺤﺩﻭﺩﺓ ﻤﻥ ﺍﻷﻋﻠﻰ ﺇﺫﻥ ﻫﻲ ﻤﺘﻘﺎﺭﺒﺔ ‪.‬‬ ‫‪ (1‬ﻤﻥ ﺃﺠل ﻜل ∗‪ n 2 + 1 > 1 ، n ∈ ℕ‬ﻤﻌﻨﺎﻩ ‪n 2 + 1 > 1‬‬
‫‪1‬‬
‫‪. lim u n = lim‬‬
‫‪3‬‬
‫‪−‬‬
‫‪1‬‬ ‫‪3‬‬
‫‪= (3‬‬ ‫‪ ،‬ﺇﺫﻥ ‪ 1‬ﻋﻨﺼﺭ ﺤﺎﺩ ﻤﻥ ﺍﻷﻋﻠﻰ‬ ‫ﺃﻱ ‪< 1‬‬
‫∞‪n →+‬‬ ‫‪n →+∞ 2‬‬ ‫‪2×3‬‬ ‫‪n‬‬ ‫‪−‬‬‫‪1‬‬
‫‪2‬‬ ‫‪n 2 +1‬‬
‫‪ 129‬ﻝﺘﻜﻥ ﺍﻝﻤﺘﺘﺎﻝﻴﺔ ) ‪ (u n‬ﺍﻝﻤﻌﺭﻓﺔ ﺒﺤﺩﻫﺎ ﺍﻷﻭل ‪ u 0‬ﻭﻤﻥ‬ ‫ﻝﹻ ) ‪. (u n‬‬

‫ﺃﺠل ﻜل ﻋﺩﺩ ﻁﺒﻴﻌﻲ ‪. u n +1 = e −u n ، n‬‬ ‫‪ (2‬ﻤﻥ ﺃﺠل ﻜل ∗‪ n 2 + 1 > n 2 ، n ∈ ℕ‬ﻤﻌﻨﺎﻩ‬


‫ﻨﺴﺘﻌﻤل ﺍﻝﺒﺭﻫﺎﻥ ﺒﺎﻝﺘﺭﺍﺠﻊ ﻹﺜﺒﺎﺕ ﺃﻨﻪ ﻤﻥ ﺃﺠل ﻜل ﻋﺩﺩ‬ ‫‪1‬‬ ‫‪1‬‬
‫<‬ ‫‪ n 2 + 1 > n‬ﺃﻱ‬
‫‪n +1 n‬‬
‫‪2‬‬
‫ﻁﺒﻴﻌﻲ ‪. 0 < u n < 1 ، n ≥ 2‬‬
‫‪. 0 < u n < v n < 1 (3‬‬
‫ﻝﺩﻴﻨﺎ ‪ u 1 = e −u 0‬ﻭﻤﻥ ﺃﺠل ﻜل ﻋﺩﺩ ﺤﻘﻴﻘﻲ ‪e −u 0 > 0 u 0‬‬
‫ﺇﺫﻥ ‪ u 1 > 0‬ﻭﺒﻤﺎ ﺃﻥ ﺍﻝﺩﺍﻝﺔ ‪ x ֏ e − x‬ﻤﺘﻨﺎﻗﺼﺔ ﺘﻤﺎﻤﺎ‬ ‫‪ (u n ) 127‬ﻤﻌﺭﻓﺔ ﻤﻥ ﺃﺠل ﻜل ﻋﺩﺩ ﻁﺒﻴﻌﻲ ﻏﻴﺭ ﻤﻌﺩﻭﻡ‬
‫ﻋﻠﻰ ‪ ℝ‬ﻓﺈﻥ ‪ 0 < e −u1 < e −0‬ﺃﻱ ‪ 0 < e −u1 < 1‬ﻭﺒﺎﻝﺘﺎﻝﻲ‬ ‫‪ 1‬‬ ‫‪ 1‬‬
‫ﺒِـ ‪. u n = ln (1 + 1) + ln 1 +  + ... + ln 1 +  :‬‬
‫‪ 2‬‬ ‫‪ n‬‬
‫‪. 0 < u2 < 1‬‬
‫‪‬‬ ‫‪1 ‬‬ ‫∗‬
‫ﻨﻔﺘﺭﺽ ﺃﻨﻪ ﻤﻥ ﺃﺠل ‪ k‬ﻋﺩﺩ ﻁﺒﻴﻌﻲ ﻜﻴﻔﻲ ‪0 < u k < 1 ,‬‬ ‫‪u n +1 − u n = ln 1 +‬‬ ‫‪ (1‬ﻤﻥ ﺃﺠل ﻜل ‪ ، n ∈ ℕ‬‬
‫‪‬‬ ‫‪n‬‬ ‫‪+‬‬ ‫‪1‬‬ ‫‪‬‬
‫ﻭﻝﻨﺒﺭﻫﻥ ﺃﻥ ‪. 0 < u k +1 < 1‬‬
‫‪ln10−6‬‬ ‫ﺒﻤﺎ ﺃﻥ ﺍﻝﺩﺍﻝﺔ ‪ x ֏ e − x‬ﻤﺘﻨﺎﻗﺼﺔ ﺘﻤﺎﻤﺎ ﻋﻠﻰ ‪ ℝ‬ﻓﺈﻨﻪ ﻤﻥ‬
‫> ‪ n‬ﻭﻤﻨﻪ‬ ‫)‪ n ln 0, 25 < ln ( 0, 000001‬ﺃﻱ‬
‫‪ln 0, 25‬‬ ‫ﻓﺭﻀﻴﺔ ﺍﻝﺘﺭﺍﺠﻊ ‪ 0 < u k < 1‬ﻴﻨﺘﺞ ‪e −0 > e −u k > e −1 :‬‬
‫‪ n ≥ 10‬ﺇﺫﻥ ﻝﻴﺱ ‪ 1,333333‬ﻋﻨﺼﺭﺍ ﺤﺎﺩﺍ ﻝﻠﻤﺘﺘﺎﻝﻴﺔ ) ‪. (u n‬‬ ‫‪1‬‬
‫ﺃﻱ ‪ ، < e −u k < 1‬ﻭﻤﻥ ﺘﻌﺭﻴﻑ ﺍﻝﻤﺘﺘﺎﻝﻴﺔ ﻝﺩﻴﻨﺎ ‪u k +1 = e −u k‬‬
‫‪ (u n ) 131‬ﺍﻝﻤﻌﺭﻓﺔ ﺒﹻ ‪ u 0‬ﻋﺩﺩ ﺤﻘﻴﻘﻲ ﻤﻌﻁﻰ ﻭ ﻤﻥ‬ ‫‪e‬‬
‫‪1‬‬
‫ﺃﺠل ﻜل ﻋﺩﺩ ﻁﺒﻴﻌﻲ ‪. u n +1 = u n 2 − 3u n + 5 ، n‬‬ ‫ﺇﺫﻥ ‪ < u k +1 < 1‬ﻭﻤﻨﻪ ‪. 0 < u k +1 < 1‬‬
‫‪e‬‬
‫‪u n +1 − u n − 1 = u n 2 − 4u n + 4 = (u n + 2 ) (1‬‬
‫‪2‬‬
‫ﺇﺫﻥ ﺤﺴﺏ ﻤﺒﺩﺃ ﺍﻝﺘﺭﺍﺠﻊ ﻨﺴﺘﻨﺘﺞ ﺃﻥ ﻤﻥ ﺃﺠل ﻜل ﻋﺩﺩ‬
‫ﺇﺫﻥ ﻤﻥ ﺃﺠل ﻜل ﻋﺩﺩ ﻁﺒﻴﻌﻲ ‪، u n +1 − u n ≥ 1 ، n‬‬ ‫ﻁﺒﻴﻌﻲ ‪. 0 < u n < 1 , n ≥ 2‬‬
‫ﻨﺴﺘﻨﺘﺞ ﺃﻥ ) ‪ (u n‬ﻤﺘﺯﺍﻴﺩﺓ‪.‬‬
‫‪1 1‬‬ ‫‪1‬‬
‫‪ (u n ) 130‬ﻤﻌﺭﻓﺔ ﺒﹻ ‪. u n = 1 + + 2 + ... + n :‬‬
‫‪4 4‬‬ ‫‪4‬‬
‫‪ (2‬ﺇﺫﺍ ﻜﺎﻨﺕ ) ‪ (u n‬ﻤﺘﻘﺎﺭﺒﺔ ﻭﻨﻬﺎﻴﺘﻬﺎ ‪ l‬ﻓﺈﻥ‬ ‫‪1‬‬
‫‪ (1‬ﻤﻥ ﺃﺠل ﻜل ﻋﺩﺩ ﻁﺒﻴﻌﻲ ‪ n‬ﻝﺩﻴﻨﺎ ‪u n +1 = u n + n +1 :‬‬
‫‪ l = l 2 − 3l + 5‬ﻤﻌﻨﺎﻩ ‪. l 2 − 4l + 5 = 0‬‬ ‫‪4‬‬
‫‪1‬‬
‫‪ (3‬ﺍﻝﻤﻤﻴﺯ ﺍﻝﻤﺨﺘﺼﺭ ﻝﹻ ‪ l 2 − 4l + 5 = 0‬ﻫﻭ ‪ −1‬ﺇﺫﻥ ﻻ‬ ‫ﻭﻤﻨﻪ ‪ u n +1 − u n = n +1‬؛ ﺇﺫﻥ ﻤﻥ ﺃﺠل ﻜل ‪، n ∈ ℕ‬‬
‫‪4‬‬
‫ﻴﻭﺠﺩ ﺃﻱ ﻋﺩﺩ ﺤﻘﻴﻘﻲ ‪ l‬ﻴﺤﻘﻕ ﺍﻝﻤﻌﺎﺩﻝﺔ ‪l 2 − 4l + 5 = 0‬‬ ‫‪ ، u n +1 − u n > 0‬ﻭﺒﺎﻝﺘﺎﻝﻲ ) ‪ (u n‬ﻤﺘﺯﺍﻴﺩﺓ ﺘﻤﺎﻤﺎ ‪.‬‬
‫ﻭﻤﻨﻪ ﺍﻝﻤﺘﺘﺎﻝﻴﺔ ) ‪ (u n‬ﻤﺘﺒﺎﻋﺩﺓ‪.‬‬
‫‪ u n (2‬ﻫﻭ ﻤﺠﻤﻭﻉ ﺤﺩﻭﺩ ﻤﺘﺘﺎﺒﻌﺔ ﻝﻠﻤﺘﺘﺎﻝﻴﺔ ﺍﻝﻬﻨﺩﺴﻴﺔ ﺫﺍﺕ‬
‫ﺇﺫﺍ ﻜﺎﻨﺕ ) ‪ (u n‬ﻤﺤﺩﻭﺩﺓ ﺒﻤﺎ ﺃﻨﻬﺎ ﻤﺘﺯﺍﻴﺩﺓ ﻓﺘﻜﻭﻥ ﻤﺘﻘﺎﺭﺒﺔ‬ ‫‪1‬‬
‫ﺍﻷﺴﺎﺱ ﻭﺍﻝﺤﺩ ﺍﻷﻭل ‪ 1‬؛‬
‫ﻭﻫﺫﺍ ﺘﻨﺎﻗﺽ ﺇﺫﻥ ) ‪ (u n‬ﻝﻴﺴﺕ ﻤﺤﺩﻭﺩﺓ ﻤﻥ ﺍﻷﻋﻠﻰ‪.‬‬ ‫‪4‬‬
‫‪n +1‬‬
‫‪1‬‬
‫ﻤﻤﺎ ﺴﺒﻕ ﻨﺴﺘﻨﺘﺞ ﺃﻥ ∞‪lim u n = +‬‬ ‫‪  −1‬‬ ‫‪4  1 ‬‬
‫‪n +1‬‬
‫‪‬‬
‫∞‪n →+‬‬
‫= ‪ u n‬ﺃﻱ‬ ‫‪‬‬ ‫‪4‬‬
‫ﺇﺫﻥ ‪= −   − 1‬‬
‫‪ 132‬ﻝﺘﻜﻥ ﺍﻝﻤﺘﺘﺎﻝﻴﺔ ) ‪ (u n‬ﺍﻝﻤﻌﺭﻓﺔ ﺒﹻ ‪ u 0 = 11‬ﻭﻤﻥ ﺃﺠل‬
‫‪1‬‬ ‫‪3  4 ‬‬
‫‪−1‬‬ ‫‪‬‬
‫‪4‬‬ ‫‪4‬‬
‫ﻜل ﻋﺩﺩ ﻁﺒﻴﻌﻲ ‪. u n +1 = 3u n − 4 ، n‬‬ ‫‪4 41‬‬
‫‪n +1‬‬
‫‪4 11‬‬
‫‪n‬‬

‫‪ . u n = −  ‬ﺒﻤﺎ ﺃﻥ‬ ‫‪= −  ‬‬


‫‪35‬‬ ‫‪17‬‬ ‫‪3 34‬‬ ‫‪3 3 4‬‬
‫= ‪. u2‬‬ ‫= ‪ u1‬ﻭ‬ ‫‪(1‬‬ ‫‪n‬‬
‫‪4‬‬ ‫‪4‬‬ ‫‪4‬‬ ‫‪1‬‬ ‫‪1‬‬
‫= ‪. lim u n‬‬ ‫‪ −1 < < 1‬ﻓﺈﻥ ‪ lim   = 0‬ﻭﻤﻨﻪ‬
‫‪ (2‬ﺍﺴﺘﻌﻤﺎل ﺍﻻﺴﺘﺩﻻل ﺒﺎﻝﺘﺭﺍﺠﻊ ﻝﻠﺒﺭﻫﺎﻥ ﻋﻠﻰ ﺍﻝﺨﺎﺼﻴﺔ‬ ‫∞‪n →+‬‬ ‫‪3‬‬ ‫‪ ‬‬
‫‪n →+∞ 4‬‬ ‫‪4‬‬
‫‪. u n +1 ≥ u n‬‬ ‫‪ (3‬ﺒﻤﺎ ﺃﻥ ﺍﻝﻤﺘﺘﺎﻝﻴﺔ ) ‪ (u n‬ﻤﺘﺯﺍﻴﺩﺓ ﺘﻤﺎﻤﺎ ﻓﺈﻨﻬﺎ ﻤﺤﺩﻭﺩﺓ ﻤﻥ‬

‫‪ (v n ) (3‬ﻤﻌﺭﻓﺔ ﻋﻠﻰ ‪ ℕ‬ﺒﹻ ‪ v n = 4u n + α :‬ﺤﻴﺙ ‪α‬‬ ‫ﺍﻷﺴﻔل ﺒﺤﺩﻫﺎ ﺍﻷﻭل ‪ . u 0‬ﺯﻴﺎﺩﺓ ﻋﻥ ﻫﺫﺍ ﻓﺈﻨﻬﺎ ﺘﺘﻘﺎﺭﺏ‬
‫‪4‬‬ ‫‪4‬‬
‫ﻋﺩﺩ ﺤﻘﻴﻘﻲ ‪.‬‬ ‫ﻭﺒﺎﻝﺘﺎﻝﻲ ﻝﺩﻴﻨﺎ‬ ‫ﺇﻝﻰ ﺇﺫﻥ ﻫﻲ ﻤﺤﺩﻭﺩﺓ ﻤﻥ ﺍﻷﻋﻠﻰ ﺒﺎﻝﻌﺩﺩ‬
‫‪3‬‬ ‫‪3‬‬
‫‪ v n +1 = 4u n +1 + α = 4 ( 3u n − 4 ) + α‬؛‬ ‫ﺃـ‬ ‫‪4‬‬
‫ﻤﻥ ﺃﺠل ﻜل ‪. 1 ≤ u n < , n ∈ ℕ‬‬
‫‪v − α ‬‬ ‫‪3‬‬
‫‪ v n +1 = 12u n − 16 + α = 12  n‬؛‬ ‫‪ − 16 + α‬‬ ‫‪4 11‬‬
‫‪n‬‬
‫‪ 4 ‬‬ ‫‪ u n > 1,333333‬ﻤﻌﻨﺎﻩ ‪−   > 1,333333‬‬
‫‪3 3 4‬‬
‫‪v n +1 = 3v n − 16 − 2α‬‬ ‫‪n‬‬
‫‪11‬‬ ‫‪4‬‬
‫ﺘﻜﻭﻥ ﺍﻝﻤﺘﺘﺎﻝﻴﺔ ) ‪ (v n‬ﻫﻨﺩﺴﻴﺔ ﺇﺫﺍ ﻭﻓﻘﻁ ﺇﺫﺍ ﻜﺎﻥ ‪. α = −8‬‬ ‫ﻭﻤﻌﻨﺎﻩ ‪   < − 1, 333333‬ﻭﻴﻜﺎﻓﺊ‬
‫‪3 4‬‬ ‫‪3‬‬
‫ﺏ ـ ‪ v n = 4u n − 8‬ﻭﻤﻨﻪ ‪، v 0 = 4u 0 − 8 = 3‬‬ ‫‪1‬‬
‫‪n‬‬

‫‪   < 4 − 3, 999999‬ﻭﻤﻌﻨﺎﻩ‬
‫‪4‬‬
‫=‬
‫‪1‬‬
‫‪+‬‬
‫‪1‬‬
‫‪−‬‬
‫‪1‬‬ ‫‪v n + 8 3n +1 + 8‬‬
‫= ‪.u n‬‬ ‫=‬ ‫‪ v n = 3n +1‬؛‬
‫! ‪( n + 1)! ( n + 1) × ( n + 1) ! n × n‬‬ ‫‪4‬‬ ‫‪4‬‬
‫=‬
‫‪1‬‬
‫‪+‬‬
‫‪1‬‬
‫‪−‬‬
‫‪1‬‬
‫= ‪u0‬‬
‫‪11‬‬
‫ﺝ ـ ) ‪ (u n‬ﻤﺘﺯﺍﻴﺩﺓ ﺇﺫﻥ ﻤﺤﺩﻭﺩﺓ ﻤﻥ ﺍﻷﺴﻔل ﺒﹻ‬
‫! ‪( n + 1) × n ! ( n + 1) × ( n + 1) × n ! n × n‬‬ ‫‪4‬‬
‫ﻭﻝﺩﻴﻨﺎ ∞‪ lim u n = +‬ﺇﺫﻥ ) ‪ (u n‬ﻝﻴﺴﺕ ﻤﺤﺩﻭﺩﺓ ﻤﻥ‬
‫)‪n ( n + 1) + n − ( n + 1‬‬
‫‪2‬‬
‫∞‪n →+‬‬
‫=‬
‫! ‪n ( n + 1) × n‬‬ ‫ﺍﻷﻋﻠﻰ ﻭﺒﺎﻝﺘﺎﻝﻲ ﻫﻲ ﻝﻴﺴﺕ ﻤﺤﺩﻭﺩﺓ‪.‬‬
‫‪2‬‬

‫‪n 2 + n + n − n 2 − 2n − 1‬‬ ‫‪−1‬‬ ‫ﺩ ـ ﻨﻀﻊ ﻤﻥ ﺃﺠل ﻜل ﻋﺩﺩ ﻁﺒﻴﻌﻲ ‪، n‬‬


‫=‬ ‫=‬
‫! ‪n ( n + 1) × n‬‬ ‫! ‪n ( n + 1) × n‬‬
‫‪2‬‬ ‫‪2‬‬
‫‪u1 u 2‬‬ ‫‪u‬‬
‫‪. w n = u0 +‬‬ ‫‪+ 2 + ... + nn‬‬
‫ﺇﺫﻥ ‪ v n +1 − v n < 0‬ﻭﺒﺎﻝﺘﺎﻝﻲ ) ‪ (v n‬ﻤﺘﻨﺎﻗﺼﺔ ﺘﻤﺎﻤﺎ ‪.‬‬ ‫‪4 4‬‬ ‫‪4‬‬
‫‪1‬‬ ‫‪1‬‬ ‫‪3 + 8 32 + 8 33 + 8‬‬ ‫‪3n +1 + 8‬‬
‫= ‪ v n − u n‬ﻭﻤﻨﻪ‬ ‫‪ v n = u n +‬ﻤﻌﻨﺎﻩ‬ ‫= ‪،w n‬‬ ‫‪+ 2 + 3 + ... + n +1‬‬
‫!‪n ×n‬‬ ‫!‪n ×n‬‬ ‫‪4‬‬ ‫‪4‬‬ ‫‪4‬‬ ‫‪4‬‬
‫‪1‬‬ ‫‪n +1‬‬
‫‪. lim (v n − u n ) = lim‬‬ ‫‪=0‬‬ ‫‪ 3  3 2‬‬ ‫‪3 ‬‬
‫∞‪n →+‬‬ ‫! ‪n →+∞ n × n‬‬ ‫‪w n =  +   + ... +    +‬‬
‫‪4 4‬‬ ‫‪ 4  ‬‬
‫‪‬‬
‫ﺨﻼﺼﺔ ‪ (u n ) :‬ﻭ ) ‪ (v n‬ﻤﺘﺘﺎﻝﻴﺘﺎﻥ ﻤﺘﺠﺎﻭﺭﺘﺎﻥ ‪.‬‬
‫‪1 1‬‬ ‫‪1 ‬‬
‫‪ (2‬ﺒﻤﺎ ﺃﻥ ) ‪ (u n‬ﻤﺘﺯﺍﻴﺩﺓ ﻭ ) ‪ (v n‬ﻤﺘﻨﺎﻗﺼﺔ ﻓﺈﻨﻪ ﻤﻥ ﺃﺠل‬ ‫‪8  + 2 + ... + n +1 ‬‬
‫‪4 4‬‬ ‫‪4 ‬‬
‫ﻋﺩﺩ ﻁﺒﻴﻌﻲ ﻏﻴﺭ ﻤﻌﺩﻭﻡ ‪ u n ≤ l ≤ v n ، n‬ﺃﻱ‬ ‫‪n +1‬‬ ‫‪n +1‬‬
‫‪3‬‬ ‫‪1‬‬
‫‪1‬‬ ‫‪1−  ‬‬ ‫‪1−  ‬‬
‫‪un ≤ l ≤ un +‬‬ ‫= ‪،w n‬‬
‫‪3 4‬‬
‫‪+‬‬
‫‪8 4‬‬
‫!‪n ×n‬‬ ‫‪4 1− 3‬‬ ‫‪4 1− 1‬‬
‫‪−3‬‬
‫ﻝﻜﻲ ﻴﻜﻭﻥ ‪ u n‬ﻗﻴﻤﺔ ﻤﻘﺭﺒﺔ ﺒﺎﻝﻨﻘﺼﺎﻥ ﺇﻝﻰ ‪ 10‬ﻝﻠﻌﺩﺩ ‪l‬‬ ‫‪4‬‬ ‫‪4‬‬
‫‪1‬‬ ‫‪  3 n +1  8   1 n +1 ‬‬
‫ﺃﻱ ‪n × n ! ≥ 1000‬‬ ‫ﻴﻜﻔﻲ ﺃﻥ ﻴﻜﻭﻥ ‪≤ 10−3‬‬
‫!‪n ×n‬‬ ‫‪،w n = 3  1 −    +  1 −   ‬‬
‫‪  4  3  4 ‬‬
‫ﻝﺩﻴﻨﺎ ‪ 5 × 5! = 600‬ﻭ ‪ 6 × 6! = 4320‬ﻭﻫﺫﺍ ﻴﺒﻴﻥ ﺃﻥ ‪u 6‬‬ ‫‪‬‬ ‫‪‬‬ ‫‪‬‬ ‫‪‬‬
‫‪8 17‬‬
‫ﻫﻭ ﺃﻗﺭﺏ ﻗﻴﻤﺔ ﺒﺎﻝﻨﻘﺼﺎﻥ ﺇﻝﻰ ‪ 10−3‬ﻝﻠﻌﺩﺩ ‪. l‬‬ ‫= ‪ lim w n = 3 +‬ﺇﺫﻥ ) ‪ (w n‬ﻤﺘﻘﺎﺭﺒﺔ ‪.‬‬
‫∞‪n →+‬‬ ‫‪3 3‬‬
‫‪1957‬‬
‫= ‪u6‬‬ ‫‪, u 6 ≃ 2, 718055556‬‬ ‫‪ 5‬ـ ا
‪"
""2‬ن ا
‪,"2‬ورن ‪.‬‬
‫‪720‬‬
‫‪ 133‬ﻝﺘﻜﻥ ) ‪ (u n‬ﻭ ) ‪ (v n‬ﺍﻝﻤﺘﺘﺎﻝﻴﺘﻴﻥ ﺍﻝﻤﻌﺭﻓﺘﻴﻥ ﻋﻠﻰ ∗‪ℕ‬‬
‫‪1‬‬ ‫‪1 1‬‬ ‫‪1‬‬
‫‪.v n = u n +‬‬ ‫ِـ ‪ u n = 1 + + + ... + :‬ﻭ‬
‫!‪n ×n‬‬ ‫!‪1! 2‬‬ ‫!‪n‬‬
‫‪1‬‬
‫‪ u n +1 = u n +‬ﻭﻤﻨﻪ‬ ‫‪ (1‬ﻝﻴﻜﻥ ‪ n‬ﻋﺩﺩﺍ ﻁﺒﻴﻌﻴﺎ ‪،‬‬
‫‪ v 0 = 2 ، u 0 = 0 134‬ﻭﻤﻥ ﺃﺠل ﻜل ﻋﺩﺩ ﻁﺒﻴﻌﻲ ‪: n‬‬ ‫!)‪( n + 1‬‬
‫‪3v + 1‬‬ ‫‪3u + 1‬‬ ‫‪1‬‬
‫‪. v n +1 = n‬‬ ‫‪ u n +1 = n‬ﻭ‬ ‫= ‪ u n +1 − u n‬ﻭﺒﺎﻝﺘﺎﻝﻲ ‪ u n +1 − u n > 0‬ﺇﺫﻥ‬
‫‪4‬‬ ‫‪4‬‬ ‫! )‪( n + 1‬‬
‫‪ (1‬ﺍﻝﺨﺎﺼﻴﺔ ﺍﻻﺒﺘﺩﺍﺌﻴﺔ ‪ u 0 ≤ 1 ≤ v 0‬؛ ﻨﻔﺭﺽ ‪u k ≤ 1 ≤ v k‬‬ ‫) ‪ (u n‬ﻤﺘﺯﺍﻴﺩﺓ ﺘﻤﺎﻤﺎ ‪.‬‬
‫ﻤﻌﻨﺎﻩ ‪ 3u k ≤ 3 ≤ 3v k‬ﻭﻤﻌﻨﺎﻩ ‪3u k + 1 ≤ 4 ≤ 3v k + 1‬‬ ‫ﻝﻴﻜﻥ ‪ n‬ﻋﺩﺩﺍ ﻁﺒﻴﻌﻴﺎ ‪،‬‬
‫‪3u + 1‬‬ ‫‪3v + 1‬‬ ‫‪1‬‬ ‫‪1‬‬
‫‪. k‬‬ ‫‪≤1≤ k‬‬ ‫ﺃﻱ‬ ‫‪v n +1 − v n = u n +1 − u n +‬‬ ‫‪−‬‬
‫‪4‬‬ ‫‪4‬‬ ‫! ‪( n + 1) × ( n + 1)! n × n‬‬
‫ﻭﻜﺫﻝﻙ ﻝﺩﻴﻨﺎ ﻤﻥ ﺃﺠل ﻜل ﻋﺩﺩ ﻁﺒﻴﻌﻲ ‪: n‬‬ ‫‪3u n + 1‬‬ ‫‪1−un‬‬
‫= ‪ u n +1 − u n‬ﺒﻤﺎ ﺃﻥ ‪u n ≤ 1‬‬ ‫= ‪−un‬‬ ‫‪(2‬‬
‫‪u + 4v n‬‬ ‫‪u + 4v n‬‬ ‫‪4‬‬ ‫‪4‬‬
‫‪v n +1 − v n = n‬‬ ‫‪ v n +1 = n‬ﻭﻤﻨﻪ ‪− v n‬‬
‫‪5‬‬ ‫‪5‬‬ ‫ﻓﺈﻥ ‪ u n +1 − u n ≥ 0‬ﻭﻤﻨﻪ ) ‪ (u n‬ﻤﺘﺯﺍﻴﺩﺓ‪.‬‬
‫‪u n −v n w n‬‬ ‫‪3v n + 1‬‬ ‫‪1 −v n‬‬
‫= ‪. v n +1 − v n‬‬ ‫=‬ ‫ﺃﻱ‬ ‫= ‪ v n +1 − v n‬ﺒﻤﺎ ﺃﻥ ‪ 1 ≤ v n‬ﻓﺈﻥ‬ ‫= ‪−v n‬‬
‫‪5‬‬ ‫‪5‬‬ ‫‪4‬‬ ‫‪4‬‬
‫ﻝﺩﻴﻨﺎ ﻤﻥ ﺃﺠل ﻜل ﻋﺩﺩ ﻁﺒﻴﻌﻲ ‪ w n < 0 : n‬ﺇﺫﻥ‬ ‫‪ v n +1 − v n ≤ 0‬ﻭﻤﻨﻪ ) ‪ (v n‬ﻤﺘﻨﺎﻗﺼﺔ‪.‬‬
‫‪ u n +1 − u n > 0‬ﻭ ‪ v n +1 − v n < 0‬ﻭﻤﻨﻪ ) ‪ (u n‬ﻤﺘﺯﺍﻴﺩﺓ‬ ‫‪3u n + 1 3v n + 1 3‬‬
‫= ‪ u n +1 −v n +1‬ﻭﻤﻨﻪ‬ ‫‪−‬‬ ‫) ‪= (u n −v n‬‬
‫ﺘﻤﺎﻤﺎ ﻭ ) ‪ (v n‬ﻤﺘﻨﺎﻗﺼﺔ ﺘﻤﺎﻤﺎ‪.‬‬ ‫‪4‬‬ ‫‪4‬‬ ‫‪4‬‬
‫ﺒﻤﺎ ﺃﻥ ) ‪ (u n‬ﻤﺘﺯﺍﻴﺩﺓ ‪ (v n ) ،‬ﻤﺘﻨﺎﻗﺼﺔ‬ ‫ﺍﻝﻤﺘﺘﺎﻝﻴﺔ ) ‪ (w n‬ﺍﻝﻤﻌﺭﻓﺔ ﺒﹻ ‪ w n = u n − v n‬ﻫﻨﺩﺴﻴﺔ‬

‫) ‪ (u n‬ﻭ ) ‪(v n‬‬ ‫ﻭ ‪ lim (u n − v n ) = lim w n = 0‬ﻓﺈﻥ‬


‫‪n‬‬
‫‪3‬‬ ‫‪3‬‬
‫∞‪n →+‬‬ ‫∞‪n →+‬‬ ‫‪ u n − v n = (u 0 − v 0 )  ‬ﺇﺫﻥ‬ ‫ﻭﻋﻠﻴﻪ‬ ‫ﺃﺴﺎﺴﻬﺎ‬
‫‪4‬‬ ‫‪4‬‬
‫ﻤﺘﺠﺎﻭﺭﺘﺎﻥ ﻭﺒﺎﻝﺘﺎﻝﻲ ﻝﻬﻤﺎ ﻨﻔﺱ ﺍﻝﻨﻬﺎﻴﺔ ﺍﻝﻌﺩﺩ ﺍﻝﺤﻘﻴﻘﻲ ‪. l‬‬
‫‪n‬‬
‫‪3‬‬
‫‪ (4‬ﻤﻥ ﺃﺠل ﻜل ﻋﺩﺩ ﻁﺒﻴﻌﻲ ‪ t n = 3u n + 10v n ، n‬ﻭﻤﻨﻪ‬ ‫) ‪(u n‬‬ ‫‪ lim (u n − v n ) = lim 2   = 0‬؛ ﺇﺫﻥ‬
‫∞‪n →+‬‬ ‫∞‪n →+‬‬
‫‪4‬‬
‫‪t n +1 − t n = 3u n +1 + 10v n +1 − 3u n − 10v n‬‬
‫ﻭ ) ‪ (v n‬ﻤﺘﺠﺎﻭﺭﺘﺎﻥ ‪.‬‬
‫‪= 3 (u n +1 − u n ) + 10 (v n +1 − v n ) = −2w n + 2w n = 0‬‬
‫‪3u n + 1‬‬
‫ﻭﺒﺎﻝﺘﺎﻝﻲ ﺍﻝﻤﺘﺘﺎﻝﻴﺔ ) ‪ (t n‬ﺜﺎﺒﺘﺔ ‪.‬‬ ‫= ‪ u n +1‬ﻤﻌﻨﺎﻩ ‪ 4u n +1 − 3u n = 1‬ﺇﺫﻥ‬
‫‪4‬‬
‫ﻭﻤﻨﻪ ﻤﻥ ﺃﺠل ﻜل ﻋﺩﺩ ﻁﺒﻴﻌﻲ ‪ t n = t 0 , n‬ﺃﻱ‬ ‫‪ lim ( 4u n +1 − 3u n ) = 1‬ﺃﻱ ‪. l = 1‬‬
‫∞‪n →+‬‬
‫‪ 3u n + 10v n = 3u 0 + 10v 0 = 23‬ﺇﺫﻥ‬
‫‪ v 0 = 2 ، u 0 = 1 135‬ﻭﻤﻥ ﺃﺠل ﻜل ﻋﺩﺩ ﻁﺒﻴﻌﻲ ‪: n‬‬
‫‪ lim ( 3u n + 10v n ) = 23‬ﻫﺫﺍ ﻤﻥ ﺠﻬﺔ‪ .‬ﻭﻤﻥ ﺠﻬﺔ‬
‫∞‪n →+‬‬ ‫‪u + 4v n‬‬ ‫‪u + 2v n‬‬
‫‪. v n +1 = n‬‬ ‫‪، u n +1 = n‬‬
‫ﺃﺨﺭﻯ ‪ lim ( 3u n + 10v n ) = 3l + 10l = 13l‬ﺇﺫﻥ‬ ‫‪5‬‬ ‫‪3‬‬
‫∞‪n →+‬‬

‫‪23‬‬ ‫‪w n +1 = u n +1 − v n +1‬‬ ‫‪ (1‬ﻝﻴﻜﻥ ‪ n‬ﻋﺩﺩﺍ ﻁﺒﻴﻌﻴﺎ ‪،‬‬


‫=‪. l‬‬ ‫‪ 13l = 23‬ﻭﺒﺎﻝﺘﺎﻝﻲ‬
‫‪13‬‬ ‫‪u + 2v n u n + 4v n 2u n − 2v n‬‬
‫‪= n‬‬ ‫‪−‬‬ ‫=‬
‫‪ v 0 = 4 ، u 0 = 3 136‬ﻭﻤﻥ ﺃﺠل ﻜل ﻋﺩﺩ ﻁﺒﻴﻌﻲ ‪، n‬‬ ‫‪3‬‬ ‫‪5‬‬ ‫‪15‬‬
‫‪u +v‬‬ ‫‪u +v n‬‬ ‫‪2‬‬ ‫‪2‬‬
‫ﺃﻱ ‪ ، w n +1 = (u n −v n ) = w n‬ﺇﺫﻥ ) ‪(w n‬‬
‫‪ u n +1 = n‬؛ ‪. v n +1 = n +1 n‬‬ ‫‪15‬‬ ‫‪15‬‬
‫‪2‬‬ ‫‪2‬‬
‫‪2‬‬
‫‪59‬‬ ‫‪29‬‬ ‫‪15‬‬ ‫‪7‬‬ ‫ﻫﻨﺩﺴﻴﺔ ﺃﺴﺎﺴﻬﺎ ﻭﺤﺩﻫﺎ ﺍﻷﻭل ‪ w 0 = u 0 − v 0 = −1‬ﻭﺒﻤﺎ‬
‫= ‪. v2‬‬ ‫= ‪ u2‬ﻭ‬ ‫= ‪، v1‬‬ ‫‪، u 1 = (1‬‬ ‫‪15‬‬
‫‪16‬‬ ‫‪8‬‬ ‫‪4‬‬ ‫‪2‬‬ ‫‪n‬‬
‫‪2‬‬ ‫‪2‬‬
‫‪ (2‬ﻤﻥ ﺃﺠل ﻜل ﻋﺩﺩ ﻁﺒﻴﻌﻲ ‪ ، n‬ﻨﻀﻊ ‪. w n = v n − u n :‬‬ ‫ﺃﻥ ‪ −1 < < 1‬ﻓﺈﻥ ‪. w n = −   . lim w n = 0‬‬
‫‪ 15 ‬‬ ‫‪n‬‬ ‫∞‪→+‬‬ ‫‪15‬‬
‫‪u n +1 + v n‬‬ ‫‪v − u n +1‬‬ ‫‪u + 2v n‬‬
‫= ‪w n +1 = v n +1 − u n +1‬‬ ‫‪− u n +1 = n‬‬ ‫‪u n +1 = n‬‬ ‫‪ (2‬ﻝﺩﻴﻨﺎ ﻤﻥ ﺃﺠل ﻜل ﻋﺩﺩ ﻁﺒﻴﻌﻲ ‪: n‬‬
‫‪2‬‬ ‫‪2‬‬ ‫‪3‬‬
‫‪v −un 1‬‬ ‫‪u + 2v n‬‬ ‫‪−2u n + 2v n‬‬
‫‪ w n +1 = n‬ﺇﺫﻥ ) ‪ (w n‬ﻫﻨﺩﺴﻴﺔ ‪.‬‬ ‫‪= wn‬‬ ‫‪ u n +1 − u n = n‬ﺃﻱ‬ ‫= ‪−un‬‬ ‫ﺇﺫﻥ‬
‫‪4‬‬ ‫‪4‬‬ ‫‪3‬‬ ‫‪3‬‬
‫‪n‬‬ ‫‪n‬‬
‫‪2‬‬ ‫‪2‬‬
‫‪1‬‬ ‫‪1‬‬
‫‪lim w n = lim w 0   = lim   = 0‬‬ ‫‪. u n +1 − u n = − (u n − v n ) = − w n‬‬
‫∞‪n →+‬‬ ‫∞‪n →+‬‬
‫‪4‬‬ ‫‪ ‬‬
‫‪n →+∞ 4‬‬ ‫‪3‬‬ ‫‪3‬‬
‫‪u +v n‬‬ ‫‪v −un w n‬‬
‫‪u n +1 − u n = n‬‬ ‫‪−un = n‬‬ ‫=‬ ‫‪(3‬‬
‫‪2‬‬ ‫‪2‬‬ ‫‪2‬‬
‫ﺇﺫﺍ ﻜﺎﻥ ‪ 1 ≤ u k ≤ 2‬ﻓﺈﻥ ‪ 1 ≤ f (u k ) ≤ 2‬ﺃﻱ‬ ‫= ‪v n +1 − v n‬‬
‫‪u n +1 + v n‬‬ ‫‪u −v‬‬
‫‪− v n = n +1 n = n‬‬
‫‪u −v n‬‬
‫‪2‬‬ ‫‪2‬‬ ‫‪4‬‬
‫‪. 1 ≤ u k +1 ≤ 2‬‬
‫‪w‬‬
‫* " ‪ " 1 ≤ v n ≤ 2‬؛ ﻨﻔﺱ ﺍﻝﺒﺭﻫﺎﻥ‪.‬‬ ‫‪ v n +1 − v n = − n‬ﺒﻤﺎ ﺃﻥ ‪ w n > 0‬ﻓﺈﻥ ) ‪ (u n‬ﻤﺘﺯﺍﻴﺩﺓ‬
‫‪4‬‬
‫‪3‬‬ ‫ﺘﻤﺎﻤﺎ ﻭ ) ‪ (v n‬ﻤﺘﻨﺎﻗﺼﺔ ﺘﻤﺎﻤﺎ ‪.‬‬
‫= ‪ u 1‬ﺇﺫﻥ ‪. u 0 ≤ u 1‬‬ ‫* " ‪ " u n ≤ u n +1‬؛ ‪ u 0 = 1‬ﻭ‬
‫‪2‬‬
‫ﺇﺫﻥ ) ‪(u n‬‬ ‫ﻭﻝﺩﻴﻨﺎ ‪lim (v n − u n ) = lim w n = 0‬‬
‫ﺇﺫﺍ ﻜﺎﻥ ‪ u k ≤ u k +1‬ﻓﺈﻥ ) ‪ f (u k ) ≤ f (u k +1‬ﻷﻥ ‪f‬‬ ‫∞‪n →+‬‬ ‫∞‪n →+‬‬

‫ﻭ ) ‪ (v n‬ﻤﺠﺎﻭﺭﺘﺎﻥ ‪.‬‬
‫ﻤﺘﺯﺍﻴﺩﺓ ﻭﻤﻨﻪ ‪. u k +1 ≤ u k + 2‬‬
‫‪ (4‬ﺘﺤﺫﻑ )ﺒﺭﻫﻥ ﺃﻥ( ﻤﻥ ﺍﻝﻤﻌﻁﻴﺎﺕ ‪.‬‬
‫* " ‪ " v n ≥ v n +1‬؛ ﻨﻔﺱ ﺍﻝﺒﺭﻫﺎﻥ‪.‬‬
‫‪1‬‬ ‫‪1‬‬
‫‪ (4‬ﻤﻥ ﺃﺠل ﻜل ﻋﺩﺩ ﻁﺒﻴﻌﻲ ‪ n‬ﻝﺩﻴﻨﺎ ‪:‬‬ ‫= ‪t n +1‬‬ ‫) ‪(u n +1 + 2v n +1 ) = ( 2u n +1 + v n‬‬
‫‪3‬‬ ‫‪3‬‬
‫‪2v + 1 2u n + 1‬‬ ‫‪v n −un‬‬
‫‪.v n +1 − u n +1 = n‬‬ ‫‪−‬‬ ‫=‬ ‫‪1‬‬
‫)‪v n + 1 u n + 1 (v n + 1)(u n + 1‬‬ ‫‪ t n +1 = (u n + 2v n ) = t n‬ﺇﺫﻥ ) ‪ (t n‬ﻤﺘﺘﺎﻝﻴﺔ ﺜﺎﺒﺘﺔ ‪.‬‬
‫‪3‬‬
‫‪ 1 ≤ u n ≤ 2‬ﻭ ‪ 1 ≤ v n ≤ 2‬ﺇﺫﻥ ‪2 ≤ u n + 1 ≤ 3‬‬ ‫‪1‬‬ ‫‪11‬‬
‫= ) ‪lim t n = t 0 = (u 0 + 2v 0‬‬
‫ﻭ ‪ 2 ≤ v n + 1 ≤ 3‬ﻭﻤﻨﻪ ‪، 4 ≤ (u n + 1)(v n + 1) ≤ 9‬‬ ‫∞‪n →+‬‬ ‫‪3‬‬ ‫‪3‬‬
‫‪11‬‬ ‫‪1‬‬
‫ﺇﺫﻥ ‪ v n − u n‬ﻭ ‪ v n +1 − u n +1‬ﻝﻬﻤﺎ ﻨﻔﺱ ﺍﻹﺸﺎﺭﺓ ؛‬ ‫= ‪.l‬‬ ‫ﻭ ‪ lim t n = ( l + 2l ) = l‬ﺇﺫﻥ‬
‫‪3‬‬ ‫∞‪n →+‬‬ ‫‪3‬‬
‫ﺍﺴﺘﻌﻤﺎل ﺍﻝﺘﺭﺍﺠﻊ ‪ v 0 − u 0 = 1 :‬ﻭﻤﻨﻪ ‪ ، v 0 − u 0 ≥ 0‬ﻭﺇﺫﺍ‬ ‫‪2x + 1‬‬
‫= ) ‪f (x‬‬ ‫‪ f 137‬ﻤﻌﺭﻓﺔ ﻋﻠﻰ ]‪ [ 0; 2‬ﺒِـ ‪:‬‬
‫ﻜﺎﻥ ‪ v k − u k ≥ 0‬ﻓﺈﻥ ‪.v k +1 − u k +1 ≥ 0‬‬ ‫‪x +1‬‬
‫‪.‬‬
‫ﻝﺩﻴﻨﺎ ‪ 4 ≤ (u n + 1)(v n + 1) ≤ 9‬ﻭﻤﻨﻪ‬
‫‪1‬‬ ‫‪1‬‬ ‫ﻤﺘﺯﺍﻴﺩﺓ ﺘﻤﺎﻤﺎ ﻋﻠﻰ ]‪[ 0; 2‬‬ ‫= ) ‪ f ' ( x‬ﻭﻤﻨﻪ ‪f‬‬
‫‪1‬‬
‫‪(1‬‬
‫< ‪ 0‬ﺒﻤﺎ ﺃﻥ ‪ v n − u n ≥ 0‬ﻓﺈﻥ‬ ‫≤‬ ‫)‪( x + 1‬‬
‫‪2‬‬

‫‪(u n + 1)(v n + 1) 4‬‬


‫‪v n −un‬‬ ‫‪1‬‬ ‫ﻭﺒﺎﻝﺘﺎﻝﻲ ﺇﺫﺍ ﻜﺎﻥ ‪ 1 ≤ x ≤ 2‬ﻓﺈﻥ‬
‫ﺃﻱ‬ ‫) ‪≤ (v n − u n‬‬
‫‪(u n + 1)(v n + 1) 4‬‬ ‫‪3‬‬ ‫‪5‬‬
‫) ‪ f (1) ≤ f ( x ) ≤ f ( 2‬ﺃﻱ ≤ ) ‪ ≤ f ( x‬ﻭﻤﻨﻪ‬
‫‪1‬‬ ‫‪2‬‬ ‫‪3‬‬
‫≤ ‪. v n +1 − u n +1‬‬ ‫) ‪(v n − u n‬‬ ‫]‪. f ( x ) ∈ [1; 2‬‬
‫‪4‬‬
‫ﺍﺴﺘﻌﻤﺎل ﺍﻝﺘﺭﺍﺠﻊ ﻹﺜﺒﺎﺕ ﺃﻨﻪ ﻤﻥ ﺃﺠل ﻜل ﻋﺩﺩ ﻁﺒﻴﻌﻲ ‪، n‬‬ ‫‪ v 0 = 2 ، u 0 = 1 (2‬ﻭﻤﻥ ﺃﺠل ﻜل ﻋﺩﺩ ﻁﺒﻴﻌﻲ ‪، n‬‬
‫) ‪ u n +1 = f (u n‬؛ ) ‪. v n +1 = f (v n‬‬
‫‪n‬‬
‫‪1‬‬
‫‪. v n −un ≤  ‬‬
‫‪4‬‬ ‫ﻴﺒﺩﻭ ﺃﻥ ) ‪ (u n‬ﻤﺘﺯﺍﻴﺩﺓ‬
‫‪0‬‬ ‫‪0‬‬
‫‪1‬‬ ‫‪1‬‬
‫ﻝﺩﻴﻨﺎ ‪ v 0 − u 0 = 1‬ﻭ ‪   = 1‬ﺇﺫﻥ ‪v 0 − u 0 ≤  ‬‬ ‫ﻭ ) ‪ (v n‬ﻤﺘﻨﺎﻗﺼﺔ ﻭﻝﻬﻤﺎ ﻨﻔﺱ‬
‫‪4‬‬ ‫‪4‬‬
‫‪k‬‬ ‫ﺍﻝﻨﻬﺎﻴﺔ ﻭﻫﻲ ﻓﺎﺼﻠﺔ ﻨﻘﻁﺔ‬
‫‪1‬‬
‫ﻨﻔﺭﺽ ﺃﻥ ‪. v k − u k ≤  ‬‬
‫‪4‬‬ ‫ﺘﻘﺎﻁﻊ ﺍﻝﻤﻨﺤﻨﻴﻴﻥ ‪.‬‬

‫‪1‬‬ ‫‪11‬‬
‫‪k‬‬
‫‪ (3‬ﺍﻝﺒﺭﻫﺎﻥ ﺒﺎﻝﺘﺭﺍﺠﻊ ﻋﻥ ﺍﻝﺨﻭﺍﺹ ‪:‬‬
‫‪.v k +1 − u k +1 ≤ (v k − u k ) ≤  ‬‬
‫‪4‬‬ ‫‪44‬‬ ‫* " ‪ " 1 ≤ u n ≤ 2‬؛ ‪ u 0 = 1‬ﻭﻤﻨﻪ ‪1 ≤ u 0 ≤ 2‬‬
‫=‬
‫(‬ ‫‪v n − un‬‬ ‫) ‪)( v − u )( v + u‬‬
‫‪n‬‬ ‫‪n‬‬ ‫‪n‬‬ ‫‪n‬‬ ‫‪1‬‬
‫‪n‬‬
‫‪1‬‬
‫ﻝﺩﻴﻨﺎ ‪ 0 ≤ v n − u n ≤  ‬ﻭ ‪ lim   = 0‬ﺇﺫﻥ‬
‫‪n‬‬

‫) ‪2( v + u‬‬ ‫‪ ‬‬ ‫‪4‬‬


‫‪n →+∞ 4‬‬

‫‪ lim (v n − u n ) = 0‬ﻭﺤﺴﺏ ﺍﻝﺴﺅﺍل ‪ (3‬ﻝﺩﻴﻨﺎ ‪u n ≤ u n +1‬‬


‫‪n‬‬ ‫‪n‬‬

‫) ‪1( v − u‬‬
‫∞‪n →+‬‬

‫) ‪(v − u‬‬
‫‪n‬‬ ‫‪n‬‬
‫=‬ ‫ﻭ ‪ v n ≥ v n +1‬ﻤﻌﻨﺎﻩ ) ‪ (u n‬ﻤﺘﺯﺍﻴﺩﺓ ﻭ ) ‪ (v n‬ﻤﺘﻨﺎﻗﺼﺔ ﺇﺫﻥ‬
‫) ‪2( v + u‬‬
‫‪n‬‬ ‫‪n‬‬
‫‪n‬‬ ‫‪n‬‬
‫) ‪ (u n‬ﻭ ) ‪ (v n‬ﻤﺘﺠﺎﻭﺭﺘﺎﻥ ﻭﺒﺎﻝﺘﺎﻝﻲ ﻝﻬﻤﺎ ﻨﻔﺱ ﺍﻝﻨﻬﺎﻴﺔ ‪. l‬‬
‫‪ − u n < u n‬ﻭﻤﻨﻪ ‪v n − u n < v n + u n‬‬
‫‪2u n + 1‬‬
‫‪v n − un‬‬ ‫= ‪ u n +1‬ﻤﻌﻨﺎﻩ ‪u n +1u n + u n +1 − 2u n − 1 = 0‬‬
‫ﺇﺫﻥ ‪< 1‬‬ ‫‪un +1‬‬
‫‪v n + un‬‬
‫ﻭﻤﻨﻪ ‪ lim (u n +1u n + u n +1 − 2u n − 1) = 0‬ﺃﻱ‬
‫‪1‬‬ ‫‪v − un 1‬‬ ‫∞‪n →+‬‬
‫‪(v n − u n ) n‬‬ ‫ﻭﺒﺎﻝﺘﺎﻝﻲ ) ‪< (v n − u n‬‬
‫‪1+ 5‬‬ ‫‪1− 5‬‬
‫‪2‬‬ ‫‪v n + un 2‬‬ ‫=‪l‬‬ ‫= ‪ l‬ﺃﻭ‬ ‫‪ l 2 − l − 1 = 0‬ﻭﻤﻌﻨﺎﻩ‬
‫‪1‬‬ ‫‪2‬‬ ‫‪2‬‬
‫< ‪. v n +1 − u n +1‬‬ ‫ﺃﻱ ) ‪(v n − u n‬‬ ‫‪1+ 5‬‬
‫‪2‬‬
‫= ‪.l‬‬ ‫ﺒﻴﻨﻤﺎ ‪ 1 ≤ u n ≤ 2‬ﻭ ‪ 1 ≤ v n ≤ 2‬ﺇﺫﻥ‬
‫‪1‬‬
‫≤ ‪v 0 −u0‬‬ ‫ﻨﺴﺘﻌﻤل ﺍﻝﺘﺭﺍﺠﻊ ﻭﻝﺩﻴﻨﺎ ﺍﻝﺨﺎﺼﻴﺔ ) ‪(b − a‬‬ ‫‪2‬‬
‫‪20‬‬ ‫‪ a 138‬ﻭ ‪ b‬ﻋﺩﺩﺍﻥ ﺤﻘﻴﻘﻴﺎﻥ ﺤﻴﺙ ‪. 0 < a < b‬‬
‫ﺘﻜﺎﻓﺊ ) ‪ v 0 − u 0 ≤ (b − a‬ﻭﻫﻲ ﺼﺤﻴﺤﺔ ‪.‬‬
‫ﺍﻝﻤﺘﺘﺎﻝﻴﺘﺎﻥ ) ‪ (u n‬ﻭ ) ‪ (v n‬ﻤﻌﺭﻓﺘﺎﻥ ِـ ‪v 0 = b ، u 0 = a :‬‬
‫‪1‬‬
‫ﻨﻔﺭﺽ ﺃﻥ ﺍﻝﺨﺎﺼﻴﺔ ) ‪ v k − u k ≤ k (b − a‬ﻭﻝﻨﺒﺭﻫﻥ‬ ‫‪u n +v n‬‬
‫‪2‬‬ ‫= ‪.v n +1‬‬ ‫ﻭﻤﻥ ﺃﺠل ﻜل ‪، u n +1 = u nv n ، n ∈ ℕ‬‬
‫‪1‬‬ ‫‪2‬‬
‫ﺼﺤﺔ ﺍﻝﺨﺎﺼﻴﺔ ) ‪v k ++ − u k +1 ≤ k +1 (b − a‬‬
‫‪2‬‬ ‫‪ (1‬ﻨﺴﻤﻲ ‪ p n‬ﺍﻝﺨﺎﺼﻴﺔ " ‪" 0 < u n ≤ v n‬‬
‫‪1‬‬
‫ﻝﺩﻴﻨﺎ ﻤﻤﺎ ﺴﺒﻕ ) ‪ v k +1 − u k +1 < (v k − u k‬ﻭﻤﻥ ﻓﺭﻀﻴﺔ‬ ‫ﻝﺩﻴﻨﺎ ‪ v 0 = b ، u 0 = a‬ﻭ ‪ 0 < a < b‬ﺇﺫﻥ‬
‫‪2‬‬
‫‪1‬‬ ‫‪ 0 < u 0 ≤ v 0‬ﻭﻤﻨﻪ ﺍﻝﺨﺎﺼﻴﺔ ‪ p 0‬ﺼﺤﻴﺤﺔ ‪.‬‬
‫ﺍﻝﺘﺭﺍﺠﻊ ) ‪ v k − u k ≤ k (b − a‬ﻴﻨﺘﺞ ﺃﻥ‬
‫‪2‬‬ ‫ﻨﻔﺭﺽ ﺃﻥ ﺍﻝﺨﺎﺼﻴﺔ ‪ p k‬ﺼﺤﻴﺤﺔ ﺃﻱ ‪0 < u k ≤ v k‬‬
‫‪1‬‬ ‫‪1‬‬
‫) ‪ (v k − u k ) ≤ k +1 (b − a‬ﺇﺫﻥ‬ ‫ﻭﺒﺎﻝﺘﺎﻝﻲ ‪. 0 < u k +1‬‬ ‫ﻝﺩﻴﻨﺎ ‪ u k v k > 0‬ﺇﺫﻥ ‪u k v k > 0‬‬
‫‪2‬‬ ‫‪2‬‬
‫‪ (u k + v k‬ﻤﻌﻨﺎﻩ‬ ‫‪) − (u k‬‬ ‫‪−v k‬‬ ‫)‬ ‫ﻝﺩﻴﻨﺎ ‪= 4u k v k‬‬
‫‪2‬‬ ‫‪2‬‬
‫‪1‬‬
‫) ‪ v k +1 − u k +1 ≤ k +1 (b − a‬ﻭﺤﺴﺏ ﻤﺒﺩﺃ ﺍﻝﺘﺭﺍﺠﻊ ﻴﻨﺘﺞ‬
‫‪ (u k + v k‬ﺒﻤﺎ ﺃﻥ ‪ 0 < u k ≤ v k‬ﻓﺈﻥ‬ ‫)‬ ‫‪≥ 4u k v k‬‬
‫‪2‬‬
‫‪2‬‬
‫‪1‬‬
‫ﺃﻨﻪ ﻤﻥ ﺃﺠل ﻜل ﻋﺩﺩ ﻁﺒﻴﻌﻲ ‪.v n − u n ≤ n (b − a ) ، n‬‬ ‫‪ u k + v k > 0‬ﻭﻤﻨﻪ ‪ u k + v k ≥ 2 u k v k‬ﻤﻌﻨﺎﻩ‬
‫‪2‬‬
‫‪u k +v k‬‬
‫‪ (3‬ﻝﺩﻴﻨﺎ ﻜل ﺤﺩﻭﺩ ﺍﻝﻤﺘﺘﺎﻝﻴﺔ ) ‪ (u n‬ﻤﻭﺠﺒﺔ ﺘﻤﺎﻤﺎ ﺇﺫﻥ ﻨﺩﺭﺱ‬ ‫ﺃﻱ ‪ v k +1 ≥ u k +1‬ﻭﺒﺎﻝﺘﺎﻝﻲ ‪:‬‬ ‫‪≥ u kv k‬‬
‫‪2‬‬
‫‪u n +1‬‬
‫‪.‬‬ ‫‪ p k +1‬ﺼﺤﻴﺤﺔ ‪.‬‬ ‫‪ 0 < u k +1 ≤ v k +1‬ﺇﺫﻥ ﺍﻝﺨﺎﺼﻴﺔ‬
‫‪un‬‬
‫ﻭﺤﺴﺏ ﻤﺒﺩﺃ ﺍﻝﺘﺭﺍﺠﻊ ﻴﻨﺘﺞ ﺃﻨﻪ ﻤﻥ ﺃﺠل ﻜل ﻋﺩﺩ ﻁﺒﻴﻌﻲ ‪، n‬‬
‫‪u‬‬ ‫‪uv‬‬ ‫‪v‬‬
‫‪ 0 < u n ≤ v n‬ﻓﺈﻥ‬ ‫‪ n +1 = n 2n = n‬ﺒﻤﺎ ﺃﻥ‬ ‫ﺍﻝﺨﺎﺼﻴﺔ ‪ p n‬ﺃﻱ ‪. 0 < u n ≤ v n‬‬
‫‪un‬‬ ‫‪un‬‬ ‫‪un‬‬
‫‪ (2‬ﻝﻴﻜﻥ ‪ n‬ﻋﺩﺩﺍ ﻁﺒﻴﻌﻴﺎ‬
‫‪u n +1‬‬ ‫‪vn‬‬
‫ﺃﻱ ‪≥ 1‬‬ ‫‪ 0 < u n ≤ v n‬ﻭﻤﻨﻪ ‪≥ 1‬‬ ‫‪u n + v n − 2 u nv n‬‬
‫‪un‬‬ ‫‪un‬‬ ‫= ‪v n +1 − u n +1‬‬
‫‪2‬‬
‫ﻭﺒﺎﻝﺘﺎﻝﻲ ) ‪ (u n‬ﻤﺘﺯﺍﻴﺩﺓ ‪.‬‬
‫(‬ ‫)‬
‫‪2‬‬
‫‪v n − un‬‬
‫‪2‬‬ ‫‪2‬‬
‫‪u + v n − 2 un v n‬‬
‫‪= n‬‬ ‫=‬
‫‪2‬‬ ‫‪2‬‬
‫‪3‬‬ ‫‪u n +v n‬‬ ‫‪u −v n‬‬
‫ﻭﻤﻨﻪ‬ ‫ﻨﻀﻊ ‪ (w n ) ، w n = u n − v n‬ﻫﻨﺩﺴﻴﺔ ﺃﺴﺎﺴﻬﺎ‬ ‫= ‪ v n +1 − v n‬ﺒﻤﺎ ﺃﻥ‬ ‫‪−v n = n‬‬
‫‪10‬‬ ‫‪2‬‬ ‫‪2‬‬
‫‪ 3‬‬
‫‪n‬‬
‫‪ 0 < u n ≤ v n‬ﻓﺈﻥ ‪ u n − v n ≤ 0‬ﺃﻱ ‪v n +1 − v n ≤ 0‬‬
‫‪ w n = u n − v n = (u 0 − v 0 )  ‬ﺇﺫﻥ‬
‫‪ 10 ‬‬ ‫ﻭﺒﺎﻝﺘﺎﻝﻲ ) ‪ (v n‬ﻤﺘﻨﺎﻗﺼﺔ ‪.‬‬
‫‪ lim (u n − v n ) = 0‬؛ ﻭﻤﻨﻪ ) ‪ (u n‬ﻭ ) ‪ (v n‬ﻤﺘﺠﺎﻭﺭﺘﺎﻥ ‪.‬‬ ‫ﻝﺩﻴﻨﺎ ﻤﻥ ﺃﺠل ﻜل ﻋﺩﺩ ﻁﺒﻴﻌﻲ ‪، n‬‬
‫∞‪n →+‬‬

‫‪1‬‬ ‫‪1‬‬
‫‪ (2‬ﻤﻥ ﺃﺠل ﻜل ﻋﺩﺩ ﻁﺒﻴﻌﻲ ‪ ، n‬ﻨﻀﻊ ‪x n = u n + av n‬‬ ‫‪lim‬‬
‫‪n →+∞ 2 n‬‬
‫) ‪ 0 ≤ v n − u n ≤ n (b − a‬ﻭ ‪(b − a ) = 0‬‬
‫‪2‬‬
‫ﻭ ‪ y n = u n + b v n‬ﺤﻴﺙ ‪ a‬ﻭ ‪ b‬ﻋﺩﺩﻴﻥ ﺤﻘﻴﻘﻴﻴﻥ ﻤﺘﻤﺎﻴﺯﻴﻥ‪.‬‬
‫ﺇﺫﻥ ‪. lim (v n − u n ) = 0‬‬
‫‪u n +v n‬‬ ‫‪u + 4v n‬‬ ‫∞‪n →+‬‬
‫= ‪x n +1 = u n +1 + av n +1‬‬ ‫‪+a n‬‬ ‫ﺨﻼﺼﺔ ‪ :‬ﺍﻝﻤﺘﺘﺎﻝﻴﺘﺎﻥ ) ‪ (u n‬ﻭ ) ‪ (v n‬ﻤﺘﺠﺎﻭﺭﺘﺎﻥ ‪.‬‬
‫‪2‬‬ ‫‪5‬‬

‫= ‪x n +1‬‬
‫‪( 2a + 5 )u n + ( 8a + 5)v n‬‬ ‫≤ ‪. v n −un‬‬
‫‪3‬‬
‫‪ a = 2 (4‬ﻭ ‪, b = 5‬ﺇﺫﻥ‬
‫‪10‬‬ ‫‪2n‬‬
‫‪3‬‬
‫‪x n +1‬‬ ‫=‬
‫‪( 2a + 5 )  u‬‬ ‫‪+‬‬
‫‪8a + 5 ‬‬
‫‪vn ‬‬
‫ﻤﻌﻨﺎﻩ ‪ 2n > 3000‬ﻭﻝﺩﻴﻨﺎ ‪211 = 2048‬‬ ‫‪n‬‬
‫‪< 10 −3‬‬
‫‪‬‬ ‫‪2‬‬
‫‪2a + 5 ‬‬
‫‪n‬‬
‫‪10‬‬ ‫‪‬‬
‫ﻭ ‪ 2 = 4096‬ﺇﺫﻥ ‪n ≥ 12‬‬
‫‪12‬‬

‫‪8a + 5‬‬
‫ﺃﻱ ‪2a 2 − 3a − 5 = 0‬‬ ‫) ‪ ( x n‬ﻫﻨﺩﺴﻴﺔ ﻤﻌﻨﺎﻩ ‪= a‬‬ ‫‪n‬‬ ‫‪1‬‬ ‫‪2‬‬ ‫‪3‬‬ ‫‪4‬‬ ‫‪5‬‬ ‫‪6‬‬
‫‪2a + 5‬‬ ‫‪u‬‬
‫‪v‬‬
‫‪2 3,1623 3,32686 3,3289968 3,3289971‬‬
‫‪5‬‬ ‫‪3,5 3,33114 3,3289975 3,3289971‬‬
‫‪3,328997‬‬
‫‪3,328997‬‬
‫ﻭﻜﺫﻝﻙ ) ‪ ( y n‬ﻫﻨﺩﺴﻴﺔ ﻤﻌﻨﺎﻩ ‪2b 2 − 3b − 5 = 0‬‬
‫‪6‬‬ ‫‪7‬‬ ‫‪8‬‬ ‫‪9‬‬ ‫‪10‬‬ ‫‪11‬‬
‫‪ 3,328997 3,3289971 3,328997 3,328997 3,3289971 3,329‬ﺇﺫﻥ ‪ a‬ﻭ ‪ b‬ﻫﻤﺎ ﺍﻝﺤﻼﻥ ﺍﻝﻤﺘﻤﺎﻴﺯﺍﻥ ﻝﻠﻤﻌﺎﺩﻝﺔ‬
‫‪3,328997 3,3289971 3,328997 3,328997 3,3289971‬‬ ‫‪3,329‬‬
‫‪5‬‬
‫‪ 2x 2 − 3x − 5 = 0‬ﺃﻱ ‪ a = −1‬ﻭ = ‪ b‬ﺃﻭ ‪b = −1‬‬ ‫ﻭﺍﻝﻌﺩﺩ ‪ l ≃ 3,329‬ﻫﻭ ﺍﻝﻨﻬﺎﻴﺔ ﺍﻝﻤﺸﺘﺭﻜﺔ ِـ ) ‪ (u n‬ﻭ ) ‪. (v n‬‬
‫‪2‬‬
‫ﻭ = ‪.a‬‬
‫‪5‬‬ ‫‪ v 0 = 2 ، u 0 = −1 139‬ﻭﻤﻥ ﺃﺠل ﻜل ﻋﺩﺩ ﻁﺒﻴﻌﻲ ‪، n‬‬
‫‪2‬‬ ‫‪u + 4v n‬‬ ‫‪u +v n‬‬
‫‪5‬‬ ‫‪. v n +1 = n‬‬ ‫‪ u n +1 = n‬؛‬
‫ﻨﻔﺭﺽ ‪ a = −1‬ﻭ = ‪ b‬ﺇﺫﻥ‬ ‫‪5‬‬ ‫‪2‬‬
‫‪2‬‬
‫‪ (1‬ﺃ ـ ‪ u 0 < v 0‬ﺼﺤﻴﺤﺔ ‪ .‬ﻤﻥ ﺃﺠل ‪ k‬ﻋﺩﺩ ﻁﺒﻴﻌﻲ ‪،‬‬
‫‪3‬‬ ‫‪3‬‬
‫‪x n +1 = (u n −v n ) = x n‬‬ ‫‪ u k < v k‬ﻤﻌﻨﺎﻩ ‪ . u k − v k < 0‬ﻝﺩﻴﻨﺎ‬
‫‪10‬‬ ‫‪10‬‬
‫‪‬‬ ‫‪5 ‬‬ ‫‪u + v k u k + 4v k 3u k − 3v k‬‬
‫ﻭ ‪. y n +1 =  u n + v n  = y n‬‬ ‫‪u k +1 − v k +1 = k‬‬ ‫‪−‬‬ ‫=‬
‫‪‬‬ ‫‪2 ‬‬ ‫‪2‬‬ ‫‪5‬‬ ‫‪10‬‬
‫‪3‬‬
‫ﺃﻱ ) ‪ u k +1 − v k +1 = 10 (u k −v k‬ﺇﺫﻥ ‪ u k +1 − v k +1 < 0‬ﻝﺩﻴﻨﺎ ‪ x 0 = u 0 − v 0 = −3‬ﻭ ‪ y 0 = u 0 + v 0 = 4‬ﺇﺫﻥ‬
‫‪5‬‬
‫‪2‬‬
‫‪u +v n‬‬ ‫‪v −un‬‬
‫‪u n +1 − u n = n‬‬ ‫‪−un = n‬‬ ‫ﺏـ‬
‫‪n‬‬
‫‪ 3‬‬
‫‪ x n = −3  ‬ﻭ ‪. y n = y 0 = 4‬‬ ‫‪2‬‬ ‫‪2‬‬
‫‪ 10 ‬‬
‫ﻝﺩﻴﻨﺎ ‪ u n < v n‬ﻤﻌﻨﺎﻩ ‪ v n − u n > 0‬ﺇﺫﻥ ‪u n +1 − u n > 0‬‬
‫‪ (3‬ﺇﻴﺠﺎﺩ ﺍﻝﻨﻬﺎﻴﺔ ﺍﻝﻤﺸﺘﺭﻜﺔ ﻝﻠﻤﺘﺘﺎﻝﻴﺘﻴﻥ ) ‪ (u n‬ﻭ ) ‪. (v n‬‬
‫ﻭﺒﺎﻝﺘﺎﻝﻲ ) ‪ (u n‬ﻤﺘﺯﺍﻴﺩﺓ ﺘﻤﺎﻤﺎ‪.‬‬
‫ﻝﺩﻴﻨﺎ ‪ x n = u n + av n‬ﻭ ‪ y n = u n + b v n‬ﺃﻱ‬
‫‪u + 4v n‬‬ ‫‪u −v n‬‬
‫‪n‬‬ ‫‪ v n +1 − v n = n‬ﻭﻤﻨﻪ‬ ‫‪−v n = n‬‬
‫‪5‬‬ ‫‪ 3‬‬ ‫‪5‬‬ ‫‪5‬‬
‫‪ x n = u n −v n = −3  ‬ﻭ ‪y n = u n + v n = 4‬‬
‫‪2‬‬ ‫‪ 10 ‬‬ ‫‪ v n +1 − v n < 0‬ﺇﺫﻥ ) ‪ (v n‬ﻤﺘﻨﺎﻗﺼﺔ ﺘﻤﺎﻤﺎ‪.‬‬
‫‪3‬‬
‫= ‪u n +1 − v n +1‬‬ ‫) ‪(u n −v n‬‬
‫‪10‬‬
8 8 6 3 
n
 3
n
lim v n = ‫ﺇﺫﻥ‬ vn =
7
+   ‫ ﺃﻱ‬v n = 4 + 3   ‫ﻭﻤﻨﻪ‬
n →+∞ 7 7 7  10  2  10 

‫ﻤﺴﺎﺌل‬
2
n +2
 −1 
n +2
(u n )
‫و‬6 ‫ ( !
ا  ت _' ا‬E ) @ 140
= α   + β   = u n +2
7  5  :
  ‫
ا‬fF ‫ ا‬Sa3  ‫ وا‬ℕ j(
'6 ‫ا‬
. ( E )
! ‫'  ا‬m  ‫( ه‬u n )
  ‫إذن ا‬ 3 2
. u n + 2 = u n +1 + u n
4 35 35
 6 u 1 = − ‫ و‬α + β = 3  6 u 0 = 3 (3 ، n ∈ ℕ )‫) آ‬4‫   أ‬6
T (u n ) (1
35
2 1 4 6 3 2
!   ‫ و‬10α − 7 β = −4 ‫ أي‬α − β = − . u n = 0 ‫أي‬ u n = 0  6 u n = u n + u n
7 5 35 7 35 35
، n ∈ ℕ )‫) آ‬4‫   أ‬6 r ‫س‬7^‫
ذات ا‬/ (u n )
 −1 
n n
2
. u n =   + 2   ‫ أي‬β = 2 ‫ و‬α = 1 67 3 2
7  5  un = − r ‫ أي‬u n + 2r = (u n + r ) + u n
30 35 35
−1
n
2 2
lim   = 0 ‫ إذن‬−1 < < 1 ‫ و‬−1 < < 1 . u n = 0 ‫
أي‬T (u n ) U ‫و‬
 
n →+∞ 7 5 7
، n ∈ ℕ )‫) آ‬4‫   أ‬6 q ‫س‬7^‫
ذات ا‬7 ‫( ه‬u n )
 −1 
n

. lim u n = 0   ‫ و‬lim   = 0 ‫و‬ 3 2


n →+∞ n →+∞
 5   6‫ و‬u n q 2 = (u n q ) + u n
)‫) آ‬4‫ق و أ‬2:
(R f
‫( ا ا‬1 − I 141 35 35
q = − ‫ أو‬u n = 0 ‫ أي‬u n ( 35q 2 − 3q − 2 ) = 0
2
1 −x 2
U ‫ و‬f ' ( x ) = 1 − x − =   x ≥ 0 5
1+ x 1+ x 4
. [ 0; + ∞[ j( 3
mR  f
‫ إذن ا ا‬f ' ( x ) ≤ 0 . q = ‫أو‬
7
  x ≥ 0 )‫) آ‬4‫ق و أ‬2:
(R g
‫ا ا‬ 9 Uv
‫و‬6 ‫ ( ه !
ا  ت _' ا‬E ) ‫ أن‬
1 −x
‫ إذن‬g ' ( x ) ≤ 0 U ‫ و‬g ' ( x ) = −1 = 43   ‫
؛‬/
  9‫
و‬T
    43
1+ x 1+ x 2
q = −  77‫ ( أ‬E )
! ‫ن  ا‬7 ‫ ن ه‬
. [ 0; + ∞[ j( 3
mR  g
‫ا ا‬ 5
‫ن‬v x > 0 ‫ وإذا آن‬f ( 0 ) = 0   (2 .q = ‫و‬
4
)‫) آ‬4‫ و    أ‬f ( x ) < 0 ‫ أي‬f ( x ) < f ( 0 )
7
، ‫ د‬β ‫ و‬α @ (2
1
x − x 2 − ln (1 + x ) ≤ 0 ‫ أي‬f ( x ) ≤ 0 ، x ≥ 0 3 2 3  2
n +1 n +1
 −1  
2 u n +1 + u n = α   + β    +
1 35 35 35   7   5  
. x − x 2 ≤ ln (1 + x )  6
2 2  2  −1  
n n

g ( x ) < g ( 0 ) ‫ن‬v x > 0 ‫ وإذا آن‬g ( 0 ) = 0   + α   + β   


35   7   5  
, x ≥ 0 )‫) آ‬4‫ و    أ‬g ( x ) < 0 ‫أي‬
 1  −1   −3
n n
1 2 6 
 6 ln (1 + x ) − x ≤ 0 ‫ أي‬g ( x ) ≤ 0 = α    + 2 + β    + 2
35  7   7  35  5   5 
. ln (1 + x ) ≤ x 2  4 
n
 −1   1 
n

=α    + β    
 7   49   5   25 
n
1 x2
−1 x− ≤ ln (1 + x ) ≤ x ، x ≥ 0 )‫) آ‬4‫  أ‬:
f:*
1  2 
n
1 2
‫ ؛‬Sn = = 1−  
2 1 −1 2 3
‫ 'ض أن‬. u 1 > 0 U ‫ و‬u 1 =   (1 − II
2 2
1
n
 1  1
−1 u n 1 + n +1  > 0 ‫ن‬v 1 + n +1 > 0 ‫  أن‬u n > 0
1  4 
n
1 11  2  2
. Tn = = −  
4 1 −1 3 3 4 )‫) آ‬4‫  أ‬U‫ أ‬C  A4‫أ ا 'ا‬1 Y/ ‫ إذن‬u n +1 > 0 U ‫و‬
4 . u n > 0 ، n ‫وم‬6 '_ 61Z ‫د‬
n
1  1 3 3
‫ ؛‬lim S n = lim 1 −   = 1 ‫ب ـ‬ ‫ إذن‬ln 1 +  = ‫ و‬ln u 1 = (2
n →+∞ n →+∞
2  2 2 2
 1
n
1 11 1 ‫ 'ض أن‬. ln u 1 = ln 1 + 
. lim T n = lim −   =
n →+∞ n →+∞ 3 3 4 3  2
, ‫وم‬6 '_ 61Z ‫ دا‬n @ ‫( أ ـ‬5  1  1   1 
ln u n = ln 1 +  + ln 1 + 2  + ... + ln 1 + n 
 1  1  2  2   2 
  ، u n +1 − u n = u n 1 + n +1  − u n = n +1 × u n
 2  2   ‫و‬
1  1   1 
> 0 , n ‫وم‬6 '_ 61Z ‫) آ) د‬4‫ أ‬ ln u n +1 = ln u n 1 + n +1  = ln u n + ln 1 + n +1 
2n +1  2   2 
1 : U ‫و‬
‫ أي‬n +1 × u n > 0 ‫( إذن‬1 ‫ال‬p/ ‫ ) ا‬u n > 0 ‫و‬
2  1  1 
ln u n +1 = ln 1 +  + ln 1 + 2  + ... +
. 3 ‫اة‬X (u n )   ‫ و‬u n +1 − u n > 0  2  2 
1
n
 1   1 
61Z ‫) آ) د‬4‫  أ‬U ‫ و‬S n = 1 −     ‫ب ـ‬ ln 1 + n  + ln 1 + n +1 
2  2   2 
‫ن‬v ln u n ≤ S n ‫  أن‬S n ≤ 1 ، n ‫وم‬6 '_ 61Z ‫) آ) د‬4‫  أ‬U‫ أ‬C  A4‫أ ا 'ا‬1 Y/   ‫و‬
، n ‫وم‬6 '_
 ‫ودة‬a (u n )
  ‫ إذن ا‬u n ≤ e U ‫ و‬ln u n ≤ 1
 1  1   1 
.
‫  ر‬v 3 ‫اة‬X  ‫ و أ‬j(^‫ا‬ ln u n = ln 1 +  + ln 1 + 2  + ... + ln 1 + n 
 2  2   2 
w l  ‫ د‬4 Uv
‫( ر‬u n ) ‫ـ ـ  أن‬ A| , 1 ≤ k ≤ n w 61Z ‫ دا‬k @ (3
1 1
U ‫ و‬S n − T n ≤ ln u n ≤ S n   ‫ ؛‬lim u n = l l1m3 (1)
R:6 ‫ ا‬x = k
2 n →+∞ 2
 1  1 1 1  1  1
‫ أي‬lim  S n − T n  ≤ lim ln u n ≤ lim S n − × ≤ ln 1 + k  ≤ k
n →+∞
 2  n →+∞ n →+∞ 2k 2 4 k  2  2
5 1 1 n j ‫ إ‬1  '; k ‫ ^ن‬n ‫ت ه‬R:6 ‫ ا‬P‫ود ه‬
†@‫ ≤ و‬ln l ≤ 1  6 1 − × ≤ ln l ≤ 1
6 2 3 j( )ma ‫ت‬R:6 ‫'اف آ) ا‬Z‫ أ‬A!‫و‬
5 1
.e ≤ l ≤ e6 S n − T n ≤ ln u n ≤ S n
2
: ‫ ِـ‬n ∈ ℕ )‫) آ‬4‫'ن  أ‬6 (v n ) ‫( و‬u n ) 142

 
6 ‫ود‬a ‫ ه !ن‬T n ‫ و‬S n ‫( أ ـ‬4
1 2 n 1 1
u n = sin 2
+ sin 2 + ... + sin 2 . Y3' ‫ ا‬j( ‫و‬  77‫ أ‬7 ‫ه‬
n n n 4 2
1 2 n
. v n = 2 + 2 + ... + 2 ‫و‬
n n n
1 n ( n + 1)
2 (
vn = 1 + 2 + ... + n ) = 2
، n ∈ ℕ∗ @ (1
n 2n
 6 13 + 23 + ... + n 3 ≤ n 4   ‫و‬ n +1 1 n +1
. lim v n = lim = U ‫ و‬v n = ‫أي‬
− 6 (13 + 23 + ... + n 3 ) ≥ − 6 n 4
1 1 n →+∞ n →+∞ 2n 2 2n
6n 6n ‫  ا !ل‬x )‫) آ‬4‫ ؛  أ‬f : x ֏ x − sin x (2
f ' ( x ) ≥ 0 U ‫ و‬f ' ( x ) = 1 − cos x ، [ 0; + ∞[
‫ إذن‬v n − 6 (1 + 2 + ... + n 3 ) ≥ v n − 2  6‫و‬
1 3 3 1
6n 6n ‫ وإذا‬f ( 0 ) = 0   ‫ [ و‬0; + ∞[ j( 3 ‫اة‬X f ‫إذن‬
1
. v n − 2 ≤ un ≤v n   ‫ و‬f ( x ) > 0 ‫ أي‬f ( x ) > f ( 0 ) ‫ن‬v x > 0 ‫آن‬
6n
1 1 .
14 f
‫ا ا‬
U ‫ و‬lim v n = ‫ و‬lim = 0 (4 x2
n →+∞ 2 n →+∞ 6n 2 ، x ∈ [ 0; + ∞[ )‫) آ‬4‫ ؛  أ‬g : x ֏ −1 + + cos x
1 1 1 2
. lim u n = ‫ إذن‬lim v n − 2 = ‫ن‬v
14 f ‫  أن‬g ' ( x ) = x − sin x = f ( x )
n →+∞ 2 n →+∞ 6n 2
1 g ( 0 ) = 0   ‫ و‬3 ‫اة‬X g   ‫ و‬g ' ( x ) ≥ 0
.   ‫ و‬،
‫( ر‬u n )
  ‫إذن ا‬
2 g ( x ) > 0 ‫ أي‬g ( x ) > g ( 0 ) ‫ن‬v x > 0 ‫وإذا آن‬
‫] ِـ‬0; + ∞[ j(
'6 f − I 143
x ln x
. f (x ) = .
14 g
‫و   ا ا‬
x +1
1 x +1 x3
g '(x ) = 1+ = ، x ∈ ]0; + ∞[ )‫) آ‬4‫(  أ‬1 ، x ∈ [ 0; + ∞[ )‫) آ‬4‫ ؛  أ‬h : x ֏ − x + + sin x
x x 6
. 3 ‫اة‬X g
‫ إذن ا ا‬g ' ( x ) > 0 U ‫و‬
14 g ‫  أن‬h ' ( x ) = −1 +
x2
+ cos x = g ( x )
g ( x ) = −∞ ‫] و‬0; + ∞[ j( ‫'ة‬/ g
2
lim
x 
>
→0   ‫ و‬3 ‫اة‬X h   ‫ و‬h ' ( x ) ≥ 0 ‫ن‬v
lim g ( x ) = +∞ ‫و‬ ‫ أي‬h ( x ) > h ( 0 ) ‫ن‬v x > 0 ‫ وإذا آن‬h ( 0 ) = 0
x 
→ +∞

. β ‫ وا‬: )13 g ( x ) = 0
‫د‬6 ‫ ا‬:
f:* .
14 h
‫ و   ا ا‬h ( x ) > 0
‫ إذن‬g ( 0, 28) ≃ 0.007 ‫ و‬g ( 0, 27 ) ≃ -0.039 :   w k 61Z ‫) آ) د‬4‫  أ‬، n ∈ ℕ∗ @ (3
0, 27 ≤ β ≤ 0, 28 ، 23 ≤ n 3 ، 13 ≤ n 3 ‫ أي‬1 ≤ k 3 ≤ n 3   1 ≤ k ≤ n
  x > 0 )4‫(  أ‬2 j( )ma ‫'ف‬Z j ‫' إ‬Z A! ‫ و‬n 3 ≤ n 3 ، ...

. f '(x ) =
( ln x + 1)( x + 1) − x ln x = g ( x ) . 13 + 23 + ... + n 3 ≤ n 4 ‫ أي‬13 + 23 + ... + n 3 ≤ n × n 3
f ( x ) ≥ 0 ، x Y4  ‫) آ) د‬4‫    أ‬
( x + 1) ( x + 1)
2 2

f ' ( x ) < 0 ، x ∈ ]−∞ ; β [ )4‫ و أ‬f ' ( β ) = 0 sin x ≥ x −


x3
‫ و‬x ≥ sin x  6 h ( x ) ≥ 0 ‫و‬
6
. f ' ( x ) > 0 ، x ∈ ]β ; + ∞[ )4‫و أ‬
k x3
x ln x U ‫ و‬x = 2 A| . x − ≤ sin x ≤ x ‫أي‬
lim f ( x ) = lim =0 n 6
x → 0 x + 1
> >
x →0
k k3 k k x3 k3
x ln x ln x − ≤ sin 2 ≤ 2 ‫إذن‬ =
lim f ( x ) = lim = lim ‫و‬ n 2 6n 6 n n 6 6n 6
x →+∞ x →+∞  1  x →+∞  1 
x 1 +  1 +  : j( )ma A! ‫و‬
 x   x 
+ 2 + 2 − 6 (13 + 23 + ... + n 3 ) ≤ sin 2 +
1 2 n 1 1
. lim f ( x ) = +∞ ‫أي‬ n 2
n n 6n n
x →+∞

 6 g ( β ) = 0   ‫ و‬f (1) = 0 ‫} أن‬: (1 − II 2 n 1


sin 2 + ... + sin 2 ≤ 2 + 2 + 2
2 n
n n n n n
β ln β
f (β ) = = − β ‫ إذن‬ln β = −1 − β
v n − 6 (1 + 2 + ... + n 3 ) ≤ u n ≤ v n ‫أي‬
1 3 3
β +1
6n
u ( 0 ) = 0   [ 0; + ∞[ j( 3 ‫اة‬X u   ‫و‬ j( ‫ن‬3'/ x ֏ x + 1 ‫ و‬x ֏ x sin x ‫ا ا ن‬
)‫) آ‬4‫ إذن  أ‬. u (t ) > u ( 0 ) ‫ @ن‬t > 0 )4‫و أ‬ f
‫ إذن ا ا‬x + 1 ≠ 0 x > 0 )4‫] و  أ‬0; + ∞[

. 0 ≤ (1 + t ) ln (1 + t ) − t ‫ أي‬u (t ) ≥ 0 , t ≥ 0 : 3‫;'ا‬3 ‫ول‬4 U ‫] و‬0; + ∞[ j( ‫'ة‬/


x 0 β 1 αn +∞
t2
‫ق‬29‫) ا‬13 v : t ֏ (1 + t ) ln (1 + t ) − t −
‫ا ا‬ f '(x ) - 0 + + +
2
v ' (t ) = ln (1 + t ) − t   t ≥ 0 )‫) آ‬4‫و أ‬ f (x ) 0 +∞
1 −t n
‫ @ن‬t ≥ 0 )4‫  أ‬v " (t ) = −1 = 0
1+t 1+ t −β
v ' ( 0 ) = 0 ‫ [ و‬0; + ∞[ j(
mR  v ' ‫ إذن‬v " (t ) ≤ 0
. α n ‫ وا‬: )13 f ( x ) = n
‫د‬6 ‫إذن ا‬
v ' (t ) ≤ 0 ‫ @ن‬t ≥ 0 )4‫إذن  أ‬
e n ln e n en
)4‫ إذن  أ‬v ( 0 ) = 0 ‫ [ و‬0; + ∞[ j(
mR  v ‫إذن‬   ‫ و‬f (e n ) = = n ‫( أ ـ‬2
e n +1 e n +1
‫ أي‬v (t ) ≤ 0 ‫ @ن‬t ≥ 0 ne n en
‫ إذن‬n ≤ n  6‫ و‬n ≤ 1 ‫ أي‬e n ≤ e n + 1
t2 e +1 e +1
 6 (1 + t ) ln (1 + t ) − t − ≤0
2 . f (e n ) ≤ n

. (1 + t ) ln (1 + t ) − t ≤
t2 f ‫ و أن‬f (e n ) ≤ f (α n ) ‫ن‬v f (α n ) = n ‫ أن‬
2
. e n ≤ α n ‫ن  @ن‬v [1; + ∞[ j( 3 ‫اة‬X
, t ≥ 0 )‫) آ‬4‫  أ‬:
f:*
α n ln α n
. 0 ≤ (1 + t ) ln (1 + t ) − t ≤
t2 †@3‫و‬ = n †@3 f (α n ) = n ‫ب ـ‬
2 αn +1
‫ @ن إذن‬t = ε n A| ‫ـ ـ‬  6‫ و‬α n ( ln α n − n ) = n ‫ أي‬α n ln α n = nα n + n
εn2 αn 
(1) . . . ln 
n n
  ‫ و‬0 ≤ (1 + ε n ) ln (1 + ε n ) − ε n ≤ = ‫ أي‬ln α n − ln e n =
 e  αn αn
n
2
n α
، (1 + ε n ) ln (1 + ε n ) = n
n n
e e
e lim = +∞ ‫ أن‬ ≤ n  6 e n ≤ α n  
n →+∞ n n n
n ε 2
n α
C  j ‫
ا^و‬1 ‫  ا‬0 ≤ n − ε n ≤ n ‫إذن‬ ‫ إذن‬lim = 0 U ‫ و‬lim n = +∞ ‫ن‬v
e 2 n →+∞ α n →+∞ n
n
ε 2
n 2
n 2
n αn αn
. n ≤ 2 n ‫ أي‬ε n 2 ≤ 2 n †@‫ و‬ε n ≤ n . lim = 1   ‫ و‬lim ln =0
2 2e e e n →+∞ e n n →+∞ en
αn
2
n 1 n 
‫ أي‬0 ≤ n − ε n ≤  n    ‫و‬ U ‫ و‬1 + ε n =  6 ( 2 ) ‫( أ ـ‬3
e 2e  en
αn αn
n2
0 ≤ ne − n − ε n ≤ e −2 n C  (1) ‫( و‬1 + ε n ) ln (1 + ε n ) = n
ln
2 ene
n
. (1 + ε n ) ln (1 + ε n ) = n
2
n
†@3 ( 2 ) ‫ و‬0 ≤ n − ε n e n ≤ e − n †@3 ( 3) ‫د ـ‬ e
2
2 j( ‫ق‬29‫) ا‬13 u : t ֏ (1 + t ) ln (1 + t ) − t
‫ب ـ ا ا‬
n
0 ≤ n − α n + e n ≤ e − n ‫ إذن‬ε n e n = α n − e n 1
2 u ' (t ) = ln (1 + t ) + (1 + t ) − 1   ‫ [ و‬0; + ∞[
1+ t
‫ @ن‬t ≥ 0 )4‫  أ‬، u ' (t ) = ln (1 + t ) ‫أي‬
u ' (t ) ≥ 0 ‫ إذن‬ln (1 + t ) ≥ 0 U ‫( و‬1 + t ) ≥ 1
‫‪lim (e n + n − α n ) = 0‬‬
‫‪2‬‬
‫‪ n   −2n ‬‬
‫‪1‬‬ ‫‪2‬‬
‫‪n 2 −n‬‬
‫∞‪n →+‬‬ ‫‪ lim‬إذن‬ ‫و   ‪e = lim 2    e  = 0‬‬
‫‪n →+∞ 2‬‬ ‫∞‪n →+‬‬
‫‪2 ‬‬ ‫‪‬‬

‫ا‪s+:!+ /"c‬‬
‫‪ (1 146‬ﺼﺤﻴﺤﺔ ﻷﻥ ﻜل ﻤﺘﺘﺎﻝﻴﺔ ﻤﺘﻨﺎﻗﺼﺔ ﻫﻲ ﻤﺤﺩﻭﺩﺓ‬ ‫ا‪ "c‬ر ‪!"+ +‬ـد‬
‫ﻤﻥ ﺍﻷﻋﻠﻰ ﺒﺤﺩﻫﺎ ﺍﻷﻭل‪.‬‬
‫‪ 144‬ﻨﻌﺘﺒﺭ ﺍﻝﻤﺘﺘﺎﻝﻴﺔ ) ‪ (u n‬ﺍﻝﻤﻌﺭﻓﺔ ـ ‪ u 0 = 0‬ﻭﻤﻥ ﺃﺠل‬
‫‪ (2‬ﻜل ﻤﺘﺘﺎﻝﻴﺔ ﻤﺘﻨﺎﻗﺼﺔ ﻭﻤﺤﺩﻭﺩﺓ ﻤﻥ ﺍﻷﺴﻔل ﺒﺎﻝﻌﺩﺩ ‪0‬‬
‫‪1‬‬ ‫‪2‬‬
‫ﻜل ‪. u n + 2 = u n +1 + u n ، n ∈ ℕ‬‬
‫ﻓﺘﻜﻭﻥ ﻨﻬﺎﻴﺘﻬﺎ ﻤﻌﺩﻭﻤﺔ ‪ ،‬ﺠﻤﻠﺔ ﺨﺎﻁﺌﺔ ﻷﻥ ﻨﻬﺎﻴﺘﻬﺎ ﻤﻭﺠﺒﺔ‬ ‫‪3‬‬ ‫‪3‬‬
‫ﻭﻴﻤﻜﻥ ﺃﻥ ﺘﻜﻭﻥ ﻏﻴﺭ ﻤﻌﺩﻭﻤﺔ ﻤﺜﻼ ﺍﻝﻤﺘﺘﺎﻝﻴﺔ ﺍﻝﻤﻌﺭﻓﺔ ﺒـ‬ ‫ﺘﺼﺤﻴﺢ ﺇﻀﺎﻓﺔ ‪. u 1 = 1‬‬
‫‪1‬‬ ‫‪2‬‬
‫‪.u n = 1 +‬‬ ‫‪ v n = u n +1 − u n‬ﻭ ‪. w n = u n +1 + u n‬‬
‫‪n +1‬‬ ‫‪3‬‬
‫‪ (3‬ﺇﺫﺍ ﻜﺎﻨﺕ ﻤﺘﺘﺎﻝﻴﺔ ﻤﺘﺯﺍﻴﺩﺓ ﻓﺈﻨﻬﺎ ﻤﺤﺩﻭﺩﺓ ﻤﻥ ﺍﻷﺴﻔل ﺒﺤﺩﻫﺎ‬ ‫‪ (1‬ب ـ ﺍﻝﻤﺘﺘﺎﻝﻴﺔ ) ‪ (w n‬ﺤﺴﺎﺒﻴﺔ ﺃﺴﺎﺴﻬﺎ ‪ 0‬ﻭﺤﺩﻫﺎ ﺍﻷﻭل ‪.1‬‬
‫ﺍﻷﻭل؛ ﻭﺍﻝﺠﻤﻠﺔ ﺍﻝﻤﻌﻁﺎﺓ ﺼﺤﻴﺤﺔ‪.‬‬ ‫‪2‬‬
‫‪ −‬ﻭﺤﺩﻫﺎ ﺍﻷﻭل ‪.1‬‬ ‫ـ ـ ﺍﻝﻤﺘﺘﺎﻝﻴﺔ ) ‪ (v n‬ﻫﻨﺩﺴﻴﺔ ﺃﺴﺎﺴﻬﺎ‬
‫‪ (4‬ﺍﻝﺠﻤﻠﺔ ﺼﺤﻴﺤﺔ‪.‬‬ ‫‪3‬‬
‫د ـ ﺍﻝﻤﺘﺘﺎﻝﻴﺔ ) ‪ (w n‬ﻫﻨﺩﺴﻴﺔ ﺃﺴﺎﺴﻬﺎ ‪.1‬‬
‫‪ (5‬ﺍﻝﺠﻤﻠﺔ ﺼﺤﻴﺤﺔ‪.‬‬
‫‪n‬‬
‫‪ (6‬ﺠﻤﻠﺔ ﺨﺎﻁﺌﺔ ﻷﻨﻪ ﻴﻤﻜﻥ ﺃﻥ ﺘﻜﻭﻥ ‪lim u n = lim v n‬‬ ‫‪ 2‬‬
‫∞‪n →+‬‬ ‫∞‪n →+‬‬
‫ـ ـ ‪. w n = 1‬‬ ‫‪ (2‬أ ـ ‪. v n =  − ‬‬
‫‪ 3‬‬
‫‪1‬‬ ‫‪1‬‬
‫= ‪.v n‬‬ ‫= ‪ un‬ﻭ‬ ‫ﻤﺜﻼ‬ ‫‪ (3‬أ ـ ‪. u n = v 0 + v 1 + ... + v n −1‬‬
‫‪n +1‬‬ ‫‪n +2‬‬
‫‪ (u n ) 148‬ﻤﻌﺭﻓﺔ ﻋﻠﻰ ‪ ℕ‬ﺒـ ‪ u 0 = 1,5 :‬ﻭﻤﻥ ﺃﺠل‬
‫‪3‬‬
‫ب ـ ) ‪. u n = (w n − v n‬‬
‫‪5‬‬
‫ﻜل ﻋﺩﺩ ﻁﺒﻴﻌﻲ ‪. u n +1 = 2u n − 1 ، n‬‬ ‫‪3‬‬
‫‪.‬‬ ‫د ـ ﺍﻝﻤﺘﺘﺎﻝﻴﺔ ) ‪ (u n‬ﻤﺘﻘﺎﺭﺒﺔ ﻭﻨﻬﺎﻴﺘﻬﺎ‬
‫‪ (1‬ﺼﺤﻴﺤﺔ ﻷﻨﻪ ) ‪ u n +1 = f (u n‬ﻭ ‪، f ( x ) = 2x − 1‬‬ ‫‪5‬‬
‫)‪2n + ( −1‬‬
‫‪n‬‬
‫‪ f ( x ) = x‬ﻤﻌﻨﺎﻩ ‪. x = 1‬‬ ‫‪ .‬ـ ـ ‪ n sin 1‬ﻭ ‪. n > 0‬‬
‫‪n‬‬
‫‪ (1 145‬ب ـ‬
‫‪n‬‬ ‫‪n +1‬‬
‫‪ v n +1 = u n +1 − 1 = 2u n − 2 = 2v n (2‬ﺇﺫﻥ ﺍﻝﺠﻤﻠﺔ‬
‫‪ (2‬ب ـ ﺍﻝﻤﺘﺘﺎﻝﻴﺔ ‪ v‬ﻤﺤﺩﻭﺩﺓ ﻤﻥ ﺍﻷﺴﻔل‪.‬‬
‫ﺼﺤﻴﺤﺔ‪.‬‬
‫د ـ ﻻ ﻴﻤﻜﻥ ﻤﻌﺭﻓﺔ ﺇﻥ ﻜﺎﻨﺕ ﺍﻝﻤﺘﺘﺎﻝﻴﺔ ‪ v‬ﺘﻘﺒل ﻨﻬﺎﻴﺔ ﺃﻡ ﻻ ‪.‬‬
‫‪n −1‬‬
‫‪ v n = 2‬ﻭﻤﻨﻪ ∞‪ lim v n = +‬ﻭﺒﺎﻝﺘﺎﻝﻲ ﺍﻝﻤﺘﺘﺎﻝﻴﺔ‬ ‫‪(3‬‬
‫∞‪n →+‬‬

‫) ‪ (v n‬ﻝﻴﺴﺕ ﻤﺤﺩﻭﺩﺓ ﻤﻥ ﺍﻷﻋﻠﻰ ‪ ،‬ﻭﺍﻝﺠﻤﻠﺔ ﺍﻝﻤﻌﻁﺎﺓ ﺨﺎﻁﺌﺔ‬ ‫أ‪ N‬أم ‪ ]0c‬؟‬


‫ﺍﻷﻧﺸﻄﺔ‬
‫ا
ط اول‬
‫ ‪/ :‬‬
‫ا
ف‪ :‬ر
 م ا ‪
/‬ا‪. ℤ 
(R‬‬
‫ ت‪ :‬م ا ‪-‬ط آ*) ‪P‬ا ا ‪1‬ب و ج ‪' #‬ة " ‪
(R‬ا ‪ " ℤ 
/‬و ‪  #‬أاج‪.‬‬
‫ا
‪./ :‬‬
‫ا
ط ا
‬
‫ ‪/ :‬‬
‫ا
ف‪ >?3 :‬ا ا‪ #7‬و ا |ت‪.‬‬
‫ ت‪ :‬م ا ‪-‬ط  أاج ‪.‬‬
‫ا
‪./ :‬‬
‫ا
ط ا

‬
‫ ‪/ :‬‬
‫ا
ف‪ :‬ر
ا ‪ #7‬ا ‪'-‬ك ا^آ‪.'1‬‬
‫ ت‪ :‬م ا ‪-‬ط آ*) ('ة " ا ‪ #7‬ا ‪'-‬ك ا^آ‪ " '1‬و ‪  #‬أاج‪.‬‬
‫ا
‪../ :‬‬
‫ا
ط ا
ا‬
‫ ‪/ :‬‬
‫ا
ف‪ >?3 :‬ا ا‪ ،#7‬ا |ت‪ ،‬ا '‪6‬ت ا 
‪.… ،‬‬
‫ ت‪ : :‬م ا ‪-‬ط  أاج‪.‬‬
‫ا
‪./ :‬‬

‫ﺍﻷﻋﻤﺎﻝ ﺍﻟﻤﻮﺟﻬﺔ‬
‫ا
‪ lk‬ت ا
‪4 V‬ر‪% l‬‬
‫ ‪/ :‬‬
‫ا
ف‪ >?3 :‬ا ا‪.#7‬‬
‫ ت‪ #3 @ :‬ا ‪ )@2  )6‬أاج آ @ ا‪'R‬ا‪ U‬آا‪. X  Y4‬‬
‫ا
‪./ :‬‬

‫ا
"‪  XV‬ا‪ %0D‬ا
‪D‬ب‬
‫ ‪/ :‬‬
‫ا
ف‪ >?3 :‬ا ا‪.#7‬‬
‫ ت‪ #3 @ :‬ا ‪ )@2  )6‬أاج آ @ ا‪'R‬ا‪ U‬آا‪. X  Y4‬‬
‫ا
‪./ :‬‬

‫ﺍﻟﺘﻤﺎﺭﻳﻦ‬
‫ﺘﻤﺎﺭﻴﻥ ﺘﻁﺒﻴﻘﻴﺔ‬
‫‪ 1‬ـ ‪ % :P‬ا
‪ℤ \ %2I.‬‬
‫‪ 1‬ﻤﺠﻤﻭﻋﺔ ﻗﻭﺍﺴﻡ ﺍﻝﻌﺩﺩ ‪ 20‬ﻫﻲ ‪. {−20, −10, −5, −4, −2, −1,1, 2, 4,5,10, 20} :‬‬

‫‪ 2‬ﻤﺠﻤﻭﻋﺔ ﻗﻭﺍﺴﻡ ﺍﻝﻤﻭﺠﺒﺔ ﻝﻠﻌﺩﺩ ‪ 39‬ﻫﻲ }‪{1,3,13,39‬‬


‫})‪. ( a , b ) ∈ {(1,39 ) ; ( 39,1) ; ( 3,13) ; (13,3‬‬

‫‪ 3‬ﻝﺩﻴﻨﺎ ‪ x 2 − y 2 = 15‬ﺘﻌﻨﻲ ‪ ( x − y )( x + y ) = 15‬ﻭﻴﻜﻭﻥ ﺍﻝﻌﺩﺩﺍﻥ ﺍﻝﺼﺤﻴﺤﺎﻥ ‪ x − y‬ﻭ ‪ x + y‬ﻤﻥ ﻗﻭﺍﺴﻡ‪.15‬‬

‫‪ 4‬ﺃ ـ ‪. ( x − 2 )( y − 3) = xy − 3x − 2 y + 6‬‬

‫ﺏ ـ ‪ xy = 3x + 2 y‬ﺘﻌﻨﻲ ‪ xy − 3x − 2 y + 6 = 6‬ﺃﻱ ‪ ( x − 2 )( y − 3) = 6‬ﺜﻡ ﻨﺴﺘﻌﻤل ﻗﻭﺍﺴﻡ ‪.6‬‬

‫‪ −1027 ≤ 53k ≤ 1112 7‬ﻤﻌﻨﺎﻩ ‪−19 ≤ k ≤ 20‬‬


‫ﻋﺩﺩ ﺍﻝﻤﻀﺎﻋﻔﺎﺕ ﻝﻠﻌﺩﺩ ‪ 53‬ﺍﻝﻤﺤﺼﻭﺭﺓ ﺒﻴﻥ ‪ −1027‬ﻭ ‪ 1112‬ﻫﻭ ‪.40‬‬

‫‪ a = 7 k (1 8‬ﻭ ‪ 7 k < 50‬ﺃﻱ ‪k ≤ 7‬‬


‫}‪. a ∈ {7,14, 21, 28,35, 42, 49‬‬
‫‪33 11 11a‬‬
‫ﺤﻴﺙ ‪ a‬ﻋﺩﺩ ﺼﺤﻴﺢ ﻏﻴﺭ ﻤﻌﺩﻭﻡ ‪ 0 < 7a < 50 ،‬ﻤﻌﻨﺎﻩ ‪ 0 < a ≤ 7‬ﻭﺒﺎﻝﺘﺎﻝﻲ ‪:‬‬ ‫= =‬ ‫‪(2‬‬
‫‪21 7 7a‬‬
‫‪11 22 33 44 55 66 77‬‬
‫=‬ ‫=‬ ‫=‬ ‫=‬ ‫=‬ ‫=‬
‫‪7 14 21 28 35 42 49‬‬
‫‪ 13 9‬ﻗﺎﺴﻡ ﻝﻠﻌﺩﺩ ‪ n + 4‬ﻤﻌﻨﺎﻩ ‪ n + 4 = 13k‬ﻤﻊ ∗‪ k ∈ ℕ‬ﺃﻱ ‪ n ≤ 22 . n = 13k − 4‬ﻤﻌﻨﺎﻩ ‪−24 ≤ n ≤ 22‬‬
‫‪24‬‬ ‫‪22‬‬
‫‪ −‬ﺃﻱ }‪، k ∈ {−1,0,1‬‬ ‫≤ ‪≤k‬‬ ‫ﻭﻴﻜﺎﻓﺊ ‪ −24 ≤ 13k ≤ 22‬ﻭﻤﻌﻨﺎﻩ‬
‫‪13‬‬ ‫‪13‬‬
‫ﻭﻤﻨﻪ }‪. n ∈ {−17 , − 4,9‬‬
‫‪ , 12 = 22 × 4 10‬ﻤﺠﻤﻭﻋﺔ ﻗﻭﺍﺴﻡ ‪ 12‬ﻫﻲ ‪D12 = {−12, −6, −4, −3, −2, −1,1, 2,3, 4, 6,12} :‬‬
‫‪1 1 2 3 4 6 12‬ـ‪2‬ـ ‪3‬ـ ‪4‬ـ ‪6‬ـ ‪12‬ـ ‪5n + 7‬‬
‫‪1 5‬ـ ‪3‬ـ ‪4‬ـ‪5‬ـ‪6‬ـ‪8‬ـ‪9‬ـ‪10‬ـ‪11‬ـ‪13‬ـ‪19‬ـ ‪5n‬‬
‫‪n‬‬ ‫‪2‬ـ‬ ‫‪1‬ـ‬ ‫‪1‬‬
‫ﺍﻝﻌﺩﺩ ‪ n + 6‬ﻴﻘﺒل ﺍﻝﻘﺴﻡ ﻋﻠﻰ ‪ n‬ﻤﻌﻨﺎﻩ ‪ n + 6 = nk‬ﻤﻊ ∗‪ k ∈ ℕ‬ﻭﻴﻜﺎﻓﺊ )‪ 6 = n ( k − 1‬ﺇﺫﻥ ‪ n‬ﻴﻘﺴﻡ ‪. 6‬‬
‫ﻭﺒﺎﻝﺘﺎﻝﻲ }‪ . n ∈ {1; 2;3;6‬ﻭﺒﺎﻝﻌﻜﺱ ﻜل ﺍﻝﻘﻴﻡ ﺍﻝﻤﻌﻴﻨﺔ ﺘﺤﻘﻕ ﺍﻝﻤﻁﻠﻭﺏ ‪.‬‬
‫‪ 34 = 2 × 17 (1‬ﻭﻤﻨﻪ ﻤﺠﻤﻭﻋﺔ ﻗﻭﺍﺴﻡ ‪ 34‬ﻫﻲ ‪:‬‬
‫}‪. D34 = {−34, −17, −2, −1,1, 2,17,34‬‬

‫‪1 1 2 17 34‬ـ ‪2‬ـ ‪17‬ـ ‪34‬ـ ‪5n + 6‬‬


‫‪4 11 28‬ـ‪5‬ـ‪9‬ـ ‪8‬ـ ‪23‬ـ ‪40‬ـ ‪5n‬‬
‫‪n‬‬ ‫‪8‬ـ‬ ‫‪1‬ـ‬
‫‪ 5n + 6 (2‬ﻗﺎﺴﻡ ﻝﻠﻌﺩﺩ ‪ n + 8‬ﻭﻤﻨﻪ ‪ 5n + 6‬ﻴﻘﺴﻡ ‪ 5n + 40‬ﺇﺫﻥ ‪ 5n + 6‬ﻴﻘﺴﻡ ) ‪ ( 5n + 40 ) − ( 5n + 6‬ﺃﻱ ‪5n + 6‬‬
‫ﻴﻘﺴﻡ ‪ 34‬ﻭﻤﻨﻪ ‪ n = −1‬ﺃﻭ ‪. n = −8‬‬
‫ﻭﺒﺎﻝﻌﻜﺱ ﺇﺫﺍ ﻜﺎﻥ ‪ n = −1‬ﻓﺈﻥ ‪ 1‬ﻴﻘﺴﻡ ‪ 7‬ﻭﺇﺫﺍ ﻜﺎﻥ ‪ n = −8‬ﻓﺈﻥ ‪ −34‬ﻴﻘﺴﻡ ‪ 0‬ﺇﺫﻥ ﻜﻼ ﺍﻝﻨﺘﻴﺠﺘﻴﻥ ﺘﺤﻘﻕ ﺍﻝﻤﻁﻠﻭﺏ ‪.‬‬
‫‪ n 14‬ﻋﺩﺩ ﺼﺤﻴﺢ ‪ .‬ﻨﻀﻊ ‪ a = 3n + 7‬ﻭ ‪b = 7 n + 2‬‬

‫ﺇﺫﺍ ﻜﺎﻥ ﺍﻝﻌﺩﺩ ‪ d‬ﻗﺎﺴﻤﺎ ِـ ‪ a :‬ﻭ ‪ b‬ﻓﺈﻥ ‪ d‬ﻴﻘﺴﻡ ‪ 7a‬ﻭ ‪3b‬‬


‫ﻭﻤﻨﻪ ‪ d‬ﻴﻘﺴﻡ ‪. 7a − 3b = 49‬‬
‫‪ n 15‬ﻋﺩﺩ ﻁﺒﻴﻌﻲ ﻏﻴﺭ ﻤﻌﺩﻭﻤﺎ ﻭﻴﺨﺘﻠﻑ ﻋﻥ ﺍﻝﻌﺩﺩ ‪. 1‬‬
‫)‪ . n 3 − n = n ( n − 1)( n + 1‬ﺒﻌﺽ ﺍﻝﻘﻭﺍﺴﻡ ﻝﻠﻌﺩﺩ ‪، n 2 − 1 ، n 2 + n ، n 2 − n ، n + 1 ، n ، n − 1 ، 1 : n 3 − n‬‬

‫‪n3 − n‬‬
‫‪ 17‬ﻝﻴﻜﻥ ‪ a‬ﻭ ‪ b‬ﻋﺩﺩﻥ ﺼﺤﻴﺤﻴﻥ ﻏﻴﺭ ﻤﻌﺩﻭﻤﻴﻥ ‪.‬‬
‫) ‪(a + b‬‬ ‫ﺃ( ‪= a 3 + 3a 2b + 3ab 2 + b 3‬‬
‫‪3‬‬

‫ﺏ( ﻨﻔﺭﺽ ﺃﻥ ‪ a 3 + b 3 = 3k‬ﺇﺫﻥ‬

‫) ‪(a + b‬‬ ‫) ‪= 3k + 3a 2b + 3ab 2 = 3 ( k + a 2b + ab 2‬‬


‫‪3‬‬

‫‪ 2‬ـ ا
‪ %2I.‬ا‪%- :P‬‬
‫‪ 18‬ﺘﻌﻴﻴﻥ ﺒﺎﻗﻲ ﺍﻝﻘﺴﻤﺔ ﺍﻷﻗﻠﻴﺩﻴﺔ ﻝﻠﻌﺩﺩ ‪ a‬ﻋﻠﻰ ‪: b‬‬
‫أ ـ ‪ a = 118‬و ‪ 118 = 5 × 23 + 3 . b = 5‬ا ‪ R1‬ه ‪. 3‬‬
‫ب ـ ‪ a = 152‬و ‪ 152 = 7 × 21 + 5 . b = 7‬ا ‪ R1‬ه ‪. 5‬‬
‫‪4‬ـ ـ ‪ a = −118‬و ‪−118 = 5 ( −24 ) + 2 . b = 5‬‬
‫د ـ ‪ a = −152‬و ‪−152 = 7 ( −22 ) + 2 . b = 7‬‬

‫‪ 19‬ﻋﻴﻥ ﺍﻷﻋﺩﺍﺩ ﺍﻝﻁﺒﻴﻌﻴﺔ ‪ n = 41k + 5‬ﻤﻊ ‪ 41k + 5 < 100‬ﺃﻱ ‪ k ≤ 2‬ﻭﻤﻨﻪ }‪n ∈ {5, 46,87‬‬

‫‪ a 20‬ﻭ ‪ b‬ﻋﺩﺩﺍﻥ ﻁﺒﻴﻌﻴﺎﻥ ﻏﻴﺭ ﻤﻌﺩﻭﻤﻴﻥ ﺤﻴﺙ ‪ a = 17b + 3‬ﻭ ‪ b > 3‬ﻭ ‪، a = 23b + 27‬‬
‫ﺇﺫﻥ ‪ 6b − 24 = 0‬ﻭﻤﻨﻪ ‪ b = 4‬ﻭ ‪a = 71‬‬
‫‪ n 21‬ﻋﺩﺩ ﻁﺒﻴﻌﻲ ‪ ،‬ﺒﻘﺴﻤﺘﻪ ﻋﻠﻰ ‪ 7‬ﺃﻭ ﻋﻠﻰ ‪ 3‬ﻨﺠﺩ ﻨﻔﺱ ﺍﻝﺒﺎﻗﻲ ﺃﻱ ‪ n = 7 k + r‬ﻭ ‪ n = 3k '+ r‬ﻤﻊ ‪. 0 ≤ r < 3‬‬
‫‪ n − r‬ﻴﻘﺒل ﺍﻝﻘﺴﻤﺔ ﻋﻠﻰ ‪ 3‬ﻭ ‪ 7‬ﻭﻫﻤﺎ ﻋﺩﺩﺍﻥ ﺃﻭﻝﻴﺎﻥ ‪،‬‬
‫ﺇﺫﻥ ﻤﻭﺠﻭﺩﻴﻥ ﻓﻲ ﺘﺤﻠﻴﻠﻪ ﻭﺒﺎﻝﺘﺎﻝﻲ ‪ 21‬ﻴﻜﻭﻥ ﻗﺎﺴﻡ ﻝﻪ ‪،‬‬
‫ﺃﻱ ‪ n − r = 21α‬ﻤﻌﻨﺎﻩ ‪ n = 21α + r‬ﺒﻤﺎ ﺃﻥ ‪0 ≤ r < 3‬‬
‫ﻓﺈﻥ ‪ n = 21α + 1 ، n = 21α‬ﺃﻭ ‪. α ∈ ℕ , n = 21α + 2‬‬
‫‪ a 24‬ﻭ ‪ b‬ﻋﺩﺩﺍﻥ ﻁﺒﻴﻌﻴﺎﻥ ﻏﻴﺭ ﻤﻌﺩﻭﻤﻴﻥ ﺤﻴﺙ ‪:‬‬
‫‪ a + b = 416‬ﻭ ‪ a = bk + 61‬ﻤﻊ ‪. b > 61‬‬
‫ﻭﻤﻨﻪ ‪ bk + 61 + b = 416‬ﺃﻱ ‪ b ( k + 1) = 355‬ﺇﺫﻥ ‪ b‬ﻗﺎﺴﻡ ﻝﻠﻌﺩﺩ ‪ 355‬ﻭﻝﺩﻴﻨﺎ ‪. 355 = 5 × 71‬ﻗﻭﺍﺴﻡ ‪ 355‬ﻫﻲ ‪، 1‬‬
‫‪ 71 ، 5‬ﻭ ‪ 355‬ﺒﻤﺎ ﺃﻥ ‪ b > 61‬ﻓﺈﻥ ‪ b = 71‬ﺃﻭ ‪. b = 355‬‬
‫ﺇﺫﺍ ﻜﺎﻥ ‪ b = 71‬ﻓﺈﻥ ‪. a = 416 − 71 = 345‬‬
‫ﺇﺫﺍ ﻜﺎﻥ ‪. a = 416 − 355 = 61 b = 355‬‬
‫‪ 25‬ﺍﺴﺘﻌﻤﺎل ﺨﻭﺍﺭﺯﻤﻴﺔ ﺃﻗﻠﻴﺩﺱ ﻝﺘﻌﻴﻴﻥ ) ‪: PGCD ( a , b‬‬
‫أ ـ ‪ a = 315‬و ‪315 = 117 × 2 + 81 . b = 117‬‬
‫‪ 117 = 81× 2 + 36‬؛ ‪ 81 = 36 × 2 + 9‬؛ ‪ . 36 = 9 × 4 + 0‬و ‪. PGCD ( 315,117 ) = 9 U‬‬
‫ب ـ ‪ a = 1260‬و ‪ 1260 = 528 × 2 + 204 . b = 528‬؛ ‪ 528 = 204 × 2 + 120‬؛ ‪ 204 = 120 × 1 + 84‬؛‬
‫‪ 120 = 84 × 1 + 36‬؛ ‪ 84 = 36 × 2 + 12‬؛ ‪ 36 = 12 × 3 + 0‬و ‪. PGCD (1260,528) = 12 U‬‬
‫‪4‬ـ ـ ‪ a = 1380‬و ‪. b = 972‬‬
‫‪ 1380 = 972 ×1 + 408‬؛ ‪ 972 = 408 × 2 + 156‬؛‬
‫‪ 408 = 156 × 2 + 96‬؛ ‪ 156 = 96 ×1 + 60‬؛ ‪ 96 = 60 × 1 + 36‬؛ ‪ 60 = 36 × 1 + 24‬؛ ‪ 36 = 24 ×1 + 12‬؛‬
‫‪ 24 = 12 × 2 + 0‬و ‪. PGCD (1380,972 ) = 12 U‬‬
‫‪ n 26‬ﻋﺩﺩ ﻁﺒﻴﻌﻲ ﻏﻴﺭ ﻤﻌﺩﻭﻡ ‪.‬‬
‫‪ PGCD ( 3n , n ) = n‬؛ ‪PGCD ( n 2 , n ) = n‬‬

‫‪ 27‬ﺍﻝﺒﺭﻫﺎﻥ ﺃﻥ ﻤﺠﻤﻭﻋﺔ ﺍﻝﻘﻭﺍﺴﻡ ﺍﻝﻤﺸﺘﺭﻜﺔ ﻝﻠﻌﺩﺩﻴﻥ ‪ a‬ﻭ ‪ b‬ﻫﻲ ﻨﻔﺴﻬﺎ ﻤﺠﻤﻭﻋﺔ ﻗﻭﺍﺴﻡ ﺍﻝﻌﺩﺩ ) ‪. p gcd ( a , b‬‬

‫ﻨﻀﻊ ) ‪. δ = p gcd ( a , b‬‬


‫ﻜل ﻋﺩﺩ ‪ d‬ﻗﺎﺴﻡ ﻝﻠﻌﺩﺩ ‪ δ‬ﻫﻭ ﻗﺎﺴﻡ ﻝﻠﻌﺩﺩﻴﻥ ‪ a‬ﻭ ‪ b‬ﻷﻥ ‪ δ‬ﻴﻘﺴﻡ ‪ a‬ﻭ ‪. b‬‬
‫ﺍﻝﻌﻜﺱ ﻨﻔﺭﺽ ﺃﻥ ‪ d‬ﻗﺎﺴﻡ ﻝﻠﻌﺩﺩﻴﻥ ‪ a‬ﻭ ‪ b‬ﻭﻤﻨﻪ ‪ a = α d‬ﻭ ‪ b = β d‬ﻤﻊ ‪ α‬ﻭ ‪ β‬ﻋﺩﺩﻴﻥ ﻁﺒﻴﻌﻴﻴﻥ ﻏﻴﺭ ﻤﻌﺩﻭﻤﻴﻥ ‪.‬‬
‫ﺇﺫﺍ ﻜﺎﻥ ‪ p gcd (α , β ) = 1‬ﻓﺈﻥ ‪ p gcd ( a , b ) = d‬ﻭﻤﻨﻪ ‪ d = δ‬ﻭﺒﺎﻝﺘﺎﻝﻲ ‪ d‬ﻴﻘﺴﻡ ‪. δ‬‬
‫ﺇﺫﺍ ﻜﺎﻥ ‪ p gcd (α , β ) = λ‬ﻤﻊ ‪ λ ≠ 1‬ﻓﺈﻨﻪ ﻴﻭﺠﺩ ﻋﺩﺩﻴﻥ ﻁﺒﻴﻌﻴﻴﻥ ﻏﻴﺭ ﻤﻌﺩﻭﻤﻴﻥ ﻭﺃﻭﻝﻴﻴﻥ ﻓﻴﻤﺎ ﺒﻴﻨﻬﻤﺎ ' ‪ α‬ﻭ ' ‪ β‬ﺤﻴﺙ‬
‫' ‪ α = λα‬ﻭ ' ‪ β = λβ‬ﻭﻤﻨﻪ ' ‪ a = d λα‬ﻭ ' ‪ b = d λβ‬ﺇﺫﻥ ‪ p gcd ( a , b ) = d λ‬ﻭﻤﻨﻪ ‪ δ = d λ‬ﻭﻤﻨﻪ ‪ d‬ﻴﻘﺴﻡ ‪. δ‬‬
‫‪1‬‬ ‫‪1‬‬ ‫‪2‬‬ ‫‪1‬‬ ‫‪4‬‬ ‫‪28‬‬
‫‪792 456 336 120 96 24 0‬‬
‫ﺇﺫﻥ ‪ . PGCD ( 792, 456 ) = 24‬ﻝﺩﻴﻨﺎ ‪. 24 = 23 × 3‬‬
‫ﻭﻤﻨﻪ ﻤﺠﻤﻭﻋﺔ ﺍﻝﻘﻭﺍﺴﻡ ﺍﻝﻤﺸﺘﺭﻜﺔ ﻝﻠﻌﺩﺩﻴﻥ ‪ 456‬ﻭ ‪ 792‬ﻫﻲ‪:‬‬
‫}‪. D24 = {1, 2,3, 4, 6,8,12, 24‬‬
‫‪1‬‬ ‫‪2‬‬ ‫‪5‬‬ ‫ـ‬ ‫‪29‬‬
‫‪448 308 140‬‬ ‫‪28‬‬ ‫‪0‬‬
‫إذن ‪28 = 22 × 7   . PGCD ( 448,308 ) = 28‬‬
‫ـ ﻤﺠﻤﻭﻋﺔ ﺍﻝﻘﻭﺍﺴﻡ ﺍﻝﻤﺸﺘﺭﻜﺔ ﻝﻠﻌﺩﺩﻴﻥ ‪ 448‬ﻭ ‪ 308‬ﻫﻲ ‪. D28 = {1, 2, 4, 7,14, 28} :‬‬

‫‪ 4294 = nk + 10‬ﻭ ‪ 3521 = nk '+ 11‬ﻭﻤﻨﻪ‬


‫‪ 4284 = nk‬ﻭ ' ‪ 3510 = nk‬ﺇﺫﻥ ‪ n‬ﻫﻭ ﻗﺎﺴﻡ ﻝﻠﻌﺩﺩﻴﻥ ‪ 4284‬ﻭ ‪. 3510‬‬
‫‪1‬‬ ‫‪4‬‬ ‫‪1‬‬ ‫‪1 6 1 2‬‬
‫‪4284 3510 774 414 360 54 36 18 0‬‬
‫‪ PGCD ( 4284,3510 ) = 18‬ﻭﻝﺩﻴﻨﺎ ‪. 18 = 2 × 32 :‬‬
‫ﺇﺫﻥ }‪. n ∈ {1, 2,3, 6,9,18‬‬
‫‪ n 31‬ﻋﺩﺩ ﻁﺒﻴﻌﻲ ﻤﻜﻭﻥ ﻤﻥ ﺃﺭﺒﻌﺔ ﺃﺭﻗﺎﻡ ﺤﻴﺙ‪:‬‬
‫‪ 21685 = nk + 37‬ﻭ ‪33509 = nk '+ 53‬‬
‫ﻭﻤﻨﻪ ‪ 21648 = nk‬ﻭ ' ‪33456 = nk‬‬
‫ﺇﺫﻥ ‪ n‬ﻫﻭ ﻗﺎﺴﻡ ﻝﻠﻌﺩﺩﻴﻥ ‪ 21648‬ﻭ ‪ 33456‬ﻝﺩﻴﻨﺎ ) ‪PGCD ( 33456, 21648) = 12 × PGCD ( 2788,1804‬‬
‫‪1‬‬ ‫‪1‬‬ ‫‪1‬‬ ‫‪5‬‬
‫‪2788 1804 984 820 164 0‬‬
‫‪PGCD ( 33456, 21648 ) = 12 ×164 = 1968‬‬
‫‪ 1968 = 2 × 984‬ﺇﺫﻥ ﺍﻝﻘﺎﺴﻡ ﺍﻝﻭﺤﻴﺩ ﺍﻝﻤﻜﻭﻥ ﻤﻥ ﺃﺭﺒﻌﺔ ﺃﺭﻗﺎﻡ ﻝﻠﻌﺩﺩ ) ‪ PGCD ( 33456, 21648‬ﻫﻭ ﻨﻔﺴﻪ ؛‬
‫ﺇﺫﻥ ‪. n = 1968‬‬
‫‪1‬‬ ‫‪2‬‬ ‫‪4‬‬ ‫‪(1 32‬‬
‫‪182 126 56 14‬‬ ‫‪0‬‬
‫ﺇﺫﻥ ‪. PGCD (182,126 ) = 14‬‬
‫‪ (2‬ﺍﺴﺘﻌﻤﺎل ﺨﻭﺍﺭﺯﻤﻴﺔ ﺃﻗﻠﻴﺩﺱ ‪:‬‬
‫‪ 182 = 126 × 1 + 56‬ﻤﻌﻨﺎﻩ ‪182 − 126 = 56‬‬
‫‪ 126 = 56 × 2 + 14‬ﻤﻌﻨﺎﻩ ‪126 − 56 × 2 = 14‬‬
‫ﺇﺫﻥ ‪ 14 = 126 − 56 × 2 = 126 − (182 − 126 ) × 2 :‬ﺃﻱ ‪ . 14 = 182 ( −2 ) + 126 × 3‬ﺇﺫﻥ ‪ α = −2‬ﻭ ‪. β = 3‬‬

‫‪ 3‬ـ ‪c‬اص ا
‪ FD.‬ا
‪"2‬ك اآ‪/‬‬
‫‪ 1399 = 82 × 17 + 5 33‬ﻭﻤﻨﻪ ﺍﻝﺒﺎﻗﻲ ﻫﻭ ‪. 5‬‬
‫‪PGCD (1399,82 ) = PGCD ( 82,5 ) = 1‬‬

‫‪ 34‬ﺘﻌﻴﻴﻥ ﺍﻝﻘﺎﺴﻡ ﺍﻝﻤﺸﺘﺭﻙ ﺍﻷﻜﺒﺭ ﻝﻠﻌﺩﺩﻴﻥ ﺍﻝﺼﺤﻴﺤﻴﻥ ‪ a‬ﻭ ‪: b‬‬


‫أ ـ ‪ a = −350‬و ‪. b = −252‬‬
‫‪PGCD ( −350, − 252 ) = PGCD ( 350, 252 ) = 14‬‬
‫ب ـ ‪ a = 126‬و ‪. b = −735‬‬
‫‪PGCD (126, − 735 ) = PGCD (126, 735 ) = 21‬‬
‫‪4‬ـ ـ ‪ a = −138‬و ‪. b = 575‬‬
‫‪PGCD ( −138,575 ) = PGCD (138,575 ) = 23‬‬

‫‪PGCD ( 54,82 ) = 2 35‬‬


‫‪. PGCD ( 5400,8200 ) = 100PGCD ( 54,82 ) = 200‬‬

‫ﻤﻥ ﺍﻝﺘﻤﺭﻴﻥ ‪ 36‬ﺇﻝﻰ ﺍﻝﺘﻤﺭﻴﻥ ‪ ، 41‬ﻋﻴ‪‬ﻥ ﻜل ﺍﻝﺜﻨﺎﺌﻴﺎﺕ ) ‪ ( a , b‬ﻤﻥ ﺍﻷﻋﺩﺍﺩ ﺍﻝﻁﺒﻴﻌﻴﺔ ﺍﻝﺘﻲ ﺘﺤﻘﻕ ﺍﻝﺸﺭﻁﻴﻥ ﺍﻝﻤﻘﺘﺭﺤﻴﻥ‪.‬‬

‫ﻨﻀﻊ ‪ PGCD ( a , b ) = d :‬ﻭﻨﻁﺒﻕ ﺍﻝﺨﺎﺼﻴﺔ ' ‪ b = db ' ، a = da‬ﻤﻊ ' ‪ a‬ﻭ ' ‪ b‬ﺃﻭﻝﻴﻴﻥ ﻓﻴﻤﺎ ﺒﻴﻨﻬﻤﺎ ‪.‬‬
‫‪ 9 ( a '+ b ' ) = 54‬‬ ‫‪a + b = 54‬‬
‫‪‬‬ ‫‪ 6 ‬‬ ‫‪36‬‬
‫‪ p gcd ( a ', b ' ) = 1‬‬ ‫‪ PGCD ( a , b ) = 9‬‬
‫‪ a '+ b ' = 6‬‬
‫أي }) ‪. ( a , b ) ∈ {( 9, 45 ) ; ( 45,9‬‬ ‫})‪{(1,5) ; ( 5,1‬‬ ‫‪ ‬و@† )' ‪  3 ( a ', b‬إ ‪j‬‬ ‫و‪ 6‬‬
‫‪ p gcd ( a ', b ') = 1‬‬
‫‪a + b = 72‬‬
‫}) ‪( a , b ) ∈ {( 9, 63) ; ( 27, 45 ) ; ( 45, 27 ) ; ( 63,9‬‬ ‫‪‬؛‬ ‫‪37‬‬
‫‪ PGCD ( a , b ) = 9‬‬
‫‪a + b = 420‬‬
‫‪  PGCD a , b = 84 38‬؛ }) ‪( a , b ) ∈ {( 84,336 ) ; (168, 252 ) ; ( 252,168 ) ; ( 336,84‬‬
‫‪‬‬ ‫) (‬
‫‪ 36a 'b ' = 360‬‬ ‫‪ ab = 360‬‬
‫‪‬‬ ‫‪ ‬ﻤﻌﻨﺎﻩ‬ ‫‪39‬‬
‫‪ p gcd ( a ', b ') = 1‬‬ ‫‪PGCD ( a , b ) = 6‬‬
‫‪ a 'b ' = 10‬‬
‫‪ ‬ﻭﻴﻜﺎﻓﺊ })‪ ( a ', b ') ∈ {(1,10 ) ; ( 2,5) ; ( 5, 2 ) ; (10,1‬ﺃﻱ‬ ‫ﻭﻤﻌﻨﺎﻩ‬
‫‪ p gcd ( a ', b ') = 1‬‬
‫}) ‪(a , b ) ∈ {( 6, 60 ) ; (12,30 ) ; ( 30,12 ) ; ( 60, 6‬‬
‫‪ab = 2700‬‬
‫})‪(a , b ) ∈ {( 5,540 ) ; ( 20,135) ; ( 20,135) ; ( 540,5‬‬ ‫‪‬؛‬ ‫‪40‬‬
‫‪ PGCD ( a , b ) = 5‬‬
‫‪ a − b = 825‬‬
‫‪2‬‬ ‫‪2‬‬

‫‪ ‬ﻤﻌﻨﺎﻩ ) ‪ ( a, b ) = ( 85,80‬ﺃﻭ )‪. ( a, b ) = ( 35, 28‬‬ ‫‪41‬‬


‫‪PGCD ( a , b ) = 5‬‬
‫أ ـ ‪ a = 55‬و ‪ b = 36‬؛ ‪. PGCD ( 36,55 ) = 1‬‬
‫‪ a = 14 42‬و ‪ b = 165‬؛ ‪. PGCD (165,14 ) = 1‬‬
‫بـ‬
‫‪4‬ـ ـ ‪ a = 1155‬و ‪ b = 872‬؛ ‪PGCD (1155,872 ) = 1‬‬
‫ﻓﻲ ﻜل ﺤﺎﻝﺔ ﻨﻘﻭل ﺃﻥ ﺍﻝﻌﺩﺩﻴﻥ ‪ a‬ﻭ ‪ b‬ﺃﻭﻝﻴﺎﻥ ﻓﻴﻤﺎ ﺒﻴﻨﻬﻤﺎ ‪.‬‬
‫‪PGCD (140,143) = 1‬‬ ‫‪(1 43‬‬
‫‪ (2‬ﺍﺴﺘﻨﺘﺞ ﻓﻲ ﻜل ﺤﺎﻝﺔ ﻤﻥ ﺍﻝﺤﺎﻝﺘﻴﻥ ﺍﻝﺘﺎﻝﻴﺘﻴﻥ ‪:‬‬
‫‪a = 140 × 34‬‬
‫‪ ‬ﻭ ‪ PGCD (140,143) = 1‬ﻤﻌﻨﺎﻩ ‪. PGCD ( a , b ) = 34‬‬ ‫ﺃـ‬
‫‪b = 143 × 34‬‬
‫‪a = 143 × 82‬‬
‫‪ ‬ﻭ ‪ PGCD (140,143) = 1‬ﻤﻌﻨﺎﻩ ‪. PGCD ( a , b ) = 82‬‬ ‫ﺏـ‬
‫‪b = 140 × 82‬‬
‫‪ 44‬ﻷﻥ ‪ 7‬ﻻ ﻴﻘﺴﻡ ‪. 500‬‬

‫ﺘﻤﺎﺭﻴﻥ ﻝﻠﺘﻌﻤﻕ‬
‫‪ 1‬ـ ‪ % :P‬ا
‪ℤ \ %2I.‬‬
‫‪ 45‬ﺍﻝﻤﺴﺎﻓﺔ ﺒﻴﻥ ﺍﻝﻌﻤﻭﺩﻴﻥ ﺍﻝﻤﺘﺘﺎﻝﻴﻴﻥ ﻫﻲ ﻋﺩﺩ ﻁﺒﻴﻌﻲ ‪ x‬ﺤﻴﺙ ‪ 2 < x < 5‬ﻭﺒﺎﻝﺘﺎﻝﻲ ‪ :‬ﺇﻤﹼﺎ ‪ x = 3‬ﻭﺇﻤﺎ ‪ . x = 4‬ﻝﺩﻴﻨﺎ ‪4‬‬
‫ﻻ ﻴﻘﺴﻡ ‪ 90‬ﺒﻴﻨﻤﺎ ‪ 3‬ﻫﻭ ﻗﺎﺴﻡ ﻤﺸﺘﺭﻙ ﻝﻠﻌﺩﺩﻴﻥ ‪ 90‬ﻭ ‪ ، 156‬ﻭﻨﺄﺨﺫ ﻗﺎﺴﻤﺎ ﻤﺸﺘﺭﻜﺎ ﻷﻥ ﻜل ﺯﺍﻭﻴﺔ ﺍﻝﻘﻁﻌﺔ ﻴﻐﺭﺱ ﻋﻤﻭﺩ‪ .‬ﺇﺫﻥ‬
‫ﺍﻝﻤﺴﺎﻓﺔ ﺒﻴﻥ ﻋﻤﻭﺩﻴﻴﻥ ﻤﺘﺘﺎﻝﻴﻴﻥ ﻫﻲ ‪. 3 m‬‬
‫ﻤﺤﻴﻁ ﺍﻝﻘﻁﻌﺔ ﻫﻭ ‪ 2 ( 90 + 156 ) = 492 m‬ﻭﻝﺩﻴﻨﺎ ﻋﺩﺩ ﺍﻷﻋﻤﺩﺓ ﻫﻭ ﻨﻔﺱ ﻋﺩﺩ ﺍﻝﻔﺭﺍﻏﺎﺕ ﺍﻝﻤﻭﺠﻭﺩﺓ ﺒﻴﻥ ﻋﻤﻭﺩﻴﻥ ﻤﺘﺘﺎﻝﻴﻴﻥ ﺃﻱ‬
‫‪492‬‬
‫‪= 164‬‬
‫‪3‬‬
‫‪ 46‬ﻗﻭﺍﺴﻡ ‪ 220‬ﻫﻲ ‪.220 ،110 ،55 ،44 ،22 ، 20 ، 11 ، 10 ، 5 ، 4 ، 2 ، 1 :‬‬
‫= ‪1 + 2 + 4 + 5 + 10 + 11 + 20 + 22 + 44 + 55 + 110‬‬
‫‪284‬‬
‫ﻗﻭﺍﺴﻡ ‪ 284‬ﻫﻲ ‪. 284 ، 142 ، 71 ، 4 ، 2 ، 1 :‬‬
‫‪. 1 + 2 + 4 + 71 + 142 = 220‬‬
‫‪ 47‬ﻝﻴﻜﻥ ‪ n‬ﻋﺩﺩﺍ ﻁﺒﻴﻌﻴﺎ ﺃﻜﺒﺭ ﻤﻥ ﺃﻭ ﻴﺴﺎﻭﻱ ‪. 3‬‬
‫ﻝﺩﻴﻨﺎ ‪ n + 5 = n − 2 + 7‬ﻭﻤﻨﻪ ‪ n + 5‬ﻤﻀﺎﻋﻑ ِـ ‪n − 2‬‬
‫ﻤﻌﻨﺎﻩ ‪ n − 2‬ﻗﺎﺴﻡ ﻝﻠﻌﺩﺩ ‪ 7‬ﻭﺒﺎﻝﺘﺎﻝﻲ ‪ n − 2 = 1‬ﺃﻭ ‪ n − 2 = 7‬ﺃﻱ ‪ n = 3‬ﺃﻭ ‪. n = 9‬‬
‫ﻋﻜﺴﻴﺎ ﺇﺫﺍ ﻜﺎﻥ ‪ n = 3‬ﺃﻭ ‪ n = 9‬ﻓﺈﻥ ‪ n + 5 = 8‬ﺃﻭ ‪ n + 5 = 14‬ﻭ ‪ n − 2 = 1‬ﺃﻭ ‪ n − 2 = 7‬ﻭﺒﺎﻝﺘﺎﻝﻲ ﻓﻲ ﻜﻼ ﺍﻝﺤﺎﻝﺘﻴﻥ‬
‫‪ n + 5‬ﻤﻀﺎﻋﻑ ِـ ‪. n − 2‬‬
‫‪ (1 48‬ﻗﻭﺍﺴﻡ ‪ 8‬ﻫﻲ ‪ 8 ، 4 ، 2 ، 1‬؛ ﻭﻤﻨﻪ ﻤﺠﻤﻭﻉ ﻗﻭﺍﺴﻡ ﺍﻝﻌﺩﺩ ‪ 8‬ﻫﻭ ‪. 15 :‬‬
‫ﻗﻭﺍﺴﻡ ‪ 81‬ﻫﻲ ‪ 81 ، 27 ، 9 ، 3 ، 1‬؛ ﻭﻤﻨﻪ ﻤﺠﻤﻭﻉ ﻗﻭﺍﺴﻡ ﺍﻝﻌﺩﺩ ‪ 81‬ﻫﻭ ‪. 121 :‬‬

‫‪ (2‬ﻋﺩﺩ ﻗﻭﺍﺴﻡ ‪ 8‬ﻫﻭ ‪ 4‬ﻭﻋﺩﺩ ﻗﻭﺍﺴﻡ ‪ 81‬ﻫﻭ ‪ 5‬ﺇﺫﻥ ﻋﺩﺩ ﻗﻭﺍﺴﻡ ﺍﻝﻌﺩﺩ ‪ 8 × 81‬ﻫﻭ ‪. 4 × 5 = 20‬‬
‫‪n + 2 n −1 + 3 n −1‬‬
‫‪(1 49‬‬
‫‪3‬‬
‫=‬ ‫=‬ ‫‪+‬‬
‫‪n −1‬‬ ‫‪n −1‬‬ ‫‪n −1 n −1‬‬
‫‪3‬‬ ‫‪n +2‬‬ ‫‪n +2‬‬ ‫‪3‬‬
‫ﻋﺩﺩﺍ ﺼﺤﻴﺤﺎ ﻭﻝﻬﺫﺍ ﻴﺠﺏ ﺃﻥ ﻴﻜﻭﻥ‬ ‫ﻋﺩﺩﺍ ﺼﺤﻴﺤﺎ ﻴﻜﻔﻲ ﺃﻥ ﻴﻜﻭﻥ‬ ‫ﻭﺒﺎﻝﺘﺎﻝﻲ ﻝﻜﻲ ﻴﻜﻭﻥ‬ ‫‪= 1+‬‬
‫‪n −1‬‬ ‫‪n −1‬‬ ‫‪n −1‬‬ ‫‪n −1‬‬
‫ﺍﻝﻌﺩﺩ )‪ ( n − 1‬ﻗﺎﺴﻤﺎ ﻝﻠﻌﺩﺩ ‪. 3‬‬
‫ﻗﻭﺍﺴﻡ ﺍﻝﻌﺩﺩ ‪ 3‬ﻫﻲ ‪ 1 ، −3 ، −1‬ﻭ ‪ 3‬ﻭﺒﺎﻝﺘﺎﻝﻲ )‪ ( n − 1 = 1) ، ( n − 1 = −3) ، ( n − 1 = −1‬ﺃﻭ )‪( n − 1 = 3‬‬
‫ﻤﻌﻨﺎﻩ ) ‪ ( n = 2 ) ، ( n = −2 ) ، ( n = 0‬ﺃﻭ ) ‪ ( n = 4‬ﻭﺒﻤﺎ ﺃﻥ ‪ n ∈ ℕ‬ﻓﺈﻥ ﻗﻴﻤﻪ ﺍﻝﻤﻤﻜﻨﺔ ﻫﻲ ‪ 2 ، 0 :‬ﻭ ‪. 4‬‬
‫‪ (2‬ﻝﻴﻜﻥ ‪ α‬ﻭ ‪ β‬ﻋﺩﺩﻴﻥ ﻁﺒﻴﻌﻴﻴﻥ ﺤﻴﺙ ‪ a = 2α × 3β‬ﻭﻤﻨﻪ ‪ a 2 = 22α × 32 β‬ﻋﺩﺩ ﻗﻭﺍﺴﻡ ‪ a 2‬ﻫﻭ )‪( 2α + 1)( 2β + 1‬‬
‫ﻭﻋﺩﺩ ﻗﻭﺍﺴﻡ ‪ a‬ﻫﻭ )‪ (α + 1)( β + 1‬ﻭﻤﻥ ﺍﻝﻤﻌﻁﻴﺎﺕ ﻝﺩﻴﻨﺎ ‪ ( 2α + 1)( 2β + 1) = 3 (α + 1)( β + 1) :‬ﻤﻌﻨﺎﻩ‬
‫‪ 4αβ + 2α + 2 β + 1 = 3αβ + 3α + 3β + 3‬ﻭﻤﻌﻨﺎﻩ ‪ αβ − α = β + 2‬ﻴﻜﺎﻓﺊ ‪ α ( β − 1) = β + 2‬ﺃﻱ‬
‫‪α = 4‬‬ ‫‪α = 2‬‬ ‫‪β +2‬‬
‫‪ ‬؛ ﺇﺫﻥ ‪ a = 22 × 34 = 324‬ﺃﻭ ‪. a = 24 × 32 = 144‬‬ ‫‪ ‬ﺃﻭ‬ ‫= ‪ . α‬ﻭﺤﺴﺏ ﺍﻝﺴﺅﺍل ﺍﻝﺴﺎﺒﻕ ﻴﻨﺘﺞ ﺃﻥ‬
‫‪β = 2‬‬ ‫‪β = 4‬‬ ‫‪β −1‬‬
‫‪ xy − 4 y − 12 = 0 50‬ﺇﺫﺍ ﻜﺎﻥ ‪ x = 4‬ﻓﺈﻥ ﺍﻝﻤﻌﺎﺩﻝﺔ ﺘﺼﺒﺢ ‪ −12 = 0‬ﻭﻫﺫﺍ ﻏﻴﺭ ﻤﻤﻜﻥ ﺇﺫﻥ ‪. x ≠ 4‬‬
‫‪12‬‬
‫= ‪ y‬ﻭﻤﻨﻪ ‪ x − 4‬ﻴﻘﺴﻡ ‪ 12‬ﻭﻝﺩﻴﻨﺎ ‪. 12 = 22 × 3‬‬ ‫‪ xy − 4 y − 12 = 0‬ﻤﻌﻨﺎﻩ‬
‫‪x −4‬‬
‫‪1‬ـ ‪2‬ـ ‪3‬ـ ‪4‬ـ ‪6‬ـ ‪12‬ـ ‪x − 4‬‬
‫‪1 2 3 4 6 12‬‬

‫‪2‬ـ ‪8‬ـ ‪x‬‬


‫‪0 1 2 3 5 6 7 8 10 16‬‬

‫‪12 12 6 4 3 2 1‬ـ ‪6‬ـ ‪4‬ـ ‪3‬ـ ‪2‬ـ ‪1‬ـ ‪y‬‬


‫‪ (1 51‬ﻝﻴﻜﻥ ]‪x ∈ [ −3;1[ ∪ ]1;3‬‬
‫‪2x 2 − 3x − 3 2x 2 − 3x + 1 − 4‬‬
‫= ) ‪f (x‬‬ ‫=‬
‫‪x −1‬‬ ‫‪x −1‬‬
‫= ) ‪f (x‬‬
‫‪( x − 1)( 2x − 1) − 4 = 2x − 1 − 4‬‬
‫‪x −1‬‬ ‫‪x −1‬‬ ‫‪x −1‬‬
‫‪ M ∈C f‬ﻤﻌﻨﺎﻩ ]‪x ∈ [ −3;1[ ∪ ]1;3‬‬ ‫‪ (2‬ﻝﺘﻜﻥ ) ‪ M ( x , y‬ﻨﻘﻁﺔ ﻤﻥ ﺍﻝﻤﺴﺘﻭﻱ ﺇﺤﺩﺍﺜﻴﺘﻴﻬﺎ ﺃﻋﺩﺍﺩ ﺼﺤﻴﺤﺔ ‪.‬‬
‫‪4‬‬ ‫‪4‬‬
‫‪. y − 2x + 1 = −‬‬ ‫‪ y = 2x − 1 −‬ﺃﻱ‬ ‫ﻭ‬
‫‪x −1‬‬ ‫‪x −1‬‬
‫ﺇﺫﻥ ‪ x − 1‬ﻴﻘﺴﻡ ‪4‬‬

‫‪1‬ـ ‪2‬ـ ‪4‬ـ ‪x − 1‬‬


‫‪1‬‬ ‫‪2‬‬ ‫‪4‬‬

‫‪y − 2x + 1 1‬‬ ‫‪2‬‬ ‫‪1‬ـ ‪2‬ـ ‪4‬ـ ‪4‬‬

‫‪1‬ـ ‪3‬ـ ‪x‬‬


‫‪0‬‬ ‫‪2‬‬ ‫‪3‬‬ ‫‪5‬‬
‫‪y‬‬ ‫‪1‬ـ ‪6‬ـ‬ ‫‪3‬‬ ‫‪1‬ـ‬ ‫‪3‬‬ ‫‪8‬‬
‫‪ n 52‬ﻋﺩﺩ ﻁﺒﻴﻌﻲ ‪ .‬ﻨﻀﻊ ) ‪. a = n ( n 2 + 5‬‬

‫‪ (1‬ﺇﺫﺍ ﻜﺎﻥ ‪ n‬ﻋﺩﺩﺍ ﺯﻭﺠﻴﺎ ﻓﺈﻥ ‪ a‬ﻋﺩﺩ ﺯﻭﺠﻲ‪.‬‬


‫ﺇﺫﺍ ﻜﺎﻥ ‪ n‬ﻋﺩﺩﺍ ﻓﺭﺩﻴﺎ ﻓﺈﻥ ‪ n = 2k + 1‬ﻭﻤﻨﻪ ‪ n 2 + 5 = 4k 2 + 4k + 6‬ﻭﻫﻭ ﻋﺩﺩ ﺯﻭﺠﻲ ﺇﺫﻥ ‪ a‬ﻋﺩﺩ ﺯﻭﺠﻲ‪.‬‬
‫‪ (2‬ﺒﻨﻔﺱ ﺍﻝﻁﺭﻴﻘﺔ ﻨﻤﻴﺯ ﺍﻝﺤﺎﻻﺕ ‪. n = 3k + 2 ، n = 3k + 1 ، n = 3k‬‬
‫‪ a 53‬ﻋﺩﺩ ﻁﺒﻴﻌﻲ؛ ﻝﻠﺒﺭﻫﺎﻥ ﺃﻥ ﺍﻝﻌﺩﺩ )‪ a ( a 2 − 1‬ﻤﻀﺎﻋﻑ ﻝﻠﻌﺩﺩ ‪ 6‬ﻴﻜﻔﻲ ﺃﻥ ﻨﺒﺭﻫﻥ )‪ a ( a 2 − 1‬ﻤﻀﺎﻋﻑ ِـ ‪ 2‬ﻭ ‪3‬‬
‫ﻷﻥ ‪ 2‬ﻭ ‪ 3‬ﺃﻭﻝﻴﺎﻥ ﻓﻴﻤﺎ ﺒﻴﻨﻬﻤﺎ ﺜﻡ ﻨﻤﻴﺯ ﺍﻝﺤﺎﻻﺕ‪.‬‬
‫‪ 54‬ﺭﻗﻡ ﺁﺤﺎﺩ ﺍﻝﻌﺩﺩ ‪ n 5 − n‬ﻫﻭ ‪ 0‬ﻤﻌﻨﺎﻩ ‪ n 5 − n‬ﻴﻘﺒل ﺍﻝﻘﺴﻤﺔ ﻋﻠﻰ ‪ 10‬ﻭﻝﺩﻴﻨﺎ ﻤﻥ ﺒﻴﻥ ﺍﻝﻘﻭﺍﺴﻡ ﻝﻠﻌﺩﺩ ‪ 10‬ﻗﺎﺴﻤﻴﻥ ﺃﻭﻝﻴﻴﻥ‬
‫ﻓﻘﻁ ﻫﻤﺎ ‪ 2‬ﻭ ‪. 5‬‬
‫)‪ n 5 − n = n ( n 4 − 1) = n ( n 2 − 1)( n 2 + 1‬ﺃﻱ )‪. n 5 − n = n ( n − 1)( n + 1) ( n 2 + 1‬‬
‫ﻝﺩﻴﻨﺎ )‪ n ( n + 1‬ﻫﻭ ﺠﺩﺍﺀ ﻋﺩﺩﻴﻥ ﻁﺒﻴﻌﻴﻴﻥ ﻤﺘﻭﺍﻝﻴﻴﻥ ﺇﺫﻥ ﻫﻭ ﻋﺩﺩ ﺯﻭﺠﻲ ﺃﻱ ﻤﻀﺎﻋﻑ ِـ ‪. 2‬‬
‫‪ n 5 − n‬ﻤﻀﺎﻋﻑ ِـ )‪ , n ( n + 1‬ﻭ )‪ n ( n + 1‬ﻤﻀﺎﻋﻑ ِـ ‪ 2‬ﺇﺫﻥ ‪ n 5 − n‬ﻤﻀﺎﻋﻑ ِـ ‪. 2‬‬
‫ﻝﺩﻴﻨﺎ ﻜل ﻋﺩﺩ ﻁﺒﻴﻌﻲ ‪ n‬ﻫﻭ ﺇﻤﹼﺎ ﻤﻀﺎﻋﻔﺎ ِـ ‪ 5‬ﻭﺇﻤﺎ ﻝﻴﺱ ﻤﻀﺎﻋﻔﺎ ِـ ‪. 5‬‬
‫ﺇﺫﺍ ﻜﺎﻥ ‪ n‬ﻤﻀﺎﻋﻔﺎ ﻝﹻ ‪ , 5‬ﺒﻤﺎ ﺃﻥ ‪ n 5 − n‬ﻤﻀﺎﻋﻑ ﻝﹻ ‪ n‬ﻓﺈﻥ ‪ n 5 − n‬ﻤﻀﺎﻋﻑ ﻝﹻ ‪. 5‬‬
‫ﺇﺫﺍ ﻜﺎﻥ ‪ n‬ﻝﻴﺱ ﻤﻀﺎﻋﻔﺎ ﻝﹻ ‪ 5‬ﻓﺈﻥ ﺒﻭﺍﻗﻲ ﻗﺴﻤﺘﻪ ﻋﻠﻰ ‪ 5‬ﻫﻲ ‪. 4 , 3 , 2 ، 1‬‬
‫ﺇﺫﺍ ﻜﺎﻥ ﺒﺎﻗﻲ ﻗﺴﻤﺔ ‪ n‬ﻋﻠﻰ ‪ 5‬ﻫﻭ ‪ 1‬ﻓﺈﻥ ‪ n − 1‬ﻴﻜﻭﻥ ﻤﻀﺎﻋﻑ ﻝـ ‪ 5‬ﻭﺒﻤﺎ ﺃﻥ ‪ n 5 − n‬ﻤﻀﺎﻋﻑ ﻝﹻ ‪ n − 1‬ﻓﺈﻨﻪ ﻴﻜﻭﻥ‬
‫ﻤﻀﺎﻋﻑ ﻝـ ‪. 5‬‬
‫ﺇﺫﺍ ﻜﺎﻥ ﺒﺎﻗﻲ ﻗﺴﻤﺔ ‪ n‬ﻋﻠﻰ ‪ 5‬ﻫﻭ ‪ 4‬ﻓﺈﻥ ‪ n + 1‬ﻴﻜﻭﻥ ﻤﻀﺎﻋﻑ ﻝـ ‪ 5‬ﻭﺒﻤﺎ ﺃﻥ ‪ n 5 − n‬ﻤﻀﺎﻋﻑ ﻝﹻ ‪ n + 1‬ﻓﺈﻨﻪ ﻴﻜﻭﻥ‬
‫ﻤﻀﺎﻋﻑ ﻝـ ‪. 5‬‬
‫ﺇﺫﺍ ﻜﺎﻥ ﺒﺎﻗﻲ ﻗﺴﻤﺔ ‪ n‬ﻋﻠﻰ ‪ 5‬ﻫﻭ ‪ r‬ﺤﻴﺙ }‪ r ∈ {2;3‬ﻓﺈﻥ ‪ n = 5k + r‬ﻭﻤﻨﻪ ‪n 2 = 25k 2 + 10k + r 2‬‬
‫ﺃﻱ ‪n 2 + 1 = 5 ( 5k 2 + 2k ) + r 2 + 1‬‬
‫ﻭﺒﺎﻝﺘﺎﻝﻲ ﺇﺫﺍ ﻜﺎﻥ }‪ r ∈ {2;3‬ﻓﺈﻥ ‪ n 2 + 1 = 5 ( 5k 2 + 2k ) + 5‬ﺃﻭ ‪ n 2 + 1 = 5 ( 5k 2 + 2k ) + 10‬ﺇﺫﻥ ﻓﻲ ﺍﻝﺤﺎﻝﺘﻴﻥ‬
‫‪ n 2 + 1‬ﻤﻀﺎﻋﻑ ﻝﹻ ‪ 5‬ﻭﺒﻤﺎ ﺃﻥ ‪ n 5 − n‬ﻤﻀﺎﻋﻑ ﻝـ ‪ n 2 + 1‬ﻓﺈﻥ ‪ n 5 − n‬ﻤﻀﺎﻋﻑ ﻝـ ‪. 5‬‬
‫ﺇﺫﻥ ﻤﻥ ﺃﺠل ﻜل ﻋﺩﺩ ﻁﺒﻴﻌﻲ ‪ n 5 − n ، n‬ﻤﻀﺎﻋﻑ ـ ‪ . 5‬ﻭﺒﺎﻝﺘﺎﻝﻲ ﺘﺤﻠﻴل ﺍﻝﻌﺩﺩ ‪ n 5 − n‬ﻴﺸﻤل ﺍﻝﻌﺩﺩﻴﻥ ﺍﻷﻭﻝﻴﻴﻥ ‪ 2‬ﻭ ‪5‬‬
‫ﺇﺫﻥ ‪ n 5 − n‬ﻫﻭ ﻤﻀﺎﻋﻑ ﻝﻠﻌﺩﺩ ‪. 10‬‬
‫‪ n p +1‬ﻭ ‪ n p +5‬ﻝﻬﻤﺎ ﻨﻔﺱ ﺭﻗﻡ ﺍﻵﺤﺎﺩ ﻤﻌﻨﺎﻩ ﺃﻥ ﺭﻗﻡ ﺁﺤﺎﺩ ‪ n p +5 − n p +1‬ﻫﻭ ‪ n p +5 − n p +1 = n p ( n 5 − n ) . 0‬ﻭﻤﻨﻪ‬
‫‪ n p +5 − n p +1‬ﻤﻀﺎﻋﻑ ﻝﻠﻌﺩﺩ ‪. 10‬‬
‫‪ 55‬ﻝﻠﺒﺭﻫﺎﻥ ﺃﻥ ﻤﻥ ﺃﺠل ﻜل ﻋﺩﺩ ﻁﺒﻴﻌﻲ ‪ n‬ﻴﻜﻭﻥ ‪ n 7 − n‬ﻴﻘﺒل ﺍﻝﻘﺴﻤﺔ ﻋﻠﻰ ‪ 14‬ﻴﻜﻔﻲ ﺃﻥ ﻨﺒﺭﻫﻥ ﺃﻨﻪ ﻴﻘﺒل ﺍﻝﻘﺴﻤﺔ ﻋﻠﻰ ‪2‬‬
‫ﻭﻋﻠﻰ ‪ 7‬ﻷﻥ ‪ 2‬ﻭ ‪ 7‬ﺃﻭﻝﻴﺎﻥ ﻓﻴﻤﺎ ﺒﻴﻨﻬﻤﺎ‪.‬‬
‫‪ (1 56‬ﻤﻥ ﺃﺠل ﻜل ﻋﺩﺩ ﻁﺒﻴﻌﻲ ‪ a = n 2 + 5n + 4 = ( n + 1)( n + 4 ) ، n‬ﻭ ) ‪b = n 2 + 3n + 2 = ( n + 1)( n + 2‬‬

‫ﺇﺫﻥ ﺍﻝﻌﺩﺩ ‪ n + 1‬ﻫﻭ ﻗﺎﺴﻡ ﻤﺸﺘﺭﻙ ﻝﻠﻌﺩﺩﻴﻥ ‪ a‬ﻭ ‪. b‬‬


‫‪ (2‬ﻝﺩﻴﻨﺎ ‪3n 2 + 15n + 20 = ( n + 1)( 3n + 12 ) + 8‬‬

‫ﺇﺫﻥ ﺍﻝﻌﺩﺩ ‪ n + 1‬ﻗﺎﺴﻤﺎ ﻝﻠﻌﺩﺩ ‪ 3n 2 + 15n + 20‬ﻤﻌﻨﺎﻩ ﺍﻝﻌﺩﺩ ‪ n + 1‬ﻗﺎﺴﻤﺎ ﻝﻠﻌﺩﺩ ‪ 8‬ﻭﻤﻨﻪ }‪ n + 1∈ {1; 2; 4;8‬ﺃﻱ‬
‫}‪. n ∈ {0;1;3; 7‬‬

‫ﻭﻋﻜﺴﻴﺎ ﺒﺘﻌﻭﻴﺽ ‪ n‬ﺒﻘﻴﻡ ﺍﻝﻤﺠﻤﻭﻋﺔ }‪ {0;1;3; 7‬ﻨﺠﺩ ﺍﻝﻌﺩﺩ ‪ n + 1‬ﻗﺎﺴﻤﺎ ﻝﻠﻌﺩﺩ ‪3n 2 + 15n + 20‬‬

‫‪ n 57‬ﻭ ‪ a‬ﻋﺩﺩﺍﻥ ﺼﺤﻴﺤﺎﻥ ﺤﻴﺙ ‪ a‬ﻴﻘﺴﻡ ‪ n − 1‬ﻭ ‪. n 2 + n + 3‬‬

‫أ ـ ﻝﺩﻴﻨﺎ )‪ n 2 − 2n + 1 = ( n − 1‬ﻭ ‪ a‬ﻴﻘﺴﻡ ‪ n − 1‬ﺇﺫﻥ ‪ a‬ﻴﻘﺴﻡ )‪ ( n − 1‬ﺃﻱ ‪ a‬ﻴﻘﺴﻡ ‪. n 2 − 2n + 1‬‬


‫‪2‬‬ ‫‪2‬‬

‫ب ـ ‪ a‬ﻴﻘﺴﻡ ‪ n 2 + n + 3‬ﻭ ‪ n 2 − 2n + 1‬ﺇﺫﻥ ‪ a‬ﻴﻘﺴﻡ ﺍﻝﻔﺭﻕ )‪ ( n 2 + n + 3) − ( n 2 − 2n + 1‬ﺃﻱ ﻴﻘﺴﻡ ‪. 3n + 2‬‬

‫‪4‬ـ ـ ‪ a‬ﻴﻘﺴﻡ ‪ n − 1‬ﻭﻤﻨﻪ ‪ a‬ﻴﻘﺴﻡ ‪ 3n − 3‬ﻭﺒﻤﺎ ﺃﻥ ‪ a‬ﻴﻘﺴﻡ ‪ 3n + 2‬ﻓﺈﻨﻪ ﻴﻘﺴﻡ ﺍﻝﻔﺭﻕ )‪ ( 3n + 2 ) − ( 3n − 3‬ﺃﻱ ‪a‬‬
‫ﻴﻘﺴﻡ ‪. 5‬‬
‫د ـ }‪. a ∈ {−5; −1;1;5‬‬

‫‪ 58‬ﻨﻔﺘﺭﺽ ﺃﻥ ﺍﻝﺜﻨﺎﺌﻴﺔ ) ‪ ( x ; y‬ﻴﻜﻭﻥ ﻤﻥ ﺃﺠﻠﻬﺎ ﺍﻝﻌﺩﺩ ‪ xy‬ﻗﺎﺴﻤﺎ ﻝﻠﻌﺩﺩ ‪ x + y‬ﺇﺫﻥ ‪ x ≠ 0‬ﻭ ‪y ≠ 0‬‬
‫ﻭﻝﺩﻴﻨﺎ ‪ x + y = xyk‬ﻤﻊ ‪ k ∈ ℕ‬ﺇﺫﻥ )‪ x = y ( xk − 1‬ﻭ )‪ y = x ( yk − 1‬ﻭﺒﺎﻝﺘﺎﻝﻲ ‪ x‬ﻴﻘﺴﻡ ‪ y‬ﻭ ‪ y‬ﻴﻘﺴﻡ ‪ x‬ﺇﺫﻥ‬
‫‪ x = y‬ﻭﺒﺎﻝﺘﺎﻝﻲ ﻴﺼﺒﺢ ‪ 2x = x 2 k‬ﺃﻱ ‪ 2 = xk‬ﻭﻤﻨﻪ ‪ x‬ﻴﻘﺴﻡ ‪ 2‬ﺇﺫﻥ ‪ x = y = 1‬ﺃﻭ ‪. x = y = 2‬‬
‫ﻭﺒﺎﻝﻌﻜﺱ ﺍﻝﺜﻨﺎﺌﻴﺘﻴﻥ )‪ (1,1‬ﻭ ) ‪ ( 2, 2‬ﺘﺤﻘﻘﺎﻥ ﺍﻝﻤﻁﻠﻭﺏ‪.‬‬

‫‪ n 59‬ﻋﺩﺩ ﻁﺒﻴﻌﻲ ﻓﺭﺩﻱ ‪ S .‬ﻤﺠﻤﻭﻉ ﺃﻋﺩﺍﺩ ﻁﺒﻴﻌﻴﺔ ﻤﺘﺘﺎﺒﻌﺔ ﻭﻋﺩﺩﻫﺎ ‪ . n‬ﻨﻌﺘﺒﺭ ﺍﻝﻌﺩﺩ ﺍﻝﻁﺒﻴﻌﻲ ‪ a‬ﻭﻨﻀﻊ‬
‫)‪ S = a + ( a + 1) + ( a + 2 ) + ... + ( a + n − 1‬ﻫﻭ ﻤﺠﻤﻭﻉ ﺤﺩﻭﺩ ﻤﺘﺘﺎﺒﻌﺔ ﻤﻥ ﻤﺘﺘﺎﻝﻴﺔ ﺤﺴﺎﺒﻴﺔ ﺃﺴﺎﺴﻬﺎ ‪. 1‬‬
‫‪‬‬ ‫‪n −1 ‬‬
‫= ‪S‬‬
‫‪n‬‬
‫(‬ ‫‪a + ( a + n − 1) ) = n  a +‬‬ ‫‪‬‬
‫‪2‬‬ ‫‪‬‬ ‫‪2 ‬‬
‫‪n −1‬‬ ‫‪n −1‬‬
‫‪ k = a +‬ﻫﻭ ﻋﺩﺩ‬ ‫ﻴﻜﻭﻥ ﻋﺩﺩ ﻁﺒﻴﻌﻲ ﻭﻤﻨﻪ‬ ‫ﺒﻤﺎ ﺃﻥ ‪ n‬ﻋﺩﺩ ﻁﺒﻴﻌﻲ ﻓﺭﺩﻱ ﻓﺈﻥ ‪ n − 1‬ﻫﻭ ﺯﻭﺠﻲ ﻭﺒﺎﻝﺘﺎﻝﻲ‬
‫‪2‬‬ ‫‪2‬‬
‫ﻁﺒﻴﻌﻲ ﻭﻤﻨﻪ ‪ S = nk‬ﻤﻊ ‪ k ∈ ℕ‬ﺇﺫﻥ ﺍﻝﻌﺩﺩ ‪ S‬ﻴﻘﺒل ﺍﻝﻘﺴﻤﺔ ﻋﻠﻰ ‪. n‬‬

‫‪ 2‬ـ ا
‪ %2I.‬ا‪%- :P‬‬
‫‪ 71 = 0 × 72 + 71 66‬ﺇﺫﻥ ﺒﺎﻗﻲ ﺍﻝﻘﺴﻤﺔ ﺍﻷﻗﻠﻴﺩﻴﺔ ﻝﻠﻌﺩﺩ ‪ 71‬ﻋﻠﻰ ‪ 72‬ﻫﻭ ‪. 71‬‬
‫‪ 67‬ﻜﺘﺎﺏ ﻤﻜﺘﻭﺏ ﻋﻠﻴﻪ ‪ 4350‬ﺴﻁﺭﺍ ‪ .‬ﻜل ﺼﻔﺤﺔ ﺘﺤﻤل ‪ 34‬ﺴﻁﺭﺍ ﻤﺎﻋﺩﺍ ﺍﻝﺼﻔﺤﺔ ﺍﻷﺨﻴﺭﺓ ﻨﺎﻗﺼﺔ ‪.‬‬
‫‪ 4350 = 34 × 127 + 32‬ﺇﺫﻥ ﺘﻭﺠﺩ ﺒﺎﻝﻜﺘﺎﺏ ‪ 127‬ﺼﻔﺤﺔ ﻜﺎﻤﻠﺔ ﻭﺍﻝﺼﻔﺤﺔ ﺍﻷﺨﻴﺭﺓ ﻤﻜﺘﻭﺏ ﻋﻠﻴﻬﺎ ‪ 32‬ﺴﻁﺭﺍ ﻓﻘﻁ ‪.‬‬
‫‪ 68‬ﻋﻠﻤﺎ ﺃﻨﻪ ﻴﻭﺠﺩ ﻋﺩﺩ ﻁﺒﻴﻌﻲ ‪ k‬ﺤﻴﺙ ‪ . 100100 = 13k + 35‬ﻭﻝﺩﻴﻨﺎ ‪100100 = 13k + 26 + 9‬‬
‫ﺃﻱ ‪ 100100 = 13 ( k + 2 ) + 9‬ﺒﻤﺎ ﺃﻥ ‪ 9 < 13‬ﻓﺈﻥ ﺒﺎﻗﻲ ﻗﺴﻤﺔ ‪ 100100‬ﻋﻠﻰ ‪ 13‬ﻫﻭ ‪. 9‬‬
‫‪ 69‬ﺍﻝﺒﺎﻗﻴﺎﻥ ﻝﻠﻘﺴﻤﺔ ﺍﻷﻗﻠﻴﺩﻴﺔ ﻝﻜل ﻤﻥ ﺍﻝﻌﺩﺩﻴﻥ ‪ m‬ﻭ ‪ n‬ﻋﻠﻰ ‪ 17‬ﻫﻤﺎ ﻋﻠﻰ ﺍﻝﺘﻭﺍﻝﻲ ‪ 8‬ﻭ ‪ . 12‬ﺃﻱ ‪m = 17 k + 8‬‬
‫ﻭ ‪ n = 17 p + 12‬ﻤﻊ ‪ k ∈ ℕ‬ﻭ ‪. p ∈ ℕ‬‬
‫‪m + n = 17 ( k + p ) + 20 = 17 ( k + p + 1) + 3‬‬
‫ﺇﺫﻥ ﺒﺎﻗﻲ ﻗﺴﻤﺔ ‪ m + n‬ﻋﻠﻰ ‪ 17‬ﻫﻭ ‪. 3‬‬
‫) ‪m × n = (17k + 8 )(17 p + 2‬‬
‫‪m × n = 17 2 kp + 17 ( 2k + 8 p ) + 16‬‬
‫‪m × n = 17 (17 kp + 2k + 8 p ) + 16‬‬
‫ﺇﺫﻥ ﺒﺎﻗﻲ ﻗﺴﻤﺔ ‪ m × n‬ﻋﻠﻰ ‪ 17‬ﻫﻭ ‪. 16‬‬
‫‪m 2 = (17 k ) + 16 × 17 k + 64‬‬
‫‪2‬‬

‫‪. m 2 = 17 (17 k 2 + 16k + 3 ) + 13‬‬


‫ﺇﺫﻥ ﺒﺎﻗﻲ ﻗﺴﻤﺔ ‪ m 2‬ﻋﻠﻰ ‪ 17‬ﻫﻭ ‪. 13‬‬
‫‪ 23×0 − 1 = 0 79‬ﻭﻫﻭ ﻴﻘﺒل ﺍﻝﻘﺴﻤﺔ ﻋﻠﻰ ‪. 7‬‬
‫ﻨﻔﺭﺽ ‪ 23 p − 1‬ﻴﻘﺒل ﺍﻝﻘﺴﻤﺔ ﻋﻠﻰ ‪ 7‬ﺃﻱ ‪ 23 p − 1 = 7 k‬ﻤﻊ ‪ k ∈ ℕ‬ﻭﻝﻨﺒﺭﻫﻥ ‪ 23( p +1) − 1‬ﻴﻘﺒل ﺍﻝﻘﺴﻤﺔ ﻋﻠﻰ ‪. 7‬‬
‫(‪2‬‬ ‫‪− 1 = 8 × 23 p − 1 = 8 ( 7 k + 1) − 1 = 56k + 7‬‬
‫)‪3 p +1‬‬

‫ﺃﻱ )‪ 23( p +1) − 1 = 7 ( 8k + 1‬ﻭﻤﻨﻪ ‪ 23( p +1) − 1‬ﻴﻘﺒل ﺍﻝﻘﺴﻤﺔ ﻋﻠﻰ ‪ . 7‬ﺇﺫﻥ ﺤﺴﺏ ﻤﺒﺩﺃ ﺍﻝﺘﺭﺍﺠﻊ ﻴﻨﺘﺞ ﺃﻨﻪ ﻤﻥ ﺃﺠل ﻜل ﻋﺩﺩ‬
‫ﻁﺒﻴﻌﻲ ‪ ، n‬ﺍﻝﻌﺩﺩ ‪ 23n − 1‬ﻴﻘﺒل ﺍﻝﻘﺴﻤﺔ ﻋﻠﻰ ‪. 7‬‬
‫أ ـ ﻤﻥ ﺃﺠل ﻜل ‪ 23n − 1 = 7 k ، n ∈ ℕ‬ﻤﻊ ‪k ∈ ℕ‬‬
‫ﺃﻱ ‪ 23n = 7 k + 1‬ﺇﺫﻥ ﺍﻝﺒﺎﻗﻲ ﻫﻭ ‪. 1‬‬
‫ب ـ ‪ a = 23n +1 = 2 ( 7 k + 1) = 7 ( 2k ) + 2‬ﺇﺫﻥ ﺍﻝﺒﺎﻗﻲ ‪. 2‬‬
‫ـ ـ ‪ a = 23n + 2 = 4 ( 7k + 1) = 7 ( 4k ) + 4‬ﺍﻝﺒﺎﻗﻲ ﻫﻭ ‪. 3‬‬

‫‪ 80‬ﺇﺫﺍ ﻜﺎﻥ ‪ d‬ﻗﺎﺴﻤﺎ ﻤﺸﺘﺭﻜﺎ ﻝﻠﻌﺩﺩﻴﻥ ‪ a‬ﻭ ‪ b‬ﻓﻬﻭ ﻗﺎﺴﻡ ‪ a 2‬ﻭﺒﺎﻝﺘﺎﻝﻲ ﻫﻭ ﻗﺎﺴﻡ ﻝﻠﻌﺩﺩ ‪ a 2 + b‬ﻭﻤﻨﻪ ‪ d‬ﻴﻜﻭﻥ ﻗﺎﺴﻤﺎ ﻤﺸﺘﺭﻜﺎ‬
‫‪ a‬ﻭ ) ‪. (a 2 + b‬‬
‫ﺇﺫﺍ ﻜﺎﻥ ‪ d‬ﻗﺎﺴﻤﺎ ﻤﺸﺘﺭﻜﺎ ﻝﻠﻌﺩﺩﻴﻥ ‪ a‬ﻭ ) ‪ ( a 2 + b‬ﻓﻬﻭ ﻗﺎﺴﻡ ‪ a 2‬ﻭﺒﺎﻝﺘﺎﻝﻲ ﻫﻭ ﻗﺎﺴﻡ ﻝﻠﻌﺩﺩ ‪ ( a 2 + b ) − a 2‬ﺃﻱ ﻗﺎﺴﻡ ﻝﻠﻌﺩﺩ ‪b‬‬
‫ﻭﻤﻨﻪ ‪ d‬ﻴﻜﻭﻥ ﻗﺎﺴﻤﺎ ﻤﺸﺘﺭﻜﺎ ﻝﻠﻌﺩﺩﻴﻥ ‪ a‬ﻭ ‪. b‬‬
‫ﻨﺴﺘﻨﺘﺞ ﻤﻥ ﻫﺫﺍ ﺃﻥ ﺍﻝﻘﻭﺍﺴﻡ ﺍﻝﻤﺸﺘﺭﻜﺔ ﻝﻠﻌﺩﺩﻴﻥ ‪ a‬ﻭ ) ‪ ( a 2 + b‬ﻫﻲ ﻨﻔﺱ ﺍﻝﻘﻭﺍﺴﻡ ﺍﻝﻤﺸﺘﺭﻜﺔ ﻝﻠﻌﺩﺩﻴﻥ ‪ a‬ﻭ ‪. b‬‬
‫ﻭﺒﺎﻷﺨﺹ ) ‪. PGCD ( a ; a 2 + b ) = PGCD ( a ; b‬‬
‫‪ (2‬ﻜل ﻗﺎﺴﻡ ﻤﺸﺘﺭﻙ ﻝﻠﻌﺩﺩﻴﻥ ‪ a‬ﻭ ‪ b‬ﻫﻭ ﻗﺎﺴﻡ ﻝﻜل ﻤﻥ ﺍﻷﻋﺩﺍﺩ ‪ 3b ، 2a ، a + b :‬ﻭ ‪2a + 3b‬‬
‫ﺇﺫﻥ ﻜل ﻗﺎﺴﻡ ﻤﺸﺘﺭﻙ ﻝﻠﻌﺩﺩﻴﻥ ‪ a‬ﻭ ‪ b‬ﻫﻭ ﻗﺎﺴﻡ ﻤﺸﺘﺭﻙ ﻝﻠﻌﺩﺩﻴﻥ ‪ a + b‬ﻭ ‪. 2a + 3b‬‬
‫ﻭﺒﺎﻝﻌﻜﺱ ﻝﺩﻴﻨﺎ ﻜل ﻗﺎﺴﻡ ﻤﺸﺘﺭﻙ ﻝﻠﻌﺩﺩﻴﻥ ‪ a + b‬ﻭ ‪ 2a + 3b‬ﻫﻭ ﻗﺎﺴﻡ ﻝﻜل ﻤﻥ ﺍﻷﻋﺩﺍﺩ ) ‪، 2 ( a + b ) ، 3 ( a + b‬‬
‫) ‪ 3 ( a + b ) − ( 2a + 3b‬ﻭ ) ‪ ( 2a + 3b ) − 2 ( a + b‬ﻭﻝﺩﻴﻨﺎ ‪ 3 (a + b ) − ( 2a + 3b ) = a‬ﻭ ‪( 2a + 3b ) − 2 ( a + b ) = b‬‬
‫ﺇﺫﻥ ﻜل ﻗﺎﺴﻡ ﻤﺸﺘﺭﻙ ﻝﻠﻌﺩﺩﻴﻥ ‪ a + b‬ﻭ ‪ 2a + 3b‬ﻫﻭ ﻗﺎﺴﻡ ﻤﺸﺘﺭﻙ ﻝﻠﻌﺩﺩﻴﻥ ‪ a‬ﻭ ‪b‬‬
‫ﻨﺴﺘﻨﺘﺞ ﻤﻥ ﻫﺫﺍ ﺃﻥ ﺍﻝﻘﻭﺍﺴﻡ ﺍﻝﻤﺸﺘﺭﻜﺔ ﻝﻠﻌﺩﺩﻴﻥ ‪ a + b‬ﻭ ‪ 2a + 3b‬ﻫﻲ ﻨﻔﺱ ﺍﻝﻘﻭﺍﺴﻡ ﺍﻝﻤﺸﺘﺭﻜﺔ ﻝﻠﻌﺩﺩﻴﻥ ‪ a‬ﻭ ‪. b‬‬
‫ﻭﺒﺎﻷﺨﺹ ) ‪. PGCD ( a + b ; 2a + 3b ) = PGCD ( a ; b‬‬

‫‪ n 81‬ﻋﺩﺩ ﻁﺒﻴﻌﻲ ‪ a = 11n + 3 .‬ﻭ ‪. b = 13n − 1‬‬


‫‪13a − 11b = 13 (11n + 3) − 11(13n − 1) (1‬‬
‫‪. 13a − 11b = 143n + 39 − 143n + 11 = 50‬‬
‫‪ PGCD ( a ; b ) (2‬ﻴﻘﺴﻡ ‪ 13a‬ﻭ ‪ 11b‬ﻭﺍﻝﻔﺭﻕ ‪ 13a − 11b‬ﺃﻱ ) ‪ PGCD ( a ; b‬ﻴﻘﺴﻡ ‪. 50‬ﻝﺩﻴﻨﺎ ‪ , 50 = 2 × 52‬ﺇﺫﻥ‬
‫) ‪ PGCD ( a ; b‬ﻴﻨﺘﻤﻲ ﺇﻝﻰ ﺍﻝﻤﺠﻤﻭﻋﺔ }‪. {1, 2,5,10, 25,50‬‬
‫‪ (3‬ﺘﻌﻴﻴﻥ ﺜﻨﺎﺌﻴﺔ ) ‪ ( a ; b‬ﺒﺤﻴﺙ ﻴﻜﻭﻥ ‪. PGCD ( a ; b ) = 50‬‬
‫‪ 50‬ﻴﻘﺴﻡ ‪ a‬ﻭ ‪ b‬ﻭﻤﻨﻪ ﻴﻘﺴﻡ ‪ 6a‬ﻭ ‪ 5b‬ﻭﻜﺫﻝﻙ ‪6a − 5b‬‬
‫ﺃﻱ ‪ 50‬ﻴﻘﺴﻡ ‪ n + 23‬ﻭﻤﻌﻨﺎﻩ ‪ n + 23 = 50k‬ﻤﻊ ∗‪k ∈ ℕ‬‬
‫) ‪( a ; b ) = ( 300;350‬‬ ‫ﻭﻤﻌﻨﺎﻩ ‪ n = 50k − 23‬ﻭﺒﺄﺨﺫ ‪ k = 1‬ﻨﺠﺩ ‪ n = 27‬ﻭﻤﻨﻪ‬
‫ﻭﺒﺎﻝﻌﻜﺱ ‪ a = 300 = 6 × 50‬ﻭ ‪ b = 350 = 7 × 50‬ﻭﻝﺩﻴﻨﺎ ‪ 6‬ﻭ ‪ 7‬ﺃﻭﻝﻴﺎﻥ ﻓﻴﻤﺎ ﺒﻴﻨﻬﻤﺎ ﺇﺫﻥ ‪PGCD (a ; b ) = 50‬‬

‫‪ 2a 2 + b 2 = 20992‬‬
‫‪ ‬ﻤﻌﻨﺎﻩ ﺘﻭﺠﺩ )' ‪ ( a '; b‬ﻤﻥ ‪ ℕ‬ﺤﻴﺙ ' ‪ b = 16b ' , a = 16a‬؛ ' ‪ a‬ﻭ ' ‪ b‬ﺃﻭﻝﻴﺎﻥ ﻓﻴﻤﺎ ﺒﻴﻨﻬﻤﺎ‬
‫‪∗2‬‬
‫‪82‬‬
‫‪PGCD ( a ; b ) = 16‬‬
‫ﻤﻥ ﺍﻝﻔﺭﻀﻴﺔ ﺍﻷﻭﻝﻰ ﻨﺤﺼل ﻋﻠﻰ ‪ 2a '2 + b '2 = 82‬ﻭﻤﻌﻨﺎﻩ ) ‪ b '2 = 2 ( 41 − a '2‬ﺇﺫﻥ ﻴﺠﺏ ‪a '2 < 41‬‬
‫‪1‬‬ ‫‪4‬‬ ‫‪9‬‬ ‫‪16 25 36‬‬
‫‪a '2‬‬
‫‪b '2 80 74 64 50 32 10‬‬
‫ﺇﺫﻥ ﺍﻝﺜﻨﺎﺌﻴﺔ ﺍﻝﻭﺤﻴﺩﺓ )' ‪ ( a '; b‬ﻫﻲ ) ‪ ( 3,8‬ﻭﻤﻨﻪ )‪( a ;b ) = ( 48,128‬‬
‫‪ a 83‬ﻭ ‪ b‬ﻋﺩﺩﺍﻥ ﻤﻥ ∗‪ ℕ‬ﻭ ‪. PGCD ( a ; b ) = d‬‬
‫ﺘﻭﺠﺩ )' ‪ ( a '; b‬ﻤﻥ ‪ ℕ∗2‬ﺤﻴﺙ ' ‪ b = db ' , a = da‬؛ ' ‪ a‬ﻭ ' ‪ b‬ﺃﻭﻝﻴﺎﻥ ﻓﻴﻤﺎ ﺒﻴﻨﻬﻤﺎ ؛ ‪ ab + 5d 2 = 35d‬ﺘﺼﺒﺢ‬
‫‪35‬‬
‫= ' ‪ a 'b‬؛ ﻗﻭﺍﺴﻡ ‪ 35‬ﻫﻲ‪1,5, 7,35 :‬‬ ‫‪ d 2 ( a ' b '+ 5 ) = 35d‬ﻤﻌﻨﺎﻩ ‪ d ( a 'b '+ 5 ) = 35‬ﻭﻤﻨﻪ ‪ d‬ﻴﻘﺴﻡ ‪ 35‬ﻭ ‪− 5‬‬
‫‪d‬‬
‫ﺇﺫﺍ ﻜﺎﻥ ‪ d = 35‬ﺃﻭ ‪ d = 7‬ﻓﺈﻥ ‪ a 'b ' ≤ 0‬ﻭﻫﺫﺍ ﻤﺭﻓﻭﺽ‬
‫ﺇﺫﺍ ﻜﺎﻥ ‪ d = 1‬ﻓﺈﻥ ‪ a 'b ' = 30‬ﻭﻝﺩﻴﻨﺎ ‪30 = 2 × 3 × 5‬‬
‫; ) ‪( a '; b ') ∈ {(1,30 ) ; ( 2,15) ; ( 3,10 ) ; ( 5, 6‬‬ ‫ﻭﻤﺠﻤﻭﻋﺔ ﻗﻭﺍﺴﻡ ‪ 30‬ﻫﻲ ‪ {1, 2,3,5, 6,10,15,30} :‬ﻭﻤﻨﻪ ‪:‬‬
‫})‪( 6,5) ; (10,3) ; (15, 2 ) ; ( 30,1‬‬
‫ﺇﺫﺍ ﻜﺎﻥ ‪ d = 5‬ﻓﺈﻥ ‪ a 'b ' = 2‬؛ })‪ ( a ';b ') ∈ {(1, 2 ) ; ( 2,1‬ﻭﻤﻨﻪ })‪. ( a ;b ) ∈ {( 5,10 ) ; (10,5‬‬
‫ﺨﻼﺼﺔ ‪. ( a ; b ) ∈ {(1,30 ) ; ( 2,15) ; ( 3,10 ) ; ( 5, 6 ) ; ( 6,5 ) ; (10,3) ; (15, 2 ) ; ( 30,1) ; ( 5,10 ) ; (10;5 )} :‬‬
‫‪ (1 84‬ﻝﻴﻜﻥ ‪ d‬ﻗﺎﺴﻤﺎ ﻤﺸﺘﺭﻜﺎ ﻝِـ ‪ b ، a‬ﺇﺫﻥ ﻫﻭ ﻗﺎﺴﻡ ﻝﻜل ﻤﻥ ‪ 4a , 5b , 7a‬ﻭ ‪ 3b‬ﻭﺒﺎﻝﺘﺎﻝﻲ ‪ d‬ﻴﻘﺴﻡ ‪, 7a − 5b‬‬
‫‪ 4a − 3b , 5b − 7a‬ﻭ ‪ 3b − 4a‬ﺃﻱ ‪ d‬ﻗﺎﺴﻡ ﻤﺸﺘﺭﻙ ِـ ‪ x‬ﻭ ‪. y‬‬
‫ﺍﻝﻌﻜﺱ ﻝﻴﻜﻥ ‪ d‬ﻗﺎﺴﻤﺎ ﻤﺸﺘﺭﻜﺎ ِـ ‪ x‬ﻭ ‪ y‬ﺇﺫﻥ ﻫﻭ ﻗﺎﺴﻡ ﻝﻜل ﻤﻥ ‪ 3x , 7 y , 4x‬ﻭ ‪ 5 y‬ﻭﺒﺎﻝﺘﺎﻝﻲ ‪ d‬ﻗﺎﺴﻡ ﻝﻠﻔﺭﻗﻴﻥ‬
‫‪ 4x − 7 y‬ﻭ ‪3x − 5 y‬‬
‫ﻝﺩﻴﻨﺎ ‪ 4x − 7 y = 4 ( 7a − 5b ) − 7 ( 4a − 3b ) = b‬ﻭ ‪3x − 5 y = 3 ( 7a − 5b ) − 5 ( 4a − 3b ) = a‬‬
‫ﺇﺫﻥ ‪ d‬ﻗﺎﺴﻡ ﻤﺸﺘﺭﻙ ِـ ‪. b ، a‬‬
‫ﻭﻤﻨﻪ‪ :‬ﻤﺠﻤﻭﻋﺔ ﺍﻝﻘﻭﺍﺴﻡ ﺍﻝﻤﺸﺘﺭﻜﺔ ﻝﻠﻌﺩﺩﻴﻥ ‪ b ، a‬ﻫﻲ ﻨﻔﺴﻬﺎ ﻤﺠﻤﻭﻋﺔ ﺍﻝﻘﻭﺍﺴﻡ ﺍﻝﻤﺸﺘﺭﻜﺔ ﻝﻠﻌﺩﺩﻴﻥ ‪ x‬ﻭ ‪ y‬؛ ﻭﺒﺎﻷﺨﺹ‬
‫) ‪. PGCD ( x ; y ) = PGCD ( x ; y ) = PGCD ( a ; b‬‬
‫‪( 7α − 5β )( 4α − 3β ) = 1300‬‬
‫‪(1)... ‬‬ ‫‪(2‬‬
‫‪‬‬ ‫‪PGCD (α ; β ) = 5‬‬
‫ﻨﻀﻊ ‪ x = 7α − 5β :‬ﻭ ‪ . y = 4α − 3β‬ﻭﺤﺴﺏ ﺍﻝﺴﺅﺍل ‪ (1‬ﻴﻜﻭﻥ ‪ α = 3x − 5 y‬ﻭ ‪β = 4x − 7 y‬‬
‫‪‬‬ ‫‪xy = 1300‬‬
‫‪‬‬ ‫ﻭﻤﻨﻪ ‪ . PGCD ( x ; y ) = PGCD (α ; β ) = 5‬ﺇﺫﻥ )‪ (1‬ﺘﺼﺒﺢ‬
‫‪PGCD ( x ; y ) = 5‬‬
‫‪ PGCD ( x ; y ) = 5‬ﻤﻌﻨﺎﻩ ﻴﻭﺠﺩ ' ‪ x‬ﻭ ' ‪ y‬ﻋﺩﺩﺍﻥ ﺼﺤﻴﺤﺎﻥ ﻏﻴﺭ ﻤﻌﺩﻭﻤﻴﻥ ﺤﻴﺙ ' ‪ x‬ﻭ ' ‪ y‬ﻭ ' ‪ x = 5x‬ﻭ ' ‪y = 5 y‬‬
‫ﻭﻤﻨﻪ ‪ 25x ' y ' = 1300‬ﺃﻱ ‪x ' y ' = 52‬‬
‫‪ 52 = 2 × 13‬ﻭﻗﻭﺍﺴﻤﻪ ﻫﻲ ‪ 26 , 13 , 4 , 2 , 1‬ﻭ ‪52‬‬
‫‪2‬‬

‫‪x ' -52 -13 -2 -1‬‬ ‫‪1‬‬ ‫‪2‬‬ ‫‪13 52‬‬


‫‪y ' -1 -2 -13 -52 52 13 2 1‬‬
‫‪x -260 -65 -10 -5‬‬ ‫‪5‬‬ ‫‪10 65 260‬‬
‫‪y‬‬ ‫‪-5 -10 -65 -260 260 65 10 5‬‬
‫‪α‬‬ ‫‪-755 -145 295 1285 -1285 -295‬‬ ‫‪145 755‬‬
‫‪β‬‬ ‫‪-1005 -190 415 1800 -1800 -415‬‬ ‫‪190 1005‬‬

‫})‪(α ; β ) ∈ {( 295, 415) ; (1285,1800 ) ; (145,190 ) ; ( 755,1005‬‬


‫ﻤﻥ ﺍﻝﺘﻤﺭﻴﻥ ‪ 85‬ﺇﻝﻰ ﺍﻝﺘﻤﺭﻴﻥ ‪ ، 88‬ﺒﺭﻫﻥ ﻤﻥ ﺃﺠل ﻜل ﻋﺩﺩ ﻁﺒﻴﻌﻲ ‪ ، n‬ﺃ ‪‬‬
‫ﻥ ﺍﻝﻌﺩﺩﻴﻥ ‪ a‬ﻭ ‪ b‬ﺃﻭﻝﻴﺎﻥ ﻓﻴﻤﺎ ﺒﻴﻨﻬﻤﺎ ‪.‬‬
‫‪ a = n + 3 85‬ﻭ ‪. b = 2n + 7‬‬
‫‪ d‬ﻴﻘﺴﻡ ‪ a‬ﻭ ‪ b‬ﺇﺫﻥ ﻴﻘﺴﻡ ‪ 2a‬ﻭﻜﺫﻝﻙ ‪ b − 2a‬ﻭﻝﺩﻴﻨﺎ ‪ b − 2a = 1 :‬ﺇﺫﻥ ‪ d‬ﻴﻘﺴﻡ ‪ 1‬ﻭﻤﻨﻪ ‪. d = 1‬‬
‫‪ a = 3n + 4 86‬ﻭ ‪. b = 8n + 11‬‬
‫‪ d‬ﻴﻘﺴﻡ ‪ a‬ﻭ ‪ b‬ﺇﺫﻥ ﻴﻘﺴﻡ ‪ 8a‬ﻭ ‪ 3b‬ﻭﻜﺫﻝﻙ ‪ 3b − 8a‬ﻭﻝﺩﻴﻨﺎ ‪ 3b − 8a = 1 :‬ﺇﺫﻥ ‪ d‬ﻴﻘﺴﻡ ‪ 1‬ﻭﻤﻨﻪ ‪. d = 1‬‬
‫‪ a = 9n + 7 87‬ﻭ ‪. b = 5n + 4‬‬
‫‪ d‬ﻴﻘﺴﻡ ‪ a‬ﻭ ‪ b‬ﺇﺫﻥ ﻴﻘﺴﻡ ‪ 5a‬ﻭ ‪ 9b‬ﻭﻜﺫﻝﻙ ‪ 9b − 5a‬ﻭﻝﺩﻴﻨﺎ ‪ 9b − 5a = 1 :‬ﺇﺫﻥ ‪ d‬ﻴﻘﺴﻡ ‪ 1‬ﻭﻤﻨﻪ ‪. d = 1‬‬
‫‪ a = 7 n 2 + 2 88‬ﻭ ‪. b = 4n 2 + 1‬‬
‫‪ d‬ﻴﻘﺴﻡ ‪ a‬ﻭ ‪ b‬ﺇﺫﻥ ﻴﻘﺴﻡ ‪ 4a‬ﻭ ‪ 7b‬ﻭﻜﺫﻝﻙ ‪ 4a − 7b‬ﻭﻝﺩﻴﻨﺎ ‪ 4a − 7b = 1 :‬ﺇﺫﻥ ‪ d‬ﻴﻘﺴﻡ ‪ 1‬ﻭﻤﻨﻪ ‪. d = 1‬‬
‫‪ n 89‬ﻋﺩﺩ ﻁﺒﻴﻌﻲ ﻏﻴﺭ ﻤﻌﺩﻭﻡ ‪.‬‬
‫‪ (1‬ﻨﻀﻊ ‪ PGCD ( 2n − 1;9n + 4 ) = d‬ﺇﺫﻥ ‪ d‬ﻴﻘﺴﻡ ) ‪ ( 9n + 4‬ﻭ )‪ ( 2n − 1‬ﻭﻤﻨﻪ ‪ d‬ﻴﻘﺴﻡ ) ‪2 ( 9n + 4‬‬
‫ﻭ )‪ 9 ( 2n − 1‬ﺇﺫﻥ ‪ d‬ﻴﻘﺴﻡ )‪2 ( 9n + 4 ) − 9 ( 2n − 1‬‬
‫ﺒﻤﺎ ﺃﻥ ‪ 2 ( 9n + 4 ) − 9 ( 2n − 1) = 17‬ﻓﺈﻥ ‪ d‬ﻴﻘﺴﻡ ‪ 17‬ﺃﻱ ‪ d = 1‬ﺃﻭ ‪. d = 17‬‬
‫‪ (2‬ﺇﺫﺍ ﻜﺎﻥ ‪ PGCD ( 2n − 1;9n + 4 ) = 17‬ﻓﺈﻥ ‪ 17‬ﻴﻘﺴﻡ ) ‪ ( 9n + 4‬ﻭ )‪ ( 2n − 1‬ﻭﻤﻨﻪ ‪ 17‬ﻴﻘﺴﻡ )‪4 ( 2n − 1‬‬
‫ﺇﺫﻥ ‪ 17‬ﻴﻘﺴﻡ ﺍﻝﻔﺭﻕ ‪. ( 9n + 4 ) − 4 ( 2n − 1) = n + 8‬‬
‫‪ (3‬ﺇﺫﺍ ﻜﺎﻥ ‪ PGCD ( 2n − 1;9n + 4 ) = 17‬ﻓﺈﻥ ‪ 17‬ﻴﻘﺴﻡ ‪ n + 8‬ﻭﻤﻨﻪ ‪ n = 17α − 8‬ﻤﻊ ∗‪. α ∈ ℕ‬‬
‫ﻝﻨﺒﺭﻫﻥ ﺍﻝﻌﻜﺱ ‪ ,‬ﻨﻔﺭﺽ ﺃﻥ ‪ n = 17α − 8‬ﻤﻊ ∗‪α ∈ ℕ‬‬
‫ﻭﻤﻨﻪ ‪ 9n + 4 = 9 (17α − 8) + 4 = 9 × 17α − 68‬ﻭ ‪2n − 1 = 2 (17α − 8 ) − 1 = 2 × 17α − 17‬‬
‫ﺃﻱ ‪ 9n + 4 = 17 ( 9α − 4 ) :‬ﻭ )‪2n − 1 = 17 ( 2α − 1‬‬
‫ﻨﻀﻊ ‪ PGCD ( 2α − 1;9α − 4 ) = δ‬ﺇﺫﻥ ‪ δ‬ﻴﻘﺴﻡ )‪ ( 2α − 1‬ﻭ ) ‪ ( 9α − 4‬ﻭﻤﻨﻪ ‪ δ‬ﻴﻘﺴﻡ )‪9 ( 2α − 1‬‬
‫ﻭ ) ‪ 2 ( 9α − 4‬ﺇﺫﻥ ‪ δ‬ﻴﻘﺴﻡ )‪2 ( 9α − 4 ) − 9 ( 2α − 1‬‬
‫ﺃﻱ ‪ δ‬ﻴﻘﺴﻡ ‪ 1‬ﻭﺒﺎﻝﺘﺎﻝﻲ ‪. δ = 1‬‬
‫‪ 9n + 4 = 17 ( 9α − 4 ) , PGCD ( 2α − 1;9α − 4 ) = 1‬ﻭ )‪ 2n − 1 = 17 ( 2α − 1‬ﻤﻌﻨﺎﻩ‬
‫‪. PGCD ( 2n − 1;9n + 4 ) = 17‬‬
‫ﺨﻼﺼﺔ ‪ n = 17α − 8 :‬ﻤﻊ ∗‪ α ∈ ℕ‬ﻤﻌﻨﺎﻩ ‪. PGCD ( 2n − 1;9n + 4 ) = 17‬‬
‫‪ n 90‬ﻋﺩﺩ ﻁﺒﻴﻌﻲ ‪.‬‬
‫ﻨﻀﻊ ‪ b = n + 2 , a = 5n 2 + 14n + 14‬ﻭ ‪. c = 5n + 3‬‬
‫‪ (1‬ﻝﺩﻴﻨﺎ ) ‪ 5n 2 + 14n + 8 = ( n + 2 )( 5n + 4‬ﻭﻤﻨﻪ ‪ b‬ﻗﺎﺴﻡ ﻝﻠﻌﺩﺩ ‪. 5n 2 + 14n + 8‬‬
‫‪ b (2‬ﻴﻘﺴﻡ ‪ a‬ﺇﺫﻥ ‪ b‬ﻴﻘﺴﻡ ) ‪ a − ( 5n 2 + 14n + 8‬ﺃﻱ ‪ b‬ﻴﻘﺴﻡ ‪. 6‬‬
‫ﻭﺒﺎﻝﻌﻜﺱ ‪ ،‬ﻨﻔﺭﺽ ﺃﻥ ‪ b‬ﻴﻘﺴﻡ ‪ 6‬ﺒﻤﺎ ﺃﻥ ‪ b‬ﻴﻘﺴﻡ ‪ 5n 2 + 14n + 8‬ﻓﺈﻨﻪ ﻴﻘﺴﻡ ﺍﻝﻤﺠﻤﻭﻉ ‪ 5n 2 + 14n + 8 + 6‬ﺃﻱ ‪b‬‬
‫ﻴﻘﺴﻡ ‪. a‬‬
‫ﺨﻼﺼﺔ ‪ b :‬ﻴﻘﺴﻡ ‪ a‬ﻤﻌﻨﺎﻩ ‪ b‬ﻴﻘﺴﻡ ‪. 6‬‬
‫‪ b (3‬ﻴﻘﺴﻡ ‪ 6‬ﻤﻌﻨﺎﻩ ‪ n + 2 = 1‬ﺃﻭ ‪ n + 2 = 2‬ﺃﻭ ‪ n + 2 = 3‬ﺃﻭ ‪ n + 2 = 6‬ﻭﻤﻌﻨﺎﻩ ‪ n = 0‬ﺃﻭ ‪ n = 1‬ﺃﻭ ‪. n = 4‬‬
‫ـ ﺇﺫﺍ ﻜﺎﻥ }‪ n ∈ {0,1, 4‬ﻓﺈﻥ ‪ b‬ﻴﻘﺴﻡ ‪ 6‬ﺃﻱ ‪ b‬ﻴﻘﺴﻡ ‪ a‬ﻭﻤﻨﻪ ﺒﺎﻗﻲ ﻗﺴﻤﺔ ‪ a‬ﻋﻠﻰ ‪ b‬ﻫﻭ ‪. 0‬‬
‫ـ ﺇﺫﺍ ﻜﺎﻥ ‪ n = 2‬ﻓﺈﻥ ‪ a = 62‬ﻭ ‪ b = 4‬ﺇﺫﻥ ﺍﻝﺒﺎﻗﻲ ‪. 2‬‬
‫ـ ﺇﺫﺍ ﻜﺎﻥ ‪ n = 3‬ﻓﺈﻥ ‪ a = 101‬ﻭ ‪ b = 5‬ﺇﺫﻥ ﺍﻝﺒﺎﻗﻲ ‪. 1‬‬
‫ـ ﺇﺫﺍ ﻜﺎﻥ ‪ n > 4‬ﻓﺈﻥ ‪ b > 6‬ﻭﻝﺩﻴﻨﺎ ‪ a = bc + 6‬ﺇﺫﻥ ﺒﺎﻗﻲ ﻗﺴﻤﺔ ‪ a‬ﻋﻠﻰ ‪ b‬ﻫﻭ ‪. 6‬‬
‫‪ c = 5n + 3‬؛‬
‫ـ ﺇﺫﺍ ﻜﺎﻥ ‪ n = 0‬ﻓﺈﻥ ‪ a = 14‬ﻭ ‪ c = 3‬ﻭﻤﻨﻪ ﺒﺎﻗﻲ ﻗﺴﻤﺔ ‪ a‬ﻋﻠﻰ ‪ c‬ﻫﻭ ‪. 2‬‬
‫ﻤﻥ ﺃﺠل ﻜل ∗‪ c > 6 ، n ∈ ℕ‬ﻭﻝﺩﻴﻨﺎ ‪ a = cb + 6‬ﺇﺫﻥ ﺒﺎﻗﻲ ﻗﺴﻤﺔ ﺍﻝﻌﺩﺩ ‪ a‬ﻋﻠﻰ ‪ c‬ﻫﻭ ‪. 6‬‬
‫‪ n ∈ ℤ − {1} (1 91‬ﻨﻀﻊ ‪ a = 3n + 5 :‬ﻭ ‪. b = n − 1‬‬
‫أ ـ ﻝﺩﻴﻨﺎ ‪ a − 3b = 3n + 5 − 3n + 3 = 8‬ﺇﺫﻥ ‪a = 3b + 8‬‬
‫‪a a‬‬ ‫‪8‬‬
‫ﻋﺩﺩﺍ ﺼﺤﻴﺤﺎ ﻤﻌﻨﺎﻩ ‪ b‬ﻴﻘﺴﻡ ‪8‬‬ ‫ب ـ ‪. = 3+‬‬
‫‪b b‬‬ ‫‪b‬‬
‫ﺃﻱ ‪ b ∈ {−8; − 4; − 2; − 1;1; 2; 4;8} :‬ﻤﻌﻨﺎﻩ }‪n ∈ {−7; − 3; − 1;0; 2;3;5;9‬‬
‫‪ (2‬ﻨﻔﺭﺽ ﺃﻥ ‪ n‬ﻋﺩﺩ ﻁﺒﻴﻌﻲ ‪.‬‬
‫أ ـ ﻨﻀﻊ ‪ d . PGCD ( a ; b ) = d‬ﻴﻘﺴﻡ ‪ a‬ﻭ ‪ b‬ﺇﺫﻥ ﻴﻘﺴﻡ ‪ 3b‬ﻭﻤﻨﻪ ﻴﻘﺴﻡ ‪ a − 3b‬ﻭﺒﺎﻝﺘﺎﻝﻲ ‪ d‬ﻴﻘﺴﻡ ‪. 8‬‬
‫ب ـ ﺇﺫﺍ ﻜﺎﻥ ‪ n = 8k‬ﻓﺈﻥ ‪ d‬ﻴﻘﺴﻡ ‪ n‬ﻭﻤﻨﻪ ‪ d‬ﻴﻘﺴﻡ ‪ n − b‬ﺃﻱ ‪ d‬ﻴﻘﺴﻡ ‪ 1‬ﻭﺒﺎﻝﺘﺎﻝﻲ ‪. d = 1‬‬
‫ـ ﺇﺫﺍ ﻜﺎﻥ ‪ n = 8k + 1‬ﻓﺈﻥ )‪ a = 8 ( 3k + 1‬ﻭ ‪ b = 8k‬ﺇﺫﻥ ‪ 8‬ﻴﻘﺴﻡ ‪ d‬ﻭﺒﻤﺎ ﺃﻥ ‪ d‬ﻴﻘﺴﻡ ‪ 8‬ﻓﺈﻥ ‪. d = 8‬‬
‫ـ ﺇﺫﺍ ﻜﺎﻥ ‪ n = 8k + 2‬ﻓﺈﻥ ‪ a = 24k + 11‬ﻭ ‪ b = 8k + 1‬ﺒﻤﺎ ﺃﻥ ‪ d‬ﻴﻘﺴﻡ ‪ , 8‬ﻭ ‪ a‬ﻭ ‪ b‬ﻓﺭﺩﻴﺎﻥ ﻓﺈﻥ ‪. d = 1‬‬
‫ـ ﺇﺫﺍ ﻜﺎﻥ ‪ n = 8k + 3‬ﻓﺈﻥ ) ‪ a = 2 (12k + 7‬ﻭ )‪ b = 2 ( 4k + 1‬؛ ﻨﻀﻊ )‪ d ' d ' = PGCD (12k + 7; 4k + 1‬ﻴﻘﺴﻡ‬
‫)‪ 3 ( 4k + 1‬ﻭﻤﻨﻪ ﻴﻘﺴﻡ )‪ 12k + 7 − 3 ( 4k + 1‬ﺃﻱ ﻴﻘﺴﻡ ‪ 4‬ﻭﺒﺎﻝﺘﺎﻝﻲ ' ‪ d‬ﻴﻘﺴﻡ ‪ 4k‬ﻭ ‪ 4k + 1‬ﺇﺫﻥ ﻴﻘﺴﻡ ﻓﺭﻗﻬﻤﺎ ‪1‬‬
‫ﻭﺒﺎﻝﺘﺎﻝﻲ ‪ d ' = 1‬ﺇﺫﻥ ‪. d = PGCD ( a ; b ) = 2‬‬
‫ـ ﺇﺫﺍ ﻜﺎﻥ ‪ n = 8k + 4‬ﻓﺈﻥ ‪ a = 24k + 17‬ﻭ ‪b = 8k + 3‬‬
‫‪ a‬ﻭ ‪ b‬ﻓﺭﺩﻴﺎﻥ ﺒﻤﺎ ﺃﻥ ‪ d‬ﻴﻘﺴﻡ ‪ 8‬ﻓﺈﻥ ‪. d = 1‬‬
‫ـ ﺇﺫﺍ ﻜﺎﻥ ‪ n = 8k + 5‬ﻓﺈﻥ ) ‪ a = 4 ( 3k + 5‬ﻭ )‪ b = 4 ( 2k + 1‬؛ ﺇﺫﺍ ﻜﺎﻥ ‪ d = 8‬ﻓﺈﻥ ‪ 2k + 1‬ﻴﻘﺒل ﺍﻝﻘﺴﻤﺔ ﻋﻠﻰ ‪2‬‬
‫ﻭﻫﺫﺍ ﺘﻨﺎﻗﺽ ﺇﺫﻥ ‪. d = 4‬‬
‫ـ ﺇﺫﺍ ﻜﺎﻥ ‪ n = 8k + 6‬ﻓﺈﻥ ‪ a = 24k + 23‬ﻭ ‪ b = 8k + 5‬؛ ‪ a‬ﻭ ‪ b‬ﻓﺭﺩﻴﺎﻥ ﺒﻤﺎ ﺃﻥ ‪ d‬ﻴﻘﺴﻡ ‪ 8‬ﻓﺈﻥ ‪d = 1‬‬
‫ـ ﺇﺫﺍ ﻜﺎﻥ ‪ n = 8k + 7‬ﻓﺈﻥ )‪ b = 2 ( 4k + 3) a = 2 ( 8k + 13‬؛ ‪ 2k + 3‬ﻫﻭ ﻓﺭﺩﻱ ‪ ,‬ﺇﺫﺍ ﻜﺎﻥ ‪ d = 8‬ﺃﻭ ‪d = 4‬‬

‫ﻓﺈﻥ ‪ 2k + 3‬ﻴﻘﺒل ﺍﻝﻘﺴﻤﺔ ﻋﻠﻰ ﺍﻷﻗل ﻋﻠﻰ ‪ 2‬ﻭﻫﺫﺍ ﺘﻨﺎﻗﺽ ﺇﺫﻥ ‪. d = 2‬‬
‫‪ n (1 92‬ﻋﺩﺩ ﻁﺒﻴﻌﻲ ‪ α = n 2 + n ،‬ﻭ ‪. β = n + 2‬‬
‫أ ـ ﻨﻀﻊ ‪ PGCD (α ; β ) = d‬ﻭ ' ‪PGCD ( n ; β ) = d‬‬
‫‪ d‬ﻴﻘﺴﻡ ‪ α‬ﻭ ‪ β‬ﺇﺫﻥ ﻴﻘﺴﻡ ﻜﺫﻝﻙ ‪ n β‬ﻭﻤﻨﻪ ﻴﻘﺴﻡ ‪ n β − β‬ﺃﻱ ﻴﻘﺴﻡ ‪ n‬ﻭﺒﺎﻝﺘﺎﻝﻲ ‪ d‬ﻴﻘﺴﻡ ' ‪. PGCD ( n ; β ) = d‬‬
‫ﺍﻝﻌﻜﺱ ‪ d ' :‬ﻴﻘﺴﻡ ‪ n‬ﻭ ‪ β‬ﺇﺫﻥ ﻴﻘﺴﻡ )‪ n ( n + 1‬ﺃﻱ ﻴﻘﺴﻡ ‪ α‬ﻭﺒﺎﻝﺘﺎﻝﻲ ' ‪ d‬ﻴﻘﺴﻡ ‪. PGCD (α ; β ) = d‬‬
‫‪ d‬ﻴﻘﺴﻡ ' ‪ d‬ﻭ ' ‪ d‬ﻴﻘﺴﻡ ‪ d‬ﻤﻌﻨﺎﻩ ' ‪ d = d‬ﺃﻱ‬
‫) ‪. PGCD (α ; β ) = PGCD ( n ; β‬‬
‫ب ـ ‪ d‬ﻴﻘﺴﻡ ‪ n + 2‬ﻭ ‪ n‬ﺇﺫﻥ ﻴﻘﺴﻡ ﻓﺭﻗﻬﻤﺎ ‪ 2‬ﻭﺒﺎﻝﺘﺎﻝﻲ ‪ PGCD (α ; β ) = 2‬ﺃﻭ ‪. PGCD (α ; β ) = 1‬‬
‫‪ (2‬أ ـ ) ‪a = 3n 3 + 5n 2 + 2n = ( 3n + 2 ) ( n 2 + n‬‬
‫) ‪b = 3n 2 + 8n + 4 = ( 3n + 2 )( n + 2‬‬
‫ﺇﺫﻥ ﺍﻝﻌﺩﺩ ) ‪ ( 3n + 2‬ﻫﻭ ﻗﺎﺴﻡ ﻤﺸﺘﺭﻙ ﻝﻠﻌﺩﺩﻴﻥ ‪ a‬ﻭ ‪. b‬‬
‫ب ـ ﻝﺩﻴﻨﺎ ) ‪ a = α ( 3n + 2‬ﻭ ) ‪. b = β ( 3n + 2‬‬
‫ـ ﺇﺫﺍ ﻜﺎﻥ ‪ n‬ﻓﺭﺩﻴﺎ ﻓﺈﻥ ‪ β‬ﻴﻜﻭﻥ ﻓﺭﺩﻴﺎ ﻭﺒﺎﻝﺘﺎﻝﻲ ‪ d ≠ 2‬ﺇﺫﻥ ‪ d = PGCD (α ; β ) = 1‬ﻭﻤﻨﻪ‬
‫) ‪. PGCD ( a ; b ) = ( 3n + 2‬‬
‫ـ ﺇﺫﺍ ﻜﺎﻥ ‪ n‬ﺯﻭﺠﻴﺎ ﻓﺈﻥ ‪ α‬ﻭ ‪ β‬ﺯﻭﺠﻴﺎﻥ ﻭﻤﻨﻪ ‪d = 2‬‬
‫ﺇﺫﻥ ﻴﻭﺠﺩ ﻋﺩﺩﺍﻥ ﻁﺒﻴﻌﻴﺎﻥ ' ‪ α‬ﻭ ' ‪ β‬ﺃﻭﻝﻴﺎﻥ ﻓﻴﻤﺎ ﺒﻴﻨﻬﻤﺎ ﺤﻴﺙ ' ‪ α = 2α‬ﻭ ' ‪ β = 2 β‬ﺃﻱ ' ‪a = 2 ( 3n + 2 ) α‬‬
‫ﻭ ' ‪ b = 2 ( 3n + 2 ) β‬ﻭﻤﻨﻪ ) ‪PGCD ( a ;b ) = 2 ( 3n + 2‬‬
‫ـ ـ ‪ PGCD ( a ; b ) = 41‬ﻫﻭ ﻋﺩﺩ ﻓﺭﺩﻱ ﺇﺫﻥ ﻻ ﻴﻤﻜﻥ ﺃﻥ ﻴﻜﻭﻥ ‪ PGCD ( a ; b ) = 2 ( 3n + 2 ) = 41‬ﻭﻨﺄﺨﺫ ﺍﻝﺤﺎﻝﺔ‬
‫ﺍﻝﻤﺘﺒﻘﻴﺔ ﺃﻱ ‪ PGCD ( a ; b ) = ( 3n + 2 ) = 41‬ﻤﻌﻨﺎﻩ ‪ n = 13‬ﻭﺒﺎﻝﺘﺎﻝﻲ ‪ α = 182‬ﻭ ‪. β = 15‬‬
‫‪ n 93‬ﻋﺩﺩ ﻁﺒﻴﻌﻲ ؛ ﻨﻀﻊ‪ a = 9n + 1 :‬ﻭ ‪b = 9n − 1‬‬
‫‪ a − b = 2 (1‬؛ ) ‪ PGCD ( a ; b‬ﻴﻘﺴﻡ ﺍﻝﻔﺭﻕ ‪ a − b‬ﺃﻱ ) ‪ PGCD ( a ; b‬ﻴﻘﺴﻡ ‪ 2‬ﻫﻭ ﺇﻤﺎ ‪ 1‬ﻭﺇﻤﺎ ‪. 2‬‬
‫ـ ﺇﺫﺍ ﻜﺎﻥ ‪ n‬ﺯﻭﺠﻴﺎ ﻓﺈﻥ ‪ a‬ﻭ ‪ b‬ﻴﻜﻭﻨﺎ ﻓﺭﺩﻴﺎﻥ ﻭﺒﺎﻝﺘﺎﻝﻲ ‪. PGCD ( a ; b ) = 1‬‬
‫ـ ﺇﺫﺍ ﻜﺎﻥ ‪ n‬ﻓﺭﺩﻴﺎ ﻓﺈﻥ ‪ a‬ﻭ ‪ b‬ﻴﻜﻭﻨﺎ ﺯﻭﺠﻴﺎﻥ ﻭﻤﻨﻪ ﻴﻘﺒﻼﻥ ﺍﻝﻘﺴﻤﺔ ﻋﻠﻰ ‪ 2‬ﻭﺒﺎﻝﺘﺎﻝﻲ ‪. PGCD ( a ; b ) = 2‬‬
‫‪ 81n 2 − 1 = ( 9n + 1)( 9n − 1) = ab (3‬ﻭﻓﻲ ﺤﺎﻝﺔ ‪ n‬ﻋﺩﺩ ﻓﺭﺩﻱ ‪ PGCD ( a ; b ) = 2 ،‬ﻤﻌﻨﺎﻩ ‪، a = 2k‬‬
‫' ‪ b = 2k‬ﻭ ‪ p gcd ( k ; k ' ) = 1‬ﺇﺫﻥ ' ‪ ab = 4kk‬ﻭﺒﺎﻝﺘﺎﻝﻲ " ‪ 81n 2 − 1 = 4k‬ﻤﻌﻨﺎﻩ ‪81n 2 = 4k "+ 1‬‬
‫ﺇﺫﻥ ﺒﺎﻗﻲ ﻗﺴﻤﺔ ﺍﻝﻌﺩﺩ ‪ 81n 2‬ﻋﻠﻰ ‪ 4‬ﻫﻭ ‪. 1‬‬
‫ﺍﻝﻤﺴﺎﺌل‬
‫ﻤﻥ ﺃﺠل ﻜل ∗‪ n ∈ ℕ‬ﻨﻀﻊ‪. s n = 13 + 23 + ... + n 3 :‬‬
‫ﻓﻲ ﻫﺫﺍ ﺍﻝﺘﻤﺭﻴﻥ ﻴﻤﻜﻥ ﺍﺴﺘﻌﻤﺎل ﺍﻝﻨﺘﻴﺠﺔ ﺍﻝﺘﺎﻝﻴﺔ ‪:‬‬
‫‪ PGCD ( a ; b ) = 1‬ﻴﻜﺎﻓﺊ ‪. PGCD ( a 2 ; b 2 ) = 1‬‬
‫‪ 1(1 + 1) ‬‬ ‫‪ 1(1 + 1) ‬‬
‫‪2‬‬ ‫‪2‬‬

‫‪ s 1 = ‬ﻭﻤﻨﻪ ﺍﻝﺨﺎﺼﻴﺔ ﺍﻝﺒﺩﺍﺌﻴﺔ ﺼﺤﻴﺤﺔ ‪.‬‬ ‫‪ ‬ﺇﺫﻥ ‪‬‬ ‫‪ s n = 1 = 1 (1‬ﻭ ‪ = 1‬‬


‫‪3‬‬

‫‪ 2 ‬‬ ‫‪ 2 ‬‬


‫‪ ( k + 1)( k + 2 ) ‬‬ ‫‪ k ( k + 1) ‬‬
‫‪2‬‬ ‫‪2‬‬
‫∗‬
‫‪. s k +1 = ‬‬ ‫‪ s k = ‬ﻤﻥ ﺃﺠل ‪ k ∈ ℕ‬ﻭﻝﻨﺒﺭﻫﻥ ﺼﺤﺔ ﺍﻝﺨﺎﺼﻴﺔ ‪‬‬ ‫ﻨﻔﺭﺽ ‪‬‬
‫‪‬‬ ‫‪2‬‬ ‫‪‬‬ ‫‪‬‬ ‫‪2‬‬ ‫‪‬‬
‫‪ k ( k + 1) ‬‬
‫‪2‬‬

‫‪ s k +1 = s k + ( k + 1) = ‬ﺃﻱ‬ ‫)‪ + ( k + 1‬‬


‫‪3‬‬

‫‪‬‬ ‫‪2‬‬ ‫‪‬‬


‫‪(k‬‬ ‫‪+ 1) ( k + 2 )  ( k + 1)( k + 2 ) ‬‬
‫‪2‬‬
‫) ‪( k + 1) ( k 2 + 4k + 4‬‬
‫‪2‬‬ ‫‪2‬‬
‫)‪k 2 ( k + 1) + 4 ( k + 1‬‬
‫‪2‬‬ ‫‪3‬‬ ‫‪2‬‬

‫‪. s k +1‬‬ ‫=‬ ‫‪=‬‬ ‫= ‪ s k +1‬‬ ‫=‬


‫‪4‬‬ ‫‪‬‬ ‫‪2‬‬ ‫‪‬‬ ‫‪4‬‬ ‫‪4‬‬

‫‪ n ( n + 1) ‬‬
‫‪2‬‬

‫‪. sn = ‬‬ ‫ﻭﺤﺴﺏ ﻤﺒﺩﺃ ﺍﻝﺘﺭﺍﺠﻊ ﻴﻨﺘﺞ ﺃﻨﻪ ﻤﻥ ﺃﺠل ﻜل ﻋﺩﺩ ﻁﺒﻴﻌﻲ ﻏﻴﺭ ﻤﻌﺩﻭﻡ ‪ ، n‬‬
‫‪‬‬ ‫‪2‬‬ ‫‪‬‬
‫‪ k (2‬ﻭ ‪ k + 1‬ﻋﺩﺩﺍﻥ ﻤﺘﻭﺍﻝﻴﺎﻥ ﺇﺫﻥ ﻫﻤﺎ ﺃﻭﻝﻴﺎﻥ ﻓﻴﻤﺎ ﺒﻴﻨﻬﻤﺎ ﻭﺒﺎﻝﺘﺎﻝﻲ ‪. PGCD ( k ; k + 1) = 1‬‬
‫‪ 2k ( 2k + 1) ‬‬
‫‪2‬‬

‫‪s 2k‬‬ ‫‪=‬‬ ‫ـ ﻝﻴﻜﻥ ‪ k‬ﻋﺩﺩ ﻁﺒﻴﻌﻲ ﻏﻴﺭ ﻤﻌﺩﻭﻡ ‪ ,‬‬


‫‪‬‬ ‫‪2‬‬ ‫‪‬‬
‫‪ ( 2k + 1)( 2k + 2 ) ‬‬
‫‪2‬‬

‫ﺃﻱ )‪ s 2 k = k ( 2k + 1‬؛ ‪‬‬


‫‪2‬‬ ‫‪2‬‬
‫‪s 2 k +1 = ‬‬
‫‪‬‬ ‫‪2‬‬ ‫‪‬‬
‫(‬
‫)‪PGCD k 2 ; ( k + 1‬‬
‫‪2‬‬
‫‪ . s‬ﺒﻤﺎ ﺃﻥ ‪ PGCD ( k ; k + 1) = 1‬ﻓﺈﻥ ‪) = 1‬‬ ‫‪2 k +1‬‬ ‫ﻤﻌﻨﺎﻩ )‪= ( 2k + 1) ( k + 1‬‬
‫‪2‬‬ ‫‪2‬‬

‫ﻭﺒﺎﻝﺘﺎﻝﻲ ‪PGCD ( s 2 k ; s 2 k +1 ) = ( 2k + 1) :‬‬


‫‪2‬‬

‫‪ PGCD ( 2k + 1; 2k + 3) (3‬ﻴﻘﺴﻡ ﺍﻝﻔﺭﻕ ﺍﻝﺫﻱ ﻫﻭ ‪ 2‬ﺇﺫﻥ ‪PGCD ( 2k + 1; 2k + 3) = 1‬‬


‫ﺃﻭ ‪. PGCD ( 2k + 1; 2k + 3) = 1‬‬
‫‪ a 97‬ﻋﺩﺩ ﻁﺒﻴﻌﻲ ﻏﻴﺭ ﻤﻌﺩﻭﻡ ‪.‬‬
‫‪ (1‬ﺩﺭﺍﺴﺔ ﺍﻝﻤﻌﺎﺩﻝﺔ ﺫﺍﺕ ﺍﻝﻤﺠﻬﻭل ﺍﻝﻌﺩﺩ ﺍﻝﻁﺒﻴﻌﻲ ‪ a‬ﺍﻝﺘﺎﻝﻴﺔ‪ 9 + a 2 = 2n :‬ﺤﻴﺙ ‪ n‬ﻋﺩﺩ ﻁﺒﻴﻌﻲ ﺃﻜﺒﺭ ﻤﻥ ﺃﻭ ﻴﺴﺎﻭﻱ ‪. 4‬‬
‫أ ـ ﻨﻔﺘﺭﺽ ﺃﻥ ﺍﻝﻤﻌﺎﺩﻝﺔ ﺘﻘﺒل ﺤﻼ ‪ a‬ﺯﻭﺠﻴﺎ ﻭﻤﻨﻪ ‪ 2‬ﻴﻘﺴﻡ ‪ a 2‬ﺇﺫﻥ ﻴﻘﺴﻡ ﺍﻝﻔﺭﻕ ‪ 2n − a 2‬ﻭﺒﺎﻝﺘﺎﻝﻲ ‪ 2‬ﻴﻘﺴﻡ ‪ 9‬ﻭﻫﺫﺍ ﺘﻨﺎﻗﺽ‬
‫ﺇﺫﻥ ﻻ ﻴﻤﻜﻥ ﺃﻥ ﻴﻜﻭﻥ ‪ a‬ﺯﻭﺠﻴﺎ ﺇﺫﻥ ﻴﻜﻭﻥ ﻓﺭﺩﻴﺎ ‪.‬‬
‫ب ـ ﻨﻔﺘﺭﺽ ﺃﻥ ﺍﻝﻤﻌﺎﺩﻝﺔ ﺘﻘﺒل ﺤﻼ ‪ a‬ﺇﺫﻥ ﻫﻭ ﻓﺭﺩﻱ ﻭﻤﻨﻪ ﺒﺎﻗﻲ ﻗﺴﻤﺔ ‪ a 2‬ﻋﻠﻰ ‪ 4‬ﻫﻭ ‪ 1‬ﺃﻱ ‪ a 2 = 4k + 1‬ﻤﻊ ‪k ∈ ℕ‬‬
‫ﻭﻤﻨﻪ ‪ 9 + 4k + 1 = 2n‬ﺃﻱ ‪ . 10 = 2 n − 4k‬ﺒﻤﺎ ﺃﻥ ‪ 4‬ﻴﻘﺴﻡ ‪ 2n‬ﻭﻫﺫﺍ ﻤﻥ ﺃﺠل ‪ n ≥ 4‬ﻓﺈﻥ ‪ 4‬ﻴﻘﺴﻡ ‪ 2 n − 4k‬ﺃﻱ ‪4‬‬
‫ﻴﻘﺴﻡ ‪ 10‬ﻭﻫﺫﺍ ﺘﻨﺎﻗﺽ ‪ .‬ﺇﺫﻥ ﺍﻝﻤﻌﺎﺩﻝﺔ ﻻ ﺘﻘﺒل ﺤﻠﻭل‪.‬‬
‫‪ (2‬ﺩﺭﺍﺴﺔ ﺍﻝﻤﻌﺎﺩﻝﺔ ﺫﺍﺕ ﺍﻝﻤﺠﻬﻭل ﺍﻝﻌﺩﺩ ﺍﻝﻁﺒﻴﻌﻲ ‪ a‬ﺍﻝﺘﺎﻝﻴﺔ‪ 9 + a 2 = 3n :‬ﺤﻴﺙ ‪ n‬ﻋﺩﺩ ﻁﺒﻴﻌﻲ ﺃﻜﺒﺭ ﻤﻥ ﺃﻭ ﻴﺴﺎﻭﻱ ‪. 3‬‬
‫أ ـ ‪ 32 − 1 = 8‬ﻭ ‪ 8‬ﻴﻘﺒل ﺍﻝﻘﺴﻤﺔ ‪ 4‬ﺇﺫﻥ ﺍﻝﺨﺎﺼﻴﺔ ﺍﻝﺒﺩﺍﺌﻴﺔ ﺼﺤﻴﺤﺔ ‪ .‬ﻨﻔﺭﺽ ﺃﻨﻪ ﻤﻥ ﺃﺠل ∗‪ k ∈ ℕ‬ﺍﻝﻌﺩﺩ ‪ 32 k − 1‬ﻴﻘﺒل‬
‫ﺍﻝﻘﺴﻤﺔ ﻋﻠﻰ ‪ 4‬ﺃﻱ ‪ 32 k − 1 = 4P‬ﻤﻊ ∗‪. p ∈ ℕ‬‬
‫(‪3‬‬ ‫‪− 1 = 32 k + 2 − 1 = 9 × 32 k − 1 = 9 ( 4 p + 1) − 1‬‬
‫)‪2 k +1‬‬

‫) ‪ 32( k +1) − 1 = 36 p + 8 = 4 ( 9 p + 2‬ﺇﺫﻥ ‪ 32( k +1) − 1‬ﻴﻘﺒل ﺍﻝﻘﺴﻤﺔ ﻋﻠﻰ ‪ , 4‬ﻭﺤﺴﺏ ﻤﺒﺩﺃ ﺍﻝﺘﺭﺍﺠﻊ ﻴﻨﺘﺞ ﺃﻨﻪ ﻤﻥ ﺍﺠل ﻜل‬
‫∗‪ 32 n − 1 , n ∈ ℕ‬ﻴﻘﺒل ﺍﻝﻘﺴﻤﺔ ﻋﻠﻰ ‪. 4‬‬
‫ب ـ ﻝﺩﻴﻨﺎ ‪ 32 n = 4k + 1‬ﺤﻴﺙ ‪ k‬ﻋﺩﺩ ﻁﺒﻴﻌﻲ ﻭ ‪ 3 × 32 n = 4 ( 3k ) + 3‬ﺃﻱ ‪ 32 n +1 = 4k '+ 3‬ﺤﻴﺙ ' ‪ k‬ﻋﺩﺩ ﻁﺒﻴﻌﻲ ‪.‬‬
‫ﺇﺫﻥ ﺍﻝﺒﺎﻗﻴﺎﻥ ﻝﻠﻘﺴﻤﺔ ﺍﻷﻗﻠﻴﺩﻴﺔ ﻝﻜل ﻤﻥ ﺍﻝﻌﺩﺩﻴﻥ ‪ 32 n‬ﻭ ‪ 32 n +1‬ﻋﻠﻰ ‪ 4‬ﻫﻤﺎ ‪ 1‬ﻭ ‪ 3‬ﻋﻠﻰ ﺍﻝﺘﺭﺘﻴﺏ ‪.‬‬
‫ـ ـ ﺤﺴﺏ ﺍﻝﺴﺅﺍل ﺍﻝﺴﺎﺒﻕ ﻤﻥ ﺃﺠل ﻜل ﻋﺩﺩ ﻁﺒﻴﻌﻲ ﺯﻭﺠﻲ ‪ ، n‬ﺒﺎﻗﻲ ﻗﺴﻤﺔ ‪ 3n‬ﻋﻠﻰ ‪ 4‬ﻫﻭ ‪ 1‬ﻭﻤﻥ ﺃﺠل ﻜل ﻋﺩﺩ ﻁﺒﻴﻌﻲ‬
‫ﻓﺭﺩﻱ ‪ ، n‬ﺒﺎﻗﻲ ﻗﺴﻤﺔ ‪ 3n‬ﻋﻠﻰ ‪ 4‬ﻫﻭ ‪ 3‬ﺇﺫﻥ ﺍﻝﺒﺎﻗﻲ ﻴﺨﺘﻠﻑ ﻋﻥ ‪. 2‬‬
‫ﻨﻔﺘﺭﺽ ﺃﻥ ﺍﻝﻤﻌﺎﺩﻝﺔ ‪ 9 + a 2 = 3n‬ﺘﻘﺒل ﺤﻼ ‪ a‬ﻓﺭﺩﻴﺎ ﺇﺫﻥ ‪ a 2 = 4k + 1‬ﻤﻊ ‪ k ∈ ℕ‬؛ ﻭﻝﺩﻴﻨﺎ ﺇﺫﺍ ﻜﺎﻥ ‪ n‬ﻓﺭﺩﻴﺎ ﻓﺈﻥ‬
‫‪ 9 = 4 ( k '− k‬ﻭﻫﺫﺍ ﻏﻴﺭ ﻤﻤﻜﻥ ؛ ﻭﺇﺫﺍ ﻜﺎﻥ ‪ n‬ﺯﻭﺠﻴﺎ ﻓﺈﻥ ‪ 3n = 4k '+ 3‬ﻭﻤﻨﻪ ) ‪7 = 4 ( k '− k‬‬ ‫)‬ ‫‪ 3n = 4k '+ 1‬ﻭﻤﻨﻪ‬
‫ﻭﻫﺫﺍ ﻜﺫﻝﻙ ﻏﻴﺭ ﻤﻤﻜﻥ ‪ ,‬ﻭﻤﻨﻪ ﺇﺫﺍ ﻜﺎﻥ ‪ a‬ﺤﻼ ﻝﻠﻤﻌﺎﺩﻝﺔ ﻓﻼ ﻴﻤﻜﻨﻪ ﺃﻥ ﻴﻜﻭﻥ ﻓﺭﺩﻴﺎ ﻭﺒﺎﻝﺘﺎﻝﻲ ﻴﻜﻭﻥ ‪ a‬ﺯﻭﺠﻴﺎ ‪ ،‬ﻭﻤﻨﻪ ‪a = 2m‬‬
‫ﻭﺒﺎﻝﺘﺎﻝﻲ ‪ 3n = 9 + a 2 = 4(m + 2) + 1‬ﺇﺫﻥ ﺒﺎﻗﻲ ﻗﺴﻤﺔ ‪ 3n‬ﻋﻠﻰ ‪ 4‬ﻫﻭ ‪ 1‬ﻭﻫﺫﺍ ﻓﻲ ﺍﻝﺤﺎﻝﺔ ‪ n‬ﺯﻭﺠﻲ‬
‫د ـ ) ‪. 32 p − a 2 = ( 3 p − a )( 3 p + a‬‬
‫ﻨﻔﺘﺭﺽ ﺃﻥ ﺍﻝﻤﻌﺎﺩﻝﺔ ‪ 9 + a 2 = 3n‬ﺘﻘﺒل ﺤﻼ ‪ a‬ﻓﺈﻥ ‪ n‬ﺯﻭﺠﻲ ﺃﻱ ‪ n = 2 p‬ﻭ ‪ a‬ﺯﻭﺠﻲ‬
‫ﻭﻤﻨﻪ ) ‪ 9 = ( 3 p − a )( 3 p + a‬ﻭ ‪ a‬ﺯﻭﺠﻲ‪.‬‬
‫ﻗﻭﺍﺴﻡ ﺍﻝﻌﺩﺩ ‪ 9‬ﻫﻲ ‪ 3 ، 1‬ﻭ ‪ 9‬ﺇﺫﻥ ‪:‬‬
‫‪(3‬‬ ‫‪p‬‬
‫)‪−a‬‬ ‫‪1‬‬ ‫‪3‬‬ ‫‪9‬‬
‫‪(3‬‬ ‫‪p‬‬
‫)‪+ a‬‬ ‫‪9‬‬ ‫‪3‬‬ ‫‪1‬‬
‫‪2a‬‬ ‫‪8‬‬ ‫‪0‬‬ ‫‪−8‬‬
‫‪a‬‬ ‫‪4‬‬ ‫‪0 −4‬‬
‫ﺇﺫﺍ ﻜﺎﻥ ‪ a = 0‬ﻓﺈﻥ ‪ 9 = 3‬ﺃﻱ ‪ n = 2‬ﻭﻝﻜﻥ ‪n ≥ 3‬‬
‫‪n‬‬

‫ﻭﺇﺫﺍ ﻜﺎﻥ ‪ a = −4‬ﺃﻭ ‪ a = 4‬ﻓﺈﻥ ‪ 25 = 3n‬ﻭﻫﺫﺍ ﻏﻴﺭ ﻤﻤﻜﻥ‪.‬‬


‫‪ (3‬ﺩﺭﺍﺴﺔ ﺍﻝﻤﻌﺎﺩﻝﺔ ﺫﺍﺕ ﺍﻝﻤﺠﻬﻭل ﺍﻝﻌﺩﺩ ﺍﻝﻁﺒﻴﻌﻲ ‪ a‬ﺍﻝﺘﺎﻝﻴﺔ ‪ 9 + a 2 = 5n :‬ﺤﻴﺙ ‪ n‬ﻋﺩﺩ ﻁﺒﻴﻌﻲ ﺃﻜﺒﺭ ﻤﻥ ﺃﻭ ﻴﺴﺎﻭﻱ ‪. 2‬‬
‫أ ـ ﻨﻀﻊ ‪ n = 2 p + 1‬ﻭﻤﻨﻪ ‪ ، 5n = 5 × 52 p‬ﺍﻝﺒﺎﻗﻴﺎﻥ ﺍﻝﻤﻤﻜﻨﺎﻥ ﻝﻘﺴﻤﺔ ‪ 5 p‬ﻋﻠﻰ ‪ 3‬ﻫﻤﺎ ‪ 1‬ﺃﻭ ‪ 2‬ﻭﻤﻨﻪ ﺒﺎﻗﻲ ﻗﺴﻤﺔ ‪52 p‬‬
‫ﻋﻠﻰ ‪ 3‬ﻫﻭ ‪ 1‬ﻭﺒﺎﻝﺘﺎﻝﻲ ﺒﺎﻗﻲ ﻗﺴﻤﺔ ‪ 5n‬ﻋﻠﻰ ‪ 3‬ﻫﻭ ‪.2‬‬
‫ﺇﺫﺍ ﻜﺎﻥ ‪ a = 3k‬ﻓﺈﻥ ﺒﺎﻗﻲ ﻗﺴﻤﺔ ‪ 9 + a 2‬ﻋﻠﻰ ‪ 3‬ﻫﻭ ‪.0‬‬
‫ﺇﺫﺍ ﻜﺎﻥ ‪ a = 3k + 1‬ﺃﻭ ‪ a = 3k + 2‬ﻓﻨﺠﺩ ﺒﺎﻗﻲ ﻗﺴﻤﺔ ‪ 9 + a 2‬ﻋﻠﻰ ‪ 3‬ﻫﻭ‪ 1‬ﺇﺫﻥ ﻤﻥ ﺃﺠل ﻜل ﻋﺩﺩ ﻁﺒﻴﻌﻲ ‪ a‬ﻴﻜﻭﻥ ﺒﺎﻗﻲ‬
‫ﻗﺴﻤﺔ ‪ 9 + a 2‬ﻋﻠﻰ ‪ 3‬ﻫﻭ ‪ 0‬ﺃﻭ ‪ 1‬ﻭﺒﺎﻝﺘﺎﻝﻲ ﻻ ﻴﻭﺠﺩ ﻋﺩﺩ ﻁﺒﻴﻌﻲ ‪ a‬ﻴﺤﻘﻕ ‪. 9 + a 2 = 5n‬‬
‫ب ـ ﻓﻲ ﺤﺎﻝﺔ ‪ n‬ﺯﻭﺠﻲ ‪ ،‬ﻴﻜﺘﺏ ﻋﻠﻰ ﺍﻝﺸﻜل ‪ n = 2 p‬ﻭﻴﻜﻭﻥ ﻝﺩﻴﻨﺎ ) ‪9 = 52 p − a 2 = ( 5 p − a )( 5 p + a‬‬
‫ﻭﺍﻝﺤﺎﻝﺔ ﺍﻝﻭﺤﻴﺩﺓ ﻫﻲ ‪ 5 p + a = 9‬ﻭ ‪ 5 p − a = 1‬ﻭﻫﺫﺍ ﻴﻌﻨﻲ ‪ 2 × 5 p = 10‬ﻭ ‪ a = 9 − 5 p‬ﺃﻱ ‪ p = 1‬ﻭ ‪a = 4‬‬
‫‪ (1 98‬أ ـ ﺇﺫﺍ ﻜﺎﻥ ‪ d‬ﻗﺎﺴﻡ ﻝﻠﻌﺩﺩﻴﻥ ‪ a p − 1‬ﻭ ‪ a p +1 − 1‬ﻓﺈﻨﻪ ﻴﻘﺴﻡ ﻓﺭﻗﻬﻤﺎ ‪ a p +1 − a p‬ﺃﻱ ‪ d‬ﻴﻘﺴﻡ ﺍﻝﻌﺩﺩ )‪. a p ( a − 1‬‬
‫(‬
‫ب ـ ﻨﻔﺭﺽ ‪ PGCD 4 p +1 − 1, 4 p − 1 = D‬ﻭﻤﻨﻪ ‪ D = 4i‬ﺃﻭ ‪ D = 3‬ﺃﻭ ‪ D = 3 × 4i‬ﻤﻊ } ‪i ∈ {0,1,..., p‬‬ ‫)‬
‫ﻭﻝﺩﻴﻨﺎ ‪ D‬ﻻ ﻴﻤﻜﻥ ﺃﻥ ﻴﻜﻭﻥ ﺯﻭﺠﻴﺎ ﻭﺒﺎﻝﺘﺎﻝﻲ ‪ D = 1‬ﺃﻭ ‪. D = 3‬‬
‫‪ (2‬أ ـ ‪ u 3 = 21 ، u 2 = 5‬ﻭ ‪. p gcd ( 5; 21) = 1‬‬
‫ب ـ ﺍﺴﺘﻌﻤﺎل ﺍﻝﺘﺭﺍﺠﻊ ﻝﻠﺒﺭﻫﺎﻥ ﻋﻠﻰ ﺃﻨﻪ ﻤﻥ ﺃﺠل ﻜل ﻋﺩﺩ ﻁﺒﻴﻌﻲ ‪. u n +1 = 4u n + 1 ، n‬‬

‫ـ ـ ﺍﻝﺒﺭﻫﺎﻥ ﺒﺎﻝﺘﺭﺍﺠﻊ ﻨﺠﺩ ‪ ،‬ﻤﻥ ﺃﺠل ﻜل ﻋﺩﺩ ﻁﺒﻴﻌﻲ ‪ u n ، n‬ﻫﻭ ﻋﺩﺩ ﻁﺒﻴﻌﻲ ‪.‬‬

‫د ـ ‪. PGCD (u n +1 , u n ) = 1‬‬
‫‪1‬‬ ‫‪4‬‬
‫‪ (3‬أ ـ ﻝﻴﻜﻥ ‪ n‬ﻋﺩﺩﺍ ﻁﺒﻴﻌﻴﺎ‪v n +1 = u n +1 + = 4u n + ،‬‬
‫‪3‬‬ ‫‪3‬‬
‫‪1 4‬‬ ‫‪‬‬ ‫‪1 4‬‬
‫‪ v n +1 = 4 v n −  + = 4v n‬ﺇﺫﻥ ) ‪ (v n‬ﻤﺘﺘﺎﻝﻴﺔ ﻫﻨﺩﺴﻴﺔ ﺃﺴﺎﺴﻬﺎ ‪ 4‬ﻭﺤﺩﻫﺎ ﺍﻷﻭل = ‪. v 0 = u 0 +‬‬
‫‪3 3‬‬ ‫‪‬‬ ‫‪3 3‬‬
‫‪n +1‬‬
‫‪4‬‬ ‫‪1 4 −1‬‬ ‫‪4‬‬
‫= ‪u n = × 4n −‬‬ ‫ب ـ ‪ v n = × 4n‬ﻭﻤﻨﻪ‬
‫‪3‬‬ ‫‪3‬‬ ‫‪3‬‬ ‫‪3‬‬
‫ـ ـ ﻝﺩﻴﻨﺎ ‪ 4 − 1 = 3u n‬ﻭ ‪ 4 − 1 = 3u n +1‬ﻭﺤﺴﺏ ﺍﻝﺴﺅﺍل ‪ (2‬ﻝﺩﻴﻨﺎ ‪ PGCD (u n +1 , u n ) = 1‬ﻭﻫﺫﺍ ﻤﻌﻨﺎﻩ‬
‫‪n +2‬‬ ‫‪n +1‬‬

‫‪. PGCD ( 4 n + 2 − 1, 4 n +1 − 1) = 3‬‬


‫ﻤﻌﻨﺎﻩ ‪( 2 − x )( 2 + x ) = y 2‬‬ ‫‪E ... x 2 + y 2 = 4 (1 99‬‬
‫ﺇﺫﻥ ﻴﺠﺏ ﺃﻥ ﻴﻜﻭﻥ ‪ 2 − x > 0‬ﺃﻱ ‪ x = 1‬ﻭﻨﺠﺩ ‪y 2 = 3‬‬
‫ﺇﺫﻥ ﻻ ﻴﻭﺠﺩ ﻋﺩﺩ ﻁﺒﻴﻌﻲ ‪ y‬ﻴﺤﻘﻕ ﺍﻝﻤﻌﺎﺩﻝﺔ ‪. E‬‬
‫‪ (2‬أ ـ ﻨﻔﺘﺭﺽ ﺃﻥ ﺍﻝﻌﺩﺩﻴﻥ ‪ x‬ﻭ ‪ y‬ﺯﻭﺠﻴﺎﻥ ﺃﻱ ‪ x = 2n‬ﻭ ‪ y = 2m‬ﺇﺫﻥ ) ‪ p 2 = 2 ( n 2 + m 2‬ﻭﺒﺎﻝﺘﺎﻝﻲ ‪ 2‬ﻴﻘﺴﻡ ‪p 2‬‬
‫ﻭﻤﻨﻪ ﻴﻘﺴﻡ ‪ p‬ﻭﻫﺫﺍ ﺘﻨﺎﻗﺽ ﻷﻥ ‪ p‬ﺃﻭﻝﻲ ‪ p ≠ 2‬ﺃﻱ ‪ p‬ﻋﺩﺩ ﻓﺭﺩﻱ ‪.‬‬
‫ﻨﻔﺘﺭﺽ ﺃﻥ ﺍﻝﻌﺩﺩﻴﻥ ‪ x‬ﻭ ‪ y‬ﻓﺭﺩﻴﺎﻥ ﺃﻱ ‪ x = 2n + 1‬ﻭ ‪ y = 2m + 1‬ﺇﺫﻥ )‪p = 2 ( 2n + 2n + 2m + 2m + 1‬‬
‫‪2‬‬ ‫‪2‬‬ ‫‪2‬‬

‫ﻭﻫﺫﺍ ﻜﺫﻝﻙ ﺘﻨﺎﻗﺽ ﺇﺫﻥ ‪ x‬ﻭ ‪ y‬ﺃﺤﺩﻫﻤﺎ ﺯﻭﺠﻲ ﻭﺍﻵﺨﺭ ﻓﺭﺩﻱ ‪.‬‬
‫ب ـ ﻨﻔﺘﺭﺽ ﺃﻥ ‪ p‬ﻴﻘﺴﻡ ‪ x‬ﺃﻱ ‪ x = kp‬ﺇﺫﻥ ) ‪ y 2 = p 2 (1 − k 2‬ﺤﺎﻝﺘﻴﻥ ﻤﻤﻜﻨﺘﻴﻥ ‪ k = 1‬ﺃﻭ ‪ k = 0‬ﺃﻱ ‪x = 0‬‬
‫ﺃﻭ ‪ y = 0‬ﻭﻝﻜﻥ ‪ x‬ﻭ ‪ y‬ﻏﻴﺭ ﻤﻌﺩﻭﻤﻴﻥ‬
‫ﻭﺒﻨﻔﺱ ﺍﻝﻁﺭﻴﻘﺔ ﺇﺫﺍ ﺍﻓﺘﺭﻀﻨﺎ ‪ p‬ﻴﻘﺴﻡ ‪ y‬؛‬
‫ﺇﺫﻥ ‪ p‬ﻻ ﻴﻘﺴﻡ ‪ x‬ﻭﻻ ‪. y‬‬
‫‪ x + y‬ﺃﻱ ‪ d‬ﻴﻘﺴﻡ ‪. p‬‬
‫‪2‬‬ ‫‪2‬‬ ‫‪2‬‬
‫ـ ـ ﻨﻀﻊ ‪ PGCD ( x 2 , y 2 ) = d‬؛ ‪ d‬ﻴﻘﺴﻡ ﺍﻝﻤﺠﻤﻭﻉ‬
‫د ـ ‪ d = 1‬ﺃﻭ ‪ ، d = p‬ﺃﻭ ‪ d = p 2‬ﺒﻤﺎ ﺃﻥ ‪ p‬ﻻ ﻴﻘﺴﻡ ‪ x‬ﻭﻻ ‪ y‬ﻓﺈﻥ ‪ ، d ≠ p‬ﺃﻭ ‪ d ≠ p 2‬ﻭﺒﺎﻝﺘﺎﻝﻲ ‪. d = 1‬‬
‫‪(u‬‬ ‫‪(3‬أ ـ ‪−v 2 ) + ( 2uv ) = u 4 + v 4 − 2u 2v 2 + 4u 2v 2‬‬
‫‪2‬‬ ‫‪2‬‬ ‫‪2‬‬

‫) ‪ (u 2 −v 2 ) + ( 2uv ) = p 2 (u 2 − v 2 ) + ( 2uv ) = u 4 + v 4 + 2u 2v 2 = (u 2 + v 2‬ﻭﻫﺫﺍ ﻫﻭ ﺍﻝﻤﻁﻠﻭﺏ ‪.‬‬


‫‪2‬‬ ‫‪2‬‬ ‫‪2‬‬ ‫‪2‬‬ ‫‪2‬‬

‫ب ـ ‪ p = 5‬ﻤﻌﻨﺎﻩ ‪ p = 12 + 22‬ﺇﺫﻥ ) ‪ ( 3, 4‬ﻫﻲ ﺤل ِـ ‪E‬‬


‫‪ p = 13‬ﻤﻌﻨﺎﻩ ‪ p = 32 + 22‬ﺇﺫﻥ ) ‪ ( 5,12‬ﻫﻲ ﺤل ِـ ‪. E‬‬
‫‪ (4‬أ ـ ‪ p = 3‬؛ ﺇﺫﺍ ﺍﻓﺘﺭﻀﻨﺎ ﺃﻥ ‪ u 2 + v 2 = 3‬ﻓﺈﻥ ‪ u 2 = 3 − v 2‬ﻭﻴﺠﺏ ﺃﻥ ﻴﻜﻭﻥ ‪ v 2 < 3‬ﻭﺒﺎﻝﺘﺎﻝﻲ ‪ v = 1‬ﺜﻡ ﻨﺠﺩ‬
‫‪ u 2 = 2‬ﻭ ‪ 2‬ﻝﻴﺱ ﻤﺭﺒﻌﺎ ﺘﺎﻤﺎ ﺇﺫﻥ ‪ 3‬ﻝﻴﺱ ﻤﺠﻤﻭﻉ ﻤﺭﺒﻌﻴﻥ ‪.‬‬
‫‪ x 2 + y 2 = 9‬ﻤﻌﻨﺎﻩ ‪ y 2 = 9 − x 2‬ﻭﻤﻨﻪ ﻴﺠﺏ ﺃﻥ ﻴﻜﻭﻥ ‪ x 2 = 1‬ﺃﻭ ‪ x 2 = 4‬ﻭﻋﻠﻴﻪ ‪ y 2 = 8‬ﺃﻭ ‪ y 2 = 5‬ﻭ ‪ 8‬ﻭ ‪5‬‬
‫ﻝﻴﺴﺎ ﻤﺭﺒﻌﻴﻥ ﺇﺫﻥ ﺍﻝﻤﻌﺎﺩﻝﺔ ﻻ ﺘﻘﺒل ﺤﻼ ‪.‬‬
‫ب ـ ‪ p = 7‬؛ ﺇﺫﺍ ﺍﻓﺘﺭﻀﻨﺎ ﺃﻥ ‪ u 2 + v 2 = 7‬ﻓﺈﻥ ‪ u 2 = 7 − v 2‬ﻭﻴﺠﺏ ﺃﻥ ﻴﻜﻭﻥ ‪ v 2 < 7‬ﻭﺒﺎﻝﺘﺎﻝﻲ ‪ v = 1‬ﺃﻭ ‪ v = 2‬ﺜﻡ‬
‫ﻨﺠﺩ ‪ u 2 = 6‬ﺃﻭ ‪ u 2 = 3‬ﻭ ‪ 6‬ﻭ ‪ 3‬ﻝﻴﺴﺎ ﻤﺭﺒﻌﻴﻥ ﺘﺎﻤﻴﻥ ﺇﺫﻥ ‪ 7‬ﻝﻴﺱ ﻤﺠﻤﻭﻉ ﻤﺭﺒﻌﻴﻥ ‪.‬‬
‫‪ x 2 + y 2 = 49‬ﻤﻌﻨﺎﻩ ‪ y 2 = 49 − x 2‬ﻭﻤﻨﻪ ﻴﺠﺏ ﺃﻥ ﻴﻜﻭﻥ ‪ x 2 = 1‬ﺃﻭ ‪ x 2 = 4‬ﺃﻭ ‪ x 2 = 9‬ﺃﻭ ‪ x 2 = 16‬ﺃﻭ ‪x 2 = 25‬‬
‫ﺃﻭ ‪ x 2 = 36‬ﻭﻋﻠﻴﻪ ‪ y 2 = 48‬ﺃﻭ ‪ y 2 = 45‬ﺃﻭ ‪ y 2 = 40‬ﺃﻭ ‪ y 2 = 33‬ﺃﻭ ‪ y 2 = 24‬ﺃﻭ ‪ y 2 = 13‬ﻭﻓﻲ ﻜل ﺤﺎﻝﺔ ‪ y‬ﻝﻴﺴﺎ‬
‫ﻋﺩﺩﺍ ﻁﺒﻴﻌﻴﺎ ﺇﺫﻥ ﺍﻝﻤﻌﺎﺩﻝﺔ ﻻ ﺘﻘﺒل ﺤﻼ ‪.‬‬
‫‪ M 0 ( x 0 ; y 0 ) (1 100‬؛ )‪ M 0 (1;8‬ﻭﻝﺩﻴﻨﺎ ‪ 5 (1) − 8 + 3 = 0‬ﻭﻤﻨﻪ ﺍﻝﻤﻌﺎﺩﻝﺔ ﻤﺤﻘﻘﺔ ﺇﺫﻥ ) ∆ ( ∈ ‪. M 0‬‬
‫ﻨﻔﺭﺽ ﺃﻥ ) ∆ ( ∈ ‪ M k‬ﺃﻱ ‪. 5x k − y k + 3 = 0‬‬
‫ﻝﻨﺒﺭﻫﻥ ) ∆ ( ∈ ‪. M k +1‬‬
‫‪‬‬ ‫‪35‬‬ ‫‪5‬‬
‫‪ 5x k +1 = 3 x k + 3 y k + 5‬‬
‫‪ ‬ﻭﻤﻨﻪ ‪ 5x k +1 − y k +1 = 5x k − y k‬ﺃﻱ ‪5x k +1 − y k +1 + 3 = 5x k − y k + 3‬‬ ‫ﻝﺩﻴﻨﺎ‬
‫‪20‬‬
‫‪− y = − x − y − 5‬‬ ‫‪8‬‬
‫‪ k +1‬‬ ‫‪3‬‬
‫‪k‬‬
‫‪3‬‬
‫‪k‬‬

‫ﻤﻌﻨﺎﻩ ‪ 5x k +1 − y k +1 + 3 = 0‬ﺇﺫﻥ ) ∆ ( ∈ ‪M k +1‬‬


‫ﻭﺒﺎﻝﺘﺎﻝﻲ ﺤﺴﺏ ﻤﺒﺩﺃ ﺍﻝﺘﺭﺍﺠﻊ ﻴﻨﺘﺞ ﺃﻨﻪ ﻤﻥ ﺃﺠل ﻜل ﻋﺩﺩ ﻁﺒﻴﻌﻲ ‪. M n ∈ ( ∆ ) ، n‬‬
‫ـ ﻝﻴﻜﻥ ‪ n‬ﻋﺩﺩ ﻁﺒﻴﻌﻲ ‪ M n ∈ ( ∆ ) ،‬ﻤﻌﻨﺎﻩ ‪ 5x n − y n + 3 = 0‬ﺃﻱ ‪ 5x n + 3 = y n‬ﺒﺎﻝﺘﻌﻭﻴﺽ ﻓﻲ ﺍﻝﻤﻌﺎﺩﻝﺔ ﺍﻷﻭﻝﻰ‬
‫‪7‬‬ ‫‪1‬‬
‫ﻝﻠﺠﻤﻠﺔ ﻨﺠﺩ ‪x n + ( 5x n + 3) + 1‬‬
‫= ‪ x n +1‬ﻭﻤﻌﻨﺎﻩ ‪. x n +1 = 4x n + 2‬‬
‫‪3‬‬ ‫‪3‬‬
‫‪ x 0 = 1 (2‬ﻭﻤﻨﻪ ‪ x 0 ∈ ℕ‬؛ ﻨﻔﺭﺽ ‪ x k ∈ ℕ‬ﻭﻤﻨﻪ ‪ 4x k ∈ ℕ‬ﺇﺫﻥ ‪ ( 4x k + 2 ) ∈ ℕ‬ﻭﺍﻝﺘﺎﻝﻲ ‪. x k +1 ∈ ℕ‬‬
‫ﺇﺫﻥ ﺤﺴﺏ ﻤﺒﺩﺃ ﺍﻝﺘﺭﺍﺠﻊ ﻴﻨﺘﺞ ﺃﻨﻪ ﻤﻥ ﺃﺠل ﻜل ﻋﺩﺩ ﻁﺒﻴﻌﻲ ‪. x n ∈ ℕ ، n‬‬
‫ـ ﻝﺩﻴﻨﺎ ‪ 5x n + 3 = y n‬ﺒﻤﺎ ﺃﻥ ‪ x n ∈ ℕ‬ﻓﺈﻥ ‪ ( 5x n + 3) ∈ ℕ‬ﻭﺒﺎﻝﺘﺎﻝﻲ ‪. y n ∈ ℕ‬‬
‫‪ PGCD ( x n ; y n ) = d (3‬ﺇﺫﻥ ﻴﻭﺠﺩ ﻋﺩﺩﺍﻥ ﻁﺒﻴﻌﻴﺎﻥ ﻏﻴﺭ ﻤﻌﺩﻭﻤﻴﻥ ﻭﺃﻭﻝﻴﻴﻥ ﻓﻴﻤﺎ ﺒﻴﻨﻬﻤﺎ ‪ x‬ﻭ ‪ y‬ﺤﻴﺙ ‪x n = dx‬‬
‫ﻭ ‪ . y n = dy‬ﻝﺩﻴﻨﺎ ﺍﻝﺜﻨﺎﺌﻴﺔ ) ‪ ( x n ; y n‬ﺘﺤﻘﻕ ﻤﻌﺎﺩﻝﺔ ) ∆ ( ﺇﺫﻥ ‪ 5x n − y n + 3 = 0‬ﻭﻤﻨﻪ ‪ d ( 5x − y ) + 3 = 0‬ﺃﻱ‬
‫) ‪ 3 = d ( y − 5x‬ﺇﺫﻥ ‪ d‬ﻗﺎﺴﻡ ﻝﻠﻌﺩﺩ ‪ 3‬ﺃﻱ }‪. d ∈ {1;3‬‬
‫‪5‬‬ ‫‪2 5 2‬‬
‫‪ x 0 = × 40 − = − = 1 (4‬ﻭﻫﺫﺍ ﺼﺤﻴﺢ ‪.‬‬
‫‪3‬‬ ‫‪3 3 3‬‬
‫‪5‬‬ ‫‪2‬‬ ‫‪5‬‬ ‫‪2‬‬
‫ﻨﻔﺭﺽ ﺃﻥ ‪ x k = × 4k −‬ﻭﻝﻨﺒﺭﻫﻥ ‪. x k +1 = × 4 k +1 −‬‬
‫‪3‬‬ ‫‪3‬‬ ‫‪3‬‬ ‫‪3‬‬
‫‪5‬‬ ‫‪8‬‬ ‫‪5‬‬ ‫‪2‬‬ ‫‪5‬‬ ‫‪2‬‬
‫ﻝﺩﻴﻨﺎ ‪. x k +1 = × 4k +1 − + 2 = × 4k +1 − x k +1 = 4x k + 2 = 4  × 4k −  + 2‬‬
‫‪3‬‬ ‫‪3‬‬ ‫‪3‬‬ ‫‪3‬‬ ‫‪3‬‬ ‫‪3‬‬
‫‪5‬‬ ‫‪2‬‬
‫‪. x n = × 4n −‬‬ ‫ﺇﺫﻥ ﺤﺴﺏ ﻤﺒﺩﺃ ﺍﻝﺘﺭﺍﺠﻊ ﻴﻨﺘﺞ ﺃﻨﻪ ﻤﻥ ﺃﺠل ﻜل ﻋﺩﺩ ﻁﺒﻴﻌﻲ ‪, n‬‬
‫‪3‬‬ ‫‪3‬‬
‫ـ ﻤﻤﺎ ﺴﺒﻕ ﻴﻨﺘﺞ ‪ 3x n = 5 × 4n − 2‬ﺇﺫﻥ ‪ 3‬ﻗﺎﺴﻡ ﻝﻠﻌﺩﺩ ‪ . 5 × 4n − 2‬ﻝﺩﻴﻨﺎ ‪ 2‬ﻴﻘﺴﻡ ‪ 4n‬ﻭﻤﻨﻪ ‪ 2‬ﻴﻘﺴﻡ ‪ 5 × 4n‬ﻭﺒﺎﻝﺘﺎﻝﻲ‬
‫‪ 2‬ﻴﻘﺴﻡ ‪ 5 × 4n − 2‬ﺇﺫﻥ ‪ 2‬ﻭ ‪ 3‬ﻤﻭﺠﻭﺩﺍﻥ ﻓﻲ ﺘﺤﻠﻴل ﺍﻝﻌﺩﺩ ‪ 5 × 4n − 2‬ﺇﺫﻥ ‪ 6‬ﻗﺎﺴﻡ ﻝﻠﻌﺩﺩ ‪. 5 × 4n − 2‬‬

‫ﺍﺨﺘﺒﺭ ﻤﻌﻠﻭﻤﺎﺘﻙ‬
‫ا‪ "c‬ر ‪!"+ +‬ـد‬
‫‪ (1 101‬ب ـ ‪r = 5‬‬
‫‪ (2‬ـ ـ ‪. 46 = 13 × 3 + 7‬‬
‫‪ (3‬ب ـ ‪. 70 = 11× 6 + 4‬‬
‫‪ (1 102‬ب ـ ) ‪ PGCD ( a ;12‬ﻫﻭ ‪ 1‬ﺃﻭ ‪ 3‬؛‬
‫ﻷﻥ ‪ a − 12b = 15‬ﺘﻌﻨﻲ ﺃﻥ ‪a − 12 (b + 1) = 3‬‬
‫ﻭﻤﻨﻪ ) ‪ PGCD ( a ;12‬ﻫﻭ ﻗﺎﺴﻡ ﻝﻠﻌﺩﺩ ‪. 3‬‬
‫‪ (2‬ـ ـ ﺍﻝﻌﺩﺩ ‪ a‬ﻫﻭ ﺠﺩﺍﺀ ﻋﺩﺩﻴﻥ ﺃﻭﻝﻴﻴﻥ ﻓﻲ ﻤﺎ ﺒﻴﻨﻬﻤﺎ ‪،‬‬
‫ﻷﻥ ‪ a = 2835 = 34 × 5 × 7 = 81× 45‬؛ ‪ 81‬ﻭ ‪ 45‬ﺃﻭ‪‬ﻝﻴﺎﻥ ﻓﻴﻤﺎ ﺒﻴﻨﻬﻤﺎ‪.‬‬
‫‪4487‬‬ ‫‪7 × 641‬‬ ‫‪52 × 641‬‬
‫=‪F‬‬ ‫‪= 4 2‬‬ ‫=‬ ‫‪ (3‬ب ـ ﻴﻭﺠﺩ ﻜﺴﺭ ﻤﺴﺎﻭﻴﺎ ِـ ‪ F‬ﻤﻘﺎﻤﻪ ﻤﻥ ﻗﻭﻯ ﺍﻝﻌﺩﺩ ‪ 15‬ﻷﻥ‬
‫‪14175 3 × 5 × 7 ( 3 × 5 ) 4‬‬
‫‪ 103‬ـ ـ ‪. PGCD ( n ; n + 1) = 1‬‬
‫أ‪ N‬أم ‪ ]0c‬؟‬

‫‪ (1 104‬ﺨﺎﻁﺌﺔ‪ (2 .‬ﺼﺤﻴﺤﺔ‪ (3 .‬ﺼﺤﻴﺤﺔ‪.‬‬


‫‪ (6‬ﺨﺎﻁﺌﺔ‪.‬‬ ‫‪ (5‬ﺨﺎﻁﺌﺔ‪.‬‬ ‫‪ (4‬ﺨﺎﻁﺌﺔ‪.‬‬
‫‪ (6‬ﺨﺎﻁﺊ‪.‬‬ ‫‪ (3‬ﺼﺤﻴﺢ‪ (4 .‬ﺼﺤﻴﺢ‪ (5 .‬ﺨﺎﻁﺊ‪.‬‬ ‫‪ (1 105‬ﺨﺎﻁﺊ‪ (2 .‬ﺼﺤﻴﺢ‪.‬‬
‫‪ (2‬ﺨﺎﻁﺌﺔ‪ (3 .‬ﺼﺤﻴﺤﺔ‪.‬‬ ‫ﺼﺤﻴﺤﺔ‪.‬‬ ‫‪(1‬‬
‫‪106‬‬
‫‪ (6‬ﺨﺎﻁﺌﺔ‪.‬‬ ‫‪ (4‬ﺨﺎﻁﺌﺔ‪ (5 .‬ﺼﺤﻴﺤﺔ‪.‬‬
‫ﺍﻷﻧﺸﻄﺔ‬
‫ا
ط اول‬
‫ ‪/ :‬‬
‫ا
ف‪ :‬اآ‪-‬ف ‪* i6‬اص ا ‪
/‬ا‪.5j(
(R‬‬
‫ ت‪ :‬م ا ‪-‬ط آ*) ‪P‬ا ا ‪1‬ب و ج ‪' #‬ة " ا ات  ‪ ." ℤ‬و ‪  #‬أاج ‪ A‬ا‪67‬ل ‪ 4‬ز‬
‫ا ا‪.23‬‬
‫ا
‪ )6 63 @ :‬ا ‪ S1<3 #T U4‬ا ‪'1‬ه ت ل ا ‪-‬ت‪.‬‬
‫ا
ط ا
‬
‫ ‪/ :‬‬
‫ا
ف‪ >?3 :‬ا  ت‪ ،‬ا ا‪ #7‬و ا ‪1‬ا‪.R‬‬
‫ ت‪ :‬م ا ‪-‬ط  أاج‬
‫ا
‪./ :‬‬
‫ا
ط ا

‬
‫ ‪/ :‬‬
‫ا
ف‪ :‬ر
 م أ‪
o‬ا ‪6‬اد‪.‬‬
‫ ت‪ :‬م ا ‪-‬ط آ*) ('ة " ا ‪6‬اد " و ‪  #‬أاج‪.‬‬
‫ا
‪../ :‬‬
‫ا
ط ا
ا‬
‫ ‪/ :‬‬
‫ا
ف‪ >?3 :‬ا ات‪.‬‬
‫ ت‪ :‬م ا ‪-‬ط  أاج‪.‬‬
‫ا
‪../ :‬‬

‫ﺍﻷﻋﻤﺎﻝ ﺍﻟﻤﻮﺟﻬﺔ‬
‫‪ % :P‬ا
‪%2I.‬‬
‫ ‪/ :‬‬
‫ا
ف‪'2 63 :‬وط ‪
(R‬ا ‪ 10 ،9 ،5 ،4 ،3 ،2j(
/‬و ‪.11‬‬
‫ ت‪ #3 @ :‬ا ‪ )@2  )6‬أاج آ @ ا‪'R‬ا‪ U‬آا‪. X  Y4‬‬
‫ا
‪../ :‬‬
‫‪"V+‬ح &‪I‬ب‬
‫ ‪/ :‬‬
‫ا
ف‪ >?3 :‬ا ات  و‪ 
6‬د‪.
9‬‬
‫ ت‪ #3 @ :‬ا ‪ )@2  )6‬أاج آ @ ا‪'R‬ا‪ U‬آا‪. X  Y4‬‬
‫ا
‪./ :‬‬
‫& ‪!+‬د‪E‬ت ‪ +‬ا
‪ax + by = c X‬‬
‫ ‪/ :‬‬
‫ا
ف‪ >?3 :‬ا ات ‪ )a‬ا ‪6‬د‪9‬ت  ا ‪. ax + by = c )@-‬‬
‫ ت‪ :‬م ا ‪-‬ط  أاج أو آا‪. X  Y4‬‬
‫ا
‪./ :‬‬
‫ﺍﻟﺘﻤﺎﺭﻳﻦ‬
‫ﺍﻝﺘﻤﺎﺭﻴﻥ ﺍﻝﺘﻁﺒﻴﻘﻴﺔ‬
‫‪ 1‬ـ ا
‪2‬ا\‪ℤ \ %.‬‬
‫‪ 1‬أ ـ ‪ 45 − 3 = 42 = 7 × 6‬إذن ] ‪. 45 ≡ 3 [ 7‬‬
‫ب ـ ‪ 152 − 2 = 150 = 3 × 50‬إذن ]‪. 152 ≡ 2 [3‬‬
‫‪4‬ـ ـ ‪ 29 − ( −1) = 30 = 6 × 5‬إذن ]‪. 29 ≡ −1[ 6‬‬
‫د ـ ‪ 137 − ( −3) = 140 = 5 × 28‬و ‪. 137 ≡ −3[5] U‬‬
‫و ـ )‪ −13 − 2 = −15 = 5 ( −3‬و ‪. −13 ≡ 2 [5] U‬‬
‫هـ ـ )‪ −17 − ( −7 ) = −10 = 10 ( −1‬و ‪−17 ≡ −7 [10] U‬‬
‫‪ 37 ≡ x [ 4] 2‬ﻤﻌﻨﺎﻩ ‪ 37 − x = 4k‬ﻤﻊ ‪ k ∈ ℤ‬ﺃﻱ‬
‫‪ x = 37 − 4k‬ﻭﺒﺎﻝﺘﺎﻝﻲ ﻴﻤﻜﻥ ﺃﺨﺫ ‪، x = 37 − 4 × 2 = 29 ، x = 37 − 4 = 33 ، x = 37‬‬
‫‪x = 37 − 4 ( −2 ) = 45 ، x = 37 − 4 ( −1) = 42‬‬
‫ﻤﻥ ﺃﺠل ‪ k = 9‬ﻴﻜﻭﻥ ‪ x = 1‬ﻭﻫﻭ ﺍﻝﻌﺩﺩ ﺍﻝﻁﺒﻴﻌﻲ ﺍﻝﻭﺤﻴﺩ ﺍﻷﺼﻐﺭ ﺘﻤﺎﻤﺎ ﻤﻥ ‪. 4‬‬
‫‪ n ≡ 4 [ 7 ] 3‬ﻤﻌﻨﺎﻩ ‪ n = 7 k + 4‬ﻤﻊ ‪. k ∈ ℤ‬‬
‫‪4‬‬ ‫‪26‬‬
‫≤ ‪− ≤k‬‬ ‫‪ 0 ≤ n ≤ 30‬ﻤﻌﻨﺎﻩ ‪ 0 ≤ 7 k + 4 ≤ 30‬ﺃﻱ‬
‫‪7‬‬ ‫‪7‬‬
‫‪ k ∈ ℤ‬ﺇﺫﻥ }‪ k ∈ {0,1, 2,3‬ﻭﻤﻨﻪ }‪. n ∈ {4,11,18, 25‬‬
‫‪ n ≡ 140 [12] 4‬ﻭ ]‪ 140 ≡ 8 [12‬ﺇﺫﻥ ]‪n ≡ 8 [12‬‬
‫ﺒﻤﺎ ﺃﻥ ‪ 0 ≤ 8 < 12‬ﻓﺈﻥ ‪ 8‬ﻫﻭ ﺒﺎﻗﻲ ﻗﺴﻤﺔ ‪ n‬ﻋﻠﻰ ‪. 12‬‬
‫‪ x ≡ 2 [ 7 ] 5‬ﺇﺫﻥ‪:‬‬
‫] ‪ x + 5 ≡ 7 [ 7‬ﻭﻤﻨﻪ ]‪x + 5 ≡ 0 [ 7‬‬
‫] ‪ x − 5 ≡ −3[ 7‬ﻭﻤﻨﻪ ]‪x − 5 ≡ 4 [ 7‬‬
‫] ‪ 9x ≡ 18 [ 7‬ﻭﻤﻨﻪ ]‪9x ≡ 4 [ 7‬‬
‫] ‪ −15x ≡ −30 [ 7‬ﻭﻤﻨﻪ ] ‪−15x ≡ 5 [ 7‬‬
‫] ‪ x ≡ 2 [ 7‬ﻭﻤﻨﻪ ]‪ x 3 ≡ 8 [ 7‬ﺃﻱ ]‪. x 3 ≡ 1[ 7‬‬
‫‪ 6‬أ ـ ] ‪ 46 ≡ 0 [ n‬ﻤﻌﻨﺎﻩ ‪ 46 = kn‬ﻤﻊ ∗‪ k ∈ ℕ‬ﺇﺫﻥ ‪ n‬ﻴﻘﺴﻡ ‪ 46‬ﻭ ‪ n ≥ 2‬ﺃﻱ }‪. n ∈ {2, 23, 46‬‬
‫ب ـ ] ‪ 10 ≡ 1[ n‬ﻤﻌﻨﺎﻩ ‪ 9 = kn‬ﻤﻊ ∗‪ k ∈ ℕ‬ﺇﺫﻥ ‪ n‬ﻴﻘﺴﻡ ‪ 9‬ﻭ ‪ n ≥ 2‬ﺃﻱ }‪. n ∈ {3,9‬‬
‫ـ ـ ] ‪ 27 ≡ 5 [ n‬ﻤﻌﻨﺎﻩ ‪ 22 = kn‬ﻤﻊ ∗‪ k ∈ ℕ‬ﺇﺫﻥ ‪ n‬ﻴﻘﺴﻡ ‪ 22‬ﻭ ‪ n ≥ 2‬ﺃﻱ }‪. n ∈ {2,11, 22‬‬
‫‪ a ≡ b [ n ] 7‬ﻤﻌﻨﺎﻩ ‪ a − b = kn‬ﻤﻊ ‪ k ∈ ℕ‬ﻭﻴﻜﺎﻓﺊ ‪ am − bm = knm‬ﻭﻤﻌﻨﺎﻩ ] ‪. am ≡ bm [ nm‬‬
‫‪ 8‬ﻝﺩﻴﻨﺎ ] ‪ B − b ≡ 0 [ n ] ، A − a ≡ 0 [ n‬ﻭ ] ‪ C − c ≡ 0 [ n‬ﻭﻫﺫﺍ ﻤﻌﻨﺎﻩ ] ‪ B ≡ b [ n ] ، A ≡ a [ n‬ﻭ ] ‪C ≡ c [ n‬‬
‫ﺇﺫﻥ ] ‪ ABC ≡ abc [ n‬ﺃﻱ ] ‪. ABC − abc ≡ 0 [ n‬‬
‫] ‪ n ≡ 0 [ m‬ﻤﻌﻨﺎﻩ ‪ n = km‬ﻤﻊ ∗‪. k ∈ ℕ‬‬ ‫‪9‬‬
‫] ‪ a ≡ b [ n‬ﻤﻌﻨﺎﻩ ‪ a − b = k ' n‬ﻤﻊ ‪. k ' ∈ ℕ‬‬
‫ﻭﻤﻨﻪ ‪ a − b = k ' km‬ﺇﺫﻥ ] ‪. a ≡ b [ m‬‬
‫‪ 30757 ≡ 7 [10] (1 10‬ﻭﻤﻨﻪ ]‪ 15163 ≡ 3[10] . a ≡ 7 [10‬ﻭﻤﻨﻪ ]‪ 12924 ≡ 4 [10] . b ≡ 3[10‬ﻭﻤﻨﻪ‬
‫]‪. c ≡ 4 [10‬‬
‫‪ (2‬أ ـ ]‪ a + b + c ≡ 7 + 3 + 4 [10‬ﻭﻤﻨﻪ ]‪. a + b + c ≡ 4 [10‬‬
‫ب ـ ]‪ a − b + c ≡ 7 − 3 + 4 [10‬ﻭﻤﻨﻪ ]‪. a − b + c ≡ 8 [10‬‬
‫ـ ـ ]‪ a + b − c ≡ 7 + 3 − 4 [10‬ﻭﻤﻨﻪ ]‪. a + b − c ≡ 6 [10‬‬
‫د ـ ]‪ abc ≡ 7 × 3 × 4 [10‬ﻭﻤﻨﻪ ]‪. abc ≡ 4 [10‬‬
‫هـ ـ ]‪ab + ac + bc ≡ 7 × 3 + 7 × 4 + 4 × 3[10‬‬
‫]‪. ab + ac + bc ≡ 1[10‬‬
‫و ـ ]‪ a 2 + b 2 + c 2 ≡ 49 + 9 + 16 [10‬ﻭﻤﻨﻪ ]‪. a 2 + b 2 + c 2 ≡ 4 [10‬‬
‫‪ 11‬ﺍﻝﺴﺎﻋﺔ ﺍﻝﻤﻁﻠﻭﺒﺔ ﻫﻲ ‪ n‬ﺤﻴﺙ ‪0 ≤ n < 24‬‬
‫ﺃ ـ ]‪ n ≡ 3 + 112 [ 24‬ﻭﻤﻨﻪ ]‪ n ≡ 115 [ 24‬ﺃﻱ ]‪ n ≡ 19 [ 24‬ﺇﺫﻥ ﺍﻝﺴﺎﻋﺔ ﻜﺎﻨﺕ ﺘﺸﻴﺭ ﺇﻝﻰ ‪ 19‬ﺃﻱ ﺍﻝﺴﺎﺒﻌﺔ ﻤﺴﺎﺀ ‪.‬‬
‫ﺏ ـ ]‪ n ≡ 3 − 163[ 24‬ﻭﻤﻨﻪ ]‪ n ≡ −160 [ 24‬ﺃﻱ ]‪ n ≡ 8 [ 24‬ﺇﺫﻥ ﺍﻝﺴﺎﻋﺔ ﻜﺎﻨﺕ ﺘﺸﻴﺭ ﺇﻝﻰ ﺍﻝﺜﺎﻤﻨﺔ ﺼﺒﺎﺤﺎ ‪.‬‬
‫‪ 12‬ﺃ ـ ]‪ 15123 ≡ 3[5‬ﻭﻤﻨﻪ ﺍﻝﻨﻘﻁﺔ ‪ M‬ﺘﺼل ﺇﻝﻰ ﺍﻝﻨﻘﻁﺔ ‪. D‬‬
‫ﺏ ـ ]‪ −15132 ≡ 3[5‬ﻭﻤﻨﻪ ﺍﻝﻨﻘﻁﺔ ‪ M‬ﺘﺼل ﻜﺫﻝﻙ ﺇﻝﻰ ﺍﻝﻨﻘﻁﺔ ‪. D‬‬
‫‪ 12 ≡ 2 [5] 13‬ﻭﻤﻨﻪ ]‪ 124 ≡ 24 [ 5‬ﺃﻱ ]‪124 ≡ 16 [5‬‬
‫]‪ 16 ≡ 1[5‬ﻭﻤﻨﻪ ]‪1527 = 4 × 381 + 3 . 124 ≡ 1[5‬‬
‫) ‪ 121527 = 12 4×381+3 = (12 4‬ﻭﻤﻨﻪ ]‪ 121527 ≡ 1381 × 23 [5‬ﺃﻱ ]‪. 121527 ≡ 3[5‬‬
‫‪381‬‬
‫ﻝﺩﻴﻨﺎ ‪× 123‬‬
‫‪ 371 ≡ 1[5] 14‬ﻭﻤﻨﻪ ]‪. 371238 ≡ 1[5‬‬
‫]‪ 579 ≡ −1[5‬ﻭﻤﻨﻪ ]‪. 5792008 ≡ 1[5‬‬
‫]‪ 1429 ≡ −1[5‬ﻭﻤﻨﻪ ]‪ 14292009 ≡ −1[5‬ﺒﻤﺎ ﺃﻥ ]‪ −1 ≡ 4 [5‬ﻓﺈﻥ ]‪. 14292009 ≡ 4 [5‬‬
‫]‪ 1954 ≡ −1[5‬ﻭﻤﻨﻪ ]‪. 19541962 ≡ 1[5‬‬
‫‪ 1754 ≡ −1[9] # 15‬ﻭﻤﻨﻪ ]‪. 175412 ≡ 1[9‬‬
‫‪ 34572 ≡ 3[9] #‬ﻭﻤﻨﻪ ]‪ 34572457 ≡ 3457 [9‬ﻭﻝﺩﻴﻨﺎ‬
‫‪ 3457 = 3 × 3456 = 3 × 9228‬ﺇﺫﻥ ]‪ 3457 ≡ 0 [9‬ﻭﺒﺎﻝﺘﺎﻝﻲ ]‪. 34572457 ≡ 0 [9‬‬
‫)‪ ( −3‬ﻭﻤﻨﻪ‬ ‫)‪= −3 × ( −3‬‬ ‫)‪ 3752009 ≡ ( −3‬ﻭﻝﺩﻴﻨﺎ ‪= −3 × 91004‬‬ ‫‪ 375 ≡ −3[9] #‬ﻭﻤﻨﻪ ]‪[9‬‬
‫‪2009‬‬ ‫‪2×1004‬‬ ‫‪2009‬‬

‫)‪ ( −3‬ﺇﺫﻥ ]‪. 3752009 ≡ 0 [9‬‬ ‫]‪≡ 0 [9‬‬


‫‪2009‬‬

‫ﺇﺫﻥ ]‪. 42003 + 12003 ≡ 0 [5‬‬ ‫‪ 16‬أ ـ ]‪ 4 ≡ −1[5‬ﻭﻤﻨﻪ ]‪42003 ≡ −12003 [5‬‬
‫]‪ 3 ≡ −2 [5‬ﻭﻤﻨﻪ ]‪32003 ≡ −22003 [5‬‬
‫ﺇﺫﻥ ]‪. 32003 + 22003 ≡ 0 [5‬‬
‫ﻭﺒﺎﻝﺘﺎﻝﻲ ]‪12003 + 22003 + 32003 + 42003 ≡ 0 [ 5‬‬
‫ب ـ ] ‪ 6 ≡ −1[ 7‬ﻭﻤﻨﻪ ] ‪ 62007 ≡ −12007 [ 7‬ﺇﺫﻥ ] ‪. 62007 + 12007 ≡ 0 [ 7‬‬
‫]‪ 5 ≡ −2 [ 7‬ﻭﻤﻨﻪ ]‪ 52007 ≡ −22007 [ 7‬ﺇﺫﻥ ] ‪ 52007 + 22007 ≡ 0 [ 7‬؛ ] ‪ 4 ≡ −3[ 7‬ﻭﻤﻨﻪ ] ‪ 42007 ≡ −32007 [ 7‬ﺇﺫﻥ‬
‫]‪ 42007 + 32007 ≡ 0 [ 7‬؛ ﻭﺒﺎﻝﺘﺎﻝﻲ ]‪. 12007 + 22007 + 32007 + 42007 + 52007 + 62007 ≡ 0 [ 7‬‬
‫ـ ـ ﻝﺩﻴﻨﺎ ]‪ 1 ≡ −8 [9‬؛ ]‪ 7 ≡ −2 [9‬؛ ]‪ 3 ≡ −6 [9‬؛ ]‪. 5 ≡ −4 [9‬‬
‫ﻭﻤﻨﻪ ]‪ 12008 ≡ 82008 [9‬؛ ]‪ 72008 ≡ 22008 [9‬؛ ]‪ 32008 ≡ 62008 [ 9‬؛ ]‪. 52008 ≡ 42008 [9‬‬
‫ﺇﺫﻥ ]‪ 12008 − 82008 ≡ [9‬؛ ]‪ 72008 − 22008 ≡ [9‬؛ ]‪ 32008 − 62008 ≡ [ 9‬؛ ]‪. 52008 − 42008 ≡ [9‬‬
‫‪12008 − 2 2008 + 32008 − 4 2008‬‬ ‫ﻭﺒﺎﻝﺘﺎﻝﻲ ‪:‬‬
‫]‪+52008 − 62008 + 7 2008 − 82008 ≡ 0 [9‬‬
‫‪ 17‬ﻤﻥ ﺃﺠل ﻜل ﻋﺩﺩ ﻁﺒﻴﻌﻲ ‪ 42 n +1 ≡ 0 [ 4] ، n‬؛ ‪ 22 n +1 = 2 × 4n‬ﻭﻤﻨﻪ ]‪ 22 n +1 ≡ 0 [ 4‬؛ ]‪ 3 ≡ −1[ 4‬ﻭﻤﻨﻪ‬
‫]‪ 32 n +1 ≡ −12 n +1 [ 4‬ﺃﻱ ]‪ 12 n +1 + 32 n +1 ≡ 0 [ 4‬؛‬
‫ﻭﺒﺎﻝﺘﺎﻝﻲ ]‪. 12 n +1 + 22 n +1 + 32 n +1 + 42 n +1 ≡ 0 [ 4‬‬
‫‪ 18‬ﻤﺠﻤﻭﻉ ﺃﺭﻗﺎﻡ ﺍﻝﻌﺩﺩ ‪ 7254‬ﻫﻭ ‪ 18‬ﻭﻫﻭ ﻤﻀﺎﻋﻑ ِـ ‪ 9‬ﺇﺫﻥ ]‪ 7254 ≡ 0 [9‬ﻭﻤﻨﻪ ]‪. 7254n ≡ 0 [9‬‬
‫ﺍﻝﻌﺩﺩ ‪ 3532‬ﺯﻭﺠﻲ ﺇﺫﻥ ]‪ 3532 ≡ 0 [ 2‬ﻭﻤﻨﻪ ]‪. 3532n ≡ 0 [ 2‬‬
‫]‪ 1785 ≡ 0 [5‬ﻭﻤﻨﻪ ]‪. 1785n ≡ 0 [5‬‬
‫]‪ 51502 ≡ 0 [11‬ﻭﻤﻨﻪ ]‪. 51502n ≡ 0 [11‬‬
‫‪ 3286 ≡ 6 [10] (1 19‬ﻭﻤﻨﻪ ]‪ 3286374 ≡ 6374 [10‬ﻭﻝﺩﻴﻨﺎ ﻤﻥ ﺃﺠل ﻜل ∗‪6n ≡ 6 [10] n ∈ ℕ‬‬
‫ﺇﺫﻥ ]‪ 6374 ≡ 6 [10‬ﻭﺒﺎﻝﺘﺎﻝﻲ ]‪. 3286374 ≡ 6 [10‬‬
‫‪ 76 ≡ 4 [12] (2‬ﻭﻝﺩﻴﻨﺎ ﻤﻥ ﺃﺠل ﻜل ∗‪4n ≡ 4 [12] ، n ∈ ℕ‬‬
‫ﺇﺫﻥ ]‪ 4784 ≡ 4 [12‬ﻭﺒﺎﻝﺘﺎﻝﻲ ]‪. 76784 ≡ 4 [12‬‬
‫‪ (1 20‬ﻝﻴﻜﻥ ‪ n‬ﻋﺩﺩﺍ ﻁﺒﻴﻌﻴﺎ ‪ 32 n = 9n ,‬ﻭ ]‪ 9 ≡ 2 [ 7‬ﺇﺫﻥ ] ‪ 9n ≡ 2n [ 7‬ﻭﻤﻨﻪ ] ‪. 32 n ≡ 2n [ 7‬‬
‫ﻭﺒﺎﻝﺘﺎﻝﻲ ﻤﻥ ﺃﺠل ﻜل ﻋﺩﺩ ﻁﺒﻴﻌﻲ ‪. 32 n − 2n ≡ 0 [ 7] ,‬‬
‫‪ (2‬ﺒﻭﺍﻗﻲ ﻗﺴﻤﺔ ﺍﻝﻌﺩﺩ ‪ n‬ﻋﻠﻰ ‪ 3‬ﻫﻲ ‪ 1 , 0‬ﻭ ‪ 2‬ﻭﺒﻔﺭﺽ ‪ n‬ﻝﻴﺱ ﻤﻀﺎﻋﻔﺎ ﻝِـ ‪ 3‬ﻓﻴﻜﻭﻥ ‪ n = 3 p + 1‬ﺃﻭ ‪ n = 3 p + 2‬ﻤﻊ‬
‫‪. p ∈ℕ‬‬
‫ﺇﺫﺍ ﻜﺎﻥ ‪22 n + 2n + 1 = 26 p + 2 + 23 p +1 + 1 , n = 3 p + 1‬‬
‫‪ . 22 n + 2n + 1 = 4 × 82 p + 2 × 8 p + 1‬ﻝﺩﻴﻨﺎ ]‪ 8 ≡ 1[ 7‬ﻭﻤﻨﻪ ﻤﻥ ﺃﺠل ﻜل ‪ 8 p ≡ 1[ 7] , p ∈ ℕ‬ﻭ ]‪. 82 p ≡ 1[ 7‬‬
‫ﻭﺒﺎﻝﺘﺎﻝﻲ ] ‪ 22 n + 2n + 1 ≡ 7 [ 7‬ﺃﻱ ]‪22 n + 2n + 1 ≡ 0 [ 7‬‬
‫ﺇﺫﺍ ﻜﺎﻥ ‪22 n + 2n + 1 = 26 p + 4 + 23 p + 2 + 1 , n = 3 p + 2‬‬
‫‪ . 22 n + 2n + 1 = 2 × 82 p +1 + 4 × 8 p + 1‬ﻝﺩﻴﻨﺎ ﻤﻥ ﺃﺠل ﻜل ‪ 8 p ≡ 1[ 7] , p ∈ ℕ‬ﻭ ]‪. 82 p +1 ≡ 1[ 7‬‬
‫ﻭﺒﺎﻝﺘﺎﻝﻲ ] ‪ 22 n + 2n + 1 ≡ 7 [ 7‬ﺃﻱ ]‪22 n + 2n + 1 ≡ 0 [ 7‬‬
‫‪ 21‬ﻝﻴﻜﻥ ‪ n‬ﻋﺩﺩﺍ ﻁﺒﻴﻌﻴﺎ ‪.‬‬
‫‪ 33 = 27 (1‬ﻭﻤﻨﻪ ]‪ 33 ≡ 2 [5‬ﺇﺫﻥ ]‪ 33 n ≡ 2n [5‬ﻭﻤﻨﻪ ]‪ 33 n + 2 ≡ 9 × 2n [5‬ﺇﺫﻥ ]‪33 n + 2 ≡ 4 × 2n [5‬‬
‫‪ 2n + 4 = 16 × 2 n‬؛ ]‪ 16 ≡ 1[5‬ﺇﺫﻥ ]‪. 2n + 4 ≡ 2n [ 5‬‬
‫]‪ 33 n + 2 + 2n + 4 ≡ 4 × 2n + 2n [5‬ﺃﻱ ]‪ 33 n + 2 + 2n + 4 ≡ 5 × 2n [ 5‬ﻭﻤﻨﻪ ]‪. 33 n + 2 + 2n + 4 ≡ 0 [5‬‬
‫‪ 33 n ≡ 2n [5] (2‬ﻭﻤﻨﻪ ]‪3 × 33n ≡ 3 × 2n [5‬‬
‫‪ 2n +1 = 2 × 2n‬ﺇﺫﻥ ]‪33 n +1 + 2n +1 ≡ 3 × 2n + 2 × 2n [5‬‬
‫ﺃﻱ ]‪ 33 n +1 + 2n +1 ≡ 5 × 2n [5‬ﻭﻤﻨﻪ ]‪33 n +1 + 2n +1 ≡ 0 [5‬‬
‫‪ 10 ≡ 1[9] 22‬ﻭ ]‪ 9 ≡ 0 [9‬ﻭﻤﻨﻪ ﻤﻥ ﺃﺠل ﻜل ‪ 10n ≡ 1[ 9] ، n ∈ ℕ‬ﻭ ]‪9n ≡ 0 [9‬‬
‫ﺇﺫﻥ ]‪ ( 9n − 1)10n + 1 ≡ −1×1 + 1[9‬ﺃﻱ ]‪. α ≡ 0 [9‬‬
‫‪ 23‬ﻝﻴﻜﻥ ‪ n‬ﻋﺩﺩﺍ ﻁﺒﻴﻌﻴﺎ ‪.‬‬
‫‪ 26 = 64 (1‬ﻭ ]‪ 64 ≡ 13 [17‬ﺇﺫﻥ ] ‪ 26 ≡ 13[17‬ﻭﻤﻨﻪ ]‪ 26 n ≡ 13n [17‬ﻭﺒﺎﻝﺘﺎﻝﻲ ] ‪26 n +3 ≡ 8 ×13n [17‬‬
‫‪ 34 = 81‬ﻭ ] ‪ 81 ≡ 13[17‬ﺇﺫﻥ ] ‪ 34 ≡ 13[17‬ﻭﻤﻨﻪ ]‪ 34 n ≡ 13n [17‬ﻭﺒﺎﻝﺘﺎﻝﻲ ] ‪. 34 n + 2 ≡ 9 ×13n [17‬‬
‫] ‪ 26 n + 3 + 34 n + 2 ≡ 8 ×13n + 9 ×13n [17‬ﻭﻤﻨﻪ ] ‪ 26 n +3 + 34 n + 2 ≡ 17 ×13n [17‬ﺇﺫﻥ ] ‪. 26 n +3 + 34 n + 2 ≡ 0 [17‬‬
‫‪ 25 n = ( 25 ) = 32n (2‬ﻭ ]‪ 32 ≡ 3[ 29‬ﺇﺫﻥ ]‪ 25 n ≡ 3n [ 29‬ﻭﻤﻨﻪ ]‪. 25 n +1 ≡ 2 × 3n [ 29‬‬
‫‪n‬‬

‫‪ 3n +3 = 27 × 3n‬ﺇﺫﻥ ]‪ 25 n +1 + 3n +3 ≡ 2 × 3n + 27 × 3n [ 29‬ﻭﻤﻨﻪ ]‪ 25 n +1 + 3n +3 ≡ 29 × 3n [ 29‬ﺇﺫﻥ‬


‫]‪. 25 n +1 + 3n +3 ≡ 0 [ 29‬‬
‫‪ (1 25‬ﺇﺫﺍ ﻜﺎﻥ ‪ n‬ﻓﺭﺩﻴﺎ ﻓﺈﻥ ﺍﻝﺒﻭﺍﻗﻲ ﺍﻝﻤﻤﻜﻨﺔ ﻝﻘﺴﻤﺘﻪ ﻋﻠﻰ ‪ 16‬ﻫﻲ ﺍﻷﻋﺩﺍﺩ ﺍﻝﻁﺒﻴﻌﻴﺔ ﺍﻝﻔﺭﺩﻴﺔ ﺍﻷﺼﻐﺭ ﺘﻤﺎﻤﺎ ﻤﻥ ‪. 16‬‬
‫ﺇﺫﺍ ﻜﺎﻥ ]‪ n ≡ 1[16‬ﻓﺈﻥ ]‪. n 4 ≡ 1[16‬‬
‫ﺇﺫﺍ ﻜﺎﻥ ]‪ n ≡ 3[16‬ﻓﺈﻥ ]‪ n 4 ≡ 34 [16‬ﻭﻤﻨﻪ ]‪. n 4 ≡ 1[16‬‬
‫ﺇﺫﺍ ﻜﺎﻥ ]‪ n ≡ 5 [16‬ﻓﺈﻥ ]‪ n 2 ≡ ( −9 ) [16‬ﻭﻤﻨﻪ ]‪ n 2 ≡ 1[16‬ﻭﺒﺎﻝﺘﺎﻝﻲ ]‪. n 4 ≡ 1[16‬‬
‫‪2‬‬

‫ﺇﺫﺍ ﻜﺎﻥ ]‪ n ≡ 7 [16‬ﻓﺈﻥ ]‪ n 2 ≡ 7 2 [16‬ﺃﻱ ]‪ n 2 ≡ 1[16‬ﻭﺒﺎﻝﺘﺎﻝﻲ ]‪. n 4 ≡ 1[16‬‬


‫ﺇﺫﺍ ﻜﺎﻥ ]‪ n ≡ 9 [16‬ﻓﺈﻥ ]‪ n 2 ≡ 1[16‬ﻭﻤﻨﻪ ]‪. n 4 ≡ 1[16‬‬
‫ﺇﺫﺍ ﻜﺎﻥ ]‪ n ≡ 11[16‬ﻓﺈﻥ ]‪ n 2 ≡ ( −5 ) [16‬ﻭﻤﻨﻪ ]‪ n 2 ≡ 9 [16‬ﻭﺒﺎﻝﺘﺎﻝﻲ ]‪ n 4 ≡ 92 [16‬ﺇﺫﻥ ]‪. n 2 ≡ 1[16‬‬
‫‪2‬‬

‫ﺇﺫﺍ ﻜﺎﻥ ]‪ n ≡ 13[16‬ﻓﺈﻥ ]‪ n ≡ −3[16‬ﻭﻤﻨﻪ ]‪ n 4 ≡ ( −3) [16‬ﻭﻤﻨﻪ ]‪n 4 ≡ 1[16‬‬


‫‪4‬‬

‫ﺇﺫﺍ ﻜﺎﻥ ]‪ n ≡ 15 [16‬ﻓﺈﻥ ]‪ n ≡ −1[16‬ﻭﻤﻨﻪ ]‪n 4 ≡ 1[16‬‬


‫‪ (2‬ﺒﻭﺍﻗﻲ ﻗﺴﻤﺔ ‪ n‬ﻋﻠﻰ ‪ 5‬ﻫﻲ ‪ 3 , 2 , 1 , 0‬ﻭ ‪4‬‬
‫ﻝﻴﻜﻥ ‪ r‬ﻴﻨﺘﻤﻲ ﺇﻝﻰ }‪ {1, 2,3, 4‬ﺒﻭﻀﻊ ]‪ n ≡ r [5‬ﻤﻌﻨﺎﻩ ﺃﻥ ‪ n‬ﻝﻴﺱ ﻤﻀﺎﻋﻔﺎ ﻝﻠﻌﺩﺩ ‪ 5‬ﻭﺒﺎﻝﺘﺎﻝﻲ ﻴﻜﻭﻥ ]‪n ≡ r [5‬‬
‫‪4‬‬ ‫‪4‬‬

‫ﻭﻝﺩﻴﻨﺎ ]‪44 ≡ 1[5] , 34 ≡ 1[5] , 24 ≡ 1[5] , 14 ≡ 1[5‬‬


‫ﺇﺫﻥ ﻤﻥ ﺃﺠل ﻜل }‪ r 4 ≡ 1[5] r ∈ {1, 2,3, 4‬ﻭﺒﺎﻝﺘﺎﻝﻲ ]‪. n 4 ≡ 1[5‬‬
‫≡ ‪x‬‬ ‫أ ـ ]‪0 1 2 3 4 [ 5‬‬ ‫‪26‬‬
‫] [ ‪2x ≡ 0 2 4 1 3‬‬ ‫‪5‬‬

‫ب ـ ]‪ 2x ≡ 3[ 5‬ﻤﻌﻨﺎﻩ ]‪. x ≡ 4 [5‬‬


‫≡‪n‬‬ ‫]‪0 1 2 3 4 5 6 [7‬‬ ‫‪27‬‬
‫≡ ‪n3‬‬ ‫]‪0 1 1 6 1 6 6 [7‬‬
‫‪n3 + n − 2 ≡ 5 5 1 0 3 2 5‬‬
‫] ‪ n 3 + n − 2 ≡ 0 [ 7‬ﻤﻌﻨﺎﻩ ]‪. n ≡ 3[ 7‬‬
‫‪n‬‬ ‫‪0 1 2 3 4 5 6 (1 28‬‬
‫‪rn‬‬ ‫‪1 2 4 8 7 5 1‬‬
‫‪ (2‬ﻝﺩﻴﻨﺎ ]‪ 26 ≡ 1[9‬ﻭﻤﻨﻪ ﻤﻥ ﺃﺠل ﻜل ﻋﺩﺩ ﻁﺒﻴﻌﻲ ‪. 26 p ≡ 1[9] ، p‬‬
‫ﻭﻤﻨﻪ ]‪ 26 p + k ≡ rk [9‬ﻤﻊ }‪k ∈ {1, 2,3, 4,5‬‬
‫ﻭﻤﻨﻪ ﺇﺫﺍ ﻜﺎﻥ ‪ n = 6 p‬ﻓﺈﻥ ‪. rn = r0 = 1‬‬
‫ﺇﺫﺍ ﻜﺎﻥ ‪ n = 6 p + 1‬ﻓﺈﻥ ‪. rn = r1 = 2‬‬
‫ﺇﺫﺍ ﻜﺎﻥ ‪ n = 6 p + 2‬ﻓﺈﻥ ‪. rn = r2 = 4‬‬
‫ﺇﺫﺍ ﻜﺎﻥ ‪ n = 6 p + 3‬ﻓﺈﻥ ‪. rn = r3 = 8‬‬
‫ﺇﺫﺍ ﻜﺎﻥ ‪ n = 6 p + 4‬ﻓﺈﻥ ‪. rn = r4 = 7‬‬
‫ﺇﺫﺍ ﻜﺎﻥ ‪ n = 6 p + 5‬ﻓﺈﻥ ‪. rn = r5 = 5‬‬
‫‪ 65 ≡ 2 [9] (3‬ﻭﻤﻨﻪ ﻤﻥ ﺃﺠل ﻜل ﻋﺩﺩ ﻁﺒﻴﻌﻲ ‪ 65n ≡ 2n [9] , n‬ﻭﻝﺩﻴﻨﺎ ]‪ 2n ≡ rn [9‬ﺇﺫﻥ ]‪. 65n ≡ rn [9‬‬
‫‪ 2011 = 6 × 335 + 1‬ﻭﻤﻨﻪ ‪ r2011 = r1 = 2‬ﺇﺫﻥ ]‪652011 ≡ 2 [9‬‬
‫‪ 29‬أ ـ ]‪. 45 ≡ 1[11‬‬
‫ب ـ ]‪ 37 ≡ 4 [11‬ﻭﻤﻨﻪ ]‪ 375 ≡ 45 [11‬ﺇﺫﻥ ]‪ 375 ≡ 1[11‬ﻭﻤﻨﻪ ﻤﻥ ﺃﺠل ﻜل ﻋﺩﺩ ‪ 375 k ≡ 1[11] ، k ∈ ℕ‬؛‬
‫]‪ 375 k +1 ≡ 4 [11‬؛ ]‪ 375 k + 2 ≡ 5 [11‬؛ ]‪ 375 k +3 ≡ 9 [11‬ﻭ ]‪. 375 k + 4 ≡ 3[11‬‬
‫‪ 2x = 3 y 30‬ﻭﻤﻨﻪ ]‪ 2x ≡ 0 [3‬ﺃﻱ ]‪ 4x ≡ 0 [3‬ﻭﻫﺫﺍ ﻤﻌﻨﺎﻩ ]‪ x ≡ 0 [3‬ﺃﻱ ‪ x = 3k‬ﻤﻊ ‪. k ∈ ℤ‬‬
‫ﻭﺒﺎﻝﺘﻌﻭﻴﺽ ﻨﺠﺩ ‪ 2 ( 3k ) = 3 y‬ﺃﻱ ‪ y = 2k‬ﻭﻤﻨﻪ ) ‪ ( x , y ) = ( 3k , 2k‬ﻤﻊ ‪. k ∈ ℤ‬‬
‫‪ 2x − 5 y = 1 31‬ﻤﻌﻨﺎﻩ ‪ 2x = 5 y + 1‬ﺇﺫﻥ ]‪ 2x ≡ 1[ 5‬ﻭﻫﺫﺍ ﻤﻌﻨﺎﻩ ]‪ 6x ≡ 3[5‬ﺃﻱ ]‪ x ≡ 3 [5‬ﺇﺫﻥ ‪x = 5k + 3‬‬
‫ﻤﻊ ‪ k ∈ ℤ‬ﻭﺒﺎﻝﺘﻌﻭﻴﺽ ﻨﺠﺩ ‪ 10k + 6 = 5 y + 1‬ﺃﻱ ‪ 5 y = 10k + 5‬ﺃﻱ ‪y = 2k + 1‬‬
‫ﻭﻤﻨﻪ )‪ ( x , y ) = ( 5k + 3, 2k + 1‬ﻤﻊ ‪. k ∈ ℤ‬‬
‫‪x = 5α + 3‬‬ ‫]‪x ≡ 3 [5‬‬
‫‪ ‬ﺇﺫﻥ ‪ 5α − 6 β = −2‬ﻭﻤﻨﻪ ‪ 5α = 6 β − 2‬ﻭﻤﻨﻪ ]‪ 5α ≡ −2 [ 6‬ﻭﻫﺫﺍ ﻴﻌﻨﻲ‬ ‫‪ ‬ﻤﻌﻨﺎﻩ‬ ‫‪ 32‬أ ـ‬
‫‪ x = 6β + 1‬‬ ‫]‪ x ≡ 1[ 6‬‬
‫]‪ −α ≡ −2 [ 6‬ﺃﻱ ]‪ α ≡ 2 [ 6‬ﻭﻴﻌﻨﻲ ‪ α = 6k + 2‬ﻤﻊ ‪ k ∈ ℤ‬ﻭﺒﺎﻝﺘﻌﻭﻴﺽ ﻨﺠﺩ ‪5 ( 6k + 2 ) = 6β − 2‬‬
‫ﺃﻱ ‪ 6 β = 30k + 12‬ﻭﻤﻌﻨﺎﻩ ‪ . β = 5k + 2‬ﺇﺫﻥ ‪ x = 5α + 3 = 30k + 13‬ﻤﻊ ‪. k ∈ ℤ‬‬
‫]‪ 3x ≡ 3 [ 6‬‬ ‫]‪x ≡ 1[ 2‬‬ ‫] ‪2x ≡ 2 [ 4‬‬
‫‪ ‬ﻭﻤﻨﻪ ]‪. x ≡ 1[ 6‬‬ ‫‪ ‬ﻤﻌﻨﺎﻩ‬ ‫‪ ‬ﻤﻌﻨﺎﻩ‬ ‫بـ‬
‫]‪2x ≡ 2 [ 6‬‬ ‫]‪ x ≡ 1[3‬‬ ‫]‪ 4x ≡ 1[3‬‬

‫‪ 2‬ـ ا
"!اد‬

‫‪a = 12734 33‬‬


‫‪a = 10 + 2 × 10 + 7 × 10 2 + 3 × 10 + 4‬‬
‫‪4‬‬ ‫‪3‬‬

‫‪ b = 5723‬؛ ‪b = 5 ×103 + 7 ×10 2 + 2 × 10 + 3‬‬


‫‪ c = 503019‬؛ ‪. c = 5 × 105 + 3 + ×103 + 10 + 9‬‬
‫‪ a = 234 = 2 × 62 + 3 × 6 + 4 34‬؛ ‪ b = 1523 = 64 + 5 × 63 + 2 × 6 + 3‬؛‬
‫‪. c = 503012 = 5 × 65 + 3 × 63 + 6 + 2‬‬
‫‪ b = 5 × 7 2 + 2 × 7 = 520 . a = 73 + 2 × 7 2 + 3 × 7 + 5 = 1235 35‬؛ ‪. c = 6 × 73 + 2 × 7 + 1 = 6021‬‬
‫‪. N a = 4a 5 + 2a 3 + a + 3 = 40213 36‬‬
‫‪ 37‬ﺃ ـ ‪ x ≥ 7‬ﻭﻤﻨﻪ ﺃﺼﻐﺭ ﻗﻴﻤﺔ ﻫﻲ ‪. x = 7‬‬
‫ﺏ ـ ‪ 2306 = 2 × 73 + 3 × 7 2 + 0 × 7 + 6‬ﻭ ‪1035 = 73 + 0 × 7 2 + 3 × 7 + 5‬‬
‫‪، 7 = 1× 22 + 1× 2 + 1 = 111 ، 4 = 1× 22 + 0 × 2 + 0 = 100 ، 2 = 1× 2 + 0 = 10 38‬‬
‫‪. 33 = 1× 25 + 1 = 10001‬‬
‫‪ n = 26 + 25 + 23 + 2 2 + 1 = 109 39‬؛ ‪. n = 2x 2 + x + 4‬‬
‫‪15‬‬
‫‪ . x = −‬ﺇﺫﻥ ﺍﻷﺴﺎﺱ ‪. 7‬‬ ‫ﺇﺫﻥ ‪ 2x 2 + x + 4 = 109‬ﻤﻌﻨﺎﻩ ‪ 2x 2 + x − 105 = 0‬ﻭﻤﻌﻨﺎﻩ ‪، x = 7‬‬
‫‪2‬‬
‫‪ 2003 = 21× 43 40‬ﻤﻌﻨﺎﻩ ‪ x ≥ 5‬ﻭ )‪ 2x 3 + 3 = ( 2x + 1)( 4x + 3‬ﺃﻱ ‪ x ≥ 5‬ﻭ ‪2x − 8x − 10x = 0‬‬
‫‪3‬‬ ‫‪2‬‬

‫ﻭﻤﻌﻨﺎﻩ ‪ 2x 2 − 8x − 10 = 0‬ﺃﻱ ‪. x = 10‬‬


‫‪ 41‬أ ـ ‪ 411 = 15 × 23‬ﻤﻌﻨﺎﻩ )‪4x 2 + x + 1 = ( x + 5)( 2x + 3‬‬
‫ﺃﻱ ‪ 2x 2 − 12x − 14 = 0‬ﻭﻤﻌﻨﺎﻩ ‪ x 2 − 6x − 7 = 0‬ﺃﻱ ‪ x = −1‬ﺃﻭ ‪ x = 7‬ﺇﺫﻥ ﺍﻷﺴﺎﺱ ﻫﻭ ‪. x = 7‬‬
‫ب ـ ‪ 21×14 = 324‬ﻤﻌﻨﺎﻩ ‪ ( 2a + 1)( a + 4 ) = 3a 2 + 2a + 4‬ﻭﻤﻌﻨﺎﻩ ‪ a 2 − 7a = 0‬ﺇﺫﻥ ‪. a = 7‬‬
‫ـ ـ ‪ 2888 = 412 × 31‬ﻤﻌﻨﺎﻩ )‪ 2888 = ( 4x 2 + x + 2 ) ( 3x + 1‬ﻭﻤﻌﻨﺎﻩ ‪12x 3 + 7 x 2 + 7 x = 2886‬‬
‫ﻝﺩﻴﻨﺎ ‪ x ≥ 5‬ﺇﺫﺍ ﻜﺎﻥ ‪ x = 5‬ﻓﺈﻥ ]‪ 12x 3 + 7x 2 + 7x ≡ 0 [5‬ﺒﻴﻨﻤﺎ ]‪ 2886 ≡ 1[5‬ﺇﺫﻥ ‪. x ≠ 5‬‬
‫ﻭﻝﺩﻴﻨﺎ ‪ 12 × 63 + 7 × 62 + 7 × 6 = 2886 :‬ﺇﺫﻥ ﺍﻷﺴﺎﺱ ‪. x = 6‬‬
‫‪ 42‬أ ـ ‪ x 2 + 6x + 2 = 7 x + 7 + 6x + 3‬ﻤﻊ ‪ x > 7‬ﻤﻌﻨﺎﻩ ‪ x 2 − 7 x − 8 = 0‬ﻭﻤﻌﻨﺎﻩ ‪. x = 8‬‬
‫ب ـ ‪77 × 63 = ( 7 × 8 + 7 )( 6 × 8 + 3) = 3213‬‬
‫ـ ـ ‪3213 = 8 × 401 + 5 = 8 ( 8 ( 8 × 6 + 2 ) + 1) + 5‬‬
‫‪. 3213 = 6 × 83 + 2 × 82 + 8 + 5 = 6215‬‬
‫‪ 43‬أ ـ ‪ ( a + 2 )( 2a + 3) = 2a 2 + 7a + 6 . 12 × 23 = 276‬ﻭﻫﺫﺍ ﺼﺤﻴﺢ ﻤﻥ ﺃﺠل ﻜل ﻋﺩﺩ ﻁﺒﻴﻌﻲ ‪. a > 7‬‬
‫ب ـ ‪ 541 = 22 × 32‬ﻤﻌﻨﺎﻩ ) ‪ 5x 2 + 4x + 1 = ( 2x + 2 )( 3x + 2‬ﻭﻤﻌﻨﺎﻩ ‪ x 2 + 6x + 3 = 0‬ﺃﻱ ‪x = −3 + 6‬‬
‫ﺃﻭ ‪ x = −3 − 6‬ﺇﺫﻥ ﻻ ﻴﻭﺠﺩ ﺃﻱ ﺃﺴﺎﺱ ﻴﻜﺘﺏ ﻓﻴﻪ ‪541 = 22 × 32‬‬
‫‪ 10 = 1× 23 + 0 × 22 + 1× 2 + 0 = 1010 44‬؛ ‪ 100 = 64 + 32 + 4 = 26 + 25 + 22‬ﺃﻱ ‪100 = 1100101‬‬
‫(‬ ‫)‬
‫‪ 72881 = 12 12 (12 × (12 × 3 + 6 ) + 2 ) + 1 + 5 46‬ﻭﻤﻨﻪ ‪72881 = 3 × 124 + 6 × 123 + 2 × 122 + 1× 12 + 5‬‬
‫ﺇﺫﻥ ‪ 72881 = 36215‬ﻓﻲ ﺍﻷﺴﺎﺱ ‪12‬‬
‫‪ 75 < 72881 < 7 6‬ﻭﻝﺩﻴﻨﺎ ‪ 72881 = 4 × 75 + 2 × 7 4 + 2 × 73 + 3 × 7 2 + 2 × 7 + 4‬ﺃﻱ ‪72881 = 422324‬‬
‫‪. 3752 = 3 × 83 + 7 × 82 + 5 × 8 + 2 = 2026 47‬‬
‫‪ 6175 = 4523 ، 6175 = 6 × 93 + 1× 92 + 7 × 9 + 5 48‬ﻝﺩﻴﻨﺎ ‪ 123 < 4523 < 124 :‬ﻭﻤﻨﻪ‬
‫‪ 4523 = 2 × 123 + 7 × 122 + 4 × 12 + 11‬ﺇﺫﻥ ‪ 4523 = 274α‬ﺤﻴﺙ ‪. α = 11‬‬
‫‪ 234 = 2 × 52 + 3 × 5 + 4 49‬ﺃﻱ ‪ 234 = 2 × ( 7 − 2 ) + 3 × ( 7 − 2 ) + 4‬؛ ‪234 = 2 × 7 2 + 3 × 7 − 50‬‬
‫‪2‬‬

‫‪234 = 1× 7 2 + 2 × 7 + 6 = 126‬‬
‫‪ 1040 = 53 + 4 × 5 = ( 7 − 2 ) + 20‬؛ ‪ 1040 = 73 − 6 × 72 + 12 × 7 + 12‬؛ ‪1040 = 2 × 7 2 + 6 × 7 + 5 = 265‬‬
‫‪3‬‬

‫‪ a 2 = 1× a 2 + 0 × a + 0 = 100 ، a = 1× a + 0 = 10 50‬ﻭ ‪. a 3 = 1× a 3 + 0 × a 2 + 0 × a + 0 = 1000‬‬


‫‪ 51‬ﻨﻔﺭﺽ ﺃﻥ ‪ A‬ﻴﻜﺘﺏ ﻓﻲ ﺍﻝﻨﻅﺎﻡ ﺫﻱ ﺍﻷﺴﺎﺱ ﺍﻝﻌﺸﺭﻱ ﻜﻤﺎ ﻴﻠﻲ ‪ A = an an −1...a0‬ﺤﻴﺙ ‪. 0 ≤ ai ≤ 9‬‬
‫ﻭﻤﻨﻪ ‪ A = an 10n + an −110 n −1 + ... + a0‬ﻭﻝﺩﻴﻨﺎ ﻤﻥ ﺃﺠل ﻜل ‪ 10n ≡ 1[3] n ∈ ℕ‬ﻭﺒﺎﻝﺘﺎﻝﻲ‬
‫]‪ A ≡ an + an −1 + ... + a0 [3‬ﺃﻱ ]‪ A ≡ S [3‬ﺇﺫﻥ ]‪ A ≡ 0 [3‬ﻤﻌﻨﺎﻩ ]‪. S ≡ 0 [3‬‬
‫‪ 52‬ﻨﻀﻊ ‪ x = an an −1...a0‬؛ ‪ y = a0a1...an‬؛ ]‪ x ≡ an + an −1 + ... + a0 [9‬ﻭ ]‪y ≡ a0 + a1 + ... + an [9‬‬
‫ﻭﻤﻨﻪ ]‪. x − y ≡ 0 [9‬‬
‫‪+ 0 1‬‬ ‫‪2‬‬ ‫‪3‬‬ ‫‪(1 53‬‬
‫‪0 0 1‬‬ ‫‪2‬‬ ‫‪3‬‬
‫‪1 1 2‬‬ ‫‪3‬‬ ‫‪10‬‬
‫‪2 2 3‬‬ ‫‪10 11‬‬
‫‪3 3 10‬‬ ‫‪11 12‬‬
‫‪. 3 + 3 = 1× 4 + 2 = 12‬‬ ‫‪، 4 = 1× 4 + 0 = 10‬‬
‫‪111‬‬
‫‪3223‬‬
‫ﺇﺫﻥ ‪. 3223 + 132 = 10021‬‬ ‫‪+ 132‬‬ ‫‪(2‬‬
‫‪= 10 021‬‬
‫‪× 0 1‬‬ ‫‪2‬‬ ‫‪3‬‬ ‫‪(1 54‬‬
‫‪0 0 0‬‬ ‫‪0‬‬ ‫‪0‬‬
‫‪1 0 1‬‬ ‫‪2‬‬ ‫‪3‬‬
‫‪2 0 2‬‬ ‫‪10 12‬‬
‫‪3 0 3‬‬ ‫‪12 21‬‬
‫‪. 3 + 3 = 1× 4 + 2 = 12 ،‬‬ ‫‪3 × 3 = 2 × 4 + 1 = 21‬‬
‫‪2 2 2‬‬
‫‪3223‬‬
‫×‬ ‫‪123‬‬
‫‪23001‬‬
‫‪ 13112 .‬إذن ‪3223 ×123 = 1203021‬‬ ‫‪(2‬‬
‫‪3223. .‬‬
‫‪1203021‬‬
‫‪213‬‬
‫‪× 14‬‬ ‫‪431‬‬ ‫‪3421‬‬
‫‪55‬‬
‫‪. 1412 ، − 132 ، + 230‬‬
‫‪213.‬‬ ‫‪244‬‬ ‫‪4201‬‬
‫‪4042‬‬
‫‪27‬‬
‫‪× 41‬‬ ‫‪400 α‬‬ ‫‪39β 7 56‬‬
‫‪.‬‬ ‫‪27 ، − 39 β 7 ، + 213‬‬
‫‪104.‬‬ ‫‪213‬‬ ‫‪400α‬‬
‫‪1067‬‬
‫ﺘﻤﺎﺭﻴﻥ ﻝﻠﺘﻌﻤﻕ‬
‫‪ 1‬ـ ا
‪2‬ا\‪.‬ت \ ‪ℤ‬‬
‫‪ 57‬أ ـ ]‪. 2 ≡ 2 [10] ، 2 ≡ 6 [10] ، 2 ≡ 8 [10] ، 2 ≡ 4 [10] ، 2 ≡ 2 [10] ، 20 ≡ 1[10‬‬
‫‪5‬‬ ‫‪4‬‬ ‫‪3‬‬ ‫‪2‬‬

‫ﻝﺩﻴﻨﺎ ﻤﻥ ﺃﺠل ﻜل ∗‪) 6n ≡ 6 [10] ، n ∈ ℕ‬ﺒﺎﻝﺘﺭﺍﺠﻊ(‬


‫]‪ 24 ≡ 6 [10‬ﺇﺫﻥ ﻤﻥ ﺃﺠل ﻜل ‪24 p ≡ 6 p [10] ، p ∈ ℕ‬‬
‫ﺇﺫﻥ ]‪ 24 p + 4 ≡ 64 [10‬ﻭﻤﻨﻪ ]‪ 24 p + 4 ≡ 6 [10‬ﻭﻋﻠﻴﻪ ]‪ 24 p +1 ≡ 2 [10‬؛ ]‪ 24 p + 2 ≡ 4 [10‬؛ ]‪24 p +3 ≡ 8 [10‬‬
‫ب ـ ﻜل ﻋﺩﺩ ﻁﺒﻴﻌﻲ ﻴﻭﺍﻓﻕ ﺭﻗﻡ ﺁﺤﺎﺩﻩ ﺒﺘﺭﺩﻴﺩ ‪. 10‬‬
‫ﺇﺫﺍ ﻜﺎﻥ ‪ n = 0‬ﻓﺈﻥ ‪ 20 = 1‬ﻭﻫﻭ ﺭﻗﻡ ﺁﺤﺎﺩﻩ‬
‫ﺇﺫﺍ ﻜﺎﻥ ‪ n = 4k‬ﻤﻊ ∗‪ k ∈ ℕ‬ﻓﺈﻥ ﺭﻗﻡ ﺁﺤﺎﺩ ‪ 2n‬ﻫﻭ ‪. 6‬‬
‫ﺇﺫﺍ ﻜﺎﻥ ‪ n = 4k + 1‬ﻤﻊ ‪ k ∈ ℕ‬ﻓﺈﻥ ﺭﻗﻡ ﺁﺤﺎﺩ ‪ 2n‬ﻫﻭ ‪. 2‬‬
‫ﺇﺫﺍ ﻜﺎﻥ ‪ n = 4k + 2‬ﻤﻊ ‪ k ∈ ℕ‬ﻓﺈﻥ ﺭﻗﻡ ﺁﺤﺎﺩ ‪ 2n‬ﻫﻭ ‪. 4‬‬
‫ﺇﺫﺍ ﻜﺎﻥ ‪ n = 4k + 3‬ﻤﻊ ‪ k ∈ ℕ‬ﻓﺈﻥ ﺭﻗﻡ ﺁﺤﺎﺩ ‪ 2n‬ﻫﻭ ‪. 8‬‬
‫‪4‬ـ ـ ]‪35489 × 253431 ≡ 89 × 431 [10‬‬
‫ﺃﻱ ]‪. 35489 × 253431 ≡ 227 × 262 [10‬‬
‫]‪ 35489 × 253431 ≡ 289 [10‬ﻭﻝﺩﻴﻨﺎ ‪ 89 = 4 × 22 + 1‬؛‬
‫ﺇﺫﻥ ]‪ 289 ≡ 2 [10‬ﻭﻤﻨﻪ ]‪35489 × 253431 ≡ 2 [10‬‬
‫ﺇﺫﻥ ﺭﻗﻡ ﺁﺤﺎﺩ ‪ 35489 × 253431‬ﻫﻭ ‪. 2‬‬
‫) ‪ ( 512‬ﺃﻱ ]‪512008 ≡ 1[100‬‬ ‫‪ 512 = 2601 58‬ﻭﻤﻨﻪ ]‪ 512 = 1[100‬ﺇﺫﻥ ]‪= 1[100‬‬
‫‪1004‬‬

‫ﺇﺫﻥ ﺍﻝﺭﻗﻡ ﺍﻷﺨﻴﺭ ﻫﻭ ‪ 1‬ﻭﻤﺎ ﻗﺒﻠﻪ ‪. 0‬‬


‫‪ 59‬ﻝﻜل ﻋﺩﺩ ﺼﺤﻴﺢ ‪ a‬ﻝﺩﻴﻨﺎ ﺇﻤﺎ ]‪ a ≡ −1[3‬ﻭﺇﻤﺎ ]‪ a ≡ 0 [3‬ﻭﺇﻤﺎ ]‪. a ≡ 1[ 3‬‬
‫ﺇﺫﺍ ﻜﺎﻥ ]‪ x ≡ 0 [3‬ﺃﻭ ]‪ y ≡ 0 [3‬ﻓﺈﻥ ]‪xy ( x 2 − y 2 ) ≡ 0 [3‬‬
‫]‪ x 2 ≡ 1[ 3] x ≡ −1[3]  x ≡ 1[3‬‬ ‫]‪ x ≡ −1[3]  x ≡ 1[3‬‬
‫‪  2‬ﻭﻤﻨﻪ ]‪x − y ≡ 0 [3‬‬
‫‪2‬‬ ‫‪2‬‬
‫‪ ‬ﻓﺈﻥ‬ ‫‪ ‬ﺃﻭ‬ ‫‪ ‬ﺃﻭ‬ ‫‪ ‬ﺃﻭ‬ ‫ﺇﺫﺍ ﻜﺎﻥ‬
‫]‪ y ≡ 1[3]  y ≡ 1[3‬‬ ‫]‪ y ≡ −1[3]  y ≡ −1[3]  y ≡ 1[3‬‬
‫ﺇﺫﻥ ]‪. xy ( x 2 − y 2 ) ≡ 0 [3‬‬
‫‪n 3 + 3n 2 + 3n + 1 − 8 = ( n + 1) − 8 60‬‬
‫‪3‬‬

‫)‪( n + 1‬‬ ‫]‪ n 3 + 3n 2 + 3n − 7 ≡ 0 [8‬ﻤﻌﻨﺎﻩ ]‪≡ 0 [8‬‬


‫‪3‬‬

‫≡‪n‬‬ ‫‪0 1 2 3 4 5 6 7‬‬ ‫]‪[8‬‬


‫≡ ‪n +1‬‬ ‫‪1 2 3 4 5 6 7 0‬‬ ‫]‪[8‬‬
‫]‪[8‬‬
‫≡ )‪( n + 1‬‬ ‫‪1 0 3 0 5 0 7 0‬‬
‫‪3‬‬
‫ﺇﺫﻥ ]‪ n 3 + 3n 2 + 3n − 7 ≡ 0 [8‬ﻤﻌﻨﺎﻩ ]‪ n ≡ 1[8‬ﺃﻭ ]‪ n ≡ 3[8‬ﺃﻭ ]‪ n ≡ 5 [8‬ﺃﻭ ]‪. n ≡ 7 [8‬‬
‫‪ 61‬ﺃ ـ ]‪ 26 ≡ 1[9‬ﻭﻤﻨﻪ ﻤﻥ ﺃﺠل ﻜل ‪ 26 p ≡ 1[9] ، p ∈ ℕ‬ﺃﻱ ]‪ 26 p − 1 ≡ 0 [ 9‬ﻤﻌﻨﺎﻩ ‪. n = 6 p‬‬
‫) ‪ A = 2 n − 1 = ( 23‬ﺃﻱ ‪ A = 82 p − 1‬ﻭﻝﺩﻴﻨﺎ‬
‫‪2p‬‬
‫ﺏ ـ ﺇﺫﺍ ﻜﺎﻥ ‪ n = 6 p‬ﻓﺈﻥ ‪− 1‬‬
‫‪ 65‬ﻨﻀﻊ ) ‪N = ( n 2 − 1)( n 2 − 4‬‬
‫≡‪n‬‬ ‫]‪0 1 2 3 4 [5‬‬
‫≡ ‪n‬‬
‫‪2‬‬ ‫]‪0 1 4 4 1 [5‬‬
‫≡ ‪n 2 −1‬‬ ‫]‪[5‬‬
‫‪4 0 3 3 0‬‬
‫]‪n − 4 ≡ 1 2 0 0 2 [5‬‬
‫‪2‬‬

‫≡ ‪N‬‬ ‫]‪4 0 0 0 0 [5‬‬


‫ﻭﻤﻨﻪ ﺇﺫﺍ ﻜﺎﻥ ]‪ n ≡ 0 [5‬ﻓﺈﻥ ]‪N ≡ 0 [5‬‬
‫‪N = n ( 2n + 1)( 7n + 1) 66‬‬
‫≡‪n‬‬ ‫]‪0 1 2 3 4 5 [6‬‬
‫]‪2n + 1 ≡ 1 3 5 1 3 5 [6‬‬
‫]‪7 n + 1 ≡ 1 2 3 4 5 0 [6‬‬
‫≡ ‪N‬‬ ‫]‪0 0 0 0 0 0 [6‬‬
‫‪ 67‬أ ـ ‪A = n 2 − n + 1‬‬
‫≡‪n‬‬ ‫] ‪0 1 2 3 4 5 6 [7‬‬
‫≡ ‪n‬‬
‫‪2‬‬ ‫] ‪0 1 4 2 2 3 1 [7‬‬
‫‪A‬‬ ‫] ‪1 1 3 0 6 6 3 [7‬‬
‫ب ـ ] ‪ A ≡ 0 [ 7‬ﻤﻌﻨﺎﻩ ]‪. n ≡ 3[ 7‬‬
‫‪4‬ـ ـ ﻨﻀﻊ ‪ B = 27532 − 2753 + 1‬؛ ﻨﻌﺘﺒﺭ ‪ n = 2753‬ﻭﻤﻨﻪ ] ‪ n ≡ 2 [ 7‬ﺇﺫﻥ ]‪ A ≡ 3[ 7‬ﻭﺒﺎﻝﺘﺎﻝﻲ ] ‪. B ≡ 3[ 7‬‬

‫‪A = 2n 3 − n 2 + 2 68‬‬
‫≡‪n‬‬ ‫] ‪0 1 2 3 4 5 6 [7‬‬
‫≡ ‪n2‬‬ ‫] ‪0 1 4 2 2 4 1 [7‬‬
‫≡ ‪n‬‬
‫‪3‬‬ ‫] ‪0 1 1 6 1 6 6 [7‬‬
‫≡ ‪2n‬‬
‫‪3‬‬ ‫] ‪0 2 2 5 2 5 5 [7‬‬
‫‪A‬‬ ‫] ‪2 3 0 5 2 3 6 [7‬‬
‫]‪ 2n 3 − n 2 + 2 ≡ 0 [ 7‬ﻤﻌﻨﺎﻩ ] ‪n ≡ 2 [ 7‬‬
‫‪ 69‬أ ـ ] ‪ 43 ≡ 1[ 7‬ﻭﻤﻨﻪ ﻤﻥ ﺃﺠل ﻜل ‪ 43n ≡ 1[ 7 ] ، n ∈ ℕ‬ﻭﻋﻠﻴﻪ ] ‪ 43n +1 ≡ 4 [ 7‬؛ ]‪. 43n + 2 ≡ 2 [ 7‬‬
‫ب ـ ﻨﻀﻊ ‪8513n + 8512 n + 851n + 2 = N‬‬
‫] ‪ 851 ≡ 4 [ 7‬ﻭﻤﻨﻪ ﻤﻥ ﺃﺠل ﻜل ‪ 8513n ≡ 43 n [ 7] ، n ∈ ℕ‬ﺃﻱ ]‪ 8513n ≡ 1[ 7‬ﻭﻴﺼﺒﺢ ﻝﺩﻴﻨﺎ ‪:‬‬
‫] ‪ N ≡ 42 n + 4n + 3[ 7‬ﺃﻱ ] ‪N ≡ 4n ( 4 n + 1) + 3 [ 7‬‬
‫≡‪n‬‬ ‫‪0 1 2‬‬ ‫]‪[3‬‬
‫≡ ‪4n‬‬ ‫‪1 4 2‬‬ ‫]‪[7‬‬
‫≡ ‪4n + 1‬‬ ‫‪2 5 3‬‬ ‫]‪[7‬‬
‫≡ ‪N‬‬ ‫‪5 2 1‬‬ ‫]‪[7‬‬
‫]‪ 73 ≡ 1[ 9‬ﻭﻤﻨﻪ ﻤﻥ ﺃﺠل ﻜل ‪ 73k ≡ 1[9] ، k ∈ ℕ‬ﻭﻋﻠﻴﻪ ]‪ 73k +1 ≡ 7 [9‬؛ ]‪. 73k + 2 ≡ 4 [ 9‬‬ ‫‪ 70‬أ ـ‬
‫ب ـ ﻨﻀﻊ ‪7 n + 3n − 1 = A‬‬
‫ﺇﺫﺍ ﻜﺎﻥ ‪ n = 3k‬ﻓﺈﻥ ‪ A = 73k + 9k − 1‬ﻭﻤﻨﻪ ]‪ A ≡ 1 + 0 − 1[9‬ﺃﻱ ]‪. A ≡ 0 [9‬‬
‫ﺇﺫﺍ ﻜﺎﻥ ‪ n = 3k + 1‬ﻓﺈﻥ ‪ A = 73k +1 + 9k + 2‬ﻭﻤﻨﻪ ]‪ A ≡ 7 + 0 + 2 [9‬ﺃﻱ ]‪. A ≡ 0 [9‬‬
‫ﺇﺫﺍ ﻜﺎﻥ ‪ n = 3k + 2‬ﻓﺈﻥ ‪ A = 73k + 2 + 9k + 5‬ﻭﻤﻨﻪ ]‪ A ≡ 4 + 0 + 5 [9‬ﺃﻱ ]‪. A ≡ 0 [9‬‬
‫‪x ≡ 0 1 2 3 4 5 6 7 [8] 71‬‬
‫]‪3x ≡ 0 3 6 1 4 7 2 5 [8‬‬
‫]‪ 3x ≡ 7 [8‬ﻤﻌﻨﺎﻩ ]‪. x ≡ 5 [8‬‬
‫‪ 8x 2 ≡ 16 [3] 72‬ﻤﻌﻨﺎﻩ ]‪. 2x 2 ≡ 1[3‬‬
‫ﺍﻝﺒﻭﺍﻗﻲ ﺍﻝﻤﻤﻜﻨﺔ ﻝﻜل ﻋﺩﺩ ﺼﺤﻴﺢ ‪ x‬ﻋﻠﻰ ‪ 3‬ﻫﻲ ‪ 2 ، 1 ، 0‬ﻭﻤﻨﻪ ]‪ x 2 ≡ 0 [3‬ﺃﻭ ]‪ x 2 ≡ 1[ 3‬ﻭﺒﺎﻝﺘﺎﻝﻲ ]‪2x 2 ≡ 0 [3‬‬
‫ﺃﻭ ]‪2x 2 ≡ 2 [3‬‬
‫ﺇﺫﻥ ﻤﻥ ﺃﺠل ﻜل ﻋﺩﺩ ﺼﺤﻴﺢ ‪ x‬ﻴﻜﻭﻥ ﺇﻤﺎ ]‪ 2x 2 ≡ 0 [3‬ﻭﺇﻤﺎ ]‪ 2x 2 ≡ 2 [3‬ﻭﺒﺎﻝﺘﺎﻝﻲ ﻻ ﻴﻭﺠﺩ ﺃﻱ ﻋﺩﺩ ﺼﺤﻴﺢ ‪ x‬ﻴﺤﻘﻕ‬
‫]‪. 2x 2 ≡ 1[3‬‬
‫‪ 73‬أ ـ ] ‪ 23 ≡ 1[ 7‬ﻭﻤﻨﻪ ﻤﻥ ﺃﺠل ﻜل ‪ 23k ≡ 1[ 7 ] ، k ∈ ℕ‬ﻭﺒﺎﻝﺘﺎﻝﻲ ]‪23k + 2 ≡ 4 [ 7 ] ، 23k +1 ≡ 2 [ 7‬‬
‫]‪ 36 ≡ 1[ 7‬ﻭﻤﻨﻪ ﻤﻥ ﺃﺠل ﻜل ‪ 36 k ≡ 1[ 7 ] ، k ∈ ℕ‬ﻭﺒﺎﻝﺘﺎﻝﻲ ]‪، 36 k +3 ≡ 6 [ 7] ، 36 k + 2 ≡ 2 [ 7] ، 36 k +1 ≡ 3[ 7‬‬
‫]‪ 36 k + 4 ≡ 4 [ 7‬ﻭ ]‪. 36 k +5 ≡ 5 [ 7‬‬
‫≡‪n‬‬ ‫‪0‬‬ ‫ب ـ ]‪1 2 3 4 5 [ 6‬‬
‫≡ ‪2n‬‬ ‫‪1‬‬ ‫]‪2 4 1 2 4 [ 7‬‬
‫≡ ‪3n‬‬ ‫‪1‬‬ ‫]‪3 2 6 4 5 [ 7‬‬
‫≡ ‪2 n + 3n‬‬ ‫‪2‬‬ ‫]‪5 6 0 6 2 [ 7‬‬
‫] ‪ 2x + 3x ≡ 0 [ 7‬ﻤﻌﻨﺎﻩ ]‪. x ≡ 3 [ 6‬‬
‫‪. 55 ≡ 1[11] ، 35 ≡ 1[11] 74‬‬
‫]‪ 5x − 3x + 6 ≡ 0 [11‬ﻤﻌﻨﺎﻩ ]‪5x − 3x ≡ 5 [11‬‬
‫≡ ‪x‬‬ ‫]‪0 1 2 3 4 [ 5‬‬
‫≡ ‪5x‬‬ ‫]‪1 5 3 4 9 [11‬‬
‫≡ ‪3x‬‬ ‫]‪1 3 9 5 4 [11‬‬
‫]‪5x − 3x ≡ 0 2 5 10 5 [11‬‬
‫]‪ 5x − 3x ≡ 5 [11‬ﻤﻌﻨﺎﻩ ]‪ x ≡ 2 [5‬ﺃﻭ ]‪. x ≡ 4 [5‬‬
‫‪x ≡ 0 1 2 3‬‬ ‫]‪4 [ 5‬‬ ‫‪ 75‬أ ـ‬
‫‪x2≡ 0 1 4 4‬‬ ‫]‪1 [ 5‬‬
‫ب ـ ‪ x 2 − 5 y 2 = 3‬ﻤﻌﻨﺎﻩ ‪ x 2 = 5 y 2 + 3‬ﺇﺫﻥ ﻝﻜﻲ ﺘﻜﻭﻥ ﺍﻝﺜﻨﺎﺌﻴﺔ ) ‪ ( x , y‬ﺤﻼ ﻝﻠﻤﻌﺎﺩﻝﺔ ‪ x 2 = 5 y 2 + 3‬ﻴﺠﺏ ﺃﻥ‬
‫ﻴﻜﻭﻥ ]‪ x 2 ≡ 3 [5‬ﻭﻫﺫﺍ ﻏﻴﺭ ﻤﻤﻜﻥ ‪.‬‬
‫≡ ‪y‬‬ ‫أ ـ ]‪0 1 2 3 4 5 6 [ 7‬‬ ‫‪76‬‬
‫]‪y 3 ≡ 0 1 1 6 1 1 6 [ 7‬‬
‫]‪2 y 3 ≡ 0 2 2 5 2 2 5 [ 7‬‬
‫ب ـ ‪ 7 x 2 + 2 y 3 = 3‬ﻤﻌﻨﺎﻩ ‪ ، 2 y 3 = −7 x 2 + 3‬ﺇﺫﺍ ﻜﺎﻨﺕ ﺍﻝﺜﻨﺎﺌﻴﺔ ) ‪ ( x , y‬ﺤﻼ ﻝﻠﻤﻌﺎﺩﻝﺔ ‪ 2 y 3 = −7 x 2 + 3‬ﻓﺈﻥ‬
‫] ‪ 2 y 3 ≡ 3[ 7‬ﻭﻫﺫﺍ ﻏﻴﺭ ﻤﻤﻜﻥ ﺇﺫﻥ ﺍﻝﻤﻌﺎﺩﻝﺔ ‪ 7 x 2 + 2 y 3 = 3‬ﻻ ﺘﻘﺒل ﺤﻼ ‪.‬‬
‫‪ (1 77‬ﺇﺫﺍ ﻜﺎﻥ ‪ x‬ﺯﻭﺠﻴﺎ ﻓﺈﻥ ]‪ 3x ≡ 1[8‬ﻭﺇﺫﺍ ﻜﺎﻥ ‪ x‬ﻓﺭﺩﻴﺎ ﻓﺈﻥ ]‪. 3x ≡ 3[8‬‬
‫‪y‬‬ ‫‪1 2 3 4 5 6 7 [8] (2‬‬
‫]‪y 2 1 4 1 0 1 4 1 [ 8‬‬
‫‪ (3‬ﺇﺫﺍ ﻜﺎﻥ ‪ x‬ﻓﺭﺩﻴﺎ ﻓﺈﻥ ]‪ 3x ≡ 3[8‬ﻭﻤﻨﻪ ]‪ y 2 + 8 ≡ 3[8‬ﺇﺫﻥ ]‪ y 2 ≡ 3[8‬ﻭﻫﺫﺍ ﻏﻴﺭ ﻤﻤﻜﻥ ‪.‬‬
‫‪ x = 2n (4‬؛ ) ‪ ، 32 n − y 2 = ( 3n − y )( 3n + y‬ﺇﺫﻥ ) ‪ 8 = ( 3n − y )( 3n + y‬ﻭﻤﻨﻪ ) ‪ ( 3n + y‬ﻗﺎﺴﻡ ﻝﻠﻌﺩﺩ ‪. 8‬‬
‫ﺇﺫﻥ ‪ 3n + y ≤ 8‬ﺒﻤﺎ ﺃﻥ ‪ y‬ﻋﺩﺩ ﻁﺒﻴﻌﻲ ﻓﺈﻥ ‪. 3n ≤ 8‬‬
‫‪ 3n ≤ 8 (5‬ﺇﺫﻥ ‪ n = 0‬ﺃﻭ ‪ n = 1‬ﻭﻝﺩﻴﻨﺎ ‪ y 2 = 32 n − 8‬ﺇﺫﻥ ‪ y 2 = 1 − 8 = −7‬ﺃﻭ ‪y 2 = 9 − 8 = 1‬‬
‫ﻭﺒﺎﻝﺘﺎﻝﻲ ‪ y = 1‬ﻭﻝﺩﻴﻨﺎ ‪ x = 2n = 2‬ﻭﺒﺎﻝﺘﺎﻝﻲ ﺍﻝﺜﻨﺎﺌﻴﺔ ﺍﻝﻭﺤﻴﺩﺓ ﺍﻝﺘﻲ ﺘﺤﻘﻕ ﺍﻝﻤﻌﺎﺩﻝﺔ ﻫﻲ )‪. ( 2,1‬‬
‫‪ (1 78‬ﺒﻭﺍﻗﻲ ﻗﺴﻤﺔ ﻜل ﻋﺩﺩ ﻁﺒﻴﻌﻲ ‪ p‬ﻋﻠﻰ ‪ 3‬ﻫﻲ ‪ 1 ، 0‬ﻭ ‪ 2‬ﻭﺇﺫﺍ ﻜﺎﻥ ‪ p‬ﺃﻭﻝﻴﺎ ﻓﺈﻥ ]‪ p ≡ 1[ 3‬ﺃﻭ ]‪p ≡ 2 [ 3‬‬
‫ﺇﺫﻥ ]‪ p ≡ 1[ 3‬ﺃﻭ ]‪. p ≡ −1[3‬‬
‫‪ p (2‬ﻋﺩﺩ ﻁﺒﻴﻌﻲ ﺃﻭﻝﻲ ﺇﺫﻥ ﻻ ﻴﻘﺒل ﺍﻝﻘﺴﻤﺔ ﻋﻠﻰ ‪ 2‬ﺇﺫﻥ ﻫﻭ ﻓﺭﺩﻱ ﻭﻤﻨﻪ ﻴﻭﺠﺩ ﻋﺩﺩ ﻁﺒﻴﻌﻲ ‪ k‬ﺤﻴﺙ ﻴﻜﻭﻥ ‪p = 2k + 1‬‬
‫ﺃﻱ ‪ p 2 = 4k 2 + 4k + 1‬ﻭﻴﻜﺎﻓﺊ )‪p 2 − 1 = 4k ( k + 1‬‬
‫)‪n = p 4 − 1 = ( p 2 − 1)( p 2 + 1‬‬
‫)‪ p 2 + 1 = 4k 2 + 4k + 2 = 2 ( 2k 2 + 2k + 1‬ﺇﺫﻥ ‪ p 2 + 1 = 2α‬ﻤﻊ ‪. α ∈ ℕ‬‬
‫)‪ k ( k + 1‬ﻫﻭ ﻋﺩﺩ ﺯﻭﺠﻲ ﺇﺫﻥ ‪ k ( k + 1) = 2β‬ﻤﻊ ‪ β ∈ ℕ‬ﻭﻤﻨﻪ ‪. p 2 − 1 = 8β‬‬
‫ﻭﺒﺎﻝﺘﺎﻝﻲ ‪. n = ( p 2 − 1)( p 2 + 1) = 16αβ‬‬
‫‪p‬‬ ‫‪1 2 3 4 [5] (3‬‬
‫‪p‬‬ ‫‪4‬‬ ‫]‪1 1 1 1 [5‬‬
‫‪p 4 −1‬‬ ‫]‪0 0 0 0 [5‬‬
‫‪ (1 79‬أ ـ ]‪ 999 ≡ 0 [111‬ﻭﻤﻨﻪ ]‪ 1000 ≡ 1[111‬ﺇﺫﻥ ﻤﻥ ﺃﺠل ﻜل ﻋﺩﺩ ﻁﺒﻴﻌﻲ ‪. 1000n ≡ n [111] ، n‬‬
‫ب ـ ‪ 111 111 = 111 000 + 111‬ﺒﻭﻀﻊ ‪ n = 111‬ﻨﺤﺼل ﻋﻠﻰ ]‪ n ≡ 0 [111‬ﻭ ]‪ 1000n ≡ n [111‬ﻭﻤﻨﻪ‬
‫]‪ 1000n ≡ 0 [111‬ﺇﺫﻥ ]‪. 1000n + n ≡ 0 [111‬‬
‫‪ 100 010 001 = 1000 10000 + 1‬؛ ‪100 010 001 = 1000 00000 + 10000 + 1‬‬
‫‪100 010 001 = 1000 (1000 × 100 + 10 ) + 1‬‬
‫]‪ 1000 ×100 + 10 ≡ 100 + 10 [111‬ﻭﻤﻨﻪ ]‪ 1000 (1000 ×100 + 10 ) ≡ 110 [111‬ﺇﺫﻥ‬
‫]‪ 1000 (1000 × 100 + 10 ) + 1 ≡ 111[111‬؛ ]‪ 1000 (1000 ×100 + 10 ) + 1 ≡ 0 [111‬؛ ]‪. 100 010 001 ≡ 0 [111‬‬
‫‪. α = 100 010 000 001‬‬
‫‪ α = 100 010 000 000 + 1‬؛ ‪ α = 10002 (100 0 × 100 + 10 ) + 1‬ﺇﺫﻥ ]‪ α ≡ 100 + 10 + 1[111‬ﺃﻱ ]‪. α ≡ 0 [111‬‬
‫‪ 99999 ≡ 0 [11 111] (2‬ﻭﻤﻨﻪ ]‪ . 100000 ≡ 1[11 111‬ﻨﻀﻊ ‪n = 100000‬‬
‫‪ β = 1 001 001 001 001‬؛ ‪ β = 1 001 001 000 000 + 1000 + 1‬؛ ‪ β = (1 001 000 0 + 10 ) n + 1000 + 1‬؛‬
‫‪β = (1 000 000 0 + 1 000 0 + 10 ) n + 1000 + 1‬‬
‫‪β = (1 00n + 1 000 0 + 10 ) n + 1000 + 1‬‬
‫]‪ β ≡ 10110 + 1001[11 111‬؛ ]‪. β ≡ 0 [11 111] ، β ≡ 11111[11 111‬‬
‫‪ a = 8k + 2 (1 80‬؛ ‪ a = 104k '+ r‬ﻭﻤﻨﻪ ‪ 104k '+ r = 8k + 2‬ﺃﻱ ‪ r = 8 ( k − 13k ') + 2‬ﺇﺫﻥ ]‪r ≡ 2 [8‬‬
‫‪102‬‬
‫< ‪ α‬؛ ‪. α < 12‬‬ ‫ب ـ ‪ r = 8α + 2‬ﻤﻊ ‪ α ∈ ℕ‬ﻭ ‪ 8α + 2 < 104‬ﺃﻱ‬
‫‪8‬‬
‫‪ a = 13k + 3 (2‬؛ ‪ a = 104k '+ r‬ﻭﻤﻨﻪ ‪ 104k '+ r = 13k + 3‬ﺃﻱ ‪ r = 13 ( k − 8k ') + 3‬ﺇﺫﻥ ]‪. r ≡ 3[13‬‬
‫‪101‬‬
‫< ‪ β‬؛ ‪.β <7‬‬ ‫ﺏ ـ ‪ r = 13β + 3‬ﻤﻊ ‪ β ∈ ℕ‬ﻭ ‪ 13β + 3 < 104‬ﺃﻱ‬
‫‪13‬‬
‫‪ (3‬ﻤﻥ ‪ (1‬ﻨﻼﺤﻅ ﺃﻥ ‪ r‬ﻋﺩﺩ ﺯﻭﺠﻲ ﺇﺫﻥ ﻤﻥ ‪ (2‬ﻴﺠﺏ ﺃﻥ ﻴﻜﻭﻥ ‪ β‬ﻓﺭﺩﻴﺎ ﺇﺫﻥ }‪r ∈ {16, 42, 68,94‬‬
‫‪r −2‬‬
‫ﻭﺍﻝﻘﻴﻤﺔ ﺍﻝﻭﺤﻴﺩﺓ ﺍﻝﺘﻲ ﺘﺤﻘﻕ ﻫﻲ ‪. r = 42‬‬ ‫ﻭﻝﻜﻥ ﻴﺠﺏ ﺃﻥ ﻴﻜﻭﻥ ‪∈ ℕ‬‬
‫‪8‬‬
‫‪. 55 ≡ 1[11] , 54 ≡ 9 [11] , 53 ≡ 4 [11] , 52 ≡ 3[11] , 5 ≡ 5 [11] (1 81‬‬
‫‪ 55 ≡ 1[11] (2‬ﻭﻤﻨﻪ ﻤﻥ ﺃﺠل ﻜل ﻋﺩﺩ ﻁﺒﻴﻌﻲ ‪. 55 p ≡ 1[11] , p‬‬
‫ﻭﺇﺫﺍ ﻜﺎﻥ }‪ k ∈ {1, 2,3, 4‬ﻓﺈﻥ ]‪ 5k × 55 p ≡ 5k [11‬ﺃﻱ ]‪. 55 p + k ≡ 5k [11‬‬
‫ﻭﻤﻨﻪ ]‪. 55 p + 4 ≡ 54 [11] , 55 p +3 ≡ 53 [11] , 55 p + 2 ≡ 52 [11] , 55 p +1 ≡ 5 [11‬‬
‫ﺇﺫﻥ ]‪. 55 p + 4 ≡ 9 [11] , 55 p +3 ≡ 4 [11] , 55 p + 2 ≡ 3[11] , 55 p +1 ≡ 5 [11‬‬
‫‪ 2008 = 5 × 401 + 3 , 1428 = 5 × 245 + 3 (3‬ﻭﻤﻨﻪ ]‪ 52008 − 51428 ≡ ( 4 − 4 ) [11‬ﺃﻱ ]‪. 52008 − 51428 ≡ 0 [11‬‬
‫‪ 35 ≡ 5 [ 7] , 34 ≡ 4 [ 7] , 33 ≡ 6 [ 7 ] , 32 ≡ 2 [ 7] , 3 ≡ 3[ 7 ] (1 82‬ﻭ ]‪. 36 ≡ 1[ 7‬‬
‫‪ 36 ≡ 1[ 7] (2‬ﻭﻤﻨﻪ ﻤﻥ ﺃﺠل ﻜل ﻋﺩﺩ ﻁﺒﻴﻌﻲ ‪36 P ≡ 1[ 7 ] , p‬‬
‫ﻭﺇﺫﺍ ﻜﺎﻥ }‪ k ∈ {1, 2,3, 4,5‬ﻓﺈﻥ ] ‪ 3k × 36 P ≡ 3k [ 7‬ﺃﻱ ] ‪. 36 P + k ≡ 3k [ 7‬‬
‫ﻭﻤﻨﻪ ] ‪. 36 P +5 ≡ 5 [ 7 ] , 36 P + 4 ≡ 4 [ 7 ] , 36 P +3 ≡ 6 [ 7 ] , 36 P + 2 ≡ 2 [ 7 ] , 36 P +1 ≡ 3[ 7‬‬
‫‪ 1988 = 6 × 331 + 2 (3‬ﻭﻤﻨﻪ ] ‪. 31988 ≡ 2 [ 7‬‬
‫] ‪ 10 ≡ 3[ 7‬ﻭﻤﻨﻪ ] ‪ 101408 ≡ 31408 [ 7‬ﻭﻝﺩﻴﻨﺎ ‪ 1408 = 6 × 234 + 4‬ﺇﺫﻥ ] ‪ 31408 ≡ 4 [ 7‬ﻭﺒﺎﻝﺘﺎﻝﻲ ]‪. 101408 ≡ 4 [ 7‬‬
‫]‪ 9 ≡ 2 [ 7‬ﻭﻤﻨﻪ ﻤﻥ ﺃﺠل ﻜل ﻋﺩﺩ ﻁﺒﻴﻌﻲ ‪. 93n + 2 ≡ 23 n + 2 [ 7 ] , n‬‬
‫ﻭﻝﺩﻴﻨﺎ ‪ 23n + 2 = 4 × 23n = 4 × 8n‬ﻭﻤﻥ ﺃﺠل ﻜل ﻋﺩﺩ ﻁﺒﻴﻌﻲ ‪ 4 × 8n ≡ 4 × 1[ 7] , n‬ﺇﺫﻥ ]‪93n + 2 ≡ 4 [ 7‬‬
‫ﺍﻻﺴﺘﻨﺘﺎﺝ ] ‪ 31988 + 101408 + 93n + 2 ≡ ( 2 + 4 + 4 ) [ 7‬ﻭﻤﻨﻪ ] ‪ 31988 + 101408 + 93n + 2 ≡ 10 [ 7‬ﺇﺫﻥ‬
‫]‪. 31988 + 101408 + 93n + 2 ≡ 3[ 7‬‬
‫ﺇﺫﻥ ﺒﺎﻗﻲ ﺍﻝﻘﺴﻤﺔ ﺍﻷﻗﻠﻴﺩﻴﺔ ﻝﻠﻌﺩﺩ ) ‪ ( 31988 + 101408 + 93n + 2‬ﻋﻠﻰ ‪ 7‬ﻫﻭ ‪. 3‬‬
‫‪. 24 ≡ 1[5] , 23 ≡ 3[5] , 22 ≡ 4 [5] , 2 ≡ 2 [ 5] (1 83‬‬
‫ﺍﻻﺴﺘﻨﺘﺎﺠﺎﺕ ‪ 24 ≡ 1[5] :‬ﻭﻤﻨﻪ ﻤﻥ ﺃﺠل ﻜل ﻋﺩﺩ ﻁﺒﻴﻌﻲ ‪24 p ≡ 1[5] , p‬‬
‫ﺇﺫﻥ ]‪. 24 p + 3 ≡ 3[5] , 24 p + 2 ≡ 4 [5] , 24 p +1 ≡ 2 [ 5‬‬
‫]‪ 3 ≡ −2 [5‬ﻭﻤﻨﻪ ﻤﻥ ﺃﺠل ﻜل ﻋﺩﺩ ﻁﺒﻴﻌﻲ ‪ 34 p ≡ 24 p [5] , p‬ﺃﻱ ]‪34 p ≡ 1[5‬‬
‫ﻭﺒﺎﻝﺘﺎﻝﻲ ]‪ 34 p + 2 ≡ 9 [ 5] , 34 p +1 ≡ 3[ 5‬ﻭﻤﻨﻪ ]‪ 34 p +3 ≡ 27 [ 5] , 34 p + 2 ≡ 4 [5‬ﻭﻤﻨﻪ ]‪. 34 p +3 ≡ 2 [5‬‬
‫‪n = 4p 4p +1 4p + 2 4p + 3 p ∈ ℕ‬‬
‫‪2n ≡ 1‬‬ ‫‪2‬‬ ‫‪4‬‬ ‫‪3‬‬ ‫]‪[ 5‬‬
‫≡ ‪3n‬‬ ‫‪1‬‬ ‫‪3‬‬ ‫‪4‬‬ ‫‪2‬‬ ‫]‪[ 5‬‬
‫‪ 14 = 4 × 3 + 2 (2‬ﺇﺫﻥ ]‪. 214 ≡ 4 [5‬‬
‫‪ 10 = 4 × 2 + 2‬ﺇﺫﻥ ]‪. 310 ≡ 4 [5‬‬
‫‪ 2 × 34 n +1 − 24 n ≡ 2 × 3 − 1[5] (3‬ﻭﻤﻨﻪ ]‪2 × 34 n +1 − 24 n ≡ 5 [ 5‬‬
‫]‪ 5 ≡ 0 [5‬ﻓﺈﻥ ]‪. 2 × 34 n +1 − 24 n ≡ 0 [5‬‬ ‫ﺒﻤﺎ ﺃﻥ‬
‫‪ 5 ≡ −2 [ 7] (1 84‬ﻭﻤﻨﻪ ]‪ 53 ≡ −8 [ 7‬ﺃﻱ ] ‪ 53 ≡ −1[ 7‬ﺇﺫﻥ ] ‪56 ≡ 1[ 7‬‬
‫ﻭﺒﺎﻝﺘﺎﻝﻲ ﻤﻥ ﺃﺠل ﻜل ﻋﺩﺩ ﻁﺒﻴﻌﻲ ‪56 k ≡ 1[ 7] , k‬‬
‫ﻭﻴﻜﻭﻥ ﻝﺩﻴﻨﺎ ‪56 k +1 ≡ 5 [ 7 ] :‬‬
‫]‪ 56 k + 2 ≡ 25 [ 7‬ﺃﻱ ] ‪56 k + 2 ≡ 4 [ 7‬‬
‫] ‪ 56 k +3 ≡ 20 [ 7‬ﺃﻱ ] ‪56 k +3 ≡ 6 [ 7‬‬
‫] ‪ 56 k + 4 ≡ 30 [ 7‬ﺃﻱ ] ‪56 k + 4 ≡ 2 [ 7‬‬
‫] ‪ 56 k +5 ≡ 10 [ 7‬ﺃﻱ ]‪. 56 k +5 ≡ 3[ 7‬‬
‫‪ 6 ≡ −1[ 7 ] (2‬ﻭﻤﻨﻪ ﻤﻥ ﺃﺠل ﻜل ﻋﺩﺩ ﻁﺒﻴﻌﻲ ‪ 62 n ≡ ( −1) [ 7 ] , n‬ﺃﻱ ] ‪. 62 n ≡ 1[ 7‬‬
‫‪2‬‬

‫‪5 n + 6 2 n + 3 ≡ ( 5 n + 4 ) [ 7 ] (3‬‬
‫]‪ 5n + 62 n + 3 ≡ 0 [ 7‬ﻤﻌﻨﺎﻩ ] ‪ 5n + 4 ≡ 0 [ 7‬ﻭﻤﻌﻨﺎﻩ ]‪ 5n ≡ −4 [ 7‬ﻭﻤﻌﻨﺎﻩ ] ‪ 5n ≡ 3[ 7‬ﻭﻤﻌﻨﺎﻩ ‪ n = 5k + 6‬ﻤﻊ‬
‫‪. k ∈ℕ‬‬
‫‪ (1 85‬ﻝﺩﻴﻨﺎ ‪ 2 4 = 16‬ﻭﻤﻨﻪ ]‪ 24 ≡ 1[5‬ﻭﺒﺎﻝﺘﺎﻝﻲ ﻤﻥ ﺃﺠل ﻜل ﻋﺩﺩ ﻁﺒﻴﻌﻲ ‪24 p ≡ 1[5] , p‬‬
‫]‪, 24 p + 2 ≡ 4 [5] , 24 p +1 ≡ 2 [ 5‬‬
‫]‪ 24 p + 3 ≡ 8 [5‬ﺃﻱ ]‪. 24 p + 3 ≡ 3[5‬‬
‫‪ (2‬ﻝﺩﻴﻨﺎ ‪ 23 = 8‬ﻭﻤﻨﻪ ] ‪ 23 ≡ 1[ 7‬ﻭﺒﺎﻝﺘﺎﻝﻲ ﻤﻥ ﺃﺠل ﻜل ﻋﺩﺩ ﻁﺒﻴﻌﻲ ‪23k ≡ 1[ 7 ] , k‬‬
‫]‪. 23k + 2 ≡ 4 [ 7 ] , 23k +1 ≡ 2 [ 7‬‬
‫]‪3n ≡ 6 [12‬‬ ‫]‪n ≡ 2 [ 4‬‬ ‫]‪≡ 4 [ 5‬‬
‫‪4 p +2‬‬
‫‪n = 4 p + 2‬‬ ‫‪ 2‬‬
‫‪ ‬ﻭﻤﻨﻪ ]‪n ≡ 2 [12‬‬ ‫‪ ‬ﻭﻤﻌﻨﺎﻩ‬ ‫‪ ‬ﻭﻤﻌﻨﺎﻩ‬ ‫‪  3k + 2‬ﻤﻌﻨﺎﻩ‬ ‫‪ (3‬ﻝﺩﻴﻨﺎ‬
‫]‪4n ≡ 8 [12‬‬ ‫]‪ n ≡ 2 [3‬‬ ‫‪ n = 3k + 2‬‬ ‫‪2‬‬ ‫]‪≡ 4 [7‬‬
‫ﻭﻋﻜﺴﻴﺎ ﻝﺩﻴﻨﺎ ﺇﺫﺍ ﻜﺎﻥ ]‪ n ≡ 2 [12‬ﻓﺈﻥ ‪ n = 12m + 2‬ﻭﻤﻨﻪ ‪ n = 3 ( 4m ) + 2‬ﻭ ‪ n = 4 ( 3m ) + 2‬ﻭﻤﻨﻪ‬
‫]‪ 2 ≡ 4 [5‬‬
‫‪n‬‬

‫‪.  n‬‬
‫] ‪2 ≡ 4 [ 7‬‬
‫‪ ( n − 1) 86‬ﻤﻀﺎﻋﻑ ﻝﻠﻌﺩﺩ ‪ 3‬ﻤﻌﻨﺎﻩ ‪ n = 3k + 1‬ﻤﻊ ‪. k ∈ ℕ‬‬
‫‪ 1 + ( n − 1) 2 n ‬ﻴﻘﺒل ﺍﻝﻘﺴﻤﺔ ﻋﻠﻰ ‪ 7‬ﻤﻌﻨﺎﻩ ]‪ 1 + ( n − 1) 2n ≡ 0 [ 7‬ﻭﻤﻨﻪ ] ‪1 + ( 3k ) 23 k +1 ≡ 0 [ 7‬‬

‫ﻴﻜﺎﻓﺊ ] ‪ 1 + 3k ( 23 ) × 2 ≡ 0 [ 7‬ﻭﻤﻌﻨﺎﻩ ] ‪ 1 + 6k ≡ 0 [ 7‬ﺃﻱ ] ‪ − k ≡ −1[ 7‬ﻴﻜﺎﻓﺊ ]‪k ≡ 1[ 7‬‬


‫‪k‬‬

‫ﺇﺫﻥ ‪ n = 3k + 1 = 3 ( 7 p + 1) + 1‬ﺃﻱ ‪. n = 21p + 4‬‬


‫ﻭﻋﻜﺴﻴﺎ ﺇﺫﺍ ﻜﺎﻥ ‪ n = 21p + 4‬ﻓﺈﻥ ‪ n − 1 = 21p + 3‬ﻭﻤﻨﻪ )‪ ( n − 1‬ﻤﻀﺎﻋﻑ ﻝﻠﻌﺩﺩ ‪ 3‬ﻭ ] ‪n − 1 ≡ 3 [ 7‬‬
‫) ‪ 2 n = 221 p + 4 = 2 ( 23‬ﻭﺒﻤﺎ ﺃﻥ ] ‪ 23 ≡ 1[ 7‬ﻓﺈﻥ ]‪ . 2n ≡ 2 [ 7‬ﻭﻤﻨﻪ ] ‪ 1 + ( n − 1) 2n ≡ (1 + 3 × 2 ) [ 7‬ﺃﻱ‬
‫‪7 p +1‬‬
‫ﻭﻜﺫﻝﻙ‬
‫]‪. 1 + ( n − 1) 2n ≡ 0 [ 7‬‬
‫‪. 45 ≡ 1[11] , 44 ≡ 3 [11] , 43 ≡ 9 [11] , 42 ≡ 5 [11] , 4 ≡ 4 [11] (1 88‬‬
‫ﻭﻤﻨﻪ ﻤﻥ ﺃﺠل ﻜل ﻋﺩﺩ ﻁﺒﻴﻌﻲ ‪ 45 p ≡ 1[11] , p‬ﻭﻋﻠﻴﻪ ]‪, 45 p +3 ≡ 9 [11] , 45 p + 2 ≡ 5 [11] , 45 p +1 ≡ 4 [11‬‬
‫]‪. 45 p + 4 ≡ 3[11‬‬
‫‪ 1995 ≡ 4 [11] (2‬ﻭﻤﻨﻪ ]‪. 1995n ≡ 4n [11‬‬
‫]‪ 26 ≡ 4 [11‬ﻭﻤﻨﻪ ]‪ 2610 n + 2 ≡ 410 n + 2 [11‬ﻭﻝﺩﻴﻨﺎ ‪ 410 n + 2 = 45( 2 n ) + 2 = 45 p + 2‬ﻭﻤﻨﻪ ]‪ 410 n + 2 ≡ 5 [11‬ﺇﺫﻥ‬
‫]‪2610 n + 2 ≡ 5 [11‬‬
‫ﺇﺫﻥ ]‪ 6 × 1995n + 2610 n + 2 + 7 ≡ ( 6 × 4 n + 5 + 7 ) [11‬ﺃﻱ ]‪6 × 1995n + 2610 n + 2 + 7 ≡ ( 6 × 4 n + 1) [11‬‬
‫ﻭﺒﺎﻝﺘﺎﻝﻲ ]‪ 6 × 1995n + 2610 n + 2 + 7 ≡ 0 [11‬ﻤﻌﻨﺎﻩ ]‪ 6 × 4n + 1 ≡ 0 [11‬ﻭﻴﻜﺎﻓﺊ ]‪ 2 ( 6 × 4 n + 1) ≡ 0 [11‬ﺃﻱ‬
‫]‪ 4n + 2 ≡ 0 [11‬ﻭﻤﻌﻨﺎﻩ ]‪4n ≡ 9 [11‬‬
‫ﻭﻤﻌﻨﺎﻩ ‪ n = 5 p + 3‬ﻤﻊ ‪. p ∈ ℕ‬‬
‫‪ 5‬ﻭﻋﻠﻴﻪ ‪:‬‬‫‪6p‬‬
‫‪ 5 ≡ −2 [ 7] (1 89‬ﻭﻤﻨﻪ ] ‪ 5 ≡ −1[ 7‬ﺇﺫﻥ ] ‪ 5 ≡ 1[ 7‬ﻭﺒﺎﻝﺘﺎﻝﻲ ﻤﻥ ﺃﺠل ﻜل ﻋﺩﺩ ﻁﺒﻴﻌﻲ ‪≡ 1[ 7 ] , p‬‬
‫‪6‬‬ ‫‪3‬‬

‫] ‪. 56 p +5 ≡ 3[ 7 ] , 56 p + 4 ≡ 2 [ 7] , 56 p +3 ≡ 6 [ 7] , 56 p + 2 ≡ 4 [ 7] , 56 p +1 ≡ 5 [ 7‬‬
‫‪ (2‬ﻝﺩﻴﻨﺎ ] ‪ 26 ≡ 5 [ 7‬ﻭ ]‪47 ≡ 5 [ 7‬‬
‫ﺇﺫﻥ ﻤﻥ ﺃﺠل ﻜل ‪ 266 n +5 ≡ 56 n +5 [ 7 ] , n ∈ ℕ‬ﻭ ] ‪ 4712 n + 2 ≡ 56( 2 n )+ 2 [ 7‬ﻭﻋﻠﻴﻪ ]‪266 n +5 ≡ 3[ 7‬‬
‫ﻭ ] ‪. 4712 n + 2 ≡ 4 [ 7‬‬
‫]‪ 266 n +5 + 2 × 4712 n + 2 + 3 ≡ ( 3 + 2 × 4 + 3) [ 7‬ﺃﻱ ]‪ 266 n +5 + 2 × 4712 n + 2 + 3 ≡ 14 [ 7‬ﻭﻴﻜﺎﻓﺊ‬
‫]‪. 266 n +5 + 2 × 4712 n + 2 + 3 ≡ 0 [ 7‬‬
‫‪ 266 n +5 + 2 × 4712 n + 2 + 5n ≡ 0 [ 7 ] (3‬ﻤﻌﻨﺎﻩ ] ‪ 4 + 5n ≡ 0 [ 7‬ﻭﻴﻜﺎﻓﺊ ] ‪ 3 ( 4 + 5n ) ≡ 0 [ 7‬ﻭﻤﻌﻨﺎﻩ ] ‪12 + n ≡ 0 [ 7‬‬
‫ﺃﻱ ] ‪. n ≡ 2 [ 7‬‬
‫‪ 33 = 27 (1 90‬ﻭﻤﻨﻪ ]‪ 33 ≡ 1[13‬ﺇﺫﻥ ﻤﻥ ﺃﺠل ﻜل ﻋﺩﺩ ﻁﺒﻴﻌﻲ ‪ 33 p ≡ 1[13] , p‬ﻭﻋﻠﻴﻪ ]‪, 33 p +1 ≡ 3[13‬‬
‫]‪. 33 p + 2 ≡ 9 [13‬‬
‫‪ 4 ( 3n +1 − 1) ≡ 0 [13] (2‬ﻴﻜﺎﻓﺊ ]‪ 40 ( 3n +1 − 1) ≡ 0 [13‬ﻭﻴﻜﺎﻓﺊ ]‪ 3n +1 − 1 ≡ 0 [13‬ﻤﻌﻨﺎﻩ ]‪ 3n +1 ≡ 1[13‬ﻭﻤﻌﻨﺎﻩ‬
‫]‪n + 1 ≡ 0 [3‬‬
‫ﺃﻱ ]‪. n ≡ 2 [3‬‬
‫‪ 16 ≡ 2 [ 7 ] 91‬ﺇﺫﻥ ]‪ 163 ≡ 23 [ 7‬ﺃﻱ ]‪ 163 ≡ 1[ 7‬ﻭﺒﺎﻝﺘﺎﻝﻲ ﻤﻥ ﺃﺠل ﻜل ﻋﺩﺩ ﻁﺒﻴﻌﻲ ‪. 163 p ≡ 1[ 7 ] , p‬‬
‫ﺇﺫﺍ ﻜﺎﻥ ‪ n = 3 p‬ﻓﺈﻥ )‪ 15 (16 n +1 − 1) = 15 (16 × 163 p − 1‬ﻭﻤﻨﻪ ] ‪ 15 (16 n +1 − 1) ≡ 1( 2 × 1 − 1) [ 7‬ﺃﻱ‬
‫] ‪15 (16 n +1 − 1) ≡ 1[ 7‬‬
‫ﺇﺫﺍ ﻜﺎﻥ ‪ n = 3 p + 1‬ﻓﺈﻥ )‪ 15 (16 n +1 − 1) = 15 (162 × 163 p − 1‬ﻭﻤﻨﻪ ] ‪15 (16 n +1 − 1) ≡ 1( 4 × 1 − 1) [ 7‬‬
‫ﺃﻱ ] ‪. 15 (16 n +1 − 1) ≡ 3 [ 7‬‬
‫ﺇﺫﺍ ﻜﺎﻥ ‪ n = 3 p + 2‬ﻓﺈﻥ )‪ 15 (16 n +1 − 1) = 15 (163 × 163 p − 1‬ﻭﻤﻨﻪ ] ‪ 15 (16 n +1 − 1) ≡ 1( 8 × 1 − 1) [ 7‬ﺃﻱ‬
‫] ‪. 15 (16 n +1 − 1) ≡ 0 [ 7‬‬
‫‪ (1 92‬ـ ﺃ ـ ‪ 34 = 81‬ﻭﻤﻨﻪ ]‪ 34 ≡ 1[10‬ﻭﺒﺎﻝﺘﺎﻝﻲ ﻤﻥ ﺃﺠل ﻜل ﻋﺩﺩ ﻁﺒﻴﻌﻲ ‪ 34 p ≡ 1[10] , p‬ﻭﻋﻠﻴﻪ‬
‫]‪34 p +3 ≡ 7 [10] , 34 p + 2 ≡ 9 [10] , 34 p +1 ≡ 3 [10‬‬
‫ـ ﺏ ـ ‪ 92001 = 34002 = 34×1000 + 2‬ﻭﻤﻨﻪ ]‪ 63 × 92001 ≡ 3 × 9 [10‬ﺃﻱ ]‪. 63 × 92001 ≡ 7 [10‬‬
‫)‪ 71422 ≡ ( −3‬ﺃﻱ ]‪ 71422 ≡ 34×355+ 2 [10‬ﻤﻌﻨﺎﻩ ]‪. 71422 ≡ 9 [10‬‬ ‫]‪[10‬‬ ‫]‪ 7 ≡ −3[10‬ﻭﻤﻨﻪ‬
‫‪1422‬‬

‫ﺇﺫﻥ ]‪ 63 × 92001 − 71422 ≡ ( 7 − 9 ) [10‬ﻤﻌﻨﺎﻩ ]‪ 63 × 92001 − 71422 ≡ −2 [10‬ﺃﻱ ]‪. 63 × 92001 − 71422 ≡ 8 [10‬‬
‫‪ (2‬ـ أ ـ ﻝﺩﻴﻨﺎ ‪3n × 9 n = 3n × 32 n = n × 32 n +1‬‬
‫)‪ 7 2 n +1 ≡ ( −3‬ﺃﻱ ]‪72 n +1 ≡ −32 n +1 [10‬‬ ‫]‪[10‬‬ ‫ﻭﻝﺩﻴﻨﺎ ]‪ 7 ≡ −3[10‬ﻭﻤﻨﻪ‬
‫‪2 n +1‬‬

‫ﺇﺫﻥ ]‪3n × 9 n + 7 2 n +1 ≡ ( n 32 n +1 − 32 n +1 ) [10‬‬


‫ﺃﻱ ]‪. 3n × 9n + 72 n +1 ≡ ( n − 1) 32 n +1 [10‬‬
‫ـ ب ـ ]‪ 3n × 9n + 72 n +1 ≡ 0 [10‬ﻤﻌﻨﺎﻩ ]‪ ( n − 1) 32 n +1 ≡ 0 [10‬ﻭﻤﻌﻨﺎﻩ ]‪ 32 n +3 ( n − 1) 32 n +1 ≡ 0 [10‬ﺃﻱ‬
‫]‪ ( n − 1) 34 n + 4 ≡ 0 [10‬ﻭﻴﻜﺎﻓﺊ ]‪ n − 1 ≡ 0 [10‬ﺃﻱ ]‪. n ≡ 1[10‬‬
‫‪ 27 ≡ −1[ 7 ] (1 93‬ﺃﻱ ] ‪ 33 ≡ −1[ 7‬ﻭﻤﻨﻪ ]‪ 36 ≡ 1[ 7‬ﻭﺒﺎﻝﺘﺎﻝﻲ ﻤﻥ ﺃﺠل ﻜل ﻋﺩﺩ ﻁﺒﻴﻌﻲ ‪ 36 k ≡ 1[ 7 ] , k‬ﻭﻋﻠﻴﻪ‬
‫]‪. 36 k +5 ≡ 5 [ 7] , 36 k + 4 ≡ 4 [ 7] , 36 k +3 ≡ 6 [ 7] , 36 k + 2 ≡ 2 [ 7] , 36 k +1 ≡ 3[ 7‬‬
‫]‪ 64 ≡ 1[ 7‬ﻭﻤﻨﻪ ] ‪ 43 ≡ 1[ 7‬ﻭﺒﺎﻝﺘﺎﻝﻲ ﻤﻥ ﺃﺠل ﻜل ﻋﺩﺩ ﻁﺒﻴﻌﻲ ‪ 43 p ≡ 1[ 7] , p‬ﻭﻋﻠﻴﻪ ] ‪, 43 p +1 ≡ 4 [ 7‬‬
‫] ‪. 43 p + 2 ≡ 2 [ 7‬‬
‫‪ 2006 ≡ 4 [ 7] (2‬؛ ] ‪ 1424 ≡ 3[ 7‬ﻭﻤﻨﻪ ﻤﻥ ﺃﺠل ﻜل ‪ 20063n + 2 ≡ 43n + 2 [ 7 ] , n ∈ ℕ‬؛ ]‪14246 n +1 ≡ 36 n +1 [ 7‬‬
‫ﻭﻤﻨﻪ ]‪ 20063n + 2 ≡ 2 [ 7‬؛ ]‪ 14246 n +1 ≡ 3[ 7‬ﺇﺫﻥ ] ‪ 2 × 20063n + 2 + 14246 n +1 ≡ ( 2 × 2 + 3) [ 7‬ﺃﻱ‬
‫] ‪. 2 × 20063n + 2 + 14246 n +1 ≡ 0 [ 7‬‬
‫‪2 × 30 + 2 × 31 + ... + 2 × 3n = 2 ( 30 + 31 + ... + 3n ) (3‬‬

‫‪2 × 30 + 2 × 31 + ... + 2 × 3n = 3n +1 − 1‬‬


‫) ‪3 × 40 + 3 × 41 + ... + 3 × 4 n = 3 ( 40 + 41 + ... + 4n‬‬
‫‪ 3 × 40 + 3 × 41 + ... + 3 × 4 n = 4n +1 − 1‬ﺇﺫﻥ ‪s n = u 0 + u 1 + ... + u n = 3n +1 + 4n +1 − 2‬‬
‫≡‪n‬‬ ‫‪0 1 2 3 4 5‬‬ ‫]‪[ 6‬‬
‫≡ ‪3n +1‬‬ ‫‪3 2 6 4 5 1‬‬
‫≡ ‪4n +1‬‬ ‫‪4 2 1 4 2 1‬‬ ‫]‪[ 7‬‬
‫≡ ‪sn‬‬ ‫‪5 2 5 6 5 0‬‬
‫‪ 49 ≡ −1[10] (1 94‬ﺃﻱ ]‪ 72 ≡ −1[10‬ﻭﻤﻨﻪ ]‪ 74 ≡ 1[10‬ﻭﺒﺎﻝﺘﺎﻝﻲ ﻤﻥ ﺃﺠل ﻜل ﻋﺩﺩ ﻁﺒﻴﻌﻲ ‪7 4 k ≡ 1[10] , k‬‬
‫ﻭﻋﻠﻴﻪ ]‪. 7 4 k +3 ≡ 3 [10] , 74 k + 2 ≡ 9 [10] , 7 4 k +1 ≡ 7 [10‬‬
‫]‪. 74 k + 7 4 k +1 + 7 4 k + 2 + 74 k +3 = (1 + 7 + 9 + 3) [10‬‬
‫]‪. 74 k + 7 4 k +1 + 7 4 k + 2 + 7 4 k +3 = 0 [10‬‬
‫‪ (2‬ﻤﻥ ﺃﺠل ﻜل ‪S n = 1 + 7 + 7 2 + ... + 7 n , n ∈ ℕ‬‬
‫‪S n + 4 = S n + 7 n +1 + 7 n + 2 + 7 n +3 + 7 n + 4‬‬
‫) ‪S n + 4 = S n + 7 n +1 (1 + 7 + 7 2 + 73‬‬
‫]‪ 1 + 7 + 7 2 + 73 ≡ 0 [10‬ﺇﺫﻥ ﻤﻥ ﺃﺠل ﻜل ﻋﺩﺩ ﻁﺒﻴﻌﻲ ‪. S n + 4 = S n [10] , n‬‬
‫‪ S 0 ≡ 1[10] , S 0 = 1‬؛ ‪ S 1 ≡ 8 [10] , S 1 = 8‬؛ ‪ S 2 ≡ 7 [10] , S 2 = 57‬؛ ‪ S 3 ≡ 0 [10] , S 3 = 400‬؛‬
‫‪. S 4 ≡ 1[10] , S 4 = 2801‬‬
‫ﻝﻨﺭﻫﻥ ﺃﻨﻪ ﻤﻥ ﺃﺠل ﻜل ‪. S 4 n ≡ 1[10] n ∈ ℕ‬‬
‫]‪ . S 0 ≡ 1[10‬ﻭﻨﻔﺭﺽ ﺃﻥ ]‪. S 4 p ≡ 1[10‬‬
‫ﻝﺩﻴﻨﺎ ‪ S 4( p +1) = S 4 p + 4‬ﻭﺤﺴﺏ ﺍﻝﺴﺅﺍل ﺍﻝﺴﺎﺒﻕ ]‪ S 4 p + 4 = S 4 p [10‬ﺇﺫﻥ ]‪ S 4( p +1) ≡ 1[10‬ﻭﺒﺎﻝﺘﺎﻝﻲ ﺤﺴﺏ ﻤﺒﺩﺃ ﺍﻝﺘﺭﺍﺠﻊ‬
‫ﻴﻨﺘﺞ ﻤﻥ ﺃﺠل ﻜل ‪. S 4 n ≡ 1[10] n ∈ ℕ‬‬
‫‪ S 4 n +1 = S 4 n + 7 4 n +1‬ﻭﻤﻨﻪ ]‪ S 4 n +1 ≡ 1 + 7 [10‬ﺃﻱ ]‪. S 4 n +1 ≡ 8 [10‬‬
‫‪ S 4 n + 2 = S 4 n +1 + 7 4 n + 2‬ﻭﻤﻨﻪ ]‪ S 4 n + 2 ≡ ( 8 + 9 ) [10‬ﺃﻱ ]‪. S 4 n + 2 ≡ 7 [10‬‬
‫‪ S 4 n +3 = S 4 n + 2 + 7 4 n +3‬ﻭﻤﻨﻪ ]‪ S 4 n + 3 ≡ ( 7 + 3) [10‬ﺃﻱ ]‪. S 4 n +3 ≡ 0 [10‬‬
‫‪ 2‬ـ أ‪ %2w‬ا
"!اد‬
‫‪ 95‬ﻨﻔﺭﺽ ‪ 0 < x < 7‬ﻭ ‪ yx = 10 y + x . 0 < y < 7‬ﻭ ‪ xy = 7 x + y‬ﺇﺫﻥ ‪ 7 x + y = 10 y + x‬ﺃﻱ‬
‫‪ 6x = 9 y‬ﻤﻌﻨﺎﻩ ‪ 2x = 3 y‬ﻭﻤﻨﻪ ]‪ 2x ≡ 0 [3‬ﺃﻱ ]‪ 4x ≡ 0 [3‬ﻤﻌﻨﺎﻩ ]‪ x ≡ 0 [3‬ﻭﺒﺎﻝﺘﺎﻝﻲ ‪ x = 3‬ﺃﻭ ‪x = 6‬‬
‫) ‪( x , y ) = ( 6, 4‬‬ ‫ﺇﺫﻥ ) ‪ ( x , y ) = ( 3, 2‬ﺃﻭ‬
‫‪ 96‬ﺍﻝﺸﺭﻭﻁ ‪. 0 ≤ y < 7 ، 0 < z < 7 ، 0 < x < 7 :‬‬
‫‪ n = xyz = 7 2 x + 7 y + z‬ﻭ ‪ n = zyx = 112 z + 11y + x‬ﻭﻤﻨﻪ ‪ 49x + 7 y + z = 121z + 11y + x‬ﺃﻱ ‪:‬‬
‫‪ 48x − 4 y − 120z = 0‬ﻭﻫﺫﺍ ﻴﻜﺎﻓﺊ ‪ 12x − y − 30z = 0‬ﻤﻌﻨﺎﻩ ) ‪y = 12x − 30z = 6 ( 2x − 5z‬‬
‫ﺇﺫﻥ ]‪ y ≡ 0 [ 6‬ﻭﻝﺩﻴﻨﺎ ‪ 0 ≤ y < 7‬ﺇﺫﻥ ‪ y = 0‬ﺃﻭ ‪y = 6‬‬
‫ﺇﺫﺍ ﻜﺎﻥ ‪ y = 0‬ﻓﺈﻥ ‪ 6 ( 2x − 5z ) = 0‬ﺃﻱ ‪ 2x = 5z‬ﻭﻤﻨﻪ ]‪ 2x ≡ 0 [5‬ﻤﻌﻨﺎﻩ ]‪ 6x ≡ 0 [5‬ﺃﻱ ]‪ x ≡ 0 [5‬ﺒﻤﺎ‬
‫ﺃﻥ ‪ 0 < x < 7‬ﻓﺈﻥ ‪ x = 5‬ﻭﻤﻨﻪ ‪. z = 2‬‬
‫ﺇﺫﺍ ﻜﺎﻥ ‪ y = 6‬ﻓﺈﻥ ‪ 6 ( 2x − 5z ) = 6‬ﺃﻱ ‪ 2x = 5z + 1‬ﻭﻤﻨﻪ ]‪ 2x ≡ 1[ 5‬ﻤﻌﻨﺎﻩ ]‪ 6x ≡ 3[5‬ﺃﻱ ]‪ x ≡ 3 [5‬ﺒﻤﺎ‬
‫‪ ( x , y , z ) = ( 5, 0, 2 ) . z‬ﺃﻭ )‪( x , y , z ) = ( 3, 6,1‬‬ ‫ﺃﻥ ‪ 0 < x < 7‬ﻓﺈﻥ ‪ x = 3‬ﻭﻤﻨﻪ ‪= 1‬‬
‫‪ a > 6 97‬؛ ‪ b + c = 46 = 4a + 6‬؛ ‪bc = 555 = 5a + 5a + 5‬‬
‫‪2‬‬

‫‪ b‬ﻭ ‪ c‬ﻫﻤﺎ ﺤﻼ ﻝﻠﻤﻌﺎﺩﻝﺔ ‪ x 2 − 2 ( 2a + 3 ) x + ( 5a 2 + 5a + 5 ) = 0‬ﺤﻴﺙ ‪ x‬ﻫﻭ ﺍﻝﻤﺠﻬﻭل‬


‫) ‪ ∆ ' = ( 2a + 3) − ( 5a 2 + 5a + 5‬؛ ‪ ∆ ' = −a 2 + 7a + 4‬؛ ‪δ = 49 + 16 = 65‬‬
‫‪2‬‬

‫‪7 − 65‬‬ ‫‪7 + 65‬‬


‫ﻭﻤﻨﻪ ‪ 6 < a ≤ 7,53‬ﺇﺫﻥ ‪. a = 7‬‬ ‫≤ ‪≤a‬‬ ‫‪ ∆ ' ≥ 0‬ﻤﻌﻨﺎﻩ‬
‫‪2‬‬ ‫‪2‬‬
‫ﻭﻤﻨﻪ ﺍﻝﻤﻌﺎﺩﻝﺔ ﺘﺼﺒﺢ ‪ x 2 − 2 (17 ) x + 285 = 0‬ﻭ ‪ ∆ ' = 4‬ﺇﺫﻥ ‪ x ' = 17 − 2 = 15‬ﻭ ‪. x ' = 17 + 2 = 19‬‬
‫ﺒﻤﺎ ﺃﻥ ‪ 1 ≤ a ≤ b ≤ c‬ﻓﺈﻥ ) ‪ . ( a, b , c ) = ( 7,15,19‬ﻭﻴﻜﻭﻥ ‪. abc = 1995‬‬
‫‪ 45x − 28 y = 130 (1 98‬ﻤﻌﻨﺎﻩ ) ‪ 45x = 130 + 28 y = 2 ( 65 + 14 y‬؛ ]‪ 45x ≡ 0 [ 2‬ﺃﻱ ]‪. x ≡ 0 [ 2‬‬
‫‪ 45x − 28 y = 130‬ﻤﻌﻨﺎﻩ ) ‪ 28 y = 45x + 130 = 5 ( 9x + 26‬ﻭﻤﻨﻪ ]‪ 28 y ≡ 0 [5‬ﺃﻱ ]‪ 56 y ≡ 0 [5‬ﻭﻤﻌﻨﺎﻩ‬
‫]‪. y ≡ 0 [ 5‬‬
‫‪ 0 ≤ α ≤ 9 (2‬ﻭ ‪ 0 ≤ β ≤ 7‬؛ ‪ n = 2 × 93 + 92 α + 9α + 3 = 90α + 1461‬؛‬
‫‪. n = 5 × 73 + 7 2 β + 7 β + 6 = 56 β + 1721‬‬
‫ﺇﺫﻥ ‪ 90α + 1461 = 56 β + 1721‬ﻭﻤﻌﻨﺎﻩ ‪ 90α − 56 β = 260‬ﺃﻱ ‪45α − 28β = 130‬‬
‫ﺇﺫﻥ ]‪ α ≡ 0 [ 2‬ﻭ ]‪ β ≡ 0 [5‬ﺒﻤﺎ ﺃﻥ ‪ 0 ≤ β ≤ 7‬ﻓﺈﻥ ‪ β = 0‬ﺃﻭ ‪. β = 5‬‬
‫‪28‬‬
‫= ‪ α‬ﻤﺭﻓﻭﺽ‪.‬‬ ‫ﺇﺫﺍ ﻜﺎﻥ ‪ β = 0‬ﻓﺈﻥ ‪ 45α = 130‬ﺃﻱ‬
‫‪9‬‬
‫ﺇﺫﺍ ﻜﺎﻥ ‪ β = 5‬ﻓﺈﻥ ‪ 45α = 270‬ﺃﻱ ‪ . α = 6‬ﺇﺫﻥ ‪. n = 90α + 1461 = 2001‬‬
‫‪ 0 < y < 7 ، 0 < x < 7 99‬ﻭ ‪. 0 ≤ z < 7‬‬
‫‪ N = 113 x + 112 y + 11z + x‬ﺃﻱ ‪N = 1332x + 121y + 11z‬‬
‫‪ N = 73 y + 7 2 y + 7 x + z‬ﺃﻱ ‪N = 392 y + 7 x + z‬‬
‫ﺇﺫﻥ ‪ 1332x + 121y + 11z = 392 y + 7 x + z‬ﺃﻱ ‪ 1325x + 10z = 271y‬ﺃﻱ ‪5 ( 265x + 2z ) = 271y‬‬
‫ﺇﺫﻥ ]‪ 271y ≡ 0 [5‬ﻭﻤﻌﻨﺎﻩ ]‪ y ≡ 0 [5‬ﺒﻤﺎ ﺃﻥ ‪ 0 < y < 7‬ﻓﺈﻥ ‪. y = 5‬‬
‫ﻭﺒﺎﻝﺘﺎﻝﻲ ‪ 265x + 2z = 271‬ﺃﻱ ‪ 265x = 271 − 2z‬ﻭﻤﻨﻪ ]‪ 265x = 271[ 2‬ﺃﻱ ]‪x ≡ 1[ 2‬‬
‫ﺒﻤﺎ ﺃﻥ ‪ 0 < x < 7‬ﻓﺈﻥ ‪ x = 1‬ﺃﻭ ‪ x = 3‬ﺃﻭ ‪. x = 5‬‬
‫ﺇﺫﺍ ﻜﺎﻥ ‪ x = 1‬ﻓﺈﻥ ‪z = 3‬‬
‫ﺇﺫﺍ ﻜﺎﻥ ‪ x = 3‬ﻓﺈﻥ ‪ z‬ﻴﻜﻭﻥ ﺴﺎﻝﺏ‬
‫ﺇﺫﺍ ﻜﺎﻥ ‪ x = 5‬ﻓﺈﻥ ‪ z‬ﻴﻜﻭﻥ ﺴﺎﻝﺏ‬
‫ﻭﺒﺎﻝﺘﺎﻝﻲ )‪. ( x , y , z ) = (1,5,3‬‬
‫‪n = 1271x = 94 + 2 × 93 + 7 × 92 + 9 + x 100‬‬
‫‪ n ≡ 1 + 2 + 7 + 1 + x [8] (1‬ﺃﻱ ]‪. n ≡ 3 + x [8‬‬
‫ﻴﻜﻭﻥ ]‪ n ≡ 0 [8‬ﻤﻌﻨﺎﻩ ]‪ x + 3 ≡ 0 [8‬ﺃﻱ ]‪ x ≡ 5 [8‬ﻭﺒﻤﺎ ﺃﻥ ‪ 0 ≤ x < 9‬ﻓﺈﻥ ‪. x = 5‬‬
‫‪n = 1271x = 94 + 2 × 93 + 7 × 92 + 9 + x (2‬‬
‫‪ n = 93 ( 9 + 2 ) + 7 × 92 + 9 + x‬؛ ]‪n ≡ 4 + x [11‬‬
‫ﻴﻜﻭﻥ ]‪ n ≡ 0 [11‬ﻤﻌﻨﺎﻩ ]‪ x + 4 ≡ 0 [11‬ﺃﻱ ]‪ x ≡ 7 [11‬ﻭﺒﻤﺎ ﺃﻥ ‪ 0 ≤ x < 9‬ﻓﺈﻥ ‪. x = 7‬‬
‫‪ 0 ≤ x ≤ 9‬ﻭ ‪ n = 27 x 85 y . 0 ≤ y ≤ 9‬؛ ‪n = 2 ×105 + 7 ×104 + x ×103 + 8 ×102 + 5 ×10 + y‬‬ ‫‪101‬‬
‫]‪ n ≡ 2 + 7 + x + 8 + 5 + y [3‬ﻭ ]‪n ≡ −2 + 7 − x + 8 − 5 + y [11‬‬
‫ﻤﻌﻨﺎﻩ ]‪ n ≡ x + 1 + y [3‬ﻭ ]‪n ≡ − x + 8 + y [11‬‬
‫]‪ n ≡ 0 [3‬ﻭ ]‪ n ≡ 0 [11‬ﻤﻌﻨﺎﻩ ]‪ x + 1 + y ≡ 0 [3‬ﻭ ]‪− x + 8 + y ≡ 0 [11‬‬
‫ﺃﻱ ]‪ x + y ≡ 2 [3‬ﻭ ]‪. x − y ≡ 8 [11‬‬
‫]‪ x − y ≡ 8 [11‬ﻤﻌﻨﺎﻩ ]‪x − y ≡ −3[11‬‬
‫ﻭﻤﻨﻪ ‪( x , y ) ∈ {( 0,3) , (1, 4 ) , ( 2,5) , ( 3, 6 ) , ( 4, 7 ) ,‬‬
‫})‪( 5,8 ) , ( 6,9 ) , ( 8, 0 ) , ( 9,1‬‬
‫ﺒﻤﺎ ﺃﻥ ]‪ x + y ≡ 2 [3‬ﻓﺈﻥ }) ‪( x , y ) ∈ {(1, 4 ) , ( 4, 7 ) , (8, 0‬‬
‫‪ n = 271854‬ﺃﻭ ‪ n = 274857‬ﺃﻭ ‪. n = 278850‬‬
‫‪ (1 102‬ﺇﺫﺍ ﻜﺎﻥ ]‪ 3x ≡ 0 [ 7‬ﻓﺈﻥ ] ‪ 3 × 5x ≡ 0 × 5 [ 7‬ﻭﺒﻤﺎ ﺃﻥ ]‪ 15 ≡ 1[ 7‬ﻓﺈﻥ ]‪. x ≡ 0 [ 7‬‬
‫ﺍﻝﻌﻜﺱ ﺇﺫﺍ ﻜﺎﻥ ]‪ x ≡ 0 [ 7‬ﻓﺈﻥ ] ‪ 3x ≡ 3 × 0 [ 7‬ﻭﻤﻨﻪ ]‪. 3x ≡ 0 [ 7‬‬
‫‪ N = an an −1...a1a0 (2‬ﺃﻱ ‪N = an 10n + an −110n −1 + ... + a110 + a0‬‬
‫‪ N ' = an an −1...a1‬ﺃﻱ ‪ N ' = an 10 n −1 + an −110n − 2 + ... + a210 + a1‬ﻭﻤﻌﻨﺎﻩ‬
‫‪ 10N ' = an 10n + an −110n −1 + ... + a2102 + a110‬ﺇﺫﻥ ‪N = 10N '+ a0‬‬
‫] ‪ N ≡ 0 [ 7‬ﻤﻌﻨﺎﻩ ]‪ 10N '+ a0 ≡ 0 [ 7‬ﺃﻱ ]‪ 3N '− 6a0 ≡ 0 [ 7‬ﻭﻴﻜﺎﻓﺊ ] ‪ 3 ( N '− 2a0 ) ≡ 0 [ 7‬ﻭﻤﻌﻨﺎﻩ‬
‫] ‪ N '− 2a0 ≡ 0 [ 7‬ﻭﻫﺫﺍ ﺤﺴﺏ ﺍﻝﺴﺅﺍل ﺍﻝﺴﺎﺒﻕ ‪.‬‬
‫‪ 105154 ≡ 0 [ 7 ] (3‬ﻴﻜﺎﻓﺊ ] ‪ 10515 − 8 ≡ 0 [ 7‬ﺃﻱ ] ‪ 10507 ≡ 0 [ 7‬ﻭﻴﻜﺎﻓﺊ ] ‪ 1050 − 14 ≡ 0 [ 7‬ﺃﻱ‬
‫] ‪ 1036 ≡ 0 [ 7‬ﻭﻴﻜﺎﻓﺊ ] ‪ 103 − 12 ≡ 0 [ 7‬ﺃﻱ ] ‪ 91 ≡ 0 [ 7‬ﻭﻴﻜﺎﻓﺊ ] ‪ 9 − 2 ≡ 0 [ 7‬ﺃﻱ ] ‪7 ≡ 0 [ 7‬‬
‫ﺨﻼﺼﺔ ] ‪ 7 ≡ 0 [ 7‬ﻤﻌﻨﺎﻩ ] ‪. 105154 ≡ 0 [ 7‬‬
‫] ‪ 263572 ≡ 0 [ 7‬ﻤﻌﻨﺎﻩ ] ‪ 26357 − 4 ≡ 0 [ 7‬؛ ] ‪ 26353 ≡ 0 [ 7‬ﻴﻜﺎﻓﺊ ] ‪ 2635 − 6 ≡ 0 [ 7‬؛ ] ‪ 2629 ≡ 0 [ 7‬ﻴﻜﺎﻓﺊ‬
‫] ‪ 262 − 18 ≡ 0 [ 7‬؛ ] ‪ 244 ≡ 0 [ 7‬ﻴﻜﺎﻓﺊ ]‪ 24 − 8 ≡ 0 [ 7‬ﺃﻱ ] ‪ 16 ≡ 0 [ 7‬ﻭﻫﺫﺍ ﻏﻴﺭ ﺼﺤﻴﺢ ﺇﺫﻥ ‪ 263572‬ﻻ ﻴﻘﺒل‬
‫ﺍﻝﻘﺴﻤﺔ ﻋﻠﻰ ‪. 7‬‬
‫‪n −1‬‬ ‫‪n −2‬‬ ‫‪n −1‬‬
‫‪. N ' = an 10‬‬ ‫‪+ an −110‬‬ ‫‪+ ... + a210 + a1 ، N = an 10 + an −110‬‬
‫‪n‬‬
‫‪+ ... + a110 + a0 (1 103‬‬
‫‪ N ≡ 0 [13] . N = 10N '+ a0‬ﻤﻌﻨﺎﻩ ]‪ 10N '+ a0 ≡ 0 [13‬ﻭﻤﻌﻨﺎﻩ ]‪ 4 (10N '+ a0 ) ≡ 0 [13‬ﻭﻴﻜﺎﻓﺊ‬
‫]‪ 40N '+ 4a0 ≡ 0 [13‬ﺃﻱ ]‪ N '+ 4a0 ≡ 0 [13‬ﻷﻥ ]‪40 ≡ 1[13‬‬
‫‪ 1631216 ≡ 0 [13] (2‬ﻤﻌﻨﺎﻩ ]‪ 163121 + 24 ≡ 0 [13‬ﺃﻱ ]‪ 163144 ≡ 0 [13‬ﻤﻌﻨﺎﻩ ]‪ 16314 + 16 ≡ 0 [13‬ﺃﻱ‬
‫]‪ 16330 ≡ 0 [13‬ﻭﻤﻌﻨﺎﻩ ]‪ 1633 ≡ 0 [13‬ﻭﻤﻌﻨﺎﻩ ]‪ 163 + 12 ≡ 0 [13‬ﺃﻱ ]‪ 175 ≡ 0 [13‬ﻭﻤﻌﻨﺎﻩ ]‪17 + 20 ≡ 0 [13‬‬
‫ﺃﻱ ]‪ 37 ≡ 0 [13‬ﻭﻫﺫﺍ ﺘﻨﺎﻗﺽ ﺇﺫﻥ ‪ 1631216‬ﻻ ﻴﻘﺒل ﺍﻝﻘﺴﻤﺔ ﻋﻠﻰ ‪. 13‬‬
‫]‪ 48662029 ≡ 0 [13‬ﻤﻌﻨﺎﻩ ]‪ 4866202 + 36 ≡ 0 [13‬ﺃﻱ ]‪ 4866238 ≡ 0 [13‬ﻤﻌﻨﺎﻩ ]‪ 486623 + 32 ≡ 0 [13‬ﺃﻱ‬
‫]‪ 486655 ≡ 0 [13‬ﻤﻌﻨﺎﻩ ]‪ 48665 + 20 ≡ 0 [13‬ﺃﻱ ]‪ 48685 ≡ 0 [13‬ﻤﻌﻨﺎﻩ ]‪ 4868 + 20 ≡ 0 [13‬ﺃﻱ‬
‫]‪ 4888 ≡ 0 [13‬ﻤﻌﻨﺎﻩ ]‪ 488 + 32 ≡ 0 [13‬ﺃﻱ ]‪ 520 ≡ 0 [13‬ﻤﻌﻨﺎﻩ ]‪ 52 ≡ 0 [13‬ﺃﻱ ]‪ 5 + 8 ≡ 0 [13‬ﻤﻌﻨﺎﻩ‬
‫]‪ 13 ≡ 0 [13‬ﻭﻫﺫﺍ ﺼﺤﻴﺢ ﺇﺫﻥ ]‪. 48662029 ≡ 0 [13‬‬
‫‪. ( a 2 + a + 1)( a 2 − a + 1) = a 4 + a 2 + 1 (1 105‬‬
‫‪ (2‬ﻝﻴﻜﻥ ‪ a‬ﻋﺩﺩ ﻁﺒﻴﻌﻲ ﺤﻴﺙ ‪ 111 = a 2 + a + 1 . a > 1‬ﻭﻝﺩﻴﻨﺎ ‪ 10101 = 1× a 4 + 0 × a 3 + 1× a 2 + 0 × a + 1‬ﺃﻱ‬
‫‪ 10101 = a 4 + a 2 + 1‬ﻤﻌﻨﺎﻩ )‪ 10101 = ( a 2 + a + 1)( a 2 − a + 1‬ﺃﻱ )‪ 10101 = 111( a 2 − a + 1‬ﻭﻤﻨﻪ ‪ 111‬ﻴﻘﺴﻡ‬
‫‪. 10101‬‬
‫ﺍﻝﺤﺎﺼل ﻫﻭ ‪ a 2 − a + 1 = ( a − 1) a + 1‬ﺃﻱ ‪. a 2 − a + 1 = β a + 1 = β 1‬‬
‫‪ (1 106‬ﻓﻲ ﺍﻝﻨﻅﺎﻡ ﺍﻝﺘﻌﺩﺍﺩ ﺫﻱ ﺍﻷﺴﺎﺱ ‪ a‬ﻝﺩﻴﻨﺎ ‪ 11 = a + 1‬ﻭ )‪1001 = a 3 + 1 = ( a + 1) ( a 2 − a + 1‬‬
‫ﺃﻱ )‪ 1001 = 11( a 2 − a + 1‬ﻭﻤﻨﻪ ﺍﻝﻌﺩﺩ ‪ 1001‬ﻴﻘﺒل ﺍﻝﻘﺴﻤﺔ ﻋﻠﻰ ‪. 11‬‬
‫‪ (2‬ﺍﻝﺤﺎﺼل ﻫﻭ ﺃﻱ ‪. a 2 − a + 1 = ( a − 1) a + 1 = β 1‬‬
‫‪1001 = 11× β 1 (3‬‬
‫ﻓﻲ ﺍﻝﻨﻅﺎﻡ ﺫﻱ ﺍﻷﺴﺎﺱ ‪ 10‬ﻴﻜﻭﻥ ‪1001 = 11× 91‬‬
‫ﻓﻲ ﺍﻝﻨﻅﺎﻡ ﺫﻱ ﺍﻷﺴﺎﺱ ‪ 12‬ﻝﺩﻴﻨﺎ ‪. β 1 = 11× 12 + 1 = 133 ، 11 = 12 + 1 = 13 ، 1001 = 123 + 1 = 1729‬‬
‫ﻭﻝﺩﻴﻨﺎ ‪. 13 ×133 = 1729‬‬
‫‪ (1 107‬ﻝﻴﻜﻥ ‪ a‬ﻋﺩﺩ ﻁﺒﻴﻌﻲ ‪ ( a + 1) = a 3 + 3a 2 + 3a + 1 ،‬ﻭﻤﻨﻪ ﺇﺫﺍ ﻜﺎﻥ ‪ a > 3‬ﻓﻴﻜﻭﻥ ﻓﻲ ﺍﻷﺴﺎﺱ ‪، a‬‬
‫‪3‬‬

‫‪. ( a + 1) = 1331‬‬
‫‪3‬‬

‫)‪( a + 1‬‬ ‫‪ ( a + 1) = ( a 3 + 3a 2 + 3a + 1) ( a + 1) (2‬؛ ‪= a 4 + 4a 3 + 6a 2 + 4a + 1‬‬


‫‪4‬‬ ‫‪4‬‬

‫ﻭﻤﻨﻪ ﺇﺫﺍ ﻜﺎﻥ ‪ a > 6‬ﻓﻴﻜﻭﻥ ﻓﻲ ﺍﻷﺴﺎﺱ ‪. ( a + 1) = 14641 ، a‬‬


‫‪4‬‬

‫‪ n 2 + 2 = ( n 2 + 1) + 1 = 11 (1 108‬؛ )‪ n 2 + 2n = ( n 2 + 1) + ( 2n − 1) = 1( 2n − 1‬؛‬

‫‪ ( n 2 + 2 ) = ( n 2 + 1) + 1‬؛ ‪ ( n 2 + 2 ) = ( n 2 + 1) + 2 ( n 2 + 1) + 1 = 121‬؛‬


‫‪2‬‬ ‫‪2‬‬ ‫‪2‬‬ ‫‪2‬‬

‫‪ n 4 = ( n 4 − 1) + 1 = ( n 2 − 1)( n 2 + 1) + 1‬؛ ‪. n 4 = ( n 2 − 1)1‬‬


‫ـ ﺍﻝﺘﺤﻘﻴﻕ ﻤﻥ ﺃﺠل ﺍﻷﺴﺎﺱ ‪ a = 5‬ﺃﻱ ‪. n = 2‬‬
‫‪ n 2 + 2 = 6‬ﻭ ‪. 11 = 5 + 1 = 6‬‬
‫‪ n 2 + 2n = 8‬ﻭ ‪. 1( 2n − 1) = 5 + 3 = 8‬‬
‫‪ ( n 2 + 2 ) = 36‬ﻭ ‪. 121 = 52 + 2 × 5 + 1 = 36‬‬
‫‪2‬‬

‫‪ n 4 = 16‬ﻭ ‪. ( n 2 − 1)1 = 3 × 5 + 1 = 16‬‬


‫ـ ﺍﻝﺘﺤﻘﻴﻕ ﻤﻥ ﺃﺠل ﺍﻷﺴﺎﺱ ‪ a = 10‬ﺃﻱ ‪. n = 3‬‬
‫‪ n 2 + 2 = 11‬ﻭ ‪. 11 = 10 + 1 = 11‬‬
‫‪ n 2 + 2n = 15‬ﻭ ‪. 1( 2n − 1) = 10 + 5 = 15‬‬
‫‪ ( n 2 + 2 ) = 121‬ﻭ ‪. 121 = 121‬‬
‫‪2‬‬

‫‪ n 4 = 81‬ﻭ ‪. ( n 2 − 1)1 = 8 × 10 + 1 = 81‬‬


‫‪u = n ( n 2 + 2 ) = n ( a + 1) = na + n = nn (2‬‬
‫‪v = n 2 ( n 2 + 2 ) = n 2 ( a + 1) = n 2a + n 2 = n 2 n 2‬‬
‫‪ x = u 2 = n 2 ( a + 1) = n 2a 2 + 2n 2a + n 2‬؛ ‪x = ( a − 1) a 2 + 2 ( a − 1) a + n 2 = a 3 + a 2 − 2a + n 2‬‬
‫‪2‬‬

‫‪ x = a 3 + ( a − 2 ) a + n 2 = a 3 + ( n 2 − 1) a + n 2‬؛ ‪. x = 10 ( n 2 − 1) n 2‬‬

‫‪ y = v 2 = ( n 2a + n 2 ) = n 4a 2 + 2n 4a + n 4‬؛ )‪ y = ( a − 1) a 2 + 2 ( a − 1) a + ( a − 1‬؛‬
‫‪2‬‬ ‫‪2‬‬ ‫‪2‬‬ ‫‪2‬‬
‫‪ y = a 4 − 2a 3 + a 2 + 2a 3 − 4a 2 + 2a + a 2 − 2a + 1‬؛ ‪y = a 4 − 2a 2 + 1 = ( n 2 + 1) a 3 − 2a 2 + 1‬‬
‫‪ y = a 4 − 2a 2 + 1 = n 2a 3 + a 2 ( a − 2 ) + 1‬؛ ‪y = a 4 − 2a 2 + 1 = n 2a 3 + a 2 ( n 2 − 1) + 1‬‬

‫‪. y = a 4 − 2a 2 + 1 = n 2 ( n 2 − 1) 01‬‬

‫ﺍﻝﻤﺴﺎﺌل‬
‫‪ (1 (I 109‬ﻨﻔﺘﺭﺽ ﺃﻥ ‪ a‬ﻭ ‪ b‬ﺯﻭﺠﻴﺎﻥ ﻤﻌﺎ ‪ ،‬ﻤﻌﻨﺎﻩ ]‪ a ≡ 0 [ 2‬ﻭ ]‪ b ≡ 0 [ 2‬ﻭﻤﻨﻪ ]‪ a 2 ≡ 0 [ 2‬ﻭ ]‪b 2 ≡ 0 [ 2‬‬
‫ﻭﺒﺎﻝﺘﺎﻝﻲ ]‪ a 2 − b 2 ≡ 0 [ 2‬ﺃﻱ ]‪. N ≡ 0 [ 2‬‬
‫ﻨﻔﺘﺭﺽ ﺃﻥ ‪ a‬ﻭ ‪ b‬ﻓﺭﺩﻴﺎﻥ ﻤﻌﺎ ‪ ،‬ﻤﻌﻨﺎﻩ ]‪ a ≡ 1[ 2‬ﻭ ]‪ b ≡ 1[ 2‬ﻭﻤﻨﻪ ]‪ a 2 ≡ 1[ 2‬ﻭ ]‪b 2 ≡ 1[ 2‬‬
‫ﻭﺒﺎﻝﺘﺎﻝﻲ ]‪ a 2 − b 2 ≡ 0 [ 2‬ﺃﻱ ]‪N ≡ 0 [ 2‬‬
‫ﺇﺫﻥ ﺇﺫﺍ ﻜﺎﻥ ‪ a‬ﻭ ‪ b‬ﻤﻥ ﻨﻔﺱ ﺍﻝﺸﻔﻌﻴﺔ ﻓﻴﻜﻭﻥ ‪ N‬ﻋﺩﺩﺍ ﻁﺒﻴﻌﻴﺎ ﺯﻭﺠﻴﺎ ﻭﻫﺫﺍ ﺘﻨﺎﻗﺽ ﻷﻥ ‪ N‬ﻋﺩﺩ ﻁﺒﻴﻌﻲ ﻓﺭﺩﻱ ﻭﺒﺎﻝﺘﺎﻝﻲ ‪a‬‬
‫ﻭ ‪ b‬ﻝﻴﺱ ﻤﻥ ﺸﻔﻌﻴﺔ ﻭﺍﺤﺩﺓ ‪.‬‬
‫‪ N = a 2 − b 2 = ( a − b )( a + b ) (2‬؛ ﺒﻭﻀﻊ ‪ a − b = p‬ﻭ ‪ a + b = q‬ﻴﻜﻭﻥ ‪. N = pq‬‬
‫‪ (3‬ﺒﻤﺎ ﺃﻥ ‪ a‬ﻭ ‪ b‬ﻝﻴﺱ ﻤﻥ ﺸﻔﻌﻴﺔ ﻭﺍﺤﺩﺓ ﻓﺈﻥ ﻤﺠﻤﻭﻋﻬﻤﺎ ﻭﻓﺭﻗﻬﻤﺎ ﻴﻜﻭﻨﺎ ﻓﺭﺩﻴﻴﻥ ﺃﻱ ‪ p‬ﻭ ‪ q‬ﻓﺭﺩﻴﻴﻥ ﻤﻌﺎ ‪.‬‬
‫‪ (1‬أ ـ ‪x 0 1 2 3 4 5 6 7 8 [9] (II‬‬
‫]‪x 2 0 1 4 0 7 7 0 4 1 [ 9‬‬
‫ب ـ ]‪a 2 − 250507 ≡ a 2 − 1[9‬‬
‫‪b‬‬ ‫]‪0 1 2 3 4 5 6 7 8 [ 9‬‬
‫‪b2‬‬ ‫]‪0 1 4 0 7 7 0 4 1 [ 9‬‬
‫]‪a 2 − 250507 0 1 4 0 7 7 0 4 1 [9‬‬
‫‪a 2 −1‬‬ ‫]‪0 1 4 0 7 7 0 4 1 [ 9‬‬
‫‪a2‬‬ ‫]‪1 2 5 1 8 8 1 5 2 [ 9‬‬
‫‪4‬ـ ـ ﻤﻥ ﺍﻝﺴﺅﺍل ﺃ ـ ﻻ ﻴﻤﻜﻥ ﺃﻥ ﺘﻜﻭﻥ ﺍﻷﻋﺩﺍﺩ ‪ 5 ، 2‬ﻭ ‪ 8‬ﺒﻭﺍﻗﻲ ﻝﻤﺭﺒﻊ ﺒﺘﺭﺩﻴﺩ ‪ 9‬ﻭﺒﺎﻝﺘﺎﻝﻲ ]‪ a 2 ≡ 1[9‬ﻫﻲ ﺍﻝﺤﺎﻝﺔ ﺍﻝﻭﺤﻴﺩﺓ‬
‫ﺍﻝﻤﻤﻜﻨﺔ ﻭﻴﻨﺘﺞ ﺃﻥ ]‪ a ≡ 1[ 9‬ﺃﻭ ]‪. a ≡ 8 [9‬‬
‫‪ (2‬ﺃ ـ ‪ a 2 − 250507 = b 2‬ﻭﻤﻨﻪ ‪ a 2 − 250507 ≥ 0‬ﻭﻴﻜﺎﻓﺊ ‪ a ≤ − 250507‬ﺃﻭ ‪ a ≥ 250507‬ﺒﻴﻨﻤﺎ ‪ a‬ﻋﺩﺩ‬
‫ﻁﺒﻴﻌﻲ ﺇﺫﻥ ‪ a ≥ 250507‬ﺃﻱ ‪ a ≥ 500,51‬ﻭﻤﻨﻪ ‪. a ≥ 501‬‬
‫ﺏ ـ ‪ a 2 − 250507 = 5012 − 250507 = 494‬ﺇﺫﻥ ‪ b 2 = 494‬ﺃﻱ ‪ b = 22, 23‬ﺇﺫﻥ ‪. a ≠ 501‬‬
‫‪ (3‬ﺃ ـ ﻨﻔﺭﺽ ]‪ ، a ≡ 8 [9‬ﻝﺩﻴﻨﺎ ]‪ 503 ≡ 8 [9‬ﻤﻌﻨﺎﻩ ]‪ 8 ≡ 503[9‬ﻭﻤﻨﻪ ]‪. a ≡ 503[9‬‬
‫ﻨﻔﺭﺽ ]‪ ، a ≡ 1[ 9‬ﻝﺩﻴﻨﺎ ]‪ 505 ≡ 1[9‬ﻤﻌﻨﺎﻩ ]‪ 1 ≡ 505 [9‬ﻭﻤﻨﻪ ]‪. a ≡ 505 [9‬‬
‫ﺏ ـ ‪ a = 505 + 9k‬ﻤﻌﻨﺎﻩ ‪ a 2 − 250507 = 81k 2 + 9090k+4518‬ﻭﻴﻜﺎﻓﺊ ‪ b 2 = 81k 2 + 9090k+4518‬ﺃﻱ‬
‫) ‪b 2 = 9 ( 9k 2 + 1010k+502‬‬
‫ﻤﻥ ﺃﺠل ‪ k = 0‬ﻝﺩﻴﻨﺎ ‪ b 2 = 4518‬ﺃﻱ ‪ b = 3 502‬ﻤﺭﻓﻭﺽ ‪.‬‬
‫ﻤﻥ ﺃﺠل ‪ k = 1‬ﻝﺩﻴﻨﺎ ‪ b 2 = 9 × 1521=117 2‬ﺃﻱ ‪ b =117‬ﺇﺫﻥ ﺃﺼﻐﺭ ﻋﺩﺩ ﻁﺒﻴﻌﻲ ‪ k‬ﺤﻴﺙ ﺘﻜﻭﻥ ﺍﻝﺜﻨﺎﺌﻴﺔ‬
‫) ‪ ( 505 + 9k , b‬ﺘﺤﻘﻕ ﺍﻝﻌﻼﻗﺔ ) ‪ ( E‬ﻫﻭ ‪ k = 1‬ﻭﺒﺎﻝﺘﺎﻝﻲ ) ‪. ( a, b ) = ( 514,117‬‬
‫‪ a 2 − 250507 = b 2 (1 (III‬ﻤﻌﻨﺎﻩ ) ‪ 250507 = a 2 − b 2 = ( a − b )( a + b‬ﺃﻱ ‪. 250507 = 397 × 631‬‬
‫‪1‬‬ ‫‪1‬‬ ‫‪1 2 3 2 1 1 1 2‬‬ ‫‪(2‬‬
‫‪631 397 234 163 71 21 8 5 3 2 1 0‬‬
‫ﺇﺫﻥ ‪. p gcd ( 631,397 ) = 1‬‬
‫‪ 110‬ﺠﺯﺀ ‪ I‬ـ‬
‫‪ (1‬ﻝﺩﻴﻨﺎ ‪ 12 + 32 + 52 = 35 = 4 × 8 + 3‬ﻭﻤﻨﻪ ]‪ 12 + 32 + 52 ≡ 3 [ 4‬ﺃﻱ ‪12 + 32 + 52 ≡ 22 − 1  22 ‬‬
‫‪ (2‬ﻨﻔﺘﺭﺽ ﺃﻥ ‪. n = 3‬‬
‫أـ ‪r 0 1 2 3 4 5 6 7‬‬
‫‪R 0 1 4 1 0 1 4 1‬‬
‫ب ـ ﻤﻥ ﺃﺠل ﻜل ﺜﻼﺙ ﺃﻋﺩﺍﺩ ‪ R 2 ، R1‬ﻭ ‪ R 3‬ﻤﻥ ﺍﻝﻤﺠﻤﻭﻋﺔ }‪ {0,1, 4‬ﻴﻜﻭﻥ ‪. R1 + R 2 + R 3 ≠ 0‬‬
‫ﺇﺫﻥ ﻻ ﻴﻤﻜﻥ ﺇﻴﺠﺎﺩ ‪ y ، x‬ﻭ ‪ z‬ﺤﻴﺙ ]‪. x 2 + y 2 + z 2 ≡ 7 [8‬‬
‫ﺍﻝﺠﺯﺀ ‪ II‬ـ ﺩﺭﺍﺴﺔ ﺍﻝﺤﺎﻝﺔ ﺍﻝﻌﺎﻤﺔ ﻤﻊ ‪. n ≥ 3‬‬
‫‪ x + y + z = 2 p + 2 − 1 = 2‬ﻭﻤﻨﻪ‬
‫‪2‬‬ ‫‪2‬‬ ‫‪2‬‬ ‫‪n‬‬ ‫‪n‬‬ ‫‪n‬‬
‫ﻤﻌﻨﺎﻩ ‪( p + 1) − 1‬‬ ‫‪ (1‬ﻝﺩﻴﻨﺎ ‪x 2 + y 2 + z 2 ≡ ( 2n − 1)  2n ‬‬
‫‪ x 2 + y 2 + z 2‬ﻋﺩﺩ ﻓﺭﺩﻱ ‪.‬‬
‫) ‪. ( x + y + z ) = x 2 + y 2 + z 2 + 2 ( xy + xz + yz‬‬
‫‪2‬‬

‫ﺇﺫﻥ ) ‪ ( x + y + z‬ﻫﻭ ﻋﺩﺩ ﻓﺭﺩﻱ ﻭﺒﺎﻝﺘﺎﻝﻲ ‪ x + y + z‬ﻋﺩﺩ ﻓﺭﺩﻱ ‪.‬‬


‫‪2‬‬

‫ﺇﺫﻥ ﺘﻜﻭﻥ ﺍﻷﻋﺩﺍﺩ ‪ y ، x‬ﻭ ‪ z‬ﻜﻠﻬﺎ ﻓﺭﺩﻴﺔ ﺃﻭ ﻭﺍﺤﺩ ﻤﻨﻬﺎ ﻓﺭﻱ ﻭﺁﺨﺭﻴﻥ ﺯﻭﺠﻴﻴﻥ ‪.‬‬
‫‪ (2‬ﻨﻔﺘﺭﺽ ﺃﻥ ‪ x‬ﻭ ‪ y‬ﺯﻭﺠﻴﺎﻥ ﻭ ‪ z‬ﻓﺭﺩﻱ ‪.‬‬
‫أ ـ ‪ y = 2k ، x = 2 p‬ﻭ ‪ z = 2l + 1‬ﻭﻤﻨﻪ ‪ y 2 = 4k ، x 2 = 4 p 2‬ﻭ ‪ z 2 = 4l 2 + 4l + 1‬ﺇﺫﻥ ]‪، x 2 ≡ 0 [ 4‬‬
‫]‪ y 2 ≡ 0 [ 4‬ﻭ ]‪ z 2 ≡ 1[ 4‬ﻭﺒﺎﻝﺘﺎﻝﻲ ]‪. x 2 + y 2 + z 2 ≡ 1[ 4‬‬
‫ب ـ ﺒﻤﺎ ﺃﻥ ‪ n ≥ 3‬ﻓﺈﻥ ]‪ 2n ≡ 0 [ 4‬ﺃﻱ ‪ 2n = 4α‬ﻭﻝﺩﻴﻨﺎ ‪ x 2 + y 2 + z 2 = 2n p + 2n − 1 = 2n ( p + 1) − 1‬ﺃﻱ‬
‫‪ x 2 + y 2 + z 2 = 4α ( p + 1) − 1‬ﻭﺒﺎﻝﺘﺎﻝﻲ ]‪ x 2 + y 2 + z 2 ≡ −1[ 4‬ﺃﻱ ]‪ x 2 + y 2 + z 2 ≡ 3[ 4‬ﻭﻫﺫﺍ ﺘﻨﺎﻗﺽ ﻤﻊ‬
‫ﺍﻝﻨﺘﻴﺠﺔ ﺍﻝﺴﺎﺒﻘﺔ ‪.‬‬
‫‪ (3‬ﻨﻔﺘﺭﺽ ﺃﻥ ‪ y ، x‬ﻭ ‪ z‬ﻜﻠﻬﺎ ﻓﺭﺩﻴﺔ ‪.‬‬
‫أ ـ )‪ k 2 + k = k ( k + 1‬ﺠﺩﺍﺀ ﻋﺩﺩﻴﻥ ﻤﺘﺘﺎﻝﻴﻴﻥ ﻫﻭ ﻋﺩﺩ ﺯﻭﺠﻲ ﺃﻱ ]‪. k 2 + k ≡ 0 [ 2‬‬
‫ب ـ ﻝﻴﻜﻥ ‪ t‬ﻋﺩﺩ ﻁﺒﻴﻌﻲ ﻓﺭﺩﻱ ﺃﻱ ‪ t = 2k + 1‬ﻭﻤﻨﻪ ‪ t 2 = 4k 2 + 4k + 1 = 4 ( k 2 + k ) + 1‬ﻭﺒﻤﺎ ﺃﻥ‬
‫]‪ k 2 + k ≡ 0 [ 2‬ﻓﺈﻥ ' ‪ k 2 + k ≡ 2k‬ﺇﺫﻥ ‪ t 2 = 8k '+ 1‬ﺃﻱ ]‪t 2 ≡ 1[8‬‬
‫ﺒﻤﺎ ﺃﻥ ‪ y ، x‬ﻭ ‪ z‬ﻜﻠﻬﺎ ﻓﺭﺩﻴﺔ ‪ .‬ﻓﺈﻥ ]‪ y 2 ≡ 1[8] ، x 2 ≡ 1[8‬ﻭ ]‪ z 2 ≡ 1[8‬ﻭﺒﺎﻝﺘﺎﻝﻲ ]‪. x 2 + y 2 + z 2 ≡ 3[8‬‬
‫ـ ـ ﺍﻝﺨﻼﺼﺔ ‪x 2 + y 2 + z 2 = 2n ( p + 1) − 1 :‬‬
‫ﻤﻥ ﺃﺠل ‪ n ≥ 3‬ﻴﻜﻭﻥ ]‪ 2n ≡ 0 [8‬ﺇﺫﻥ ]‪ x 2 + y 2 + z 2 ≡ −1[8‬ﺃﻱ ]‪ x 2 + y 2 + z 2 ≡ 7 [8‬ﻭﻫﺫﺍ ﺘﻨﺎﻗﺽ ﻤﻊ ﺍﻝﻨﺘﻴﺠﺔ‬
‫] ‪x 2 + y 2 + z 2 ≡ 3 [8‬‬
‫ﺇﺫﻥ ﻻ ﺘﻭﺠﺩ ﺃﻱ ﺜﻼﺜﻴﺔ ) ‪ ( x , y , z‬ﺘﺤﻘﻕ ‪ x 2 + y 2 + z 2 ≡ ( 2n − 1)  2n ‬ﻤﻊ ‪ n ≥ 3‬ﺴﻭﺍﺀ ﻜﺎﻨﺕ ‪ y ، x‬ﻭ ‪ z‬ﻜﻠﻬﺎ‬
‫) ‪(x , y , z‬‬ ‫ﻓﺭﺩﻴﺔ ﺃﻭ ﻭﺍﺤﺩ ﻤﻨﻬﺎ ﻓﺭﺩﻱ ﻭﺍﻵﺨﺭﻴﻥ ﺯﻭﺠﻴﻴﻥ ؛ ﻭﺒﺎﻝﺘﺎﻝﻲ ‪ n = 2‬ﻫﻲ ﺍﻝﺤﺎﻝﺔ ﺍﻝﻭﺤﻴﺩﺓ ﺍﻝﺘﻲ ﺘﻭﺠﺩ ﻓﻴﻬﺎ ﺜﻼﺜﻴﺔ‬
‫ﺘﺤﻘﻕ ‪x 2 + y 2 + z 2 ≡ ( 2n − 1)  2n ‬‬
‫ﺍﻷﻧﺸﻄﺔ‬
‫ا
ط اول‬
‫ ‪/ :‬‬
‫ا
ف‪ :‬ر
 م ا |> ا ‪'-‬ك ا^‪.';f‬‬
‫ ت‪ :‬م ا ‪-‬ط آ*) ('ة " ا |> ا ‪'-‬ك ا^‪6 ';f‬د"‪.‬‬
‫ا
‪:‬‬
‫‪-‬ط ‪: 1‬‬
‫‪ (1‬ا (‪oa‬ت ا  '   ا |ء ا^ول إ ‪ j‬ا^*|' ه ‪ k A 90k‬د ‪. 61Z‬‬
‫‪ (2‬ا (‪oa‬ت ا  '   ا |ء ا ‪ V‬إ ‪ j‬ا^*|' ه ‪ k A 95k‬د ‪.61Z‬‬
‫‪M 90 = {0,90,180, 270,...,1710,...,86310,86400} (3‬‬
‫‪M 95 = {0,95,190, 285,...,1710,...,86260,86355} (4‬‬
‫‪. M 90 ∩ M 95 = {0,1710,3420,...,83790,85500} (5‬‬
‫‪ (6‬ا (‪oa‬ت ﺒﺎﻝﺜﺎﻨﻴﺔ ﺍﻝﺘﻲ ﻴﻤ ‪‬ﺭ ﻓﻴﻬﺎ ﺍﻝﻀﻭﺀﺍﻥ ﺇﻝﻰ ﺍﻷﺨﻀﺭ ﻓﻲ ﺁﻥ ﻭﺍﺤﺩ ه  ‪ 'f‬ا !
‪M 90 ∩ M 95‬‬
‫‪ >m  6) 1710 (7‬ا ()( ‪.‬‬
‫‪ (8‬ا ‪
/‬ا ‪3
6/‬ا‪ >m  6 25200s S‬ا () و ا ‪
/‬ا ‪
6/‬وا ‪3 >m‬ا‪ >m  6 27000s S‬ا ()‬
‫‪  14, 73 ≤ k ≤ 15, 78 ! 25200 ≤ 1710k ≤ 27000 A‬أن ‪ k‬د ‪v 61Z‬ن ‪. k = 15‬‬
‫و   ا (‪ 
oa‬ﺍﻝﺴﺎﻋﺔ ﺍﻝﺴﺎﺒﻌﺔ ﻭﺍﻝﺴﺎﻋﺔ ﺍﻝﺴﺎﺒﻌﺔ ﻭﺍﻝﻨﺼﻑ ﻫﻲ ‪ 26650s‬ﺃﻱ ﺍﻝﺴﺎﻋﺔ ‪ 7‬ﻭ ‪ 16‬ﺍﻝﺩﻗﻴﻘﺔ ﻭ ‪ 40‬ﺜﺎﻨﻴﺔ‪.‬‬
‫‪. t = 90u ( a (9‬‬
‫‪875‬‬ ‫‪45000‬‬
‫‪.‬‬ ‫= ‪ V m = 45Km / h‬ا ‪ kR‬ا ‪';/‬ق  ا |‪ D‬ه ‪s = 70s‬‬ ‫‪m / s = 12,5m / s (b‬‬
‫‪12, 5‬‬ ‫‪3600‬‬
‫إذا آن ‪ u‬د ا 'ات '   ا |ء ا^ول إ ‪ j‬ا^*|' *‪:‬ل ا ‪v t X‬ن ‪ v + 1‬ه د ا 'ات '   ا |ء ا ‪V‬‬
‫إ ‪ j‬ا^*|' و @   ا ‪f‬ل إ ‪ U‬أي *‪:‬ل ا ‪ t + 70 X‬و   )‪. t + 70 = 95 (v + 1‬‬
‫‪ 90u − 95v = 95 − 70 = 25 ! t + 70 = 95 (v + 1)  t = 90u i6 (c‬أي ‪. 18u − 19v = 5‬‬
‫‪ 18u − 19v = 5 = 95 − 90 = 19 × 5 − 18 × 5 (d‬و‪18u + 18 × 5 = 19v + 19 × 5  6‬‬
‫أي )‪. 18(u + 5) = 19(v + 5‬‬
‫‪ (e‬ا‪ u #R C 7‬ﻭ ‪ v‬ﺜﻡ ﻗﻴﻡ ‪t‬‬
‫ا ‪6‬د )‪ 18(u + 5‬ﻴﻘﺒل ﺍﻝﻘﺴﻤﺔ ﻋﻠﻰ ‪ 19‬ﻭﺒﻤﺎ ﺃﻥ ‪ 19‬ﺃﻭﻝﻲ ﻓﺈﻨﻪ ﻤﻭﺠﻭﺩ ﻓﻲ ﺘﺤﻠﻴل )‪ 18(u + 5‬ﺇﻝﻰ ﺠﺩﺍﺀ ﻋﻭﺍﻤل ﺃﻭ‪‬ﻝﻴﺔ ﻭﺒﻤﺎ ﺃﻨﻪ‬
‫ﺃﻭﻝﻲ ﻤﻊ ‪ 18‬ﻓﺈﻨﻪ ﻏﻴﺭ ﻤﻭﺠﻭﺩ ﻓﻲ ﺘﺤﻠﻴل ‪ 18‬ﺇﺫﻥ ﻫﻭ ﻤﻭﺠﻭﺩ ﻓﻲ ﺘﺤﻠﻴل )‪ (u + 5‬ﺃﻱ ‪ 19‬ﻗﺎﺴﻡ ﻝﹻ )‪(u + 5‬‬
‫ﺃﻱ ‪ u + 5 = 19α :‬ﻤﻊ ‪ α‬ﻋﺩﺩ ﻁﺒﻴﻌﻲ ﻭﺒﺎﻝﺘﻌﻭﻴﺽ ﻓﻲ )‪ 18(u + 5) = 19(v + 5‬ﻨﺠﺩ ‪v + 5 = 18α‬‬
‫ﺨﻼﺼﺔ ‪ u = 19α − 5‬ﻭ ‪ v = 18α − 5‬ﻤﻊ ‪ α‬ﻋﺩﺩ ﻁﺒﻴﻌﻲ‪t = 90u = 90 (19α − 5) = 1710α − 450 .‬‬
‫‪ 25200 ≤ 1710α − 450 ≤ 27000 (f‬أي ‪ 15 ≤ α ≤ 16, 05‬إذن ‪t = 1710 × 15 − 450 = 25200‬‬
‫ﺃﻭ ‪ t = 1710 × 16 − 450 = 26910‬ﺃﻱ ‪ :‬ﺍﻝﺴﺎﻋﺔ ‪ 7‬ﻭ ‪ 0‬ﺩﻗﻴﻘﺔ ﻭ ‪ 0‬ﺜﺎﻨﻴﺔ ﺃﻭ ﺍﻝﺴﺎﻋﺔ ‪ 7‬ﻭ ‪ 28‬ﺩﻗﻴﻘﺔ ﻭ ‪ 30‬ﺜﺎﻨﻴﺔ‪.‬‬
‫ا
ط ا
‬
‫ ‪/ :‬‬
‫ا
ف‪ >?3 :‬ا ات و ا^اد ا^و 
 و‪ 
6‬د‪.
9‬‬
‫ ت‪ :‬م  أاج‪.‬‬
‫ا
‪./ :‬‬
‫ﺍﻷﻋﻤﺎﻝ ﺍﻟﻤﻮﺟﻬﺔ‬
‫! ‪[  :+!+‬و‬
‫ ‪/ :‬‬
‫ا
ف‪! >?3 :‬ول اآ‪/‬ل ‪X (6 6‬و‪.‬‬
‫ ت‪ :‬م ا ‪ )@2  )6‬أاج ‪ A‬ا‪:;7‬ل ‪ 4‬ز ا ا‪.23‬‬
‫ا
‪ @ :‬إ‪13‬ع ‪ >(F‬ا ‪ 4‬ت ا 
‪(1‬غ ا ‪ CD‬ا *ة‪.‬‬

‫ا
‪/2‬ه ‪ %‬ا
‪ S‬ة
ِـ \ ‪+‬‬
‫ ‪/ :‬‬
‫ا
ف‪ >?3 :‬ا ات و ا^اد ا^و 
‪.‬‬
‫ ت‪ #3 @ :‬ا ‪ )@2  )6‬أاج آ @ ا‪'R‬ا‪ U‬آا‪. X  Y4‬‬
‫ا
‪.6 :‬‬

‫ﺍﻟﺘﻤﺎﺭﻳﻦ‬
‫ﺍﻝﺘﻤﺎﺭﻴﻥ ﺍﻝﺘﻁﺒﻴﻘﻴﺔ‬
‫‪ 1‬ـ ا‪$‬اد اوّ
‪. %‬‬
‫‪ 2‬ﺃ ـ ‪ 1429‬ﻻ ﻴﻘﺒل ﺍﻝﻘﺴﻤﺔ ﻋﻠﻰ ‪ 2‬ﻭﻻ ﻋﻠﻰ ‪ 3‬ﻭﻻ ﻋﻠﻰ ‪ 5‬؛ ‪، 1429 = 11× 129 + ، 1429 = 7 × 204 +‬‬
‫‪، 1429 = 29 × 49 + ، 1429 = 23 × 62 + ، 1429 = 19 × 75 + ، 1429 = 17 × 84 + ، 1429 = 13 ×109 +‬‬
‫‪ 1429 = 41× 34 + ، 1429 = 37 × 38 + ، 1429 = 31× 46 +‬؛ ﻋﺩﺩ ﺍﻝﻘﺴﻤﺎﺕ ﻫﻭ ‪.13‬‬
‫ﺏ ـ ‪ 1429‬ﺃﻭﻝﻲ ‪.‬‬
‫‪ 3‬ﺃ ـ ﺍﻝﺨﺎﺼﻴﺔ ‪ 2‬ﺼﻔﺤﺔ ‪. 90‬‬
‫ﻭ ‪ 853‬ﻻ ﻴﻘﺒل ﺍﻝﻘﺴﻤﺔ ﻋﻠﻰ ﺍﻷﻋﺩﺍﺩ ﺍﻷﻭﻝﻴﺔ ‪ ، 29 ، 23 ، 19 ، 17 ، 13 ، 11 ، 7 ، 5 ، 3 ، 2‬ﺇﺫﻥ‬ ‫ب ـ ‪853 ≈ 29, 2‬‬
‫ﺍﻝﻌﺩﺩ ‪ 853‬ﺃﻭﻝﻲ‪.‬‬
‫‪ 4‬أ ـ ‪ 251‬ﺃﻭﻝﻲ‪ .‬ب ـ ‪ 341‬ﻝﻴﺱ ﺃﻭﻝﻴﺎ‪.‬‬
‫ج ـ ‪ 1023‬ﻝﻴﺱ ﺃﻭﻝﻴﺎ‪.‬‬
‫‪ 7‬ﺇﺫﺍ ﻜﺎﻥ ‪ n = 2‬ﻓﺈﻥ ‪ n + 7 = 9‬ﻭﻫﻭ ﻏﻴﺭ ﺃﻭﻝﻲ‪.‬‬
‫ﺇﺫﺍ ﻜﺎﻥ ‪ n > 2‬ﻓﺈﻥ ‪ n‬ﻓﺭﺩﻱ ﻭﻤﻨﻪ ‪ n + 7‬ﻴﻜﻭﻥ ﺯﻭﺠﻴﺎ ﻴﻘﺒل ﺍﻝﻘﺴﻤﺔ ﻋﻠﻰ ‪ 2‬ﻭﻫﻭ ﻏﻴﺭ ﺃﻭﻝﻲ‪.‬‬
‫‪ n 2 + 8n + 15 = ( n + 3)( n + 5 ) 15‬ﻝﻴﺱ ﺃﻭﻝﻴﺎ ‪.‬‬
‫ﻭ ‪ 173‬ﻻ ﻴﻘﺒل ﺍﻝﻘﺴﻤﺔ ﻋﻠﻰ ‪ 13 ، 11 ، 7 ، 5 ، 3 ، 2‬ﺇﺫﻥ ﺍﻝﻌﺩﺩ ‪ 173‬ﺃﻭﻝﻲ ‪.‬‬ ‫‪ 17‬أ ـ ‪173 ≈ 13,15‬‬
‫ب ـ ‪ x 2 − y 2 = 173‬ﻤﻌﻨﺎﻩ ‪( x − y )( x + y ) = 173‬‬
‫ﺇﺫﻥ ‪ x − y = 1‬ﻭ ‪ x + y = 173‬ﺃﻱ ) ‪( x , y ) = ( 87,86‬‬
‫‪ p + 1 p −1 ‬‬
‫‪. (x , y ) = ‬‬ ‫‪,‬‬ ‫ﺝ ـ ‪ p‬ﻋﺩﺩ ﻁﺒﻴﻌﻲ ﺃﻭ‪‬ﻝﻲ ﻓﺭﺩﻱ ‪ x − y = 1 .‬ﻭ ‪ x + y = p‬ﻭﻤﻨﻪ ‪‬‬
‫‪ 2‬‬ ‫‪2 ‬‬
‫‪ 2‬ـ ا
‪ )$a2‬ا
‪"2‬ك ا‪!
SN‬د‪. -‬‬
‫‪ 28‬ﺃ ـ ‪. ppcm ( 26,12 ) = 156‬‬
‫ﺏ ـ ‪. ppcm (18, −15 ) = 90‬‬
‫‪4‬ـ ـ ‪. ppcm ( −12, −13) = 156‬‬
‫ﺩ ـ ‪. ppcm ( 230,128 ) = 14720‬‬
‫هـ ـ ‪ppcm ( 876,1028) = 225132‬‬
‫‪9 13 27 + 65 92‬‬
‫‪* 29‬‬
‫‪23‬‬
‫‪.‬‬ ‫‪+‬‬ ‫=‬ ‫=‬ ‫=‬
‫‪140 84‬‬ ‫‪420‬‬ ‫‪420 105‬‬
‫‪82 19 1243‬‬ ‫‪55‬‬ ‫‪23 1091‬‬
‫‪. +‬‬ ‫=‬ ‫*‬ ‫‪.‬‬ ‫‪+‬‬ ‫=‬ ‫*‬
‫‪75 210 1050‬‬ ‫‪195 216 2808‬‬
‫‪ 30‬ﻓﻲ ﻜل ﺤﺎﻝﺔ ﻤﻥ ﺍﻝﺤﺎﻻﺕ ﺍﻝﺘﺎﻝﻴﺔ ‪ ،‬ﻋﻴﻥ ﻗﻴﻡ ﺍﻝﻌﺩﺩ ﺍﻝﻁﺒﻴﻌﻲ ‪ a‬ﻏﻴﺭ ﺍﻝﻤﻌﺩﻭﻤﺔ ﺤﻴﺙ ‪:‬‬
‫أ ـ ‪ ppcm ( a,56 ) = 392‬؛ ﻨﻀﻊ ‪p gcd ( a,56 ) = d‬‬
‫ﻤﻌﻨﺎﻩ ' ‪ 56 = db ' ، a = da‬ﻭ ‪. p gcd ( a ', b ') = 1‬‬
‫ﻝﺩﻴﻨﺎ ‪ 56a = 392d‬ﺃﻱ ‪ a = 7d‬ﺇﺫﻥ ‪p gcd ( 7, b ') = 1‬‬
‫‪ 56 ‬‬
‫ﺃﻱ ‪ p gcd  7,  = 1‬ﺇﺫﻥ }‪d ∈ {7;14; 28;56‬‬
‫‪ d ‬‬
‫}‪. a ∈ {49;98;196;392‬‬
‫ب ـ ‪ ppcm ( a,18 ) = 630‬؛ }‪. a ∈ {35;70; 210;315;630‬‬
‫‪ 31‬ﺃ ـ ]‪ n ≡ 3[35‬ﻭ ]‪ n ≡ 3[ 28‬ﻤﻌﻨﺎﻩ ]‪ n − 3 ≡ 0 [35‬ﻭ ]‪ n − 3 ≡ 0 [ 28‬ﺇﺫﻥ ‪ n − 3‬ﻤﻀﺎﻋﻑ ﻤﺸﺘﺭﻙ ﻝﻠﻌﺩﺩﻴﻥ‬
‫‪ 35‬ﻭ ‪. 28‬‬
‫ﺃﺼﻐﺭ ﻗﻴﻤﺔ ﻝـ ‪ n − 3‬ﻫﻲ ‪ppcm ( 28,35) = 140‬‬
‫ﺇﺫﻥ ﺃﺼﻐﺭ ﻗﻴﻤﺔ ﻝﻠﻌﺩﺩ ‪ n‬ﻫﻲ ‪. 143‬‬
‫‪ a = 52 p + 7 = 64 p '+ 7 32‬ﻭﻤﻨﻪ ' ‪ a − 7 = 52 p = 64 p‬ﺇﺫﻥ ‪ a − 7 = ppmc ( 52, 64 ) = 832‬ﺃﻱ ‪. a = 839‬‬
‫‪ n ∈ ℕ∗ 33‬؛ ‪ p gcd ( n , 2n + 1) = 1‬ﻭﻤﻨﻪ )‪. ppcm ( n , 2n + 1) = n ( 2n + 1‬‬
‫‪ 34‬ﻝﺩﻴﻨﺎ )‪ppcm ( 2n + 2, 4n + 2 ) = 2 ppcm ( n + 1, 2n + 1‬‬
‫ﻭ ‪ p gcd ( n + 1, 2n + 1) = 1‬ﺇﺫﻥ‬
‫)‪ ppcm ( n + 1, 2n + 1) = ( n + 1)( 2n + 1‬ﻭﻤﻨﻪ‬
‫)‪. ppcm ( 2n + 2, 4n + 2 ) = 2 ( n + 1)( 2n + 1‬‬
‫‪ n ∈ ℕ∗ 35‬؛ )‪a = ( 32 n − 1)( 7 2 n − 1‬‬
‫)‪ a = ( 3n − 1)( 3n + 1)( 7 n − 1)( 7 n + 1‬؛‬
‫)‪ a = b ( 3n − 1)( 7 n − 1‬؛ ‪ a‬ﻤﻀﺎﻋﻑ ِـ ‪ b‬ﺇﺫﻥ‬
‫‪. ppcm ( a, b ) = a‬‬
‫‪ a + b = 60‬‬
‫‪ ‬؛ ‪، p gcd ( a, b ) = d‬‬ ‫‪ 40‬ﺃ ـ‬
‫‪ ppcm ( a, b ) = 40‬‬
‫' ‪ b = db ' ، a = da‬ﻤﻊ ‪. p gcd ( a ', b ') = 1‬‬
‫ﻝﺩﻴﻨﺎ ‪ ab = 40d‬ﻤﻌﻨﺎﻩ ‪. a 'b ' d = 40‬‬
‫ﻭﻝﺩﻴﻨﺎ ‪ a + b = 60‬ﻤﻌﻨﺎﻩ ‪. d ( a '+ b ') = 60‬‬
‫ﻭﺒﺎﻝﺘﺎﻝﻲ ‪ d‬ﻴﻘﺴﻡ ‪ p gcd ( 40, 60 ) = 20‬ﺃﻱ }‪. d ∈ {1, 2, 4,5,10, 20‬‬
‫‪60‬‬ ‫‪40‬‬ ‫‪40‬‬ ‫‪60‬‬
‫‪ x 2 −‬ﺃﻱ ‪dx 2 − 60x + 40 = 0‬‬ ‫‪x+‬‬ ‫= ' ‪ a 'b‬ﺇﺫﻥ ' ‪ a‬ﻭ ' ‪ b‬ﻫﻤﺎ ﺤﻼ ﺍﻝﻤﻌﺎﺩﻝﺔ ‪= 0‬‬ ‫= ' ‪ a '+ b‬ﻭ‬ ‫ﻝﺩﻴﻨﺎ‬
‫‪d‬‬ ‫‪d‬‬ ‫‪d‬‬ ‫‪d‬‬
‫ﺍﻝﻤﻤﻴﺯ ﺍﻝﻤﺨﺘﺼﺭ ﻫﻭ ‪ ∆ ' = 900 − 40d‬؛ ﺇﺫﺍ ﻜﺎﻥ }‪ d ∈ {1, 2, 4,5,10‬ﻓﺈﻥ‬
‫ﻭﺒﺎﻝﺘﺎﻝﻲ ﺍﻝﺤﻼﻥ ﻝﻴﺱ ﻁﺒﻴﻌﻴﺎﻥ‪.‬‬ ‫‪∆'∉ℕ‬‬
‫) ‪ ( a ', b ') = (1, 2‬ﺃﻭ )‪( a ', b ') = ( 2,1‬‬ ‫ﺇﺫﺍ ﻜﺎﻥ ‪ d = 20‬ﻓﺈﻥ ‪ ∆ ' = 100‬ﻭﻤﻨﻪ ‪ x ' = 1‬ﻭ ‪ x " = 2‬؛ ﻭﻤﻨﻪ‬
‫ﺇﺫﻥ ) ‪ ( a, b ) = ( 20, 40‬ﺃﻭ ) ‪. ( a, b ) = ( 40, 20‬‬
‫‪ a − b = 22932‬‬
‫‪ ‬؛ ‪ d‬ﻴﻘﺴﻡ ‪ p gcd ( 22932,98280 ) = 3276‬ﺃﻱ‬ ‫ﺏـ‬
‫‪ ppcm ( a, b ) = 98280‬‬
‫‪d ∈ {1, 2,3, 4, 6, 7,9,12,13,14,18, 21, 26, 28, 36, 39, 42,52, 63, 78,84, 91,117,126,156,182, 234,‬‬
‫}‪252, 273, 364, 468,546,819,1092,1638,3276‬‬
‫ﺍﻝﺤﺎﻝﺔ ﺍﻝﻭﺤﻴﺩﺓ ﺍﻝﺘﻲ ﺘﺤﻘﻕ ﻭﻫﻲ ‪ d = 3276‬ﻭﻨﺠﺩ ‪ a = 32760‬ﻭ ‪b = 9828‬‬
‫‪ 41‬ﺃ ـ ) ‪. ppcm ( a, b ) = 21× p gcd ( a, b‬‬
‫}) ‪ ( a, b ) ∈ {(d , 21d ) ; ( 3d , 7d ) ; ( 7d ,3d ) ; ( 21d , d‬ﻤﻊ‬ ‫‪ ab = md‬ﻤﻌﻨﺎﻩ ‪ a 'b ' d = m‬ﺃﻱ ‪ a 'b ' = 21‬ﻭﻨﺠﺩ‬
‫∗‪. d ∈ ℕ‬‬
‫ﺏ ـ ‪. ppcm ( a, b ) − p gcd ( a, b ) = 187‬‬
‫‪ m − d = 187‬ﻤﻌﻨﺎﻩ ‪d ( a 'b '− 1) = 187 = 11× 17‬‬
‫ﻭﻤﻨﻪ }‪ d ∈ {1,11,17,187‬ﺜﻡ ﻨﺩﺭﺱ ﺍﻝﺤﺎﻻﺕ‬
‫‪ 3‬ـ ‪/+‬ه ‪[  %‬و ‪.‬‬
‫‪ 46‬ﺃ ـ ‪ a = n‬؛ ‪−2a + b = 1 . b = 2n + 1‬‬
‫ﺏ ـ ‪ a = 2n + 3‬؛ ‪−3a + 2b = 1 . b = 3n + 5‬‬
‫‪ 47‬ﺘﻁﺒﻴﻕ ﻤﺒﺭﻫﻨﺔ ﺒﻴﺯﻭ‪ 11( 7n + 2 ) − 7 (11n + 3) = 1‬ﻤﻌﻨﺎﻩ ‪. PGCD (11n + 3, 7n + 2 ) = 1‬‬
‫‪. PGCD ( n , n 2 + 1) = 1‬‬
‫‪ n 49‬ﻋﺩﺩ ﻁﺒﻴﻌﻲ ﻏﻴﺭ ﻤﻌﺩﻭﻡ ‪.‬‬
‫ﺃ ـ ﺒﺎﻝﻨﺸﺭ ﻨﺠﺩ ‪. ( n 3 + 1) = n 2 ( n 4 + 2n ) + 1 :‬‬
‫‪2‬‬

‫ﺏ ـ ‪ ( n 3 + 1) − n 2 ( n 4 + 2n ) = 1‬ﺒﻭﻀﻊ ‪ α = n 3 + 1‬ﻭ ‪ β = − n 2‬ﺤﺴﺏ ﻤﺒﺭﻫﻨﺔ ﺒﻴﺯﻭ ﻴﻜﻭﻥ ﺍﻝﻌﺩﺩﺍﻥ‬


‫‪2‬‬

‫‪ n 3 + 1‬ﻭ ‪ n 4 + 2n‬ﺃﻭ‪‬ﻝﻴﻴﻥ ﻓﻴﻤﺎ ﺒﻴﻨﻬﻤﺎ ‪.‬‬


‫‪ 4‬ـ ‪/+‬ه ‪4 %‬ص ‪.‬‬
‫‪(1) . . . 2045x − 64 y‬‬ ‫‪ 60‬ﻨﻌﺘﺒﺭ ﻓﻲ ‪ ℤ 2‬ﺍﻝﻤﻌﺎﺩﻝﺔ ﺫﺍﺕ ﺍﻝﻤﺠﻬﻭل ) ‪ ( x , y‬ﺍﻝﺘﺎﻝﻴﺔ ‪= 1‬‬
‫‪. PGCD ( 2045, 64 ) = 1 (1‬‬
‫‪ (2‬ﺤﺴﺏ ﻤﺒﺭﻫﻨﺔ ﺒﻴﺯﻭ ﺍﻝﻤﻌﺎﺩﻝﺔ )‪ (1‬ﺘﻘﺒل ﻋﻠﻰ ﺍﻷﻗل ﺤﻼ ﻓﻲ ‪ ( 21, 671) . ℤ 2‬ﻫﻭ ﺤل ﺨﺎﺹ ﻝﻠﻤﻌﺎﺩﻝﺔ )‪. (1‬‬
‫‪ 2045x − 64 y = 1 (3‬ﻭ ‪ 2045 × 21 − 64 × 671 = 1‬ﺇﺫﻥ‬
‫‪ 2045 ( x − 21) − 64 ( y − 64 ) = 0‬ﺃﻱ ) ‪2045 ( x − 21) = 64 ( y − 64‬‬
‫‪ 64‬ﻴﻘﺴﻡ )‪ 2045 ( x − 21‬ﻭ ‪PGCD ( 2045, 64 ) = 1‬‬
‫ﺇﺫﻥ ﺤﺴﺏ ﻤﺒﺭﻫﻨﺔ ﻏﻭﺹ ‪ 64‬ﻴﻘﺴﻡ ‪ x − 21‬ﺃﻱ ‪ x − 21 = 64k‬ﻤﻊ ‪ k ∈ ℤ‬ﻭﺒﺎﻝﺘﻌﻭﻴﺽ ﻨﺠﺩ ‪. y − 64 = 2045k‬‬
‫‪ 5‬ـ ا
‪/2‬ه ‪ %‬ا
‪ S‬ة
‪. +V‬‬
‫‪ 710 ≡ 1[11] 66‬ﺤﺴﺏ ﻤﺒﺭﻫﻨﺔ ﻓﺭﻤﺎ‪.‬‬
‫) ‪ 7 2521 = 7 × 7 2520 = 7 × ( 710‬ﻭﻤﻨﻪ ]‪. 7 2521 ≡ 7 [11‬‬
‫‪252‬‬

‫‪ 68‬ﺤﺴﺏ ﻨﺘﻴﺠﺔ ﻓﺭﻤﺎ ‪ n 5 ≡ n [5] :‬ﻭ ]‪ n 3 ≡ n [3‬ﻭﻫﺫﺍ ﻤﻥ ﺃﺠل ‪ n ∈ ℤ‬ﻷﻥ ﻜل ﻤﻥ ‪ 5‬ﻭ ‪ 3‬ﺃﻭﻝﻲ‪.‬‬
‫]‪ n 5 ≡ n [5‬ﻤﻌﻨﺎﻩ ]‪n 5 − n ≡ 0 [5‬‬
‫ﻤﻥ ]‪ n 3 ≡ n [3‬ﻴﻨﺘﺞ ﺃﻥ ]‪ n 2 × n 3 ≡ n 2 × n [3‬ﻭﻤﻨﻪ ]‪ n 5 ≡ n [3‬ﺃﻱ ]‪ n 5 − n ≡ 0 [3‬ﺒﻤﺎ ﺃﻥ ‪ 5‬ﻭ ‪ 3‬ﺃﻭﻝﻴﺎﻥ ﻓﻴﻤﺎ‬
‫ﺒﻴﻨﻬﻤﺎ ﻓﺈﻥ ]‪. n 5 − n ≡ 0 [15‬‬
‫‪ x 3 ≡ x [3] (1 71‬ﻷﻥ ‪ 3‬ﺃﻭﻝﻲ‪.‬‬

‫≡ ‪x‬‬
‫‪0‬‬ ‫‪1‬‬ ‫‪2‬‬ ‫‪3‬‬ ‫]‪[ 4‬‬ ‫‪(2‬‬

‫≡ ‪x3‬‬
‫‪0‬‬ ‫‪1‬‬ ‫‪0‬‬ ‫‪3‬‬ ‫]‪[ 4‬‬
‫ﺇﺫﻥ ]‪ x ≡ 0 [ 4‬ﺃﻭ ]‪ x ≡ 1[ 4‬ﺃﻭ ]‪x ≡ 3 [ 4‬‬
‫‪ x 3 ≡ x [12] (3‬ﻤﻌﻨﺎﻩ ]‪ x ≡ 0 [ 4‬ﺃﻭ ]‪ x ≡ 1[ 4‬ﺃﻭ ]‪. x ≡ 3 [ 4‬‬

‫‪ 6‬ـ ‪  V‬ا


‪2:X‬ت‬
‫ﺽﺹﺵ ﺱ ﺯ ﺭ ﺫ ﺩ ﺥ ﺡ ﺝ ﺙ ﺕ ﺏ ﺃ‬
‫‪0 1 2 3 4 5 6 7 8 9 10 11 12 13 14‬‬

‫ﻱ ﻭ ﻩ ﻥ ﻡ ل ﻙ ﻕ ﻑ ﻍ ﻉ ﻅ ﻁ‬
‫‪15 16 17 18 19 20 21 22 23 24 25 26 27‬‬
‫‪ 73‬ﻨﻘﻭﻡ ﺒﻌﻤﻠﻴﺔ ﺍﻝﺘﺸﻔﻴﺭ ﺒﺎﺴﺘﻌﻤﺎل ﺍﻝﺘﺤﻭﻴل ‪ x ֏ y‬ﺤﻴﺙ ‪ y‬ﻫﻭ ﺒﺎﻗﻲ ﻗﺴﻤﺔ ‪ x + 3‬ﻋﻠﻰ ‪. 28‬‬
‫‪ (1‬ﺘﺸﻔﻴﺭ ﻜﻠﻤﺔ " ﺍﻝﺠﺯﺍﺌﺭ" ﻫﻭ " ﺜﻬﺩﺼﺜﺜﺵ "‪.‬‬
‫‪ (2‬ﻝﻴﻜﻥ ‪ y‬ﻤﻥ ﺍﻝﻤﺠﻤﻭﻋﺔ ‪ x + 3 ≡ y [ 28] ، N‬ﻤﻌﻨﺎﻩ ]‪ x ≡ y − 3[ 28‬؛ ﺇﺫﺍ ﻜﺎﻥ ‪ y ≥ 3‬ﻓﺈﻥ ‪ x = y − 3‬ﻭﺇﺫﺍ ﻜﺎﻥ‬
‫‪ y < 3‬ﻓﺈﻥ ‪x = y − 3 + 28 = y + 25‬‬
‫ﻭﺫﻭﺯ ‪ :‬ﻤﺤﻤﺩ‪.‬‬ ‫ﻝﺜﻐﻭﺍ ﺜﻬﺼﺎﺸﺜﺙ‪ :‬ﻓﺎﻁﻤﺔ ﺍﻝﺯﻫﺭﺍﺀ ؛‬ ‫‪ (3‬ﺤل ﺘﺸﻔﻴﺭ‪ :‬ﺘﺒﻀل‪ :‬ﻴﻭﺴﻑ ؛‬

You might also like